Sei sulla pagina 1di 276

POLITICAL LAW

1.

Distinguish
dominion.

sovereignty

from

Held:
Sovereignty is the right to
exercise the functions of a State to the
exclusion of any other State. It is
often referred to as the power of
imperium, which is defined as the
government authority possessed by
the State.
On the other hand,
dominion, or dominium, is the capacity
of the State to own or acquire property
such as lands and natural resources.
(Separate Opinion, Kapunan, J., in
Isagani Cruz v. Secretary of DENR,
G.R. No. 135385, Dec. 6, 2000, En
Banc, See Footnote 86)
2.
How did Spain acquire the
Philippines?
Held:
1.
The Philippines
passed to Spain by virtue of
discovery
and
conquest.
Consequently, all lands became the
exclusive patrimony and dominion of
the Spanish Crown.
The Spanish
Government
took
charge
of
distributing the lands by issuing royal
grants and concessions to Spaniards,
both military and civilian (Antonio H.
Noblejas, Land Titles and Deeds, p. 5
[1986]; These grants were better
known
as
repartimientos
and
encomiendas.
Repartimientos were
handouts to the military as fitting
reward for their services to the
Spanish crown.
The encomiendas
were given to Spaniards to administer
and develop with the right to receive
and enjoy for themselves the tributes
of the natives assigned to them.
Ponce, supra, p. 12, citing Benitez,
History of the Philippines, pp. 125126). Private land titles could only be
acquired from the government either
by purchase or by the various modes
of land grant from the Crown (Narciso
Pena, Registration of Land Titles and
Deeds, p. 2 [1994]).
(Separate
Opinion, Puno, J., in Cruz v.
Secretary of Environment and
Natural Resources, 347 SCRA 128,
166, En Banc [Per Curiam])
2. When Spain acquired sovereignty
over the Philippines by virtue of its
discovery and occupation thereof in
the 16th century and the Treaty of
Tordesillas of 1494 which it entered
into with Portugal (Under the Treaty of
Tordesillas, the world was divided
between Spain and Portugal, with the
former having exclusive power to
claim all lands and territories west of

the Atlantic Ocean demarcation line


[Lynch, The Legal Bases of Philippine
Colonial Sovereignty, 62 Phil. L J 279,
283 [1987]) the continents of Asia, the
Americas and Africa were considered
as terra nullius although already
populated by other peoples (See
Akehurst, a Modern Introduction to
International Law, 5th ed., 142-143).
The discovery and occupation by the
European States, who were then
considered as the only members of
the
international
community
of
civilized nations, of lands in the said
continents were deemed sufficient to
create title under international law
(See Cruz, International Law, 1996 ed.,
pp. 106-107)
(Separate Opinion,
Kapunan, J., in Isagani Cruz v.
Secretary of DENR, G.R. No.
135385, Dec. 6, 2000, 347 SCRA
128, 271, En Banc [Per Curiam])
3.
Discuss the concept of "jura
regalia" and how it evolved in the
Philippines. Does it negate native
title to lands held in private
ownership since time immemorial?
Held: Generally, under the concept of
jura regalia, private title to land must
be traced to some grant, express or
implied, from the Spanish Crown or its
successors, the American Colonial
government, and thereafter, the
Philippine Republic. The belief that
the Spanish Crown is the origin of all
land titles in the Philippines has
persisted because title to land must
emanate from some source for it
cannot issue forth from nowhere
(Pena, Registration of Land Titles and
Deeds, 1994 rev. ed., p. 15).
In its broad sense, the term "jura
regalia" refers to royal grants (1
Bouvier's Law Dictionary, 3rd revision,
p. 1759), or those rights which the
King has by virtue of his prerogatives
(Black's Law Dictionary, 6th ed., p.
1282). In Spanish law, it refers to a
right which the sovereign has over
anything in which a subject has a right
of property or propriedad (76 Corpus
Juris Secundum, citing Hart v. Burnett,
15 Cal. 530, 566). These were rights
enjoyed during feudal times by the
king as the sovereign.
The theory of the feudal system was
that title to all lands was originally
held by the King, and while the use of
lands was granted out to others who
were permitted to hold them under
certain
conditions,
the
King
theoretically
retained
the
title
(Washburn, p. 44; see also Williams,
Principles Of The Law On Real

Property, 6th ed. [1886], p. 2; Bigelow,


p. 2). By fiction of law, the King was
regarded as the original proprietor of
all lands, and the true and only source
of title, and from him all lands were
held
(Warvelle,
Abstracts
and
Examination of Title to Real Property
[1907], p. 18). The theory of jura
regalia was therefore nothing more
than a natural fruit of conquest (1
Dictionary of English Law [Jowitt, ed.]
p. 797).
The Regalian theory, however, does
not negate native title to lands held in
private
ownership
since
time
immemorial. In the landmark case of
Carino v. Insular Government (41 Phil.
935, 212 U.S. 449, 53 L. Ed. 594
[1909]), the United States Supreme
Court, reversing the decision of the
pre-war Philippine Supreme Court,
made the following pronouncement:
x x x Every presumption is and ought
to be taken against the Government in
a case like the present.
It might,
perhaps, be proper and sufficient to
say that when, as far back as
testimony or memory goes, the land
has been held by individuals under a
claim of private ownership, it will be
presumed to have been held in the
same way from before the Spanish
conquest, and never to have been
public land. x x x (Carino v. Insular
Government, supra note 75, at 941)
The above ruling institutionalized the
recognition of the existence of native
title to land, or ownership of land by
Filipinos by virtue of possession under
a claim of ownership since time
immemorial and independent of any
grant from the Spanish Crown, as an
exception to the theory of jura regalia.
xxx
Carino was decided by the U.S.
Supreme Court in 1909, at a time
when decisions of the U.S. Court were
binding as precedent in our jurisdiction
(Section 10, Philippine Bill of 1902).
We applied the Carino doctrine in the
1946 case of Oh Cho v. Director of
Lands (75 Phil. 890 [1946]), where we
stated that [a]ll lands that were not
acquired from the Government either
by purchase or by grant, belong to the
public domain, but [a]n exception to
the rule would be any land that should
have been in the possession of an
occupant and of his predecessors in
interest since time immemorial, for
such possession would justify the
presumption that the land had never

been part of the public domain or that


it had been private property even
before the Spanish conquest. (Id., at
892). (Separate Opinion, Kapunan,
J., in Isagani Cruz v. Secretary of
DENR, G.R. No. 135385, Dec. 6,
2000, 347 SCRA 128, 268-270, En
Banc [Per Curiam]
4.
What was the basis for the early
Spanish decrees embracing the theory
of jura regalia? Is this also the basis of
the declaration in Section 2, Article XII
of the 1987 Constitution that all lands
of the public domain are owned by the
State?
Consequently, did Spain
acquire title over all lands in the
Philippines in the 16th century?
Held: Dominium was the basis for the
early Spanish decrees embracing the
theory of jura regalia. The declaration
in Section 2, Article XII of the 1987
Constitution that all lands of the public
domain are owned by the State is
likewise founded on dominium.
If
dominium, not imperium, is the basis
of the theory of jura regalia, then the
lands which Spain acquired in the 16th
century were limited to non-private
lands, because it could only acquire
lands which were not yet privatelyowned or occupied by the Filipinos.
Hence, Spain acquired title only over
lands which were unoccupied and
unclaimed,
i.e.,
public
lands.
(Separate Opinion, Kapunan, J., in
Isagani Cruz v. Secretary of DENR,
G.R. No. 135385, Dec. 6, 2000, En
Banc, See Footnote 86)
5.
What
is
the
Doctrine
Constitutional Supremacy?

of

Held:
Under the doctrine of
constitutional supremacy, if a law or
contract violates any norm of the
Constitution, that law or contract,
whether
promulgated
by
the
legislative or by the executive branch
or entered into by private persons for
private purposes, is null and void and
without any force and effect. Thus,
since
the
Constitution
is
the
fundamental, paramount and supreme
law of the nation, it is deemed written
in every statute and contract.
(Manila Prince Hotel v. GSIS, 267
SCRA 408 [1997] [Bellosillo])
6.
What are self-executing and
non-self executing provisions of the
Constitution?
Held: A provision which lays down a
general principle, such as those found
in Article II of the 1987 Constitution, is

usually not self-executing.


But a
provision which is complete in itself
and becomes operative without the
aid of supplementary or enabling
legislation, or that which supplies
sufficient rule by means of which the
right it grants may be enjoyed or
protected, is self-executing. Thus a
constitutional
provision
is
selfexecuting if the nature and extent of
the right conferred and the liability
imposed are fixed by the Constitution
itself, so that they can be determined
by an examination and construction of
its terms, and there is no language
indicating that the subject is referred
to the legislature for action. (Manila
Prince Hotel v. GSIS, 267 SCRA
408 [1997] [Bellosillo])
7.
Are
provisions
of
the
Constitution self-executing or non-self
executing? Why?
Held: Unless it is expressly provided
that a legislative act is necessary to
enforce a constitutional mandate, the
presumption now is that all provisions
are self-executing. If the constitutional
provisions are treated as requiring
legislation instead of self-executing,
the legislature would have the power
to ignore and practically nullify the
mandate of the fundamental law. This
can be cataclysmic. (Manila Prince
Hotel v. GSIS, 267 SCRA 408
[1997] [Bellosillo])
8.
Is the Filipino First Policy
expressed in Section 10, Article XII of
the Constitution a self-executing
provision?
Held: Yes. It is a mandatory, positive
command which is complete in itself
and which needs no further guidelines
or implementing laws or rules for its
enforcement. From its very words the
provision does not require any
legislation to put it in operation. It is
per se judicially enforceable. When
our Constitution mandates that [i]n
the grant of rights, privileges, and
concessions covering the national
economy and patrimony, the State
shall give preference to qualified
Filipinos, it means just that qualified
Filipinos must be preferred. (Manila
Prince Hotel v. GSIS, G.R. No.
118295, May 2, 1997, 267 SCRA
408 [Bellosillo])
9.
Give examples of non-self
executing
provisions
of
the
Constitution.

Held: By its very nature, Article II of


the Constitution is a declaration of
principles and state policies. These
principles in Article II are not intended
to be self-executing principles ready
for enforcement through the courts.
They are used by the judiciary as aids
or as guides in the exercise of its
power of judicial review, and by the
legislature in its enactment of laws.
As held in the leading case of
Kilosbayan, Incorporated v. Morato
(246 SCRA 540, 564, July 17, 1995),
the principles and state policies
enumerated in Article II and some
sections of Article XII are not selfexecuting provisions, the disregard of
which can give rise to a cause of
action in courts. They do not embody
judicially enforceable constitutional
rights but guidelines for legislation.
(Tanada v. Angara, 272 SCRA 18
[1997], En Banc [Panganiban])
10.
When are acts of persons
considered State action covered
by the Constitution?
Held: In constitutional jurisprudence,
the act of persons distinct from the
government are considered state
action covered by the Constitution (1)
when the activity it engages in is a
public function; (2) when the
government is so significantly involved
with the private actor as to make the
government responsible for his action;
and (3) when the government has
approved or authorized the action.
(Manila Prince Hotel v. GSIS, 267
SCRA 408 [1997] [Bellosillo])
The Doctrine of State Immunity
from Suit
11.
Discuss the basis of the doctrine
of State immunity from suit.
Held: The basic postulate enshrined
in the Constitution that [t]he State
may not be sued without its consent,
reflects nothing less than a recognition
of the sovereign character of the State
and an express affirmation of the
unwritten rule effectively insulating it
from the jurisdiction of courts. It is
based on the very essence of
sovereignty.
As has been aptly
observed by Justice Holmes, a
sovereign is exempt from suit, not
because of any formal conception or
obsolete theory, but on the logical and
practical ground that there can be no
legal right as against the authority
that makes the law on which the right
depends. True, the doctrine, not too

infrequently, is derisively called the


royal
prerogative of dishonesty
because it grants the state the
prerogative to defeat any legitimate
claim against it by simply invoking its
non-suability. We have had occasion
to explain in its defense, however, that
a continued adherence to the doctrine
of non-suability cannot be deplored,
for the loss of governmental efficiency
and the obstacle to the performance
of its multifarious functions would be
far greater in severity than the
inconvenience that may be caused
private parties, if such fundamental
principle is to be abandoned and the
availability of judicial remedy is not to
be
accordingly
restricted.
(Department of Agriculture v.
NLRC, 227 SCRA 693, Nov. 11,
1993 [Vitug])
12.
Is the rule absolute, i.e.,
that the State may not be sued at
all?
How may consent of the
State to be sued given?
Held: The rule, in any case, is not
really absolute for it does not say that
the state may not be sued under any
circumstances. On the contrary, as
correctly phrased, the doctrine only
conveys, the state may not be sued
without its consent; its clear import
then is that the State may at times be
sued. The State's consent may be
given either expressly or impliedly.
Express consent may be made
through
a
general
law
(i.e.,
Commonwealth Act No. 327, as
amended by Presidential Decree No.
1445 [Sections 49-50], which requires
that all money claims against the
government must first be filed with
the Commission on Audit which must
act upon it within sixty days.
Rejection of the claim will authorize
the claimant to elevate the matter to
the Supreme Court on certiorari and,
in effect, sue the State thereby) or a
special law. In this jurisdiction, the
general law waiving the immunity of
the state from suit is found in Act No.
3083,
where
the
Philippine
government consents and submits to
be sued upon any money claim
involving liability arising from contract,
express or implied, which could serve
as a basis of civil action between the
private parties. Implied consent, on
the other hand, is conceded when the
State itself commences litigation, thus
opening itself to a counterclaim or
when it enters into a contract. In this
situation, the government is deemed
to have descended to the level of the
other contracting party and to have

divested itself of its sovereign


immunity.
This rule x x x is not,
however, without qualification. Not all
contracts
entered
into
by
the
government operate as a waiver of its
non-suability; distinction must still be
made between one which is executed
in the exercise of its sovereign
function and another which is done in
its proprietary capacity.
In United States of America v. Ruiz
(136 SCRA 487), where the questioned
transaction
dealt
with
the
improvements on the wharves in the
naval installation at Subic Bay, we
held:
The traditional rule of immunity
exempts a State from being sued in
the courts of another State without its
consent or waiver.
This rule is a
necessary
consequence
of
the
principle of independence and equality
of States.
However, the rules of
International Law are not petrified;
they are constantly developing and
evolving. And because the activities
of states have multiplied, it has been
necessary to distinguish them between sovereign and governmental
acts (jure imperii) and private,
commercial and proprietary acts (jure
gestionis). The result is that State
immunity now extends only to acts
jure
imperii.
The
restrictive
application of State immunity is now
the rule in the United States, the
United Kingdom and other states in
Western Europe.
Xxx
The restrictive application of State
immunity is proper only when the
proceedings arise out of commercial
transactions of the foreign sovereign,
its commercial activities or economic
affairs. Stated differently, a State may
be said to have descended to the level
of an individual and can thus be
deemed to have tacitly given its
consent to be sued only when it enters
into business contracts. It does not
apply where the contracts relate to the
exercise of its sovereign functions. In
this case the projects are an integral
part of the naval base which is
devoted to the defense of both the
United States and the Philippines,
indisputably
a
function
of
the
government of the highest order; they
are not utilized for nor dedicated to
commercial or business purposes.
(Department of Agriculture v.
NLRC, 227 SCRA 693, Nov. 11,
1993 [Vitug])

13.
When is a suit against a public
official deemed to be a suit against the
State? Discuss.
Held:
1.
The doctrine of state
immunity
from
suit
applies
to
complaints filed against public officials
for acts done in the performance of
their duties. The rule is that the suit
must be regarded as one against the
State where the satisfaction of the
judgment against the public official
concerned will require the State itself
to perform a positive act, such as
appropriation of the amount necessary
to pay the damages awarded to the
plaintiff.
The rule does not apply where the
public official is charged in his official
capacity for acts that are unlawful and
injurious to the rights of others. Public
officials are not exempt, in their
personal capacity, from liability arising
from acts committed in bad faith.
Neither does it apply where the public
official is clearly being sued not in his
official capacity but in his personal
capacity,
although
the
acts
complained of may have been
committed while he occupied a public
position. (Amado J. Lansang v. CA,
G.R. No. 102667, Feb. 23, 2000,
2nd Div. [Quisumbing])
2. As early as 1954, this Court has
pronounced that an officer cannot
shelter himself by the plea that he is a
public agent acting under the color of
his office when his acts are wholly
without authority.
Until recently in
1991 (Chavez v. Sandiganbayan, 193
SCRA 282 [1991]), this doctrine still
found application, this Court saying
that immunity from suit cannot
institutionalize
irresponsibility
and
non-accountability
nor
grant
a
privileged status not claimed by any
other
official
of
the
Republic.
(Republic v. Sandoval, 220 SCRA
124, March 19, 1993, En Banc
[Campos, Jr.])
14.
State instances when a suit
against the State is proper.
Held: Some instances when a suit
against the State is proper are:
1)

When the Republic is sued


by name;

2)

When the suit is against an


unincorporated government agency;
3)
When the suit is on its face
against a government officer but the

case is such that ultimate liability will


belong not to the officer but to the
government.
Republic v. Sandoval, 220 SCRA
124, March 19, 1993, En Banc
[Campos, Jr.])
15.
Has the government waived its
immunity from suit in the Mendiola
massacre, and, therefore, should
indemnify the heirs and victims of the
Mendiola incident? Consequently, is
the suit filed against the Republic by
petitioners in said case really a suit
against the State?
Held: Petitioners x x x advance the
argument that the State has impliedly
waived its sovereign immunity from
suit. It is their considered view that by
the recommendation made by the
Commission for the government to
indemnify the heirs and victims of the
Mendiola incident and by the public
addresses made by then President
Aquino in the aftermath of the killings,
the State has consented to be sued.
Xxx
This is not a suit against the State with
its consent.
Firstly, the recommendation made by
the
Commission
regarding
indemnification of the heirs of the
deceased and the victims of the
incident by the government does not
in any way mean that liability
automatically attaches to the State. It
is important to note that A.O. 11
expressly states that the purpose of
creating the Commission was to have
a
body
that
will
conduct
an
investigation of the disorder, deaths
and casualties that took place. In the
exercise of its functions, A.O. 11
provides guidelines, and what is
relevant to Our discussion reads:
1.
Its conclusions regarding the
existence of probable cause for the
commission of any offense and of the
persons probably guilty of the same
shall be sufficient compliance with the
rules on preliminary investigation and
the charges arising therefrom may be
filed directly with the proper court.
In effect, whatever may be the
findings of the Commission, the same
shall only serve as the cause of action
in the event that any party decides to
litigate his/her claim. Therefore, the
Commission is merely a preliminary
venue. The Commission is not the end
in itself. Whatever recommendation it

makes cannot in any way bind the


State
immediately,
such
recommendation not having become
final and executory. This is precisely
the essence of it being a fact-finding
body.
Secondly, whatever acts or utterances
that then President Aquino may have
done or said, the same are not
tantamount to the State having
waived its immunity from suit. The
Presidents act of joining the marchers,
days after the incident, does not mean
that there was an admission by the
State of any liability. In fact to borrow
the words of petitioner x x x, it was
an act of solidarity by the government
with
the
people.
Moreover,
petitioners rely on President Aquinos
speech promising that the government
would address the grievances of the
rallyists. By this alone, it cannot be
inferred that the State has admitted
any liability, much less can it be
inferred that it has consented to the
suit.
Although consent to be sued may be
given impliedly, still it cannot be
maintained that such consent was
given considering the circumstances
obtaining in the instant case.
Thirdly, the case does not qualify as a
suit against the State.
Xxx
While the Republic in this case is sued
by name, the ultimate liability does
not pertain to the government.
Although the military officers and
personnel, then party defendants,
were
discharging
their
official
functions when the incident occurred,
their functions ceased to be official the
moment they exceeded their authority.
Based on the Commission findings,
there was lack of justification by the
government forces in the use of
firearms. Moreover, the members of
the police and military crowd dispersal
units committed a prohibited act
under B.P. Blg. 880 as there was
unnecessary firing by them in
dispersing the marchers.
As early as 1954, this Court has
pronounced that an officer cannot
shelter himself by the plea that he is a
public agent acting under the color of
his office when his acts are wholly
without authority.
Until recently in
1991 (Chavez v. Sandiganbayan, 193
SCRA 282 [1991]), this doctrine still
found application, this Court saying

that immunity from suit cannot


institutionalize
irresponsibility
and
non-accountability
nor
grant
a
privileged status not claimed by any
other official of the Republic.
The
military and police forces were
deployed to ensure that the rally
would be peaceful and orderly as well
as to guarantee the safety of the very
people that they are duty-bound to
protect. However, the facts as found
by the trial court showed that they
fired at the unruly crowd to disperse
the latter.
While it is true that nothing is better
settled than the general rule that a
sovereign state and its political
subdivisions cannot be sued in the
courts except when it has given its
consent, it cannot be invoked by both
the military officers to release them
from any liability, and by the heirs and
victims to demand indemnification
from the government. The principle of
state immunity from suit does not
apply, as in this case, when the relief
demanded by the suit requires no
affirmative official action on the part of
the State nor the affirmative discharge
of any obligation which belongs to the
State in its political capacity, even
though the officers or agents who are
made defendants claim to hold or act
only by virtue of a title of the state
and as its agents and servants. This
Court has made it quite clear that
even a high position in the
government does not confer a license
to persecute or recklessly injure
another.
The inescapable conclusion is that the
State cannot be held civilly liable for
the deaths that followed the incident.
Instead, the liability should fall on the
named defendants in the lower court.
In line with the ruling of this Court in
Shauf v. Court of Appeals (191 SCRA
713 [1990]), herein public officials,
having been found to have acted
beyond the scope of their authority,
may be held liable for damages.
(Republic v. Sandoval, 220 SCRA
124, March 19, 1993, En Banc
[Campos, Jr.])
16.
May the Government validly
invoke the doctrine of State immunity
from suit if its invocation will serve as
an instrument for perpetrating an
injustice on a citizen?
Held: To our mind, it would be
the apex of injustice and highly
inequitable
for
us
to
defeat

petitioners-contractors right to be
duly compensated for actual work
performed and services rendered,
where both the government and the
public have, for years, received and
accepted benefits from said housing
project and reaped the fruits of
petitioners-contractors honest toil and
labor.
Incidentally,
respondent
likewise argues that the State may not
be sued in the instant case, invoking
the constitutional doctrine of Nonsuability of the State, otherwise known
as the Royal Prerogative of Dishonesty.
Respondents
argument
is
misplaced inasmuch as the principle of
State immunity finds no application in
the case before us.
Under
these
circumstances,
respondent may not validly invoke the
Royal Prerogative of Dishonesty and
conveniently hide under the States
cloak of invincibility against suit,
considering that this principle yields to
certain settled exceptions.
True
enough, the rule, in any case, is not
absolute for it does not say that the
state may not be sued under any
circumstances. (Citations omitted)
Thus, in Amigable v. Cuenca,
this Court, in effect, shred the
protective shroud which shields the
state from suit, reiterating our decree
in the landmark case of Ministerio v.
CFI of Cebu that the doctrine of
governmental immunity from suit
cannot serve as an instrument for
perpetrating an injustice on a citizen.
It is just as important, if not more so,
that there be fidelity to legal norms on
the part of officialdom if the rule of law
were to be maintained.
(Citations
omitted)
Although the Amigable and
Ministerio cases generously tackled
the issue of the States immunity from
suit vis a vis the payment of just
compensation
for
expropriated
property, this Court nonetheless finds
the doctrine enunciated in the

aforementioned cases applicable to


the instant controversy, considering
that the ends of justice would be
subverted if we were to uphold, in this
particular
instance,
the
States
immunity from suit.
To be sure, this Court as the
staunch guardian of the citizens rights
and welfare cannot sanction an
injustice so patent on its face, and
allow itself to be an instrument in the
perpetration thereof.
Justice and
equity sternly demand that the States
cloak of invincibility against suit be
shred in this particular instance, and
that petitioners-contractors be duly
compensated on the basis of
quantum meruit for construction
done on the public works housing
project. (EPG Construction Co. v.
Vigilar, 354 SCRA 566, Mar.16,
2001, 2nd Div. [Buena])
Citizenship
17.
To what citizenship principle
does the Philippines adhere to?
Explain, and give illustrative case.
Held:
The Philippine law on
citizenship adheres to the principle of
jus sanguinis.
Thereunder, a child
follows the nationality or citizenship of
the parents regardless of the place of
his/her birth, as opposed to the
doctrine of jus soli which determines
nationality or citizenship on the basis
of place of birth.
Private respondent Rosalind
Ybasco Lopez was born on May 16,
1934 in Napier Terrace, Broome,
Western Australia, to the spouses,
Telesforo Ybasco, a Filipino citizen and
native of Daet, Camarines Norte, and
Theresa Marquez, an Australian.
Historically, this was a year before the
1935 Constitution took into effect and
at that time, what served as the
Constitution of the Philippines were
the principal organic acts by which the
United States governed the country.
These were the Philippine Bill of July 1,
1902 and the Philippine Autonomy Act
of August 29, 1916, also known as the
Jones Law.
Among others, these laws
defined who were deemed to be
citizens of the Philippine Islands. x x x
Under both organic acts, all
inhabitants of the Philippines who
were Spanish subjects on April 11,
1899 and resided therein including

their children are deemed to be


Philippine
citizens.
Private
respondents father, Telesforo Ybasco,
was born on January 5, 1879 in Daet,
Camarines
Norte,
a
fact
duly
evidenced by a certified true copy of
an entry in the Registry of Births.
Thus, under the Philippine Bill of 1902
and the Jones Law, Telesforo Ybasco
was deemed to be a Philippine citizen.
By virtue of the same laws, which
were the laws in force at the time of
her birth, Telesforos daughter, herein
private respondent Rosalind Ybasco
Lopez, is likewise a citizen of the
Philippines.

Law (Act No. 2927), and by Republic


Act No. 530. (Antonio Bengson III
v. HRET, G.R. No. 142840, May 7,
2001, En Banc [Kapunan])
19.
To be naturalized, what must an
applicant prove? When and what are
the conditions before the decision
granting
Philippine
citizenship
becomes executory?
Held: To be naturalized, an applicant
has to prove that he possesses all the
qualifications and
none
of the
disqualifications provided by law to
become a Filipino citizen. The decision
granting
Philippine
citizenship
becomes executory only after two (2)
years from its promulgation when the
court is satisfied that during the
intervening period, the applicant has
(1) not left the Philippines; (2) has
dedicated himself to a lawful calling or
profession; (3) has not been convicted
of any offense or violation of
government promulgated rules; or (4)
committed any act prejudicial to the
interest of the nation or contrary to
any government announced policies
(Section 1, R.A. 530). (Antonio
Bengson III v. HRET, G.R. No.
142840, May 7, 2001, En Banc
[Kapunan])

The signing into law of the 1935


Philippine Constitution has established
the principle of jus sanguinis as basis
for the acquisition of Philippine
citizenship x x x. So also, the principle
of jus sanguinis, which confers
citizenship
by
virtue
of
blood
relationship,
was
subsequently
retained under the 1973 and 1987
Constitutions. Thus, the herein private
respondent, Rosalind Ybasco Lopez, is
a Filipino citizen, having been born to
a Filipino father. The fact of her being
born in Australia is not tantamount to
her losing her Philippine citizenship. If
Australia follows the principle of jus
soli, then at most, private respondent
can also claim Australian citizenship
resulting to her possession of dual
citizenship.
(Valles v. COMELEC,
337 SCRA 543, Aug. 9, 2000, En
Banc [Purisima])
18.
What are the ways of acquiring
citizenship? Discuss.
Held:
There are two ways of
acquiring citizenship: (1) by birth, and
(2) by naturalization. These ways of
acquiring citizenship correspond to the
two kinds of citizens: the natural-born
citizen, and the naturalized citizen. A
person who at the time of his birth is a
citizen of a particular country, is a
natural-born citizen thereof.
As defined in the x x x Constitution,
natural-born
citizens
are
those
citizens of the Philippines from birth
without having to perform any act to
acquire or perfect his Philippine
citizenship.
On the other hand, naturalized citizens
are those who have become Filipino
citizens
through
naturalization,
generally under Commonwealth Act
No. 473, otherwise known as the
Revised Naturalization Law, which
repealed the former Naturalization

20.
What qualifications must be
possessed by an applicant for
naturalization?
Held: Section 2, Act 473 provides the
following qualifications:
(a)
(b)
(c)

(d)

(e)

He must be not less than 21


years of age on the day of the hearing
of the petition;
He must have resided in the
Philippines for a continuous period of
not less than ten years;
He must be of good moral
character
and
believes
in
the
principles underlying the Philippine
Constitution,
and
must
have
conducted himself in a proper and
irreproachable manner during the
entire period of his residence in the
Philippines in his relation with the
constituted government as well as
with the community in which he is
living;
He must own real estate in
the Philippines worth not less than five
thousand pesos, Philippine currency,
or must have some known lucrative
trade, profession, or lawful occupation;
He must be able to speak
and write English or Spanish and any
of the principal languages; and

(f)

He must have enrolled his


minor children of school age, in any of
the public schools or private schools
recognized by the Bureau of Private
Schools of the Philippines where
Philippine history, government and
civic are taught or prescribed as part
of the school curriculum, during the
entire period of the residence in the
Philippines required of him prior to the
hearing
of
his
petition
for
naturalization as Philippine citizen.
(Antonio Bengson III v. HRET, G.R.
No. 142840, May 7, 2001, En Banc
[Kapunan])
21.
What are the disqualifications
under Section 4, Act 473, in an
application for naturalization?
Held: Section 4, Act 473, provides
the following disqualifications:

(a)

(b)

(c)
(d)
(e)
(f)

(g)

(h)

He must not be opposed to


organized government or affiliated
with any association or group of
persons who uphold and teach
doctrines opposing all organized
governments;
He must not be defending or
teaching the necessity or propriety of
violence,
personal
assault,
or
assassination for the success and
predominance of their ideas;
He must not be a polygamist
or believer in the practice of
polygamy;
He must not have been
convicted of any crime involving moral
turpitude;
He must not be suffering
from mental alienation or incurable
contagious diseases;
He must have, during the
period of his residence in the
Philippines (or not less than six
months before filing his application),
mingled socially with the Filipinos, or
who have not evinced a sincere desire
to learn and embrace the customs,
traditions and ideals of the Filipinos;
He must not be a citizen or
subject of a nation with whom the
Philippines is at war, during the period
of such war;
He must not be a citizen or
subject of a foreign country whose
laws do not grant Filipinos the right to
become
naturalized
citizens
or
subjects thereof.
(Antonio Bengson III v. HRET, G.R.
No. 142840, May 7, 2001, En Banc
[Kapunan])
22.
Can a legitimate child born
under the 1935 Constitution of a
Filipino mother and an alien father

validly elect Philippine citizenship


fourteen (14) years after he has
reached the age of majority?
Held: Under Article IV, Section 1(3) of
the 1935 Constitution, the citizenship
of a legitimate child born of a Filipino
mother and an alien father followed
the citizenship of the father, unless,
upon reaching the age of majority, the
child elected Philippine citizenship.
C.A. No. 625 which was enacted
pursuant to Section 1(3), Article IV of
the 1935 Constitution, prescribes the
procedure that should be followed in
order to make a valid election of
Philippine citizenship. However, the
1935 Constitution and C.A. No. 625 did
not prescribe a time period within
which the election of Philippine
citizenship should be made. The 1935
Charter only provides that the election
should be made upon reaching the
age of majority. The age of majority
then commenced upon reaching
twenty-one (21) years. In the opinions
of the Secretary of Justice on cases
involving the validity of election of
Philippine citizenship, this dilemma
was resolved by basing the time
period on the decisions of this Court
prior to the effectivity of the 1935
Constitution. In these decisions, the
proper period for electing Philippine
citizenship was, in turn, based on the
pronouncements of the Department of
State of the United States Government
to the effect that the election should
be made within a reasonable time
after attaining the age of majority.
The phrase reasonable time has
been interpreted to mean that the
election should be made within three
(3) years from reaching the age of
majority.
The span of fourteen (14) years that
lapsed from the time that person
reached the age of majority until he
finally expressed his intention to elect
Philippine citizenship is clearly way
beyond the contemplation of the
requirement
of
electing
upon
reaching the age of majority.
Philippine citizenship can never be
treated like a commodity that can be
claimed when needed and suppressed
when convenient.
One who is
privileged
to
elect
Philippine
citizenship has only an inchoate right
to such citizenship.
As such, he
should avail of the right with fervor,
enthusiasm and promptitude.
(Re:
Application for Admission to the
Philippine Bar, Vicente D. Ching,

Bar Matter No. 914, Oct. 1, 1999,


En Banc [Kapunan])
23.
How may Philippine citizenship
be renounced? Is the application for
an alien certificate of registration, and
the possession of foreign passport,
tantamount to acts of renunciation of
Philippine citizenship?
Held: Petitioner also contends that
even on the assumption that the
private respondent is a Filipino citizen,
she has nonetheless renounced her
Philippine citizenship. To buttress this
contention, petitioner cited private
respondents application for an alien
Certificate of Registration (ACR) and
Immigrant Certificate of Residence
(ICR), on September 19, 1988, and the
issuance to her of an Australian
passport on March 3, 1988.
Xxx
In order that citizenship may be
lost by renunciation, such renunciation
must
be
express.
Petitioners
contention that the application of
private respondent for an alien
certificate of registration, and her
Australian passport, is bereft of merit.
This issue was put to rest in the case
of Aznar v. COMELEC (185 SCRA 703
[1990]) and in the more recent case of
Mercado v. Manzano and COMELEC
(G.R. No. 135083, 307 SCRA 630, May
26, 1999).
In the case of Aznar, the Court
ruled that the mere fact that he is an
American did not mean that he is no
longer a Filipino, and that an
application for an alien certificate of
registration was not tantamount to
renunciation
of
his
Philippine
citizenship.
And, in Mercado v. Manzano and
COMELEC, it was held that the fact
that
respondent
Manzano
was
registered as an American citizen in
the Bureau of Immigration and
Deportation and was holding an
American passport on April 22, 1997,
only a year before he filed a certificate
of candidacy for vice-mayor of Makati,
were just assertions of his American
nationality before the termination of
his American citizenship.
Thus, the mere fact that private
respondent Rosalind Ybasco Lopez was
a holder of an Australian passport and
had an alien certificate of registration
are not acts constituting an effective
renunciation of citizenship and do not

militate against her claim of Filipino


citizenship.
For renunciation to
effectively result in the loss of
citizenship, the same must be express.
As held by this Court in the aforecited
case of Aznar, an application for an
alien certificate of registration does
not amount to an express renunciation
or repudiation of ones citizenship.
The application of the herein private
respondent for an alien certificate of
registration, and her holding of an
Australian passport, as in the case of
Mercado v. Manzano, were mere acts
of
assertion
of
her
Australian
citizenship before she effectively
renounced the same. Thus, at the
most, private respondent had dual
citizenship she was an Australian
and a Filipino, as well.
Moreover,
under
Commonwealth Act 63, the fact that a
child of Filipino parent/s was born in
another country has not been included
as a ground for losing ones Philippine
citizenship. Since private respondent
did not lose or renounce her Philippine
citizenship, petitioners claim that
respondent must go through the
process of repatriation does not hold
water.
(Valles v. COMELEC, 337
SCRA 543, Aug. 9, 2000, En Banc
[Purisima])
24.
How may Filipino citizens who
lost their citizenship reacquire the
same?
Answer: Filipino citizens who have
lost their citizenship may x x x
reacquire the same in the manner
provided by law. Commonwealth Act
No. 63 enumerates the three modes
by which Philippine citizenship may be
reacquired by a former citizen: (1) by
naturalization, (2) by repatriation, and
(3) by direct act of Congress.
(Frivaldo v. COMELEC, 257 SCRA
727, June 28, 1996, En Banc
[Panganiban]; Antonio Bengson III
v. HRET, G.R. No. 142840, May 7,
2001, En Banc [Kapunan])
25.
Distinguish naturalization from
repatriation.
Held: Naturalization is a mode for
both acquisition and reacquisition of
Philippine citizenship. As a mode of
initially
acquiring
Philippine
citizenship, naturalization is governed
by Commonwealth Act No. 473, as
amended.
On the other hand,
naturalization
as
a
mode
for
reacquiring Philippine citizenship is
governed by Commonwealth Act No.

63 (An Act Providing for the Ways in


Which Philippine Citizenship May Be
Lost or Reacquired [1936]). Under this
law, a former Filipino citizen who
wishes
to
reacquire
Philippine
citizenship must possess certain
qualifications and
none
of the
disqualifications mentioned in Section
4 of C.A. 473.
Repatriation, on the other hand, may
be had under various statutes by
those who lost their citizenship due to:
(1) desertion of the armed forces
(Section 4, C.A. No. 63); (2) service in
the armed forces of the allied forces in
World War II (Section 1, Republic Act
No. 965 [1953]); (3) service in the
Armed Forces of the United States at
any other time (Sec. 1, Republic Act
No. 2630 [1960]); (4) marriage of a
Filipino woman to an alien (Sec. 1,
Republic Act No. 8171 [1995]); and (5)
political and economic necessity (Ibid).
As distinguished from the lengthy
process of naturalization, repatriation
simply consists of the taking of an
oath of allegiance to the Republic of
the Philippines and registering said
oath in the Local Civil Registry of the
place where the person concerned
resides or last resided.
In Angat v. Republic (314 SCRA 438
[1999]), we held:
[P]arenthetically, under these statutes
(referring to RA Nos. 965 and 2630),
the person desiring to reacquire
Philippine citizenship would not even
be required to file a petition in court,
and all that he had to do was to take
an oath of allegiance to the Republic
of the Philippines and to register that
fact with the civil registry in the place
of his residence or where he had last
resided in the Philippines.
Moreover, repatriation results in the
recovery of the original nationality.
This means that a naturalized Filipino
who lost his citizenship will be restored
to his prior status as a naturalized
Filipino citizen. On the other hand, if
he was originally a natural-born citizen
before
he
lost
his
Philippine
citizenship, he will be restored to his
former status as a natural-born
Filipino.
(Antonio Bengson III v.
HRET, G.R. No. 142840, May 7,
2001, En Banc [Kapunan])
26.
Who may validly avail
repatriation under R.A. No. 8171?

Held:
R.A. No. 8171, which has
lapsed into law on October 23, 1995,
is an act providing for the repatriation
(a) of Filipino women who have lost
their Philippine citizenship by marriage
to aliens and (b) of natural-born
Filipinos who have lost their Philippine
citizenship on account of political or
economic necessity. (Gerardo Angat
v. Republic, G.R. No. 132244,
Sept. 14, 1999 [Vitug])
27.
Before what agency should
application for repatriation under R.A
8171 be filed?
Held: Under Section 1 of P.D. No.
725, dated June 5, 1975, amending
C.A. No. 63, an application for
repatriation could be filed with the
Special Committee on Naturalization
chaired by the Solicitor General with
the Undersecretary of Foreign Affairs
and the Director of the National
Intelligence Coordinating Agency as
the other members.
Although the
agency was deactivated by virtue of
President
Corazon
C.
Aquinos
Memorandum of March 27, 1987, it
was not, however, abrogated.
The
Committee was reactivated on June 8,
1995. Hence, the application should
be filed with said Agency, not with the
Regional Trial Court. (Gerardo Angat
v. Republic, G.R. No. 132244,
Sept. 14, 1999 [Vitug])
28.
May a natural-born Filipino who
became an American citizen still be
considered a natural-born Filipino upon
his
reacquisition
of
Philippine
citizenship and, therefore, qualified to
run for Congressman?
Held:
Repatriation results in the
recovery of the original nationality.
This means that a naturalized Filipino
who lost his citizenship will be restored
to his prior status as a naturalized
Filipino citizen. On the other hand, if
he was originally a natural-born citizen
before
he
lost
his
Philippine
citizenship, he will be restored to his
former status as a natural-born
Filipino.
In respondent Cruzs case, he lost his
Filipino citizenship when he rendered
service in the Armed Forces of the
United
States.
However,
he
subsequently reacquired Philippine
citizenship under R.A. No. 2630, which
provides:

of
Section 1. Any person who had lost
his Philippine citizenship by rendering
service to, or accepting commission in,

the Armed Forces of the United States,


or after separation from the Armed
Forces of the United States, acquired
United
States
citizenship,
may
reacquire Philippine citizenship by
taking an oath of allegiance to the
Republic of the Philippines and
registering the same with Local Civil
Registry in the place where he resides
or last resided in the Philippines. The
said oath of allegiance shall contain a
renunciation of any other citizenship.
Having thus taken the required oath of
allegiance to the Republic and having
registered the same in the Civil
Registry of Mangatarem, Pangasinan in
accordance
with
the
aforecited
provision, respondent Cruz is deemed
to have recovered his original status
as a natural-born citizen, a status
which he acquired at birth as the son
of a Filipino father. It bears stressing
that the act of repatriation allows him
to recover, or return to, his original
status before he lost his Philippine
citizenship.
Petitioners
contention
that
respondent Cruz is no longer a
natural-born citizen since he had to
perform an act to regain his
citizenship is untenable. [T]he term
natural-born citizen was first defined
in Article III, Section 4 of the 1973
Constitution as follows:
Section 4. A natural-born citizen is
one who is a citizen of the Philippines
from birth without having to perform
any act to acquire or perfect his
Philippine citizenship.
Two requisites must concur for a
person to be considered as such: (1) a
person must be a Filipino citizen from
birth and (2) he does not have to
perform any act to obtain or perfect
his Philippine citizenship.
Under the 1973 Constitution definition,
there were two categories of Filipino
citizens which were not considered
natural-born: (1) those who were
naturalized and (2) those born before
January 17, 1973 (the date of
effectivity of the 1973 Constitution), of
Filipino mothers who, upon reaching
the age of majority, elected Philippine
citizenship.
Those
naturalized
citizens were not considered naturalborn obviously because they were not
Filipinos at birth and had to perform an
act to acquire Philippine citizenship.
Those born of Filipino mothers before
the effectivity of the 1973 Constitution
were likewise not considered natural-

born because they also had to perform


an act to perfect their Philippine
citizenship.
The present Constitution, however,
now considers those born of Filipino
mothers before the effectivity of the
1973 Constitution and who elected
Philippine citizenship upon reaching
the majority age as natural-born. After
defining who are natural-born citizens,
Section 2 of Article IV adds a
sentence: Those who elect Philippine
citizenship
in
accordance
with
paragraph (3), Section 1 hereof shall
be deemed natural-born citizens.
Consequently,
only
naturalized
Filipinos are considered not naturalborn citizens. It is apparent from the
enumeration of who are citizens under
the present Constitution that there are
only two classes of citizens: (1) those
who are natural-born and (2) those
who are naturalized in accordance
with law.
A citizen who is not a
naturalized Filipino, i.e., did not have
to
undergo
the
process
of
naturalization to obtain Philippine
citizenship, necessarily is a naturalborn Filipino.
Noteworthy is the
absence in the said enumeration of a
separate category for persons who,
after losing Philippine citizenship,
subsequently reacquire it. The reason
therefore is clear: as to such persons,
they would either be natural-born or
naturalized depending on the reasons
for the loss of their citizenship and the
mode prescribed by the applicable law
for the reacquisition thereof.
As
respondent Cruz was not required by
law to go through naturalization
proceedings in order to reacquire his
citizenship, he is perforce a naturalborn Filipino. As such, he possessed
all the necessary qualifications to be
elected as member of the House of
Representatives. (Antonio Bengson
III v. HRET, G.R. No. 142840, May
7, 2001, En Banc [Kapunan])
29.
Distinguish dual citizenship from
dual allegiance.
Held: Dual citizenship arises when, as
a result of the concurrent application
of the different laws of two or more
states, a person is simultaneously
considered a national by the said
states. For instance, such a situation
may arise when a person whose
parents are citizens of a state which
adheres to the principle of jus
sanguinis is born in a state which
follows the doctrine of jus soli. Such a
person, ipso facto and without any
voluntary act on his part, is

concurrently considered a citizen of


both states.

Held:
1)

Dual allegiance, on the other hand,


refers to a situation in which a person
simultaneously owes, by some positive
act, loyalty to two or more states.
While dual citizenship is involuntary,
dual allegiance is the result of an
individuals volition.
(Mercado v.
Manzano, 307 SCRA 630, May 26,
1999, En Banc [Mendoza])
30.
What is the main concern of
Section
5,
Article
IV,
1987
Constitution,
on
citizenship?
Consequently, are persons with mere
dual citizenship disqualified to run for
elective local positions under Section
40(d) of the Local Government Code?

Those born of Filipino fathers


and/or mothers in foreign countries
which follow the principle of jus soli;
2)
Those born in the Philippines of
Filipino mothers and alien fathers if by
the laws of their fathers country such
children are citizens of that country;
3)
Those who marry aliens if by
the laws of the latters country the
former are considered citizens, unless
by their act or omission they are
deemed to have renounced Philippine
citizenship.
(Mercado v. Manzano, G.R. No.
135083, 307 SCRA 630, May 26,
1999 [Mendoza])
32.Does res judicata apply in cases
hinging on the issue of citizenship?

Held: In including Section 5 in Article


IV on citizenship, the concern of the
Constitutional Commission was not
with dual citizens per se but with
naturalized citizens who maintain their
allegiance to their countries of origin
even after their naturalization. Hence,
the phrase dual citizenship in R.A.
No. 7160, Section 40(d) (Local
Government
Code)
must
be
understood as referring to dual
allegiance.
Consequently, persons
with mere dual citizenship do not fall
under this disqualification.
Unlike
those with dual allegiance, who must,
x x x, be subject to strict process with
respect to the termination of their
status, for candidates with dual
citizenship, it should suffice if, upon
the filing of their certificate of
candidacy,
they
elect
Philippine
citizenship to terminate their status as
persons
with
dual
citizenship
considering that their condition is the
unavoidable
consequence
of
conflicting laws of different states.

Held:
Petitioner maintains further
that when citizenship is raised as an
issue in judicial or administrative
proceedings, the resolution or decision
thereon is generally not considered res
judicata in any subsequent proceeding
challenging the same; citing the case
of Moy Ya Lim Yao v. Commissioner of
Immigration (41 SCRA 292 [1971]).
He insists that the same issue of
citizenship may be threshed out anew.
Petitioner is correct insofar as
the general rule is concerned, i.e., the
principle of res judicata generally does
not apply in cases hinging on the issue
of citizenship. However, in the case of
Burca v. Republic (51 SCRA 248
[1973]), an exception to this general
rule was recognized. The Court ruled
in that case that in order that the
doctrine of res judicata may be
applied in cases of citizenship, the
following must be present:
1)

By electing Philippine citizenship, such


candidates at the same time forswear
allegiance to the other country of
which they are also citizens and
thereby terminate their status as dual
citizens. It may be that, from the
point of view of the foreign state and
of its laws, such an individual has not
effectively renounced his foreign
citizenship. That is of no moment.
(Mercado v. Manzano, G.R. No.
135083, 307 SCRA 630, May 26,
1999 [Mendoza])
31.
Cite instances when a citizen of
the Philippines may possess dual
citizenship considering the citizenship
clause (Article IV) of the Constitution.

a persons citizenship be
raised as a material issue in a
controversy where said person is a
party;
2)
the Solicitor General or his
authorized representative took active
part in the resolution thereof, and
3)
the finding on citizenship is
affirmed by this Court.
Although the general rule was set
forth in the case of Moy Ya Lim Yao,
the case did not foreclose the weight
of prior rulings on citizenship.
It
elucidated that reliance may somehow
be placed on these antecedent official
findings, though not really binding, to
make the effort easier or simpler.
(Valles v. COMELEC, 337 SCRA

543, Aug.
[Purisima])

9,

2000,

En

Banc

The real authority in these operations,


as stated in the LOI, is lodged with the
head of a civilian institution, the PNP,
and not with the military. Such being
the case, it does not matter whether
the AFP Chief actually participates in
the Task Force Tulungan since he does
not exercise any authority or control
over the same. Since none of the
Marines was incorporated or enlisted
as members of the PNP, there can be
no appointment to a civilian position
to speak of. Hence, the deployment of
the Marines in the joint visibility
patrols does not destroy the civilian
character of the PNP.

Civilian Supremacy Clause


33.

The President issued Letter of


Instruction
(LOI)
ordering
the
deployment of members of the
Philippine Marines in the metropolis to
conduct joint visibility patrols with
members of the Philippine National
Police in various shopping malls. Will
this not violate the civilian supremacy
clause under Section 3, Article II of the
Constitution? Does this not amount to
an "insidious incursion" of the military
in the task of law enforcement in
violation of Section 5(4), Article XVI of
the Constitution?
Held: The deployment of the Marines
does not constitute a breach of the
civilian supremacy clause. The calling
of the marines in this case constitutes
permissible use of military assets for
civilian law enforcement. x x x The
limited participation of the Marines is
evident in the provisions of the LOI
itself, which sufficiently provides the
metes and bounds of the Marines'
authority. It is noteworthy that the
local police forces are the ones in
charge of the visibility patrols at all
times, the real authority belonging to
the PNP. In fact, the Metro Manila
Police Chief is the overall leader of the
PNP-Philippine Marines joint visibility
patrols.
Under the LOI, the police
forces are tasked to brief or orient the
soldiers on police patrol procedures. It
is their responsibility to direct and
manage the deployment of the
Marines. It is, likewise, their duty to
provide the necessary equipment to
the Marines and render logistical
support to these soldiers. In view of
the foregoing, it cannot be properly
argued that military authority is
supreme over civilian authority.
Moreover, the deployment of the
Marines to assist the PNP does not
unmake the civilian character of the
police force. Neither does it amount
to an insidious incursion of the
military in the task of law enforcement
in violation of Section 5[4], Article XVI
of the Constitution.
In this regard, it is not correct to say
that General Angelo Reyes, Chief of
Staff of the AFP, by his alleged
involvement
in
civilian
law
enforcement,
has
been
virtually
appointed to a civilian post in
derogation of the aforecited provision.

Considering the above circumstances,


the Marines render nothing more than
assistance required in conducting the
patrols. As such, there can be no
insidious incursion of the military in
civilian affairs nor can there be a
violation of the civilian supremacy
clause in the Constitution.
It is worth mentioning that military
assistance to civilian authorities in
various forms persists in Philippine
jurisdiction. The Philippine experience
reveals that it is not averse to
requesting the assistance of the
military in the implementation and
execution of certain traditionally civil
functions.
x x x [S]ome of the
multifarious activities wherein military
aid has been rendered, exemplifying
the activities that bring both the
civilian and the military together in a
relationship of cooperation, are:
1.
2.

Elections;
Administration
of
the
Philippine National Red Cross;
3.
Relief and rescue operations
during calamities and disasters;
4.
Amateur sports promotion
and development;
5.
Development of the culture
and the arts;
6.
Conservation
of
natural
resources;
7.
Implementation
of
the
agrarian reform program;
8.
Enforcement
of
customs
laws;
9.
Composite
civilian-military
law enforcement activities;
10.
Conduct
of
licensure
examinations;
11.
Conduct of nationwide tests
for elementary and high school
students;
12.
Anti-drug
enforcement
activities;
13.
Sanitary inspections;
14.
Conduct of census work;

15.

Administration of the Civil


Aeronautics Board;
16.
Assistance in installation of
weather forecasting devices;
17.
Peace and order policy
formulation in local government units.
This unquestionably constitutes a
gloss on executive power resulting
from
a
systematic,
unbroken,
executive practice, long pursued to
the knowledge of Congress and, yet,
never before questioned. What we
have here is mutual support and
cooperation between the military and
civilian authorities, not derogation of
civilian supremacy.
In the United States, where a long
tradition of suspicion and hostility
towards the use of military force for
domestic purposes has persisted and
whose Constitution, unlike ours, does
not expressly provide for the power to
call, the use of military personnel by
civilian law enforcement officers is
allowed under circumstances similar to
those
surrounding
the
present
deployment of the Philippine Marines.
(IBP v. Hon. Ronaldo B. Zamora,
G.R. No. 141284, Aug. 15, 2000,
En Banc [Kapunan])
The Right to a Balanced and
Healthful Ecology
34.

Is the right to a balanced and


healthful ecology any less important
than any of the civil and political rights
enumerated in the Bill of Rights?
Explain.
Held: While the right to a balanced
and healthful ecology is to be found
under the Declaration of Principles and
State Policies and not under the Bill of
Rights, it does not follow that it is less
important than any of the civil and
political rights enumerated in the
latter.
Such a right belongs to a
different category of rights altogether
for it concerns nothing less than selfpreservation and self-perpetuation,
the advancement of which may even
be said to predate all governments
and constitutions. As a matter of fact,
these basic rights need not even be
written in the Constitution for they are
assumed to exist from the inception of
humankind. If they are now explicitly
mentioned in the fundamental charter,
it is because of the well-founded fear
of its framers that unless the rights to
a balanced and healthful ecology and
to health are mandated as state
policies by the Constitution itself,

thereby highlighting their continuing


importance and imposing upon the
state a solemn obligation to preserve
the first and protect and advance the
second, the day would not be too far
when all else would be lost not only for
the present generation, but also for
those to come generations which
stand to inherit nothing but parched
earth incapable of sustaining life.
(Oposa v. Factoran, Jr., 224 SCRA
792 [1993][Davide])
35.

The Province of Palawan and the


City of Puerto Princesa enacted
ordinances prohibiting the catching
and/or exportation of live tropical
fishes, and imposing penalties for
violations thereof, in order to stop the
illegal practice of cyanide fishing
which destroys the corals and other
marine resources. Several fishermen
apprehended
for
violating
the
ordinances in question challenged
their constitutionality contending that
the
ordinances
violated
their
preferential right as subsistence and
marginal fishermen to the use of our
communal
marine
resources
guaranteed by the Constitution, under
Section 7, Article XIII. Will you sustain
the challenge?
Held:
The preferential right of
subsistence or marginal fishermen to
the use of marine resources is not
absolute.
In accordance with the
Regalian Doctrine, marine resources
belong to the State, and, pursuant to
the first paragraph of Section 2, Article
XII
of
the
Constitution,
their
exploration,
development
and
utilization x x x shall be under the full
control and supervision of the State.
Moreover, their mandated protection,
development and conservation x x x
imply certain restrictions on whatever
right of enjoyment there may be in
favor of anyone. What must be borne
in mind is the State policy enshrined in
the Constitution regarding the duty of
the State to protect and advance the
right of the people to a balanced and
healthful ecology in accord with the
rhythm and harmony of nature
(Section 16, Article II). The ordinances
in question are meant precisely to
protect and conserve our marine
resources to the end that their
enjoyment may be guaranteed not
only for the present generation, but
also for the generations to come. The
right to a balanced and healthful
ecology carries with it a correlative
duty to refrain from impairing the
environment.
(Tano
v.
Gov.

Salvador P. Socrates,
110249, Aug. 21, 1997)

G.R.

No.

San Carlos [USC], et al., supra.), since


a contract creates reciprocal rights
and obligations, the obligation of the
school to educate a student would
imply a corresponding obligation on
the part of the student to study and
obey the rules and regulations of the
school (Capitol Medical Center, Inc., et
al. v. Court of Appeals, et al., supra.).
When a student commits a serious
breach of discipline or failed to
maintain
the
required
academic
standard, he forfeits his contractual
right. In this connection, this Court
recognizes
the
expertise
of
educational institutions in the various
fields of learning.
Thus, they are
afforded ample discretion to formulate
reasonable rules and regulations in the
admission of students (Yap Chin Fah,
et al. v. Court of Appeals, et al., G.R.
No. 90063, December 12, 1989),
including
setting
of
academic
standards.
Within the parameters
thereof, they are competent to
determine who are entitled to
admission
and
re-admission.
(University of San Agustin, Inc. v.
Court of Appeals, 230 SCRA 761,
774-775, March 7, 1994 [Nocon])

Academic Freedom
36.
How should the States power to
regulate educational institutions be
exercised?
Held: Section 4[1], Article XIV
of the Constitution recognizes the
States power to regulate educational
institutions:
The
State
recognizes
the
complementary roles of public and
private institutions in the educational
system and shall exercise reasonable
supervision and regulation of all
educational institutions.
As may be gleaned from the
above provision, such power to
regulate is subject to the requirement
of reasonableness.
Moreover, the
Constitution
allows
merely
the
regulation
and
supervision
of
educational
institutions,
not
the
deprivation of their rights. (Miriam
College Foundation, Inc. v. Court
of Appeals, 348 SCRA 265, 288,
Dec. 15, 2000, 1st Div. [Kapunan])

2. Section 5[2], Article XIV of


the
Constitution
guarantees
all
institutions
of
higher
learning
academic freedom. This institutional
academic freedom includes the right
of the school or college to decide for
itself, its aims and objectives, and how
best to attain them free from outside
coercion or interference save possibly
when the overriding public welfare
calls for some restraint (Tangonan v.
Pano, 137 SCRA 245, 256-257 [1985]).
The essential freedoms subsumed in
the
term
academic
freedom
encompasses
the
freedom
to
determine for itself on academic
grounds:

37.
Discuss the academic freedom
of institutions of higher learning.
Held: 1. Equally mandated by
Article XIV, Section 5[2] of the 1987
Constitution is that academic freedom
shall be enjoyed in all institutions of
higher learning. Academic freedom of
educational institutions has been
defined as the right of the school or
college to decide for itself, its aims
and objectives, and how best to attain
them - free from outside coercion or
interference save possibly when the
overriding public welfare calls for
some restraint. It has a wide sphere of
autonomy certainly extending to the
choice of students. Said constitutional
provision is not to be construed in a
niggardly manner or in a grudging
fashion. That would be to frustrate its
purpose and nullify its intent (Garcia v.
The Faculty Admission Committee, et
al., supra; Tangonan v. Pano, et al.,
supra.)
While it is true that
institution
of
learning
has
contractual obligation to afford
students
a
fair
opportunity
complete the course they seek
pursue (Licup, et al. v. University

an
a
its
to
to
of

1
2
3
4

Who may teach,


What may be taught,
How it shall be taught, and
Who may be admitted to study.
(Isabelo, Jr. v. Perpetual Help College
of Rizal, Inc., 227 SCRA 591, 595
[1993]; Ateneo de Manila University v.
Capulong, 222 SCRA 643, 660 [1993];
Garcia v. The Faculty Admission
Committee, Loyola School of Theology,
68 SCRA 277, 285 [1975]. The above
formulation was made by Justice Felix
Frankfurter in his concurring opinion in
Sweezy v. New Hampshire, 354 U.S.
234, 263)
The right of the school to discipline its
students is at once apparent in the

third freedom, i.e., how it shall be


taught.
A school certainly cannot
function in an atmosphere of anarchy.
Thus, there can be no doubt that the
establishment of an educational
institution
requires
rules
and
regulations
necessary
for
the
maintenance of an orderly educational
program and the creation of an
educational environment conducive to
learning. Such rules and regulations
are
equally
necessary
for
the
protection of the students, faculty, and
property (Angeles v. Sison, 112 SCRA
26, 37 [1982]).
Moreover, the school has an
interest in teaching the student
discipline,
a
necessary,
if
not
indispensable, value in any field of
learning. By instilling discipline, the
school teaches discipline. Accordingly,
the right to discipline the student
likewise finds basis in the freedom
what to teach.
Incidentally, the school not only
has the right but the duty to develop
discipline in its students.
The
Constitution no less imposes such
duty.
[All educational institutions] shall
inculcate patriotism and nationalism,
foster love of humanity, respect for
human rights, appreciation of the role
of national heroes in the historical
development of the country, teach the
rights and duties of citizenship,
strengthen ethical and spiritual values,
develop moral character and personal
discipline, encourage critical and
creative thinking, broaden scientific
and technological knowledge, and
promote vocational efficiency (Section
3[2], Article XIV, Constitution).
In Angeles v. Sison, we also said that
discipline was a means for the school
to carry out its responsibility to help
its students grow and develop into
mature, responsible, effective and
worthy citizens of the community.
(Supra, at 37)
Finally, nowhere in the above
formulation is the right to discipline
more evident than in who may be
admitted to study. If a school has the
freedom to determine whom to admit,
logic dictates that it also has the right
to determine whom to exclude or
expel, as well as upon whom to
impose lesser sanctions such as
suspension and the withholding of
graduation privileges.

Thus, in Ateneo de Manila v.


Capulong (222 SCRA 643 [1993]), the
Court upheld the expulsion of students
found guilty of hazing by petitioner
therein, holding that:
No one can be so myopic as to doubt
that the immediate reinstatement of
respondent students who have been
investigated and found guilty by the
Disciplinary Board to have violated
petitioner
universitys
disciplinary
rules and standards will certainly
undermine the authority of the
administration of the school. This we
would be most loathe to do.
More importantly, it will seriously
impair petitioner universitys academic
freedom which has been enshrined in
the 1935, 1973 and the present 1987
Constitution (Id., at 659-660).
(Miriam College Foundation, Inc.
v. Court of Appeals, 348 SCRA
265, Dec. 15, 2000, 1st Div.
[Kapunan])
38.
May a university validly revoke
a degree or honor it has conferred to a
student after the graduation of the
latter after finding that such degree or
honor was obtained through fraud?
Held: In Garcia v. Faculty Admission
Committee, Loyola School of Theology
(68 SCRA 277 [1975]), the SC pointed
out that academic freedom of
institutions of higher learning is a
freedom granted to institutions of
higher learning which is thus given a
wide sphere of authority certainly
extending to the choice of students.
If such institution of higher learning
can decide who can and who cannot
study in it, it certainly can also
determine on whom it can confer the
honor and distinction of being its
graduates.
Where it is shown that the conferment
of an honor or distinction was obtained
through fraud, a university has the
right to revoke or withdraw the honor
or distinction it has thus conferred.
This freedom of a university does not
terminate upon the graduation of a
student, for it is precisely the
graduation of such a student that is
in question. (UP Board of Regents
v. Hon. Court of Appeals and
Arokiaswamy William Margaret
Celine, G.R. No. 134625, Aug. 31,
1999, 2nd Div. [Mendoza])

39.
What are the essential freedoms
subsumed in the term academic
freedom?

the COA. Section 3, Article IX-C of the


Constitution mandates that:

Held: In Ateneo de Manila University


v. Capulong (G.R. No. 99327, 27 May
1993), this Court cited with approval
the formulation made by Justice Felix
Frankfurter of the essential freedoms
subsumed in the term academic
freedom encompassing not only the
freedom to determine x x x on
academic grounds who may teach,
what may be taught (and) how it shall
be taught, but likewise who may be
admitted to study. We have thus
sanctioned its invocation by a school
in
rejecting
students
who
are
academically delinquent (Tangonan v.
Pano, 137 SCRA 245 [1985]), or a
laywoman seeking admission to a
seminary (Garcia v. Loyola School of
Theology, 68 SCRA 277 [1975]), or
students violating School Rules on
Discipline.
(Ateneo
de
Manila
University
v.
Capulong,
supra.)
(Isabelo, Jr. v. Perpetual Help
College of Rizal, Inc., 227 SCRA
595-597, Nov. 8, 1993, En Banc
[Vitug])

Sec. 3.
No law shall be passed
exempting
any
entity
of
the
Government or its subsidiary in any
guise whatsoever, or any investment
of public funds, from the jurisdiction of
the Commission on Audit.

40.
Between the COAs findings and
conclusions and that of private
auditors, which should prevail?
Held:
Moreover,
as
the
constitutionally-mandated auditor of
all government agencies, the COAs
findings and conclusions necessarily
prevail over those of private auditors,
at least insofar as government
agencies and officials are concerned.
The superiority or preponderance of
the COA audit over private audit can
be gleaned from the records of the
Constitutional Commission x x x. The
findings and conclusions of the private
auditor may guide private investors or
creditors who require such private
audit.
Government agencies and
officials, however, remain bound by
the findings and conclusions of the
COA, whether the matter falls under
the first or second paragraph of
Section 2, unless of course such
findings and conclusions are modified
or reversed by the courts.
The power of the COA to examine and
audit government agencies, while nonexclusive, cannot be taken away from

The mere fact that private auditors


may audit government agencies does
not divest the COA of its power to
examine
and
audit
the
same
government agencies.
The COA is
neither by-passed nor ignored since
even with a private audit the COA will
still conduct its usual examination and
audit, and its findings and conclusions
will still bind government agencies and
their officials. A concurrent private
audit poses no danger whatsoever of
public funds or assets escaping the
usual scrutiny of a COA audit.
(Development
Bank
of
the
Philippines v. Commission on
Audit, 373 SCRA 356, January 16,
2002, En Banc [Carpio])
41.
Is the constitutional power of
the COA to examine and audit
government banks and agencies
exclusive?
Does it preclude a
concurrent audit by a private external
auditor?
Held: The resolution of the primordial
issue of whether or not the COA has
the sole and exclusive power to
examine and audit government banks
involves an interpretation of Section 2,
Article IX-D of the 1987 Constitution.
This Section provides as follows:
Sec. 2. (1) The Commission on Audit
shall have the power, authority, and
duty to examine, audit, and settle all
accounts pertaining to the revenue
and receipts of, and expenditures or
uses of funds and property, owned and
held in trust by, or pertaining to, the
Government, or any of its subdivisions,
agencies,
or
instrumentalities,
including
government-owned
or
controlled corporations with original
charters, x x x.

(2) The Commission shall have the


exclusive authority, subject to the
limitations in this Article, to define the
scope of its audit and examination,
establish the techniques and methods
required therefore, and promulgate
accounting and auditing rules and
regulations, including those for the
prevention
and
disallowance
of
irregular,
unnecessary,
excessive,
extravagant,
or
unconscionable
expenditures, or uses of government
funds and properties. (Emphasis
supplied)
The COA vigorously asserts that under
the first paragraph of Section 2, the
COA enjoys the sole and exclusive
power to examine and audit all
government agencies, including the
DBP. The COA contends this is similar
to its sole and exclusive authority,
under the same paragraph of the
same section, to define the scope of
its audit, promulgate auditing rules
and regulations, including rules on the
disallowance
of
unnecessary
expenditures of government agencies.
The bare language of Section 2,
however, shows that the COAs power
under the first paragraph is not
declared exclusive, while its authority
under the second paragraph is
expressly declared exclusive. There
is a significant reason for this marked
difference in language.
During the deliberations of the
Constitutional
Commission,
Commissioner
Serafin
Guingona
proposed the addition of the word
exclusive in the first paragraph of
Section 2, thereby granting the COA
the sole and exclusive power to
examine and audit all government
agencies. However, the Constitutional
Commission rejected the addition of
the word exclusive in the first
paragraph of Section 2 and Guingona
was forced to withdraw his proposal.
X x x.
Xxx

In sharp contrast, the Constitutional


Commission
placed
the
word
exclusive to qualify the authority of
the COA under the second paragraph
of the same Section 2. This word
exclusive did not appear in the
counterpart provisions of Section 2 in
the 1935 and 1973 Constitutions.
There is no dispute that the COAs
authority under the second paragraph
of Section 2 is exclusive as the
language of the Constitution admits of
no other meaning. Thus, the COA has
the exclusive authority to decide on
disallowances
of
unnecessary
government expenditures.
Other
government
agencies
and
their
officials, as well as private auditors
engaged by them, cannot in any way
intrude into this exclusive function of
the COA.
The qualifying word exclusive in the
second paragraph of Section 2 cannot
be applied to the first paragraph which
is another sub-section of Section 2. A
qualifying word is intended to refer
only to the phrase to which it is
immediately associated, and not to a
phrase distantly located in another
paragraph or sub-section (Felipe v. De
la Cruz, 99 Phil. 940 [1956]; Tirona v.
Cudiamat, 14 SCRA 264 [1965]).
Thus, the first paragraph of Section 2
must be read the way it appears,
without
the
word
exclusive,
signifying that non-COA auditors can
also examine and audit government
agencies. Besides, the framers of the
Constitution intentionally omitted the
word exclusive in the first paragraph
of Section 2 precisely to allow
concurrent audit by private external
auditors.
The clear and unmistakable conclusion
from a reading of the entire Section 2
is that the COAs power to examine
and audit is non-exclusive. On the
other hand, the COAs authority to
define the scope of its audit,
promulgate
auditing
rules
and
regulations, and disallow unnecessary
expenditures is exclusive.
Xxx

Manifestly,
the
express
language of the Constitution, and the
clear intent of its framers, point to
only one indubitable conclusion the
COA does not have the exclusive
power
to
examine
and
audit
government agencies. The framers of
the Constitution were fully aware of
the need to allow independent private
audit of certain government agencies
in addition to the COA audit, as when
there is a private investment in a
government-controlled corporation, or
when a government corporation is
privatized or publicly listed, or as in
the case at bar when the government
borrows money from abroad.
In
these
instances
the
government enters the marketplace
and competes with the rest of the
world in attracting investments or
loans. To succeed, the government
must abide with the reasonable
business practices of the marketplace.
Otherwise no investor or creditor will
do business with the government,
frustrating government efforts to
attract investments or secure loans
that may be critical to stimulate
moribund industries or resuscitate a
badly shattered national economy as
in the case at bar. By design the
Constitution is flexible enough to meet
these exigencies.
Any attempt to
nullify this flexibility in the instances
mentioned, or in similar instances, will
be ultra vires, in the absence of a
statute limiting or removing such
flexibility.
The deliberations of the Constitutional
Commission reveal eloquently the
intent of Section 2, Article IX-D of the
Constitution. As this Court has ruled
repeatedly, the intent of the law is the
controlling factor in the interpretation
of the law (People v. Purisima, 86
SCRA 542 [1978]; others omitted). If a
law needs interpretation, the most
dominant influence is the intent of the
law (De Jesus v. City of Manila, 29 Phil.
73 [1914]). The intent of the law is
that which is expressed in the words of
the law, which should be discovered

within its four corners aided, if


necessary, by its legislative history
(Manila Lodge No. 761 v. Court of
Appeals, 73 SCRA 162 [1976]). In the
case of Section 2, Article IX-D of the
Constitution, the intent of the framers
of the Constitution is evident from the
bare language of Section 2 itself. The
deliberations of the Constitutional
Commission confirm expressly and
even elucidate further this intent
beyond any doubt whatsoever.
There is another constitutional
barrier to the COAs insistence of
exclusive power to examine and audit
all government agencies. The COAs
claim clashes directly with the Central
Banks
constitutional
power
of
supervision
over
banks
under
Section 20, Article XII of the
Constitution. X x x
Historically, the Central Bank
has been conducting periodic and
special examination and audit of
banks to determine the soundness of
their operations and the safety of the
deposits of the public. Undeniably,
the
Central
Banks
power
of
supervision includes the power to
examine and audit banks, as the
banking laws have always recognized
this power of the Central Bank.
Hence, the COAs power to examine
and audit government banks must be
reconciled with the Central Banks
power to supervise the same banks.
The inevitable conclusion is that the
COA and the Central Bank have
concurrent jurisdiction, under the
Constitution, to examine and audit
government banks.
However, despite the Central
Banks concurrent jurisdiction over
government banks, the COAs audit
still prevails over that of the Central
Bank
since
the
COA
is
the
constitutionally mandated auditor of
government banks. And in matters
falling under the second paragraph of
Section 2, Article IX-D of the
Constitution, the COAs jurisdiction is
exclusive. Thus, the Central Bank is
devoid of authority to allow or disallow

expenditures of government banks


since this function belongs exclusively
to the COA. (Development Bank of
the Philippines v. Commission on
Audit, 373 SCRA 356, January 16,
2002, En Banc [Carpio])

Economic Policy
42.
Does the Constitutional policy of
a
self-reliant
and
independent
national economy rule out foreign
competition?
Held: The constitutional policy of a
self-reliant and independent national
economy does not necessarily rule
out the entry of foreign investments,
goods and services. It contemplates
neither economic seclusion nor
mendicancy in the international
community.
Aside from envisioning a trade policy
based on equality and reciprocity,
the fundamental law encourages
industries that are competitive in
both domestic and foreign markets,
thereby demonstrating a clear policy
against a sheltered domestic trade
environment, but one in favor of the
gradual
development
of
robust
industries that can compete with the
best in the foreign markets. (Tanada
v. Angara, 272 SCRA 18 [1997])
43.
Is
PHILSECO
(Philippine
Shipyard
and
Engineering
Corporation), as a shipyard, a public
utility and, hence, could be operated
only by a corporation at least 60% of
whose capital is owned by Filipino
citizens in accordance with Article XII,
Section 10 of the Constitution?
Held: Petitioner asserts that a
shipyard is a public utility pursuant to
Section 13 (b) of Commonwealth Act
No. 146. Respondents, on the other
hand, contend that shipyards are no
longer public utilities by express
provision of Presidential Decree No.
666, which provided incentives to the
shipbuilding and ship repair industry.
Indeed, P.D. No. 666 dated
March 5, 1975 explicitly stated that a
shipyard was not a public utility. x
xx
However, Section 1 of P.D. No.
666 was expressly repealed by Section
20 of Batas Pambansa Blg. 391, the
Investment Incentive Policy Act of

1983. Subsequently, Executive Order


No. 226, the Omnibus Investments
Code of 1987, was issued and Section
85 thereof expressly repealed B.P. Blg.
391.
The express repeal of B.P. Blg.
391 by E.O. No. 226 did not revive
Section 1 of P.D. No. 666, declassifying
the shipbuilding and ship repair
industry as a public utility, as said
executive order did not provide
otherwise.
When a law which
expressly repeals a prior law is itself
repealed, the law first repealed shall
not be thereby revived unless
expressly so provided (Administrative
Code of 1987, Book I, Chapter 5,
Section 21). Consequently, when the
APT [Asset Privatization Trust] drafted
the ASBR [Asset Specific Bidding
Rules] sometime in 1993, P.D. No. 666
no longer existed in our statute books.
While it is true that the repeal of a
statute does not operate to impair
rights that have become vested or
accrued while the statute was in force,
there are no vested rights of the
parties that should be protected in the
case at bar. The reason is simple: said
decree was already inexistent when
the ASBR was issued.
A shipyard such as PHILSECO
being a public utility as provided by
law, the following provision of the
Article XII of the Constitution applies:
Sec. 11. No franchise, certificate, or
any other form of authorization for the
operation of a public utility shall be
granted except to citizens of the
Philippines or to corporations or
associations organized under the laws
of the Philippines at least sixty per
centum of whose capital is owned by
such citizens, nor shall such franchise,
certificate,
or
authorization
be
exclusive in character or for a longer
period than fifty years. Neither shall
any such franchise or right be granted
except under the condition that it shall
be subject to amendment, alteration,
or repeal by the Congress when the
common good so requires. The State
shall encourage equity participation in
public utilities by the general public.
The participation of foreign investors
in the governing body of any public
utility enterprise shall be limited to
their proportionate share in its capital,
and all the executive and managing
officers of such corporation or
association shall be citizens of the
Philippines.

The
progenitor
of
this
constitutional provision, Article XIV,
Section 5 of the 1973 Constitution,
required the same proportion of 60%40% capitalization.
The JVA [Joint
Venture Agreement] between NIDC
[National
Investment
and
Development
Corporation]
and
Kawasaki [Kawasaki Heavy Industries,
Ltd. of Kobe, Japan] entered into on
January 27, 1977 manifests the
intention of the parties to abide by the
constitutional
mandate
on
capitalization of public utilities. x x x
A joint venture is an association of
persons
or
companies
jointly
undertaking
some
commercial
enterprise with all of them generally
contributing assets and sharing risks.
x x x.
Considered more of a
partnership (Aurbach v. Sanitary
Wares Manufacturing Corporation, G.R.
No. 75875, 180 SCRA 130, 147
[1989]), a joint venture is governed by
the laws on contracts and on
partnership. The joint venture created
between NIDC and Kawasaki falls
within the purview of an association
pursuant to Section 5 of Article XIV of
the 1973 Constitution and Section 11
of Article XII of the 1987 Constitution.
Consequently, a joint venture that
would engage in the business of
operating a public utility, such as a
shipyard, must observe the proportion
of
60%-40%
Filipino-foreign
capitalization.
(JG
Summit
Holdings, Inc. v. Court of Appeals,
345 SCRA 143, Nov. 20, 2000, 1st
Div. [Ynares-Santiago])
The Rights of Indigenous Cultural
Communities/Indigenous Peoples
44.
Does R.A. 8371, otherwise
known as the Indigenous Peoples
Rights Act infringe upon the States
ownership over the natural resources
within the ancestral domains?
Held:
Petitioners posit that IPRA
deprives the State of its ownership
over mineral lands of the public
domain and other natural resources,
as well as the States full control and
supervision over the exploration,
development and utilization of natural
resources.
Specifically, petitioners
and the Solicitor General assail
Sections 3[a], 5, and 7 of IPRA as
violative of Section 2, Article XII of the
Constitution which states, in part, that
[a]ll lands of the public domain,
waters, minerals, coal, petroleum, and
other mineral oils, all forces of

potential energy, fisheries, forests or


timber, wildlife, flora and fauna, and
other natural resources are owned by
the State. (Section 2, Article XII,
Constitution) They would have the
Court declare as unconstitutional
Section 3[a] of IPRA because the
inclusion of natural resources in the
definition
of
ancestral
domains
purportedly results in the abdication of
State ownership over these resources.
Xxx
Section 3[a] merely defines the
coverage of ancestral domains, and
describes the extent, limit and
composition of ancestral domains by
setting forth the standards and
guidelines in determining whether a
particular area is to be considered as
part of and within the ancestral
domains. In other words, Section 3[a]
serves only as a yardstick which points
out what properties are within the
ancestral domains. It does not confer
or recognize any right of ownership
over the natural resources to the
indigenous peoples.
Its purpose is
definitional and not declarative of a
right or title.
The specification of what areas
belong to the ancestral domains is, to
our mind, important to ensure that no
unnecessary encroachment on private
properties
outside
the
ancestral
domains
will
result during
the
delineation process. The mere fact
that Section 3[a] defines ancestral
domains to include the natural
resources found therein does not ipso
facto convert the character of such
natural resources as private property
of the indigenous peoples. Similarly,
Section 5 in relation to Section 3[a]
cannot be construed as a source of
ownership rights of indigenous people
over the natural resources simply
because
it
recognizes
ancestral
domains
as
their
private
but
community property.
The
phrase
private
but
community
property
is
merely
descriptive of the indigenous peoples
concept of ownership as distinguished
from that provided in the Civil Code. x
x x.
In contrast, the indigenous
peoples
concept
of
ownership
emphasizes
the
importance
of
communal or group ownership. By
virtue of the communal character of
ownership, the property held in
common cannot be sold, disposed or
destroyed because it was meant to
benefit
the
whole
indigenous

community and not


individual member.

merely

the

That IPRA is not intended to


bestow
ownership
over
natural
resources to the indigenous peoples is
also clear from the deliberations of the
bicameral conference committee on
Section 7 which recites the rights of
indigenous
peoples
over
their
ancestral domains x x x.
Further, Section 7 makes no
mention of any right of ownership of
the indigenous peoples over the
natural resources. In fact, Section 7[a]
merely recognizes the right to claim
ownership over lands, bodies of water
traditionally and actually occupied by
indigenous peoples, sacred places,
traditional
hunting
and
fishing
grounds, and all improvements made
by them at any time within the
domains. Neither does Section 7[b],
which enumerates certain rights of the
indigenous peoples over the natural
resources found within their ancestral
domains, contain any recognition of
ownership
vis--vis
the
natural
resources.
What is evident is that the IPRA
protects the indigenous peoples rights
and welfare in relation to the natural
resources found within their ancestral
domains, including the preservation of
the ecological balance therein and the
need to ensure that the indigenous
peoples will not be unduly displaced
when the State-approved activities
involving the natural resources located
therein are undertaken. (Separate
Opinion, Kapunan, J., in Cruz v.
Secretary of Environment and
Natural Resources, 347 SCRA 128,
284-293, Dec. 6, 2000, En Banc
[Per Curiam])
45.
Has the concept of native title
to natural resources, like native title to
land,
been
recognized
in
the
Philippines?
Held: The concept of native
title to natural resources, unlike native
title to land, has not been recognized
in the Philippines. NCIP and Flavier, et
al. invoke the case of Reavies v. Fianza
(40 Phil. 1017 [1909], 215 US 16, 54 L
Ed 72) in support of their thesis that
native title to natural resources has
been upheld in this jurisdiction. They
insist that it is possible for rights over
natural resources to vest on a private
(as opposed to a public) holder if these
were
held
prior
to the
1935
Constitution. However, a judicious

examination of Reavies reveals that,


contrary to the position of NCIP and
Flavier, et al., the Court did not
recognize native title to natural
resources. Rather, it merely upheld the
right of the indigenous peoples to
claim ownership of minerals under the
Philippine Bill of 1902.
While x x x native title to land
or private ownership by Filipinos of
land by virtue of time immemorial
possession in the concept of an owner
was acknowledged and recognized as
far
back
during
the
Spanish
colonization of the Philippines, there
was no similar favorable treatment as
regards natural resources. The unique
value of natural resources has been
acknowledged by the State and is the
underlying reason for its consistent
assertion of ownership and control
over said natural resources from the
Spanish regime up to the present.
Natural resources, especially minerals,
were considered by Spain as an
abundant source of revenue to finance
its battle in wars against other
nations. Hence, Spain, by asserting its
ownership over minerals wherever
these may be found, whether in public
or private lands, recognized the
separability of title over lands and that
over minerals which may be found
therein (Noblejas, Philippine Law on
Natural Resources 1961 Revised Ed.,
p. 6).
On the other hand, the United
States viewed natural resources as a
source of wealth for its nationals. As
the owner of natural resources over
the Philippines after the latters
cession from Spain, the United States
saw it fit to allow both Filipino and
American citizens to explore and
exploit minerals in public lands, and to
grant patents to private mineral lands.
x x x. Although the United States
made a distinction between minerals
found in public lands and those found
in private lands, title in these minerals
was in all cases sourced from the
State.
The framers of the 1935
Constitution found it necessary to
maintain the States ownership over
natural resources to insure their
conservation for future generations of
Filipinos, to prevent foreign control of
the
country
through
economic
domination; and to avoid situations
whereby the Philippines would become
a source of international conflicts,
thereby posing danger to its internal
security and independence.

The
declaration
of
State
ownership and control over minerals
and other natural resources in the
1935 Constitution was reiterated in
both the 1973 and 1987 Constitutions.
(Separate Opinion, Kapunan, J., in
Cruz v. Secretary of Environment
and Natural Resources, 347 SCRA
128, 284-293, Dec. 6, 2000, En
Banc [Per Curiam])
46.
Enumerate the Constitutional
provisions recognizing and protecting
the rights and interests of the
indigenous peoples.
Held:
The framers of the 1987
Constitution, looking back to the long
destitution of our less fortunate
brothers, fittingly saw the historic
opportunity to actualize the ideals of
people empowerment and social
justice, and to reach out particularly to
the marginalized sectors of society,
including the indigenous peoples.
They incorporated in the fundamental
law several provisions recognizing and
protecting the rights and interests of
the indigenous peoples, to wit:
Section 22. The State recognizes and
promotes the rights of indigenous
peoples within the framework of
national unity and development.
(Article II of the Constitution, entitled
State Principles and Policies)
Section 5. The State, subject to the
provisions of the Constitution and
national development policies and
programs, shall protect the rights of
indigenous cultural communities to
their ancestral lands to ensure their
economic, social, and cultural wellbeing.
The Congress may provide for the
applicability
of
customary
laws
governing
property
rights
and
relations in determining the ownership
and extent of ancestral domains.
(Article XII of the Constitution, entitled
National Economy and Patrimony)
Section 1. The Congress shall give the
highest priority to the enactment of
measures that protect and enhance
the right of all the people to human
dignity, reduce social, economic and
political inequalities, and remove
cultural inequalities by equitably
diffusing wealth and political power for
the common good.
To this end, the State shall regulate
the acquisition, ownership, use and
disposition of property and its

increments. (Article
Constitution, entitled
and Human Rights)

XIII of the
Social Justice

Section 6. The State shall apply the


principles of agrarian reform or
stewardship, whenever applicable in
accordance with law, in the disposition
and utilization of other natural
resources, including lands of the public
domain under lease or concession,
subject to prior rights, homestead
rights of small settlers, and the rights
of indigenous communities to their
ancestral lands. (Ibid.)
Section 17. The State shall recognize,
respect, and protect the rights of
cultural communities to preserve and
develop their cultures, traditions, and
institutions. It shall consider these
rights in the formulation of national
plans and policies. (Article XIV of the
Constitution,
entitled
Education,
Science, Technology, Arts, Culture, and
Sports)
Section 12. The Congress may create
a consultative body to advise the
President
on
policies
affecting
indigenous cultural communities, the
majority of the members of which shall
come from such communities. (Article
XVI of the Constitution, entitled
General Provisions)
(Separate Opinion, Kapunan, J., in
Isagani Cruz v. Secretary of
Environment
and
Natural
Resources,
et
al.,
G.R.
No.
135385, Dec. 6, 2000, En Banc)
47.
Discuss the Indigenous Peoples
Rights Act (R.A. No. 8371).
Held:
Republic Act No. 8371 is
entitled "An Act to Recognize, Protect
and Promote the Rights of Indigenous
Cultural
Communities/Indigenous
Peoples,
Creating
a
National
Commission on Indigenous Peoples,
Establishing
Implementing
Mechanisms,
Appropriating
Funds
Therefor, and for Other Purposes." It
is simply known as "The Indigenous
Peoples Rights Act of 1997" or the
IPRA.
The IPRA recognizes the existence of
the indigenous cultural communities
or indigenous peoples (ICCs/IPs) as a
distinct sector in Philippine society. It
grants these people the ownership
and possession of their ancestral
domains and ancestral lands, and
defines the extent of these lands and
domains. The ownership given is the
indigenous concept of ownership

under customary law which traces its


origin to native title.
Xxx
Within their ancestral domains and
ancestral lands, the ICCs/IPs are given
the right to self-governance and
empowerment (Sections 13 to 20),
social justice and human rights
(Sections 21 to 28), the right to
preserve and protect their culture,
traditions, institutions and community
intellectual rights, and the right to
develop their own sciences and
technologies (Sections 29 to 37).
(Separate Opinion, Puno, J., in
Isagani Cruz v. Secretary of DENR,
et al., G.R. No. 135385, Dec. 6,
2000, En Banc)
48.
Define
peoples/indigenous
communities."

"indigenous
cultural

Held: 1. Drawing inspiration from


both our fundamental law and
international law, IPRA now employs
the politically-correct conjunctive term
"indigenous
peoples/indigenous
cultural communities" as follows:
Section 3. Definition of Terms. - For
purposes of this Act, the following
terms shall mean:
(i)

Indigenous
peoples/Indigenous
cultural
communities. - refer to a group of
people or homogenous societies
identified
by
self-ascription
and
ascription by others, who have
continuously
lived
as
organized
community on communally bounded
and defined territory, and who have,
under claims of ownership since time
immemorial, occupied, possessed and
utilized
such
territories,
sharing
common bonds of language, customs,
traditions,
and
other
distinctive
cultural traits, or who have, through
resistance to political, social and
cultural inroads of colonization, nonindigenous religions and cultures,
became historically differentiated from
the majority of Filipinos. Indigenous
peoples shall likewise include peoples
who are regarded as indigenous on
account of their descent from the
populations
which
inhabited
the
country at the time of conquest or
colonization, or at the time of inroads
of
non-indigenous
religions
and
cultures, or the establishment of
present State boundaries, who retain
some or all of their own social,
economic,
cultural
and
political
institutions, but who may have been

displaced
from
their
traditional
domains or who may have resettled
outside their ancestral domains x x x.
(Separate Opinion, Kapunan, J.,
in Isagani Cruz v. Secretary of
Environment
and
Natural
Resources,
et
al.,
G.R.
No.
135385, Dec. 6, 2000, En Banc)
2. The IPRA is a law dealing with a
specific group of people, i.e., the
Indigenous
Cultural
Communities
(ICCs) or the Indigenous Peoples (IPs).
The term "ICCs" is used in the 1987
Constitution while that of "IPs" is the
contemporary international language
in the International Labor Organization
(ILO) Convention 169 (Convention
Concerning Indigenous and Tribal
Peoples in Independent Countries,
June 27, 1989) and the United Nations
(UN) Draft Declaration on the Rights of
Indigenous Peoples (Guide to R.A.
8371, published by the Coalition for
IPs Rights and Ancestral Domains in
cooperation with the ILO and BilanceAsia Department, p. 4 [1999] hereinafter referred to as Guide to R.A.
8371).
Indigenous Cultural Communities or
Indigenous Peoples refer to a group of
people or homogeneous societies who
have
continuously
lived
as
an
organized community on communally
bounded and defined territory. These
groups of people have actually
occupied, possessed and utilized their
territories under claim of ownership
since time immemorial. They share
common bonds of language, customs,
traditions and other distinctive cultural
traits, or, they, by their resistance to
political, social and cultural inroads of
colonization, non-indigenous religions
and cultures, became historically
differentiated
from
the
Filipino
majority.
ICCs/IPs also include
descendants of ICCs/IPs who inhabited
the country at the time of conquest or
colonization, who retain some or all of
their own social, economic, cultural
and political institutions but who may
have been displaced from their
traditional territories or who may have
resettled
outside
their
ancestral
domains. (Separate Opinion, Puno,
J., in Isagani Cruz v. Secretary of
DENR, et al., G.R. No. 135385,
Dec. 6, 2000, En Banc)
49.
Define "ancestral domains" and
"ancestral lands." Do they constitute
part of the land of the public domain?
Held:
ancestral

Ancestral domains and


lands are the private

property of indigenous peoples and do


not constitute part of the land of the
public domain.
The IPRA grants to ICCs/IPs a distinct
kind of ownership over ancestral
domains
and
ancestral
lands.
Ancestral lands are not the same as
ancestral domains. These are defined
in Section 3(a) and (b) of the
Indigenous Peoples Rights Act x x x.
Ancestral domains are all areas
belonging to ICCs/IPs held under a
claim of ownership, occupied or
possessed by ICCs/IPs by themselves
or
through
their
ancestors,
communally or individually since time
immemorial, continuously until the
present, except when interrupted by
war, force majeure or displacement by
force, deceit, stealth or as a
consequence of government projects
or any other voluntary dealings with
government and/or private individuals
or corporations.
Ancestral domains
comprise lands, inland waters, coastal
areas, and natural resources therein
and includes ancestral lands, forests,
pasture, residential, agricultural, and
other
lands
individually
owned
whether alienable or not, hunting
grounds, burial grounds, worship
areas, bodies of water, mineral and
other natural resources. They also
include lands which may no longer be
exclusively occupied by ICCs/IPs but
from which they traditionally had
access to for their subsistence and
traditional activities, particularly the
home ranges of ICCs/IPs who are still
nomadic and/or shifting cultivators
(Section 3[a], IPRA).
Ancestral lands are lands held by the
ICCs/IPs under the same conditions as
ancestral domains except that these
are limited to lands and that these
lands are not merely occupied and
possessed but are also utilized by the
ICCs/IPs under claims of individual or
traditional group ownership.
These
lands include but are not limited to
residential lots, rice terraces or
paddies, private forests, swidden
farms and tree lots (Section 3[b],
IPRA). (Separate Opinion, Puno, J.,
in Isagani Cruz v. Secretary of
DENR, et al., G.R. No. 135385,
Dec. 6, 2000, En Banc)
50.
How may ICCs/IPs acquire rights
to their ancestral domains and
ancestral lands?
Held: The rights of the ICCs/IPs to
their ancestral domains and ancestral

lands may be acquired in two modes:


(1) by native title over both ancestral
lands and domains; or (2) by torrens
title under the Public Land Act and the
Land Registration Act with respect to
ancestral lands only.
(Separate
Opinion, Puno, J., in Isagani Cruz
v. Secretary of DENR, et al., G.R.
No. 135385, Dec. 6, 2000, En
Banc)
51.
What is the concept of "native
title"?
What is a Certificate of
Ancestral Domain Title (CADT)?
Held: Native title refers to ICCs/IPs
preconquest rights to lands and
domains held under a claim of private
ownership as far back as memory
reaches.
These lands are deemed
never to have been public lands and
are indisputably presumed to have
been held that way since before the
Spanish Conquest.
The rights of
ICCs/IPs to their ancestral domains
(which also include ancestral lands) by
virtue of native title shall be
recognized and respected (Section 11,
IPRA).
Formal recognition, when
solicited by ICCs/IPs concerned, shall
be embodied in a Certificate of
Ancestral Domain Title (CADT), which
shall recognize the title of the
concerned ICCs/IPs over the territories
identified and delineated.
Like a torrens title, a CADT is evidence
of private ownership of land by native
title. Native title, however, is a right
of
private
ownership
peculiarly
granted
to
ICCs/IPs
over
their
ancestral lands and domains.
The
IPRA categorically declares ancestral
lands and domains held by native title
as never to have been public land.
Domains and lands held under native
title
are,
therefore,
indisputably
presumed to have never been public
lands and are private.
The concept of native title in the IPRA
was taken from the 1909 case of
Carino v. Insular Government (41 Phil.
935 [1909], 212 U.S. 449, 53 L. Ed.
594).
Carino firmly established a
concept of private land title that
existed irrespective of any royal grant
from the State. (Separate Opinion,
Puno, J., in Isagani Cruz v.
Secretary of DENR, et al., G.R. No.
135385, Dec. 6, 2000, En Banc)
53.
Distinguish ownership of land
under native title and ownership by
acquisitive prescription against the
State.

Held: Ownership by virtue of native


title presupposes that the land has
been held by its possessor and his
predecessor-in-interest in the concept
of an owner since time immemorial.
The land is not acquired from the
State, that is, Spain or its successor-ininterest, the United States and the
Philippine Government.
There has
been no transfer of title from the State
as the land has been regarded as
private in character as far back as
memory goes. In contrast, ownership
of land by acquisitive prescription
against
the
State
involves
a
conversion of the character of the
property from alienable public land to
private land, which presupposes a
transfer of title from the State to a
private person. (Separate Opinion,
Kapunan, J., in Isagani Cruz v.
Secretary of DENR, G.R. No.
135385, Dec. 6, 2000, En Banc)
The Right of the State to Recover
Properties Unlawfully Acquired by
Public Officials or Employees
54.
Does the right of the State to
recover properties unlawfully acquired
by public officials or employees which
may not be barred by prescription,
laches, or estoppel under Section 15,
Article XI of the Constitution apply to
criminal cases for the recovery of illgotten wealth?
Held: Section 15, Article XI, 1987
Constitution provides that [T]he right
of the State to recover properties
unlawfully acquired by public officials
or employees, from them or from their
nominees as transferees, shall not be
barred by prescription, laches, or
estoppel. From the proceedings of
the Constitutional Commission of
1986, however, it was clear that this
provision applies only to civil actions
for recovery of ill-gotten wealth, and
not to criminal cases.
Thus, the
prosecution of offenses arising from,
relating or incident to, or involving illgotten wealth contemplated in Section
15, Article XI of the Constitution may
be
barred
by
prescription.
(Presidential Ad Hoc Fact-Finding
Committee on Behest Loans, et al.
v. Hon. Aniano A. Desierto, et al.,
G.R. No. 130140, Oct. 25, 1999, En
Banc [Davide, C.J.])

STRUCTURE OF GOVERNMENT

The Doctrine of Separation of


Powers
55.
May the Government, through
the PCGG, validly bind itself to cause
the dismissal of all cases against the
Marcos heirs pending before the
Sandiganbayan and other courts in a
Compromise Agreement entered into
between the former and the latter?
Held: This is a direct encroachment
on judicial power, particularly in
regard to criminal jurisdiction. Wellsettled is the doctrine that once a case
has been filed before a court of
competent jurisdiction, the matter of
its dismissal or pursuance lies within
the full discretion and control of the
judge. In a criminal case, the manner
in which the prosecution is handled,
including the matter of whom to
present as witnesses, may lie within
the
sound
discretion
of
the
government prosecutor; but the court
decides, based on the evidence
proffered, in what manner it will
dispose of the case. Jurisdiction, once
acquired by the trial court, is not lost
despite a resolution, even by the
justice secretary, to withdraw the
information
or
to
dismiss
the
complaint. The prosecutions motion
to withdraw or to dismiss is not the
least binding upon the court. On the
contrary, decisional rules require the
trial court to make its own evaluation
of the merits of the case, because
granting such motion is equivalent to
effecting a disposition of the case
itself.
Thus, the PCGG, as the government
prosecutor of ill-gotten wealth cases,
cannot guarantee the dismissal of all
such criminal cases against the
Marcoses pending in the courts, for
said dismissal is not within its sole
power and discretion.
(Chavez v.
PCGG, 299 SCRA 744, Dec. 9, 1998
[Panganiban])
Delegation of Powers
56.
What are the tests of a valid
delegation of power?
Held: Empowering the COMELEC, an
administrative body exercising quasijudicial functions, to promulgate rules
and regulations is a form of delegation
of legislative authority x x x.
However, in every case of permissible
delegation, there must be a showing
that the delegation itself is valid. It is
valid only if the law (a) is complete in

itself, setting forth therein the policy


to be executed, carried out, or
implemented by the delegate; and (b)
fixes a standard the limits of which
are
sufficiently
determinate
and
determinable to which the delegate
must conform in the performance of
his functions. A sufficient standard is
one which defines legislative policy,
marks its limits, maps out its
boundaries and specifies the public
agency to apply it. It indicates the
circumstances
under
which
the
legislative command is to be effected.
(Santiago v. COMELEC, 270 SCRA
106, March 19, 1997)
The Legislative Department
57.
May
the
Supreme
Court
properly inquire into the motives of
the
lawmakers
in
conducting
legislative investigations?
Can it
enjoin the Congress or any of its
regular and special committees from
making inquiries in aid of legislation?
Held:
In
its
comment,
respondent Committee claims that this
Court cannot properly inquire into the
motives
of
the
lawmakers
in
conducting legislative investigations,
much less can it enjoin the Congress
or any of its regular and special
committees x x x from making
inquiries in aid of legislation, under the
doctrine of separation of powers,
which obtains in our present system of
government.
The contention is untenable. X x x
The allocation of constitutional
boundaries is a task that this Court
must perform under the Constitution.
Moreover, as held in a recent case
(Neptali A. Gonzales, et al. v. Hon.
Catalino Macaraig, Jr., et al., G.R. No.
87636, 19 November 1990, 191 SCRA
452, 463), [t]he political question
doctrine
neither
interposes
an
obstacle to judicial determination of
the rival claims. The jurisdiction to
delimit constitutional boundaries has
been given to this Court. It cannot
abdicate that obligation mandated by
the 1987 Constitution, although said
provision by no means does away with
the applicability of the principle in
appropriate cases. (Section 1, Article
VIII of the 1987 Constitution)

The Court is thus of the


considered view that it has jurisdiction
over the present controversy for the
purpose of determining the scope and
extent of the power of the Senate Blue
Ribbon Committee to conduct inquires
into private affairs in purported aid of
legislation. (Bengzon, Jr. v. Senate
Blue Ribbon Committee, 203 SCRA
767, Nov. 20, 1991, En Banc
[Padilla])
58.
Is the power of both houses of
Congress to conduct inquiries in aid of
legislation absolute or unlimited?
Held: The 1987 Constitution
expressly recognizes the power of
both houses of Congress to conduct
inquiries in aid of legislation (In
Arnault v. Nazareno, 87 Phil. 29, this
Court held that although there was no
express
provision
in
the
1935
Constitution giving such power to both
houses of Congress, it was so
incidental to the legislative function as
to be implied.).
Thus, Section 21,
Article VI provides x x x.
The power of both houses of
Congress to conduct inquiries in aid of
legislation is not, therefore, absolute
or unlimited.
Its exercise is
circumscribed by the afore-quoted
provision of the Constitution. Thus, as
provided therein, the investigation
must be in aid of legislation in
accordance with its duly published
rules of procedure and that the
rights of persons appearing in or
affected by such inquiries shall be
respected. It follows then that the
rights of persons under the Bill of
Rights must be respected, including
the right to due process and the right
not to be compelled to testify against
ones self.
The power to conduct formal
inquiries
or
investigations
is
specifically provided for in Sec. 1 of
the Senate Rules of Procedure
Governing
Inquiries
in
Aid
of
Legislation. Such inquiries may refer
to
the
implementation
or
reexamination of any law or in
connection
with
any
proposed
legislation or the formulation of future
legislation. They may also extend to
any and all matters vested by the
Constitution in Congress and/or in the
Senate alone.
As held in Jean L. Arnault v.
Leon Nazareno, et al, (No. L-3820, July
18, 1950, 87 Phil. 29), the inquiry, to
be within the jurisdiction of the

legislative body making it, must be


material or necessary to the exercise
of a power in it vested by the
Constitution, such as to legislate or to
expel a member.
Under
Sec.
4
of
the
aforementioned Rules, the Senate may
refer to any committee or committees
any speech or resolution filed by any
Senator which in its judgment requires
an appropriate inquiry in aid of
legislation.
In order therefore to
ascertain the character or nature of an
inquiry, resort must be had to the
speech or resolution under which such
an inquiry is proposed to be made.
(Bengzon, Jr. v. Senate Blue
Ribbon Committee, 203 SCRA 767,
Nov. 20, 1991, En Banc [Padilla])
59.
On 13 September 1988, the
Senate Minority Floor Leader, Hon.
Juan Ponce Enrile delivered a speech
on a matter of personal privilege
before the Senate on the alleged
take-over of SOLOIL Incorporated, the
flagship
on
the
First
Manila
Management of Companies (FMMC) by
Ricardo Lopa and called upon the
Senate to look into the possible
violation of the law in the case,
particularly with regard to Republic Act
No. 3019, the Anti-Graft and Corrupt
Practices Act.
On motion of Senator Orlando
Mercado, the matter was referred by
the Senate to the Committee on
Accountability of Public Officers (Blue
Ribbon Committee). Thereafter, the
Senate Blue Ribbon Committee started
its investigation on the matter.
Petitioners and Ricardo Lopa were
subpoenaed by the Committee to
appear before it and testify on what
they know regarding the sale of the
thirty-six (36) corporations belonging
to Benjamin Kokoy Romualdez.
At the hearing held on 23 May
1989, Ricardo Lopa declined to testify
on the ground that his testimony may
unduly prejudice the defendants in
Civil Case No. 0035 before the
Sandiganbayan.
Petitioner Jose F.S.
Bengzon, Jr. likewise refused to testify
invoking his constitutional right to due
process, and averring that the
publicity generated by respondent
Committees inquiry could adversely
affect his rights as well as those of the
other petitioners who are his co-

defendants in Civil Case No. 0035


before the Sandiganbayan.
The
Senate
Blue
Ribbon
Committee, thereupon, suspended its
inquiry and directed the petitioners to
file their memorandum on the
constitutional issues raised, after
which, it issued a resolution dated 5
June 1989 rejecting the petitioners
plea to be excused from testifying, and
the Committee voted to pursue and
continue its investigation of the
matter. X x x
Claiming that the Senate Blue
Ribbon Committee is poised to
subpoena and require their attendance
and testimony in proceedings before
the Committee, in excess of its
jurisdiction and legislative rights, and
that there is no appeal nor any other
plain, speedy and adequate remedy in
the ordinary course of law, the
petitioners filed the present petition
for prohibition with a prayer for
temporary restraining order and/or
injunctive relief.
Held: A perusal of the speech
of Senator Enrile reveals that he
(Senator Enrile) made a statement
which was published in various
newspapers on 2 September 1988
accusing Mr. Ricardo Baby Lopa of
having taken over the FMMC Group of
Companies. X x x
Verily, the speech of Senator
Enrile contained no suggestion of
contemplated legislation; he merely
called upon the Senate to look into a
possible violation of Sec. 5 of RA No.
3019, otherwise known as The AntiGraft and Corrupt Practices Act. In
other words, the purpose of the
inquiry to be conducted by respondent
Blue Ribbon Committee was to find
out whether or not the relatives of
President Aquino, particularly Mr.
Ricardo Lopa, had violated the law in
connection with the alleged sale of the
36 or 39 corporations belonging to
Benjamin Kokoy Romualdez to the
Lopa Group. There appears to be,
therefore, no intended legislation
involved.
Xxx
It appears, therefore, that the
contemplated inquiry by respondent
Committee is not really in aid of
legislation because it is not related to
a purpose within the jurisdiction of
Congress, since the aim of the
investigation is to find out whether or

not the relatives of the President or Mr.


Ricardo Lopa had violated Section 5 of
RA No. 3019, the Anti-Graft and
Corrupt Practices Act, a matter that
appears more within the province of
the courts rather than of the
legislature. Besides, the Court may
take judicial notice that Mr. Ricardo
Lopa died during the pendency of this
case. In John T. Watkins v. United
States (354 U.S. 178, 1 L. ed. 2D 1273
[1957]), it was held:
x x x. The power of Congress to
conduct inquiries in aid of legislation is
inherent in the legislative process.
That power is broad. It encompasses
inquiries
concerning
the
administration of existing laws as well
as proposed or possibly needed
statutes.
It includes surveys of
defects in our social, economic, or
political system for the purpose of
enabling Congress to remedy them. It
comprehends probes into departments
of the Federal Government to expose
corruption, inefficiency or waste. But
broad as is this power of inquiry, it is
not unlimited. There is no general
authority to expose the private affairs
of individuals without justification in
terms of the functions of Congress.
This was freely conceded by the
Solicitor General in his arguments in
this case. Nor is the Congress a law
enforcement or trial agency. These
are functions of the executive and
judicial departments of government.
No inquiry is an end in itself; it must
be related to and in furtherance of a
legislative
task
of
Congress.
Investigations conducted solely for the
personal
aggrandizement
of
the
investigators or to punish those
investigated are indefensible. (italics
supplied)
It cannot be overlooked that
when respondent Committee decided
to conduct its investigation of the
petitioners, the complaint in Civil Case
No. 0035 had already been filed with
the Sandiganbayan. A perusal of that
complaint shows that one of its
principal causes of action against
herein petitioners, as defendants
therein, is the alleged sale of the 36
(or 39) corporations belonging to
Benjamin Kokoy Romualdez. Since
the issues in said complaint had long
been joined by the filing of petitioners
respective answers thereto, the issue
sought to be investigated by the
respondent Committee is one over
which jurisdiction had been acquired
by the Sandiganbayan. In short, the
issue has been pre-empted by that

court.
To allow the respondent
Committee to conduct its own
investigation of an issue already
before the Sandiganbayan would not
only pose the possibility of conflicting
judgments between a legislative
committee and a judicial tribunal, but
if the Committees judgment were to
be reached before that of the
Sandiganbayan, the possibility of its
influence being made to bear on the
ultimate
judgment
of
the
Sandiganbayan can not be discounted.
In fine, for the respondent Committee
to probe and inquire into the same
justiciable controversy already before
the Sandiganbayan, would be an
encroachment into the exclusive
domain of judicial jurisdiction that had
much earlier set in. (Bengzon, Jr. v.
Senate Blue Ribbon Committee,
203 SCRA 767, Nov. 20, 1991, En
Banc [Padilla])
60.
Petitioners contention is that
Republic Act No. 7716 (The ExpandedVAT
Law)
did
not
originate
exclusively
in
the
House
of
Representatives as required by Art. VI,
Sec. 24 of the Constitution, because it
is in fact the result of the consolidation
of two distinct bills, H. No. 11197 and
S. No. 1630.
In this connection,
petitioners point out that although Art.
VI, Sec. 24 was adopted from the
American Federal Constitution, it is
notable in two respects: the verb
shall originate is qualified in the
Philippine Constitution by the word
exclusively and the phrase as on
other bills in the American version is
omitted.
This means, according to
them, that to be considered as having
originated in the House, Republic Act
No. 7716 must retain the essence of H.
No. 11197.
Held: This argument will not
bear analysis. To begin with, it is not
the law - but the revenue bill - which is
required by the Constitution to
originate exclusively in the House of
Representatives.
It is important to
emphasize this, because a bill
originating in the House may undergo
such extensive changes in the Senate
that the result may be a rewriting of
the whole. The possibility of a third
version by the conference committee
will be discussed later. At this point,
what is important to note is that, as a
result of the Senate action, a distinct
bill may be produced. To insist that a
revenue statute - and not only the bill
which initiated the legislative process
culminating in the enactment of the

law - must substantially be the same


as the House bill would be to deny the
Senate's power not only to concur
with amendments but also to
propose amendments. It would be
to violate the coequality of legislative
power of the two houses of Congress
and in fact make the House superior to
the Senate.
The
contention
that
the
constitutional design is to limit the
Senate's power in respect of revenue
bills in order to compensate for the
grant to the Senate of the treatyratifying power (Art. VII, Sec. 21) and
thereby equalize its powers and those
of the House overlooks the fact that
the powers being compared are
different. We are dealing here with
the legislative power which under the
Constitution is vested not only in any
particular
chamber
but
in
the
Congress of the Philippines, consisting
of a Senate and a House of
Representatives. (Art. VI, Sec. 1) The
exercise of the treaty-ratifying power
is not the exercise of legislative power.
It is the exercise of a check on the
executive power. There is, therefore,
no justification for comparing the
legislative powers of the House and of
the Senate on the basis of the
possession of a similar non-legislative
power by the Senate. The possession
of a similar power by the U.S. Senate
has never been thought of as giving it
more legislative powers than the
House of Representatives.
X x x Given, then, the power of
the Senate to propose amendments,
the Senate can propose its own
version even with respect to bills
which are required by the Constitution
to originate in the House.
It is insisted, however, that S.
No. 1630 was passed not in
substitution of H. No. 11197 but of
another Senate bill (S. No. 1129)
earlier filed and that what the Senate
did was merely to take (H. No. 11197)
into consideration in enacting S. No.
1630. There is really no difference
between the Senate preserving H. No.
11197 up to the enacting clause and
then writing its own version following
the enacting clause (which, it would
seem,
petitioners
admit
is
an
amendment by substitution), and, on
the other hand, separately presenting
a bill of its own on the same subject
matter. In either case the result are
two bills on the same subject.

Indeed, what the Constitution


simply means is that the initiative for
filing revenue, tariff, or tax bills, bills
authorizing an increase of the public
debt, private bills and bills of local
application must come from the House
of Representatives on the theory that,
elected as they are from the districts,
the members of the House can be
expected to be more sensitive to the
local needs and problems. On the
other hand, the senators, who are
elected at large, are expected to
approach the same problems from the
national perspective. Both views are
thereby made to bear on the
enactment of such laws.
Nor does the Constitution
prohibit the filing in the Senate of a
substitute bill in anticipation of its
receipt of the bill from the House, so
long as action by the Senate as a body
is withheld pending receipt of the
House bill.
The Court cannot,
therefore, understand the alarm
expressed over the fact that on March
1, 1993, eight months before the
House passed H. No. 11197, S. No.
1129 had been filed in the Senate.
After all it does not appear that the
Senate ever considered it. It was only
after the Senate had received H. No.
11197 on November 23, 1993 that the
process of legislation in respect of it
began with the referral to the Senate
Committee on Ways and Means of H.
No. 11197 and the submission by the
Committee on February 7, 1994 of S.
No. 1630.
For that matter, if the
question were simply the priority in
the time of filing of bills, the fact is
that it was in the House that a bill (H.
No. 253) to amend the VAT law was
first filed on July 22, 1992. Several
other bills had been filed in the House
before S. No. 1129 was filed in the
Senate, and H. No. 11197 was only a
substitute of those earlier bills.
(Tolentino v. Secretary of Finance,
235 SCRA 630, 661-663, Aug. 25,
1994, En Banc [Mendoza])
61.
Discuss the nature of the PartyList system.
Is it, without any
qualification, open to all?
Held: 1. The party-list system is a
social justice tool designed not only to
give more law to the great masses of
our people who have less in life, but
also to enable them to become
veritable
lawmakers
themselves,
empowered to participate directly in
the enactment of laws designed to
benefit them. It intends to make the

marginalized
and
the
underrepresented not merely passive
recipients of the States benevolence,
but
active
participants
in
the
mainstream
of
representative
democracy.
Thus, allowing all
individuals and groups, including those
which now dominate district elections,
to have the same opportunity to
participate in party-list elections would
desecrate this lofty objective and
mongrelize
the
social
justice
mechanism into an atrocious veneer
for traditional politics. (Ang Bagong
Bayani OFW Labor Party v.
COMELEC, G.R. No. 147589, June
26, 2001, En Banc [Panganiban])
2. Crucial to the resolution of this
case is the fundamental social justice
principle that those who have less in
life should have more in law. The
party-list system is one such tool
intended to benefit those who have
less in life. It gives the great masses
of our people genuine hope and
genuine power. It is a message to the
destitute and the prejudiced, and even
to those in the underground, that
change is possible. It is an invitation
for them to come out of their limbo
and seize the opportunity.
Clearly, therefore, the Court cannot
accept the submissions x x x that the
party-list system is, without any
qualification, open to all.
Such
position does not only weaken the
electoral chances of the marginalized
and
underrepresented;
it
also
prejudices them.
It would gut the
substance of the party-list system.
Instead of generating hope, it would
create a mirage. Instead of enabling
the marginalized, it would further
weaken them and aggravate their
marginalization.
(Ang
Bagong
Bayani OFW Labor Party v.
COMELEC, G.R. No. 147589, June
26, 2001, En Banc [Panganiban])
62.
Are political parties even the
major
ones

prohibited
from
participating
in
the
party-list
elections?
Held: Under the Constitution and RA
7941, private respondents cannot be
disqualified
from
the
party-list
elections, merely on the ground that
they are political parties. Section 5,
Article VI of the Constitution, provides
that members of the House of
Representatives may be elected
through
a
party-list
system
of
registered national, regional, and
sectoral parties or organizations.

Furthermore, under Sections 7 and 8,


Article IX [C] of the Constitution,
political parties may be registered
under the party-list system. X x x
During the deliberations in the
Constitutional Commission, Comm.
Christian S. Monsod pointed out that
the participants in the party-list
system may be a regional party, a
sectoral party, a national party,
UNIDO, Magsasaka, or a regional party
in Mindanao. x x x.
Indeed, Commissioner Monsod stated
that the purpose of the party-list
provision was to open up the system,
in order to give a chance to parties
that consistently place third or fourth
in congressional district elections to
win a seat in Congress. He explained:
The purpose of this is to open the
system.
In the past elections, we
found out that there were certain
groups or parties that, if we count
their votes nationwide, have about
1,000,000 or 1,500,000 votes. But
they were always third or fourth place
in each of the districts. So, they have
no voice in the Assembly. But this
way, they would have five or six
representatives in the assembly even
if they would not win individually in
legislative districts.
So, that is
essentially
the
mechanics,
the
purpose and objective of the party-list
system.
For its part, Section 2 of RA 7941 also
provides for a party-list system of
registered national, regional and
sectoral parties or organizations or
coalitions thereof, x x x. Section 3
expressly states that a party is
either a political party or a sectoral
party or a coalition of parties. More
to the point, the law defines political
party as an organized group of
citizens advocating an ideology or
platform, principles and policies for
the general conduct of government
and which, as the most immediate
means of securing their adoption,
regularly nominates and supports
certain of its leaders and members as
candidates for public office.
Furthermore, Section 11 of RA 7941
leaves no doubt as to the participation
of political parties in the party-list
system. X x x
Indubitably, therefore, political parties
even the major ones may
participate in the party-list elections.

That political parties may participate


in the party-list elections does not
mean, however, that any political
party or any organization or group
for that matter may do so. The
requisite character of these parties or
organizations must be consistent with
the purpose of the party-list system,
as laid down in the Constitution and
RA 7941. X x x (Ang Bagong Bayani
OFW Labor Party v. COMELEC,
G.R. No. 147589, June 26, 2001,
En Banc [Panganiban])
63.
Who are the marginalized and
underrepresented
sectors
to
be
represented
under
the
party-list
system?
Held:
The
marginalized
and
underrepresented
sectors
to
be
represented
under
the
party-list
system are enumerated in Section 5 of
RA 7941 x x x.
While the enumeration of marginalized
and underrepresented sectors is not
exclusive, it demonstrates the clear
intent of the law that not all sectors
can be represented under the partylist system. X x x
[W]e stress that the party-list system
seeks to enable certain Filipino citizens
specifically those belonging to
marginalized and underrepresented
sectors, organizations and parties to
be
elected
to
the
House
of
Representatives. The assertion x x x
that the party-list system is not
exclusive to the marginalized and
underrepresented disregards the clear
statutory policy. Its claim that even
the super-rich and overrepresented
can participate desecrates the spirit of
the party-list system.
Indeed, the law crafted to address the
peculiar disadvantage of Payatas
hovel dwellers cannot be appropriated
by the mansion owners of Forbes Park.
The interests of these two sectors are
manifestly disparate; hence, the x x x
position to treat them similarly defies
reason and common sense. X x x
While the business moguls and the
mega-rich are, numerically speaking, a
tiny minority, they are neither
marginalized nor underrepresented,
for the stark reality is that their
economic clout engenders political
power more awesome than their
numerical limitation.
Traditionally,
political power does not necessarily
emanate from the size of ones
constituency; indeed, it is likely to

arise more directly from the number


and amount of ones bank accounts.
It is ironic, therefore, that the
marginalized and underrepresented in
our midst are the majority who wallow
in poverty, destitution and infirmity. It
was for them that the party-list system
was enacted to give them not only
genuine hope, but genuine power; to
give them opportunity to be elected
and to represent the specific concerns
of their constituencies; and simply to
give them a direct vote in Congress
and in the larger affairs of the State.
In its noblest sense, the party-list
system truly empowers the masses
and ushers a new hope for genuine
change.
Verily, it invites those
marginalized and underrepresented in
the past the farm hands, the fisher
folk, the urban poor, even those in the
underground movement to come out
and participate, as indeed many of
them came out and participated
during the last elections. The State
cannot now disappoint and frustrate
them by disabling the desecrating this
social justice vehicle.
Because
the
marginalized
and
underrepresented had not been able
to win in the congressional district
elections normally dominated by
traditional politicians and vested
groups, 20 percent of the seats in the
House of Representatives were set
aside for the party-list system.
In
arguing that even those sectors who
normally controlled 80 percent of the
seats in the House could participate in
the
party-list
elections
for
the
remaining 20 percent, the OSG and
the
Comelec
disregard
the
fundamental difference between the
congressional district elections and
the party-list elections.
As earlier noted, the purpose of the
party-list provision was to open up the
system, in order to enhance the
chance of sectoral groups and
organizations to gain representation in
the House of Representatives through
the simplest scheme possible. Logic
shows that the system has been
opened to those who have never
gotten a foothold within it those who
cannot otherwise win in regular
elections and who therefore need the
simplest scheme possible to do so.
Conversely, it would be illogical to
open the system to those who have
long been within it those privileged
sectors that have long dominated the
congressional district elections.

Xxx
Verily, allowing the non-marginalized
and overrepresented to vie for the
remaining seats under the party-list
system would not only dilute, but also
prejudice
the
chance
of
the
marginalized and underrepresented,
contrary to the intention of the law to
enhance it. The party-list system is a
tool
for
the
benefit
of
the
underprivileged; the law could not
have given the same tool to others, to
the
prejudice
of
the
intended
beneficiaries. (Ang Bagong Bayani
OFW Labor Party v. COMELEC, G.R.
No. 147589, June 26, 2001, En
Banc [Panganiban])
64.
Section 5(2), Article VI of the
Constitution provides that "[t]he partylist representatives shall constitute
twenty per centum of the total number
of representatives including those
under the party-list."
Does the
Constitution require all such allocated
seats to be filled up all the time and
under all circumstances?
Held: The Constitution simply states
that "[t]he party-list representatives
shall constitute twenty per centum of
the total number of representatives
including those under the party-list."
Xxx
We rule that a simple reading of
Section
5,
Article
VI
of
the
Constitution,
easily
conveys
the
equally simple message that Congress
was vested with the broad power to
define and prescribe the mechanics of
the party-list system of representation.
The Constitution explicitly sets down
only the percentage of the total
membership
in
the
House
of
Representatives reserved for party-list
representatives.
In the exercise of its constitutional
prerogative, Congress enacted RA
7941.
As said earlier, Congress
declared therein a policy to promote
"proportional representation" in the
election of party-list representatives in
order to enable Filipinos belonging to
the
marginalized
and
underrepresented
sectors
to
contribute legislation that would
benefit them. It however deemed it
necessary
to
require
parties,
organizations
and
coalitions
participating in the system to obtain at
least two percent of the total votes
cast for the party-list system in order
to be entitled to a party-list seat.

Those garnering more than this


percentage could have "additional
seats in proportion to their total
number of votes." Furthermore, no
winning party, organization or coalition
can have more than three seats in the
House of Representatives. X x x
Considering the foregoing statutory
requirements, it will be shown x x x
that Section 5(2), Article VI of the
Constitution is not mandatory.
It
merely provides a ceiling for party-list
seats
in
Congress.
(Veterans
Federation Party v. COMELEC, G.R.
No. 136781, Oct. 6, 2000, En Banc
[Panganiban])
65.
What
are
the
inviolable
parameters to determine the winners
in a Philippine-style party-list election?
Held: To determine the winners in a
Philippine-style party-list election, the
Constitution and Republic Act No.
7941 mandate at least four inviolable
parameters. These are:
First, the twenty percent allocation
- the combined number of all party-list
congressmen shall not exceed twenty
percent of the total membership of the
House of Representatives, including
those elected under the party list.
Second, the two percent threshold
- only those garnering a minimum of
two percent of the total valid votes
cast for the party-list system are
"qualified" to have a seat in the House
of Representatives.
Third, the three seat limit - each
qualified party, regardless of the
number of votes it actually obtained,
is entitled to a maximum of three
seats; that is, one "qualifying" and two
additional seats.
Fourth,
proportional
representation - the additional seats
which a qualified party is entitled to
shall be computed "in proportion to
their
total
number
of
votes."
(Veterans Federation Party v.
COMELEC, G.R. No. 136781 and
Companion Cases, Oct. 6, 2000,
En Banc [Panganiban])
66.
State
the
guidelines
screening Party-List Participants.

for

Held: In this light, the Court finds it


appropriate to lay down the following
guidelines, culled from the law and the
Constitution, to assist the Comelec in
its work.

First, the political party, sector,


organization
or
coalition
must
represent
the
marginalized
and
underrepresented groups identified in
Section 5 of RA 7941. In other words,
it must show through its constitution,
articles of incorporation, bylaws,
history, platform of government and
track record that it represents and
seeks to uplift marginalized and
underrepresented sectors.
Verily,
majority of its membership should
belong to the marginalized and
underrepresented.
And it must
demonstrate that in a conflict of
interest, it has chosen or is likely to
choose the interest of such sectors.

2)
3)
4)

5)
Second, while even major political
parties are expressly allowed by RA
7941
and
the
Constitution
to
participate in the party-list system,
they must comply with the declared
statutory policy of enabling Filipino
citizens belonging to marginalized and
underrepresented sectors x x x to be
elected
to
the
House
of
Representatives.
In other words,
while they are not disqualified merely
on the ground that they are political
parties, they must show, however,
that they represent the interests of the
marginalized and underrepresented. X
x x.
Third, in view of the objections
directed against the registration of
Ang Buhay Hayaang Yumabong, which
is allegedly a religious group, the
Court notes the express constitutional
provision that the religious sector may
not be represented in the party-list
system. x x x
Furthermore, the Constitution provides
that religious denominations and
sects shall not be registered. (Sec. 2
[5], Article IX [C]) The prohibition was
explained by a member of the
Constitutional Commission in this
wise:
[T]he prohibition is on any
religious organization registering as a
political party.
I do not see any
prohibition here against a priest
running as a candidate. That is not
prohibited here; it is the registration of
a religious sect as a political party.
Fourth, a party or an organization
must not be disqualified under Section
6 of RA 7941, which enumerates the
grounds for disqualification as follows:
1)

It is a religious sect or
denomination,
organization
or

6)
7)
8)

association organized for


religious purposes;
It advocates violence
or unlawful means to
seek its goal;
It is a foreign party or
organization;
It is receiving support
from
any
foreign
government,
foreign
political
party,
foundation, organization,
whether
directly
or
through any of its officers
or members or indirectly
through third parties for
partisan
election
purposes;
It violates or fails to
comply with laws, rules or
regulations relating to
elections;
It declares untruthful
statements in its petition;
It has ceased to exist
for at least one (1) year;
or
It fails to participate in
the last two (2) preceding
elections or fails to obtain
at least two per centum
(2%) of the votes cast
under
the
party-list
system in the two (2)
preceding elections for
the constituency in which
it had registered.

Note should be taken of paragraph 5,


which disqualifies a party or group for
violation of or failure to comply with
election laws and regulations. These
laws include Section 2 of RA 7941,
which states that the party-list system
seeks to enable Filipino citizens
belonging
to
marginalized
and
underrepresented
sectors,
organizations and parties x x x to
become members of the House of
Representatives.
A
party
or
organization, therefore, that does not
comply with this policy must be
disqualified.
Fifth, the party or organization must
not be an adjunct of, or a project
organized or an entity funded or
assisted by, the government. By the
very nature of the party-list system,
the party or organization must be a
group of citizens, organized by citizens
and operated by citizens. It must be
independent of the government. The
participation of the government or its
officials in the affairs of a party-list
candidate is not only illegal and unfair
to other parties, but also deleterious

to the objective of the law: to enable


citizens belonging to marginalized and
underrepresented
sectors
and
organization to be elected to the
House of Representatives.
Sixth, the party must not only comply
with the requirements of the law; its
nominees must likewise do so. x x x
Seventh, not only the candidate party
or
organization
must
represent
marginalized and underrepresented
sectors; so also must its nominees. To
repeat, under Section 2 of RA 7941,
the nominees must be Filipino citizens
who belong to marginalized and
underrepresented
sectors,
organizations and parties. Surely, the
interests of the youth cannot be fully
represented by a retiree; neither can
those of the urban poor or the working
class, by an industrialist.
To allow
otherwise is to betray the State policy
to give genuine representation to the
marginalized and underrepresented.

implication
considerations.

or

equitable

The 1935 Constitution provided in its


Article
VI
on
the
Legislative
Department:
Sec. 15. The Senators and Members
of the House of Representatives shall
in all cases except treason, felony, and
breach of the peace, be privileged
from arrest during their attendance at
the sessions of Congress, and in going
to and returning from the same; x x x.
Because of the broad coverage of
felony and breach of the peace, the
exemption applied only to civil arrests.
A congressman like the accusedappellant, convicted under Title Eleven
of the Revised Penal Code could not
claim parliamentary immunity from
arrest. He was subject to the same
general laws governing all persons still
to be tried or whose convictions were
pending appeal.

Eighth, x x x while lacking a welldefined political constituency, the


nominee must likewise be able to
contribute to the formulation and
enactment of appropriate legislation
that will benefit the nation as a whole.
x x x (Ang Bagong Bayani OFW
Labor Party v. COMELEC, G.R. No.
147589, June 26, 2001, En Banc
[Panganiban])

The 1973 Constitution broadened the


privilege of immunity as follows:

67.
Accused-appellant
Congressman Romeo G. Jalosjos filed a
motion before the Court asking that he
be allowed to fully discharge the
duties of a Congressman, including
attendance at legislative sessions and
committee meetings despite his
having been convicted in the first
instance of a non-bailable offense. He
contended that his reelection being an
expression of popular will cannot be
rendered inutile by any ruling, giving
priority to any right or interest not
even the police power of the State.
Resolve.

For offenses punishable by more than


six years imprisonment, there was no
immunity from arrest. The restrictive
interpretation of immunity and the
intent to confine it within carefully
defined parameters is illustrated by
the
concluding
portion
of
the
provision, to wit:

Held: The immunity from arrest or


detention of Senators and members of
the House of Representatives x x x
arises from a provision of the
Constitution.
The history of the
provision shows that the privilege has
always been granted in a restrictive
sense.
The provision granting an
exemption as a special privilege
cannot be extended beyond the
ordinary meaning of its terms. It may
not be extended by intendment,

Article VIII, Sec. 9. A Member of the


Batasang Pambansa shall, in all
offenses punishable by not more than
six years imprisonment, be privileged
from arrest during his attendance at
its sessions and in going to and
returning from the same.

X x x but the Batasang Pambansa shall


surrender the member involved to the
custody of the law within twenty four
hours after its adjournment for a
recess or for its next session,
otherwise such privilege shall cease
upon its failure to do so.
The present Constitution adheres to
the same restrictive rule minus the
obligation of Congress to surrender
the subject Congressman to the
custody of the law. The requirement
that he should be attending sessions
or committee meetings has also been
removed.
For relatively minor
offenses, it is enough that Congress is
in session.

The accused-appellant argues that a


member of Congress function to
attend sessions is underscored by
Section 16(2), Article VI of the
Constitution which states that
(2) A majority of each House shall
constitute a quorum to do business,
but a smaller number may adjourn
from day to day and may compel the
attendance of absent Members in such
manner, and under such penalties, as
such House may provide.
However, the accused-appellant has
not given any reason why he should
be exempted from the operation of
Section
11, Article
VI
of
the
Constitution.
The members of
Congress cannot compel
absent
members to attend sessions if the
reason for the absence is a legitimate
one.
The
confinement
of
a
Congressman charged with a crime
punishable by imprisonment of more
than six years is not merely authorized
by
law,
it
has
constitutional
foundations.
Accused-appellants reliance on the
ruling in Aguinaldo v. Santos (212
SCRA 768, at 773 [1992]), which
states, inter alia, that
The Court should never remove a
public officer for acts done prior to his
present term of office. To do otherwise
would be to deprive the people of their
right to elect their officers. When the
people have elected a man to office, it
must be assumed that they did this
with the knowledge of his life and
character, and that they disregarded
or forgave his fault or misconduct, if
he had been guilty of any. It is not for
the Court, by reason of such fault or
misconduct, to practically overrule the
will of the people.
will not extricate him from his
predicament. It can be readily seen x
x x that the Aguinaldo case involves
the administrative removal of a public
officer for acts done prior to his
present term of office. It does not
apply to imprisonment arising from
the enforcement of criminal law.
Moreover, in the same way that
preventive suspension is not removal,
confinement pending appeal is not
removal. He remains a Congressman
unless expelled by Congress or,
otherwise, disqualified.
One rationale behind confinement,
whether pending appeal or after final
conviction, is public self-defense.

Society must protect itself. It also


serves as an example and warning to
others.
A person charged with crime is taken
into custody for purposes of the
administration of justice. As stated in
United States v. Gustilo (19 Phil. 208,
212), it is the injury to the public
which State action in criminal law
seeks to redress. It is not the injury to
the complainant. After conviction in
the Regional Trial Court, the accused
may be denied bail and thus subjected
to incarceration if there is risk of his
absconding.
The accused-appellant states that the
plea of the electorate which voted him
into office cannot be supplanted by
unfounded fears that he might escape
eventual punishment if permitted to
perform congressional duties outside
his regular place of confinement.
It will be recalled that when a warrant
for accused-appellants arrest was
issued, he fled and evaded capture
despite a call from his colleagues in
the House of Representatives for him
to attend the sessions ands to
surrender
voluntarily
to
the
authorities. Ironically, it is now the
same body whose call he initially
spurned which accused-appellant is
invoking to justify his present motion.
This can not be countenanced
because, x x x aside from its being
contrary to well-defined Constitutional
restrains, it would be a mockery of the
aims of the States penal system.
Accused-appellant argues that on
several occasions, the Regional Trial
Court of Makati granted several
motions to temporarily leave his cell at
the Makati City Jail, for official or
medical reasons x x x.
He also calls attention to various
instances, after his transfer at the New
Bilibid Prison in Muntinlupa City, when
he was likewise allowed/permitted to
leave the prison premises x x x.
There is no showing that the above
privileges are peculiar to him or to a
member of Congress. Emergency or
compelling temporary leaves from
imprisonment are allowed to all
prisoners, at the discretion of the
authorities or upon court orders.
What the accused-appellant seeks is
not of an emergency nature. Allowing
accused-appellant
to
attend
congressional sessions and committee

meetings for five (5) days or more in a


week will virtually make him a free
man
with
all
the
privileges
appurtenant to his position. Such an
aberrant situation not only elevates
accused-appellants status to that of a
special class, it also would be a
mockery of the purposes of the
correction system. X x x
The accused-appellant avers that his
constituents in the First District of
Zamboanga del Norte want their
voices to be heard and that since he is
treated as bona fide member of the
House of Representatives, the latter
urges
a
co-equal
branch
of
government to respect his mandate.
He also claims that the concept of
temporary
detention
does
not
necessarily
curtail
his
duty
to
discharge his mandate and that he has
always
complied
with
the
conditions/restrictions when he is
allowed to leave jail.
We remain unpersuaded.
Xxx
When the voters of his district elected
the accused-appellant to Congress,
they did so with full awareness of the
limitations on his freedom of action.
They did so with the knowledge that
he could achieve only such legislative
results which he could accomplish
within the confines of prison. To give a
more drastic illustration, if voters elect
a person with full knowledge that he is
suffering from a terminal illness, they
do so knowing that at any time, he
may no longer serve his full term in
office.
(People v. Jalosjos, 324
SCRA 689, Feb. 3, 2000, En Banc
[Ynares-Santiago])
68.
Discuss
the
objectives
of
Section 26(1), Article VI of the 1987
Constitution, that "[e]very bill passed
by the Congress shall embrace only
one subject which shall be expressed
in the title thereof."
Held: The objectives of Section 26(1),
Article VI of the 1987 Constitution are:
1)

To prevent hodgepodge or log-rolling legislation;


2)
To prevent surprise or
fraud upon the legislature by
means of provisions in bills of
which the titles gave no
information, and which might
therefore be overlooked and
carelessly and unintentionally
adopted; and

3)

To fairly apprise the


people,
through
such
publication
of
legislative
proceedings as is usually made,
of the subjects of legislation
that are being considered, in
order that they may have
opportunity of being heard
thereon by petition or otherwise
if they shall so desire.
Section 26(1) of Article VI of the 1987
Constitution is sufficiently complied
with where x x x the title is
comprehensive enough to embrace
the general objective it seeks to
achieve, and if all the parts of the
statute are related and germane to the
subject matter embodied in the title or
so long as the same are not
inconsistent with or foreign to the
general subject and title. (Agripino
A. De Guzman, Jr., et al. v.
COMELEC, G.R. No. 129118, July
19, 2000, en Banc [Purisima])
69.
Section 44 of R.A. No. 8189 (The
Voter's Registration Act of 1996) which
provides for automatic transfer to a
new station of any Election Officer who
has already served for more than four
years
in
a
particular
city
or
municipality was assailed for being
violative of Section 26(1) of Article VI
of the Constitution allegedly because
it has an isolated and different subject
from that of RA 8189 and that the
same is not expressed in the title of
the law.
Should the challenge be
sustained?
Held: Section 44 of RA 8189 is not
isolated considering that it is related
and germane to the subject matter
stated in the title of the law. The title
of RA 8189 is "The Voter's Registration
Act of 1996" with a subject matter
enunciated in the explanatory note as
"AN ACT PROVIDING FOR A GENERAL
REGISTRATION OF VOTERS, ADOPTING
A
SYSTEM
OF
CONTINUING
REGISTRATION,
PRESCRIBING
THE
PROCEDURES
THEREOF
AND
AUTHORIZING THE APPROPRIATION OF
FUNDS THEREFOR." Section 44, which
provides for the reassignment of
election officers, is relevant to the
subject matter of registration as it
seeks to ensure the integrity of the
registration process by providing
guideline for the COMELEC to follow in
the reassignment of election officers.
It is not an alien provision but one
which is related to the conduct and
procedure of continuing registration of
voters.
In this regard, it bears
stressing that the Constitution does

not require Congress to employ in the


title of an enactment, language of
such precision as to mirror, fully index
or catalogue, all the contents and the
minute details therein. (Agripino A.
De Guzman, Jr., et al. v. COMELEC,
G.R. No. 129118, July 19, 2000, En
Banc [Purisima])
70.
Do courts have the power to
inquire into allegations that, in
enacting a law, a House of Congress
failed to comply with its own rules?
Held:
The cases, both here and
abroad, in varying forms of expression,
all deny to the courts the power to
inquire into allegations that, in
enacting a law, a House of Congress
failed to comply with its own rules, in
the absence of showing that there was
a violation of a constitutional provision
or the right of private individuals. In
Osmena v. Pendatun (109 Phil. At 870871), it was held: At any rate, courts
have declared that the rules adopted
by deliberative bodies are subject to
revocation, modification or waiver at
the pleasure of the body adopting
them.
And it has been said that
Parliamentary
rules
are
merely
procedural, and with their observance,
the courts have no concern. They may
be waived or disregarded by the
legislative body. Consequently, mere
failure to conform to parliamentary
usage will not invalidate that action
(taken by a deliberative body) when
the requisite number of members
have agreed to a particular measure.
It must be realized that each of the
three departments of our government
has its separate sphere which the
others may not invade without
upsetting the delicate balance on
which our constitutional order rests.
Due regard for the working of our
system of government, more than
mere comity, compels reluctance on
the part of the courts to enter upon an
inquiry into an alleged violation of the
rules of the House.
Courts must
accordingly decline the invitation to
exercise their power. (Arroyo v. De
Venecia, 277 SCRA 268, Aug. 14,
1997 [Mendoza])
71.
What
is
the
Bicameral
Conference Committee? Discuss the
nature of its function and its
jurisdiction.
Held:
While it is true that a
conference
committee
is
the
mechanism
for
compromising
differences between the Senate and

the House, it is not limited in its


jurisdiction to this question.
Its
broader function is described thus:
A conference committee may deal
generally with the subject matter or it
may be limited to resolving the precise
differences between the two houses.
Even where the conference committee
is not by rule limited in its jurisdiction,
legislative custom severely limits the
freedom with which new subject
matter can be inserted into the
conference bill.
But occasionally a
conference
committee
produces
unexpected results, results beyond its
mandate.
These excursions occur
even where the rules impose strict
limitations on conference committee
jurisdiction. This is symptomatic of
the authoritarian power of conference
committee.
(Philippine
Judges
Association v. Prado, 227 SCRA
703, Nov. 11, 1993, En Banc
[Cruz])
72.
Discuss
Doctrine.

the

Enrolled

Bill

Held: Under the enrolled bill doctrine,


the signing of H. Bill No. 7189 by the
Speaker of the House and the
President of the Senate and the
certification by the secretaries of both
Houses of Congress that it was passed
on November 21, 1996 are conclusive
of its due enactment. x x x To be
sure, there is no claim either here or in
the decision in the EVAT cases
(Tolentino v. Secretary of Finance) that
the enrolled bill embodies a conclusive
presumption. In one case (Astorga v.
Villegas, 56 SCRA 714 [1974]) we
went behind an enrolled bill and
consulted the Journal to determine
whether certain provisions of a statute
had been approved by the Senate.
But, where as here there is no
evidence to the contrary, this Court
will respect the certification of the
presiding officers of both Houses that
a bill has been duly passed. Under
this rule, this Court has refused to
determine claims that the threefourths vote needed to pass a
proposed
amendment
to
the
Constitution had not been obtained,
because a duly authenticated bill or
resolution imports absolute verity and
is binding on the courts. x x x
This Court has refused to even look
into allegations that the enrolled bill
sent to the President contained
provisions
which
had
been
surreptitiously
inserted
in
the

conference committee x x x. (Tolentino


v. Secretary of Finance)

evidence to the contrary, the Journals


have also been accorded conclusive
effects. Thus, in United States v. Pons
(34 Phil. 729, 735 [1916]], quoting ex
rel. Herron v. Smith, 44 Ohio 348
[1886]), this Court spoke of the
imperatives of public policy for
regarding the Journals as public
memorials of the most permanent
character, thus: They should be
public, because all are required to
conform to them; they should be
permanent, that rights acquired today
upon the faith of what has been
declared to be law shall not be
destroyed tomorrow, or at some
remote period of time, by facts resting
only in the memory of individuals. X
x x. (Arroyo v. De Venecia, 277
SCRA 268, 298-299, Aug. 14, 1997
[Mendoza])

It has refused to look into charges that


an amendment was made upon the
last reading of a bill in violation of Art.
VI, Sec. 26(2) of the Constitution that
upon the last reading of a bill, no
amendment
shall
be
allowed.
(Philippine Judges Assn v. Prado, 227
SCRA 703, 710 [1993])
In other cases, this Court has denied
claims that the tenor of a bill was
otherwise than as certified by the
presiding officers of both Houses of
Congress.
The enrolled bill doctrine, as a rule of
evidence, is well-established.
It is
cited with approval by text writers
here and abroad. The enrolled bill rule
rests on the following considerations:
X x x.
As the President has no
authority to approve a bill not passed
by Congress, an enrolled Act in the
custody of the Secretary of State, and
having the official attestations of the
Speaker
of
the
House
of
Representatives, of the President of
the Senate, and of the President of the
United States, carries, on its face, a
solemn assurance by the legislative
and executive departments of the
government, charged, respectively,
with the duty of enacting and
executing the laws, that it was passed
by Congress.
The respect due to
coequal and independent departments
requires the judicial department to act
upon that assurance, and to accept, as
having passed Congress, all bills
authenticated in the manner stated;
leaving the court to determine, when
the question properly arises, whether
the Act, so authenticated, is in
conformity with the Constitution.
(Marshall Field & Co. v. Clark, 143 U.S.
649, 672, 36 L. Ed. 294, 303 [1891])
To overrule the doctrine now, x x x is
to repudiate the massive teaching of
our
cases
and
overthrow
an
established rule of evidence. (Arroyo
v. De Venecia, 277 SCRA 268, Aug.
14, 1997 [Mendoza])
73.
When should the Legislative
Journal be regarded as conclusive
upon the courts, and why?
Held:
The Journal is regarded as
conclusive with respect to matters
that are required by the Constitution
to be recorded therein. With respect
to other matters, in the absence of

74.
What matters are required to be
entered on the Journal?
Held:
1)

The yeas and nays on the


third and final reading of a bill (Art. VI,
Sec. 26[2]);
2)
The yeas and nays on any
question, at the request of one-fifth of
the members present (Id., Sec. 16[4]);
3)
The yeas and nays upon
repassing a bill over the Presidents
veto (Id., Sec. 27[1]); and
4)
The Presidents objection to
a bill he had vetoed (Id.).
(Arroyo v. De Venecia, 277 SCRA
268,
298,
Aug.
14,
1997
[Mendoza])
75.
A disqualification case was filed
against a candidate for Congressman
before the election with the COMELEC.
The latter failed to resolve that
disqualification
case
before
the
election and that candidate won,
although he was not yet proclaimed
because
of
that
pending
disqualification case. Is the COMELEC
now ousted of jurisdiction to resolve
the pending disqualification case and,
therefore, should dismiss the case,
considering that jurisdiction is now
vested
with
the
House
of
Representatives
Electoral
Tribunal
(HRET)?
Held: 1. In his first assignments of
error, petitioner vigorously contends
that after the May 8, 1995 elections,
the COMELEC lost its jurisdiction over
the
question
of
petitioners
qualifications to run for member of the
House of Representatives. He claims
that jurisdiction over the petition for

disqualification is exclusively lodged


with the House of Representatives
Electoral Tribunal (HRET). Given the
yet-unresolved question of jurisdiction,
petitioner avers that the COMELEC
committed serious error and grave
abuse of discretion in directing the
suspension of his proclamation as the
winning candidate in the Second
Congressional District of Makati City.
We disagree.
Petitioner conveniently confuses the
distinction between an unproclaimed
candidate
to
the
House
of
Representatives and a member of the
same. Obtaining the highest number
of votes in an election does not
automatically vest the position in the
winning candidate.
Section 17 of
Article VI of the 1987 Constitution
reads:
The Senate and the House of
Representatives
shall
have
an
Electoral Tribunal which shall be the
sole judge of all contests relating to
the election, returns and qualifications
of their respective Members.
Under the above-stated provision, the
electoral tribunal clearly assumes
jurisdiction over all contests relative to
the election, returns and qualifications
of candidates for either the Senate or
the House only when the latter
become members of either the Senate
or the House of Representatives. A
candidate
who
has
not
been
proclaimed and who has not taken his
oath of office cannot be said to be a
member
of
the
House
of
Representatives subject to Section 17
of Article VI of the Constitution. While
the proclamation of a winning
candidate in an election is ministerial,
B.P. Blg. 881 in conjunction with Sec. 6
of R.A. 6646 allows suspension of
proclamation under circumstances
mentioned therein. Thus, petitioners
contention that after the conduct of
the election and (petitioner) has been
established the winner of the electoral
exercise from the moment of election,
the
COMELEC
is
automatically
divested of authority to pass upon the
question of qualification finds no
basis in law, because even after the
elections the COMELEC is empowered
by Section 6 (in relation to Section 7)
of R.A. 6646 to continue to hear and
decide
questions
relating
to
qualifications of candidates. X x x.
Under the above-quoted provision, not
only is a disqualification case against a
candidate allowed to continue after

the election (and does not oust the


COMELEC of its jurisdiction), but his
obtaining the highest number of votes
will not result in the suspension or
termination of the proceedings against
him when the evidence of guilt is
strong. While the phrase when the
evidence of guilt is strong seems to
suggest that the provisions of Section
6 ought to be applicable only to
disqualification cases under Section 68
of the Omnibus Election Code, Section
7 of R.A. 6646 allows the application of
the provisions of Section 6 to cases
involving disqualification based on
ineligibility under Section 78 of BP. Blg.
881. X x x. (Aquino v. COMELEC,
248 SCRA 400, 417-419, Sept. 18,
1995, En Banc [Kapunan, J.])
2. As to the House of Representatives
Electoral
Tribunals
supposed
assumption of jurisdiction over the
issue of petitioners qualifications after
the May 8, 1995 elections, suffice it to
say that HRETs jurisdiction as the sole
judge of all contests relating to the
elections, returns and qualifications of
members of Congress begins only
after a candidate has become a
member
of
the
House
of
Representatives (Art. VI, Sec. 17, 1987
Constitution). Petitioner not being a
member
of
the
House
of
Representatives, it is obvious that the
HRET at this point has no jurisdiction
over the question.
(RomualdezMarcos v. COMELEC, 248 SCRA
300, 340-341, Sept. 18, 1995, En
Banc [Kapunan, J.])
76.
Will the rule be the same if that
candidate wins and was proclaimed
winner and already assumed office as
Congressman?
Held: While the COMELEC is vested
with the power to declare valid or
invalid a certificate of candidacy, its
refusal to exercise
that power
following
the
proclamation
and
assumption of the position by Farinas
is a recognition of the jurisdictional
boundaries separating the COMELEC
and the Electoral Tribunal of the House
of Representatives (HRET).
Under
Article
VI,
Section
17
of
the
Constitution, the HRET has sole and
exclusive jurisdiction over all contests
relative to the election, returns, and
qualifications of members of the
House of Representatives. Thus, once
a winning candidate has been
proclaimed, taken his oath, and
assumed office as a member of the
House of Representatives, COMELECs
jurisdiction over election contests

relating to his election, returns, and


qualifications ends, and the HRETs
own jurisdiction begins.
Thus, the
COMELECs decision to discontinue
exercising jurisdiction over the case is
justifiable, in deference to the HRETs
own jurisdiction and functions.
Xxx
Petitioner further argues that the HRET
assumes jurisdiction only if there is a
valid proclamation of the winning
candidate.
He contends that if a
candidate fails to satisfy the statutory
requirements to qualify him as a
candidate,
his
subsequent
proclamation is void ab initio. Where
the proclamation is null and void,
there is no proclamation at all and the
mere assumption of office by the
proclaimed candidate does not deprive
the COMELEC at all of its power to
declare such nullity, according to
petitioner. But x x x, in an electoral
contest where the validity of the
proclamation of a winning candidate
who has taken his oath of office and
assumed his post as congressman is
raised, that issue is best addressed to
the HRET. The reason for this ruling is
self-evident, for it avoids duplicity of
proceedings and a clash of jurisdiction
between constitutional bodies, with
due regard to the peoples mandate.
(Guerrero v. COMELEC, 336 SCRA
458, July 26, 2000, En Banc
[Quisumbing])

The Court did recognize, of course, its


power of judicial review in exceptional
cases. In Robles v. HRET (181 SCRA
780), the Court has explained that
while the judgments of the Tribunal
are beyond judicial interference, the
Court may do so, however, but only
in the exercise of this Courts socalled extraordinary jurisdiction x x x
upon a determination that the
Tribunals decision or resolution was
rendered without or in excess of its
jurisdiction, or with grave abuse of
discretion or paraphrasing Morrero
(Morrero v. Bocar [66 Phil. 429]), upon
a clear showing of such arbitrary and
improvident use by the Tribunal of its
power as constitutes a denial of due
process
of
law,
or
upon
a
demonstration
of
a
very
clear
unmitigated
error,
manifestly
constituting such grave abuse of
discretion that there has to be a
remedy for such abuse.
The Court does not x x x venture into
the perilous area of correcting
perceived errors of independent
branches of the Government; it comes
in only when it has to vindicate a
denial of due process or correct an
abuse of discretion so grave or glaring
that no less than the Constitution itself
calls for remedial action. (Libanan v.
HRET, 283 SCRA 520, Dec. 22,
1997 [Vitug])

77.
Is there an appeal from a
decision of the Senate or House of
Representatives Electoral Tribunal?
What then is the remedy, if any?

78.
What are the limitations on the
veto power of the President?

Held: The Constitution mandates that


the House of Representatives Electoral
Tribunal and the Senate Electoral
Tribunal shall each, respectively, be
the sole judge of all contests relating
to
the
election,
returns
and
qualifications of their respective
members.

Held: The act of the Executive


in vetoing the particular provisions is
an exercise of a constitutionally vested
power. But even as the Constitution
grants the power, it also provides
limitations to its exercise. The veto
power is not absolute.

The Court has stressed that x x x so


long as the Constitution grants the
HRET the power to be the sole judge
of all contests relating to the election,
returns and qualifications of members
of the House of Representatives, any
final action taken by the HRET on a
matter within its jurisdiction shall, as a
rule, not be reviewed by this Court.
The power granted to the Electoral
Tribunal x x x excludes the exercise of
any authority on the part of this Court
that would in any wise restrict it or
curtail it or even affect the same.

The Executive Department

Xxx
The OSG is correct when it
states that the Executive must veto a
bill in its entirety or not at all. He or
she cannot act like an editor crossing
out specific lines, provisions, or
paragraphs in a bill that he or she
dislikes. In the exercise of the veto
power, it is generally all or nothing.
However,
when
it
comes
to
appropriation, revenue or tariff bills,
the Administration needs the money
to run the machinery of government

and it can not veto the entire bill even


if it may contain objectionable
features. The President is, therefore,
compelled to approve into law the
entire bill, including its undesirable
parts. It is for this reason that the
Constitution has wisely provided the
item
veto
power
to
avoid
inexpedient riders being attached to
an indispensable appropriation or
revenue measure.
The Constitution provides that
only a particular item or items may be
vetoed. The power to disapprove any
item or items in an appropriate bill
does not grant the authority to veto a
part of an item and to approve the
remaining portion of the same item.
(Gonzales v. Macaraig, Jr., 191 SCRA
452, 464 [1990])
(Bengzon v.
Drilon, 208 SCRA 133, 143-145,
April
15,
1992,
En
Banc
[Gutierrez])
79.
Distinguish an item from a
provision in relation to the veto
power of the President.
Held: The terms item and provision in
budgetary legislation and practice are
concededly different. An item in a bill
refers to the particulars, the details,
the distinct and severable parts x x x
of the bill (Bengzon, supra, at 916). It
is an indivisible sum of money
dedicated to a stated purpose
(Commonwealth v. Dodson, 11 S.E., 2d
120, 124, 125, etc., 176 Va. 281). The
United States Supreme Court, in the
case of Bengzon v. Secretary of Justice
(299 U.S. 410, 414, 57 Ct 252, 81 L.
Ed., 312) declared that an item of an
appropriation bill obviously means an
item which in itself is a specific
appropriation of money, not some
general provision of law, which
happens
to
be
put
into
an
appropriation bill.
(Bengzon v.
Drilon, 208 SCRA 133, 143-145,
April
15,
1992,
En
Banc
[Gutierrez])
80.
May the President veto a law?
May she veto a decision of the SC
which has long become final and
executory?
Held:
We need no lengthy
justifications or citations of authorities
to declare that no President may veto
the provisions of a law enacted thirtyfive (35) years before his or her term
of office. Neither may the President
set aside or reverse a final and
executory judgment of this Court
through the exercise of the veto

power.
(Bengzon v. Drilon, 208
SCRA 133, 143-145, April 15,
1992, En Banc [Gutierrez])
81.
Did former President Estrada
resign as President or should be
considered resigned as of January 20,
2001 when President Gloria Macapagal
Arroyo took her oath as the 14th
President of the Republic?
Held: Resignation x x x is a factual
question and its elements are beyond
quibble: there must be an intent to
resign and the intent must be coupled
by acts of relinquishment. The validity
of a resignation is not governed by
any formal requirement as to form. It
can be oral. It can be written. It can
be express. It can be implied. As long
as the resignation is clear, it must be
given legal effect.
In the cases at bar, the facts show that
petitioner did not write any formal
letter of resignation before he
evacuated Malacanang Palace in the
afternoon of January 20, 2001 after
the oath-taking of respondent Arroyo.
Consequently,
whether
or
not
petitioner
resigned
has
to
be
determined
from
his
acts
and
omissions before, during and after
January 20, 2001 or by the totality of
prior, contemporaneous and posterior
facts and circumstantial evidence
bearing a material relevance on the
issue.
Using this totality test, we hold that
petitioner resigned as President.
Xxx
In sum, we hold that the resignation of
the petitioner cannot be doubted. It
was
confirmed
by
his
leaving
Malacanang.
In the press release
containing his final statement, (1) he
acknowledged the oath-taking of the
respondent as President of the
Republic albeit with reservation about
its legality; (2) he emphasized he was
leaving the Palace, the seat of the
presidency, for the sake of peace and
in order to begin the healing process
of our nation. He did not say he was
leaving the Palace due to any kind of
inability and that he was going to reassume the presidency as soon as the
disability disappears; (3) he expressed
his gratitude to the people for the
opportunity to serve them. Without
doubt, he was referring to the past
opportunity given him to serve the
people as President; (4) he assured
that he will not shirk from any future

challenge that may come ahead on


the same service of our country.
Petitioners reference is to a future
challenge after occupying the office of
the president which he has given up;
and (5) he called on his supporters to
join him in the promotion of a
constructive
national
spirit
of
reconciliation and solidarity. Certainly,
the national spirit of reconciliation and
solidarity could not be attained if he
did not give up the presidency. The
press
release
was
petitioners
valedictory, his final act of farewell.
His presidency is now in the past
tense. (Estrada v. Desierto, G.R.
Nos. 146710-15, March 2, 2001,
en Banc [Puno])
82.
Discuss our legal history on
executive immunity.
Held:
The doctrine of executive
immunity in this jurisdiction emerged
as a case law.
In the 1910 case of
Forbes, etc. v. Chuoco Tiaco and
Crossfield (16 Phil. 534 [1910]), the
respondent Tiaco, a Chinese citizen,
sued petitioner W. Cameron Forbes,
Governor-General of the Philippine
Islands,
J.E.
Harding
and
C.R.
Trowbridge, Chief of Police and Chief of
the Secret Service of the City of
Manila, respectively, for damages for
allegedly conspiring to deport him to
China.
In granting a writ of
prohibition, this Court, speaking thru
Mr. Justice Johnson, held:
The principle of nonliability x x x does
not mean that the judiciary has no
authority to touch the acts of the
Governor-General; that he may, under
cover of his office, do what he will,
unimpeded and unrestrained. Such a
construction would mean that tyranny,
under the guise of the execution of the
law, could walk defiantly abroad,
destroying rights of person and of
property, wholly free from interference
of courts or legislatures. This does not
mean, either, that a person injured by
the executive authority by an act
unjustifiable under the law has no
remedy, but must submit in silence.
On the contrary, it means, simply, that
the Governor-General, like the judges
of the courts and the members of the
Legislature, may not be personally
mulcted in civil damages for the
consequences of an act executed in
the performance of his official duties.
The judiciary has full power to, and
will, when the matter is properly
presented to it and the occasion justly
warrants it, declare an act of the
Governor-General illegal and void and

place as nearly as possible in status


quo any person who has been
deprived his liberty or his property by
such act. This remedy is assured to
every person, however humble or of
whatever country, when his personal
or property rights have been invaded,
even by the highest authority of the
state. The thing which the judiciary
can not do is mulct the GovernorGeneral personally in damages which
result from the performance of his
official duty, any more than it can a
member of the Philippine Commission
or the Philippine Assembly.
Public
policy forbids it.
Neither
does
this
principle
of
nonliability mean that the chief
executive may not be personally sued
at all in relation to acts which he
claims to perform as such official. On
the contrary, it clearly appears from
the
discussion
heretofore
had,
particularly that portion which touched
the liability of judges and drew an
analogy between such liability and
that of the Governor-General, that the
latter is liable when he acts in a case
so plainly outside of his power and
authority that he can not be said to
have
exercised
discretion
in
determining whether or not he had the
right to act. What is held here is that
he will be protected from personal
liability for damages not only when he
acts within his authority, but also
when he is without authority, provided
he actually used discretion and
judgment, that is, the judicial faculty,
in determining whether he had
authority to act or not. In other words,
he is entitled to protection in
determining the question of his
authority. If he decide wrongly, he is
still protected provided the question of
his authority was one over which two
men, reasonably qualified for that
position, might honestly differ; but he
is not protected if the lack of authority
to act is so plain that two such men
could not honestly differ over its
determination. In such case, he acts,
not as Governor-General but as a
private individual, and, as such, must
answer for the consequences of his
act.
Mr. Justice Johnson underscored the
consequences if the Chief Executive
was not granted immunity from suit,
viz: x x x. Action upon important
matters of state delayed; the time and
substance of the chief executive spent
in wrangling litigation; disrespect
engendered for the person of one of
the highest officials of the State and

for the office he occupies; a tendency


to unrest and disorder; resulting in a
way, in a distrust as to the integrity of
government itself.
Our 1935 Constitution took effect but
it did not contain any specific
provision on executive immunity.
Then came the tumult of the martial
law years under the late President
Ferdinand E. Marcos and the 1973
Constitution was born. In 1981, it was
amended and one of the amendments
involved executive immunity. Section
17, Article VII stated:
The President shall be immune from
suit during his tenure. Thereafter, no
suit whatsoever shall lie for official
acts done by him or by others
pursuant to his specific orders during
his tenure.
The immunities herein provided shall
apply to the incumbent President
referred to in Article XVII of this
Constitution.
In his second Vicente G. Sinco
Professorial Chair Lecture entitled,
Presidential Immunity And All The
Kings Men: The Law Of Privilege As A
Defense To Actions For Damages, (62
Phil. L.J. 113 [1987]) petitioners
learned counsel, former Dean of the
UP College of Law, Atty. Pacifico
Agabin, brightened the modifications
effected
by
this
constitutional
amendment on the existing law on
executive privilege.
To quote his
disquisition:
In the Philippines though, we
sought to do the American one better
by enlarging and fortifying the
absolute immunity concept. First, we
extended it to shield the President not
only from civil claims but also from
criminal cases and other claims.
Second, we enlarged its scope so that
it would cover even acts of the
President outside the scope of official
duties. And third, we broadened its
coverage so as to include not only the
President but also other persons, be
they government officials or private
individuals, who acted upon orders of
the President. It can be said that at
that point most of us were suffering
from AIDS (or absolute immunity
defense syndrome).
The Opposition in the then Batasang
Pambansa sought the repeal of this
Marcosian
concept
of
executive
immunity in the 1973 Constitution.
The move was led by then Member of

Parliament, now Secretary of Finance,


Alberto Romulo, who argued that the
after incumbency immunity granted to
President Marcos violated the principle
that a public office is a public trust.
He denounced the immunity as a
return to the anachronism the king
can do no wrong. The effort failed.
The 1973 Constitution ceased to exist
when President Marcos was ousted
from office by the People Power
revolution in 1986. When the 1987
Constitution was crafted, its framers
did
not
reenact
the
executive
immunity provision of the 1973
Constitution.
X x x (Estrada v.
Desierto, G.R. Nos. 146710-15,
March 2, 2001, en Banc [Puno])
83.
Can former President Estrada
still
be
prosecuted
criminally
considering that he was not convicted
in the impeachment proceedings
against him?
Held: We reject his argument that he
cannot be prosecuted for the reason
that he must first be convicted in the
impeachment proceedings.
The
impeachment
trial
of
petitioner
Estrada was aborted by the walkout of
the prosecutors and by the events that
led to his loss of the presidency.
Indeed, on February 7, 2001, the
Senate passed Senate Resolution No.
83
Recognizing
that
the
Impeachment
Court
is
Functus
Officio.
Since the Impeachment
Court is now functus officio, it is
untenable for petitioner to demand
that he should first be impeached and
then convicted before he can be
prosecuted.
The plea if granted,
would put a perpetual bar against his
prosecution. Such a submission has
nothing to commend itself for it will
place him in a better situation than a
non-sitting President who has not been
subjected
to
impeachment
proceedings and yet can be the object
of a criminal prosecution. To be sure,
the debates in the Constitutional
Commission make it clear that when
impeachment
proceedings
have
become moot due to the resignation of
the President, the proper criminal and
civil cases may already be filed
against him x x x.
This is in accord with our ruling in In
Re: Saturnino Bermudez (145 SCRA
160
[1986])
that
incumbent
Presidents are immune from suit or
from being brought to court during the
period of their incumbency and
tenure but not beyond. Considering

the peculiar circumstance that the


impeachment process against the
petitioner has been aborted and
thereafter he lost the presidency,
petitioner Estrada cannot demand as a
condition sine qua non to his criminal
prosecution before the Ombudsman
that
he
be
convicted
in
the
impeachment proceedings. (Estrada
v. Desierto, G.R. Nos. 146710-15,
Mar. 2, 2001, en Banc [Puno])
84.
State the reason why not all
appointments made by the President
under the 1987 Constitution will no
longer require confirmation by the
Commission on Appointments.
Held:
The aforecited provision
(Section 16, Article VII) of the
Constitution has been the subject of
several cases on the issue of the
restrictive function of the Commission
on Appointments with respect to the
appointing power of the President.
This Court touched upon the historical
antecedent of the said provision in the
case of Sarmiento III v. Mison (156
SCRA 549) in which it was ratiocinated
upon that Section 16 of Article VII of
the
1987
Constitution
requiring
confirmation by the Commission on
Appointments of certain appointments
issued by the President contemplates
a system of checks and balances
between the executive and legislative
branches of government. Experience
showed
that
when
almost
all
presidential appointments required the
consent of the Commission on
Appointments, as was the case under
the 1935 Constitution, the commission
became a venue of "horse trading"
and similar malpractices. On the other
hand, placing absolute power to make
appointments in the President with
hardly any check by the legislature, as
what happened under the 1973
Constitution, leads to abuse of such
power. Thus was perceived the need
to establish a "middle ground"
between
the
1935
and
1973
Constitutions.
The framers of the
1987
Constitution
deemed
it
imperative to subject certain high
positions in the government to the
power
of
confirmation
of
the
Commission on Appointments and to
allow other positions within the
exclusive appointing power of the
President. (Manalo v. Sistoza, 312
SCRA 239, Aug. 11, 1999, En Banc
[Purisima])
85.
Enumerate
the
groups
of
officers who are to be appointed by
the President under Section 16, Article

VII of the 1987 Constitution, and


identify
those
officers
whose
appointments
shall
require
confirmation by the Commission on
Appointments.
Held: Conformably, as consistently
interpreted and ruled in the leading
case of Sarmiento III v. Mison (Ibid.),
and in the subsequent cases of
Bautista v. Salonga (172 SCRA 160),
Quintos-Deles
v.
Constitutional
Commission (177 SCRA 259), and
Calderon v. Carale (208 SCRA 254),
under Section 16, Article VII, of the
Constitution, there are four groups of
officers of the government to be
appointed by the President:
First, the heads of the executive
departments,
ambassadors,
other
public ministers and consuls, officers
of the armed forces from the rank of
colonel or naval captain, and other
officers whose appointments are
vested in him in this Constitution;
Second, all other officers of the
Government whose appointments are
not otherwise provided for by law;
Third, those whom the President may
be authorized by law to appoint;
Fourth, officers lower in rank whose
appointments the Congress may by
law vest in the President alone.
It is well-settled that only presidential
appointees belonging to the first group
require the confirmation by the
Commission
on
Appointments.
(Manalo v. Sistoza, 312 SCRA 239,
Aug.
11,
1999,
En
Banc
[Purisima])
86.
Under Republic Act 6975 (the
DILG Act of 1990), the Director
General, Deputy Director General, and
other top officials of the Philippine
National
Police
(PNP)
shall
be
appointed by the President and their
appointments
shall
require
confirmation by the Commission on
Appointments.
Respondent Sistoza
was appointed Director General of the
PNP but he refused to submit his
appointment
papers
to
the
Commission on Appointments for
confirmation contending that his
appointment shall no longer require
confirmation despite the express
provision of the law requiring such
confirmation. Should his contention
be upheld?

Held:
It is well-settled that only
presidential appointees belonging to
the first group (enumerated under the
first sentence of Section 16, Article VII
of the 1987 Constitution) require the
confirmation by the Commission on
Appointments. The appointments of
respondent officers who are not within
the first category, need not be
confirmed by the Commission on
Appointments. As held in the case of
Tarrosa v. Singson (232 SCRA 553),
Congress cannot by law expand the
power
of
confirmation
of
the
Commission on Appointments and
require confirmation of appointments
of other government officials not
mentioned in the first sentence of
Section 16 of Article VII of the 1987
Constitution.
Consequently, unconstitutional are
Sections 26 and 31 of Republic Act
6975 which empower the Commission
on Appointments to confirm the
appointments of public officials whose
appointments are not required by the
Constitution to be confirmed. x x x.
(Manalo v. Sistoza, 312 SCRA 239,
Aug.
11,
1999,
En
Banc
[Purisima])
87.
Will it be correct to argue that
since the Philippine National Police is
akin to the Armed Forces of the
Philippines,
therefore,
the
appointments of police officers whose
rank is equal to that of colonel or
naval captain will require confirmation
by the Commission on Appointments?
Held:
This contention is x x x
untenable.
The Philippine National
Police is separate and distinct from the
Armed Forces of the Philippines. The
Constitution, no less, sets forth the
distinction. Under Section 4 of Article
XVI of the 1987 Constitution,
"The Armed Forces of the Philippines
shall be composed of a citizen armed
force which shall undergo military
training and service, as may be
provided by law.
It shall keep a
regular force necessary for the
security of the State."
On the other hand, Section 6 of the
same Article of the Constitution
ordains that:
"The State shall establish and
maintain one police force, which shall
be national in scope and civilian in
character to be administered and
controlled by a national police
commission. The authority of local

executives over the police units in


their jurisdiction shall be provided by
law."
To so distinguish the police force from
the armed forces, Congress enacted
Republic Act 6975 x x x.
Thereunder, the police force is
different from and independent of the
armed forces and the ranks in the
military are not similar to those in the
Philippine National Police.
Thus,
directors and chief superintendents of
the PNP x x x do not fall under the first
category of presidential appointees
requiring
confirmation
by
the
Commission
on
Appointments.
(Manalo v. Sistoza, 312 SCRA 239,
Aug.
11,
1999,
En
Banc
[Purisima])
88.
To what types of appointments
is Section 15, Article VII of the 1987
Constitution (prohibiting the President
from
making
appointments
two
months before the next presidential
elections and up to the end of his
term) directed against?
Held:
Section 15, Article VII is
directed
against
two
types
of
appointments: (1) those made for
buying votes and (2) those made for
partisan considerations.
The first
refers to those appointments made
within two months preceding the
Presidential election and are similar to
those which are declared election
offenses in the Omnibus Election
Code; while the second consists of the
so-called midnight appointments.
The SC in In Re: Hon. Mateo A.
Valenzuela and Hon. Placido B.
Vallarta, (298 SCRA 408, Nov. 9,
1998, En Banc [Narvasa C.J.])
clarified this when it held:
Section 15, Article VII has a broader
scope than the Aytona ruling. It may
not unreasonably be deemed to
contemplate not only midnight
appointments those made obviously
for partisan reasons as shown by their
number and the time of their making
but also appointments presumed
made for the purpose of influencing
the outcome of the Presidential
election.
89.
Discuss the nature of an adinterim appointment. Is it temporary
and, therefore, can be can be
withdrawn or revoked by the President
at her pleasure?

Held:
An
ad
interim
appointment
is
a
permanent
appointment because it takes effect
immediately and can no longer be
withdrawn by the President once the
appointee has qualified into office.
The fact that it is subject to
confirmation by the Commission on
Appointments does not alter its
permanent
character.
The
Constitution itself makes an ad interim
appointment permanent in character
by
making
it
effective
until
disapproved by the Commission on
Appointments or until the next
adjournment of congress. The second
paragraph of Section 16, Article VII of
the Constitution provides as follows:
The President shall have the power to
make appointments during the recess
of the Congress, whether voluntary or
compulsory, but such appointments
shall
be
effective
only
until
disapproval by the Commission on
Appointments or until the next
adjournment of the Congress.
Thus, the ad interim appointment
remains
effective
until
such
disapproval or next adjournment,
signifying that it can no longer be
withdrawn
or
revoked
by
the
President. The fear that the President
can withdraw or revoke at any time
and for any reason an ad interim
appointment is utterly without basis.
More than half a century ago,
this Court had already ruled that an ad
interim appointment is permanent in
character. In Summers v. Ozaeta (81
Phil. 754 [1948]), decided on October
25, 1948, we held that:
x x x an ad interim appointment is
one made in pursuance of paragraph
(4), Section 10, Article VII of the
Constitution, which provides that the
President shall have the power to
make appointments during the recess
of
the
Congress,
but
such
appointments shall be effective only
until disapproval by the Commission
on Appointments or until the next
adjournment of the Congress. It is an

appointment permanent in nature, and


the circumstance that it is subject to
confirmation by the Commission on
Appointments does not alter its
permanent character. An ad interim
appointment is disapproved certainly
for a reason other than that its
provisional period has expired. Said
appointment
is
of
course
distinguishable
from
an
acting
appointment
which
is
merely
temporary,
good
until
another
permanent appointment is issued.
The Constitution imposes no
condition on the effectivity of an ad
interim appointment, and thus an ad
interim appointment takes effect
immediately. The appointee can at
once assume office and exercise, as a
de jure officer, all the powers
pertaining to the office. In Pacete v.
Secretary of the Commission on
Appointments (40 SCRA 58 [1971]),
this Court elaborated on the nature of
an ad interim appointment as follows:
A distinction is thus made between
the exercise of such presidential
prerogative requiring confirmation by
the Commission on Appointments
when Congress is in session and when
it is in recess. In the former, the
President nominates, and only upon
the consent of the Commission on
Appointments may the person thus
named assume office. It is not so with
reference to ad interim appointments.
It takes effect at once. The individual
chosen may thus qualify and perform
his function without loss of time. His
title to such office is complete. In the
language of the Constitution, the
appointment
is
effective
until
disapproval by the Commission on
Appointments or until the next
adjournment of the Congress.
Petitioner cites Blacks Law
Dictionary which defines the term ad
interim to mean in the meantime or
for the time being.
Hence,
petitioner argues that an ad interim
appointment
is
undoubtedly
temporary
in
character.
This
argument is not new and was

answered by this Court in Pamantasan


ng Lungsod ng Maynila v. Intermediate
Appellate Court (140 SCRA 22 [1985]),
where we explained that:
x x x From the arguments, it is easy
to see why the petitioner should
experience difficulty in understanding
the situation. Private respondent had
been extended several ad interim
appointments
which
petitioner
mistakenly
understands
as
appointments temporary in nature.
Perhaps, it is the literal translation of
the word ad interim which creates
such belief. The term is defined by
Black to mean in the meantime or
for the time being. Thus, an officer
ad interim is one appointed to fill a
vacancy, or to discharge the duties of
the office during the absence or
temporary incapacity of its regular
incumbent (Blacks Law Dictionary,
Revised Fourth Edition, 1978). But
such is not the meaning nor the use
intended in the context of Philippine
law.
In referring to Dr. Estebans
appointments, the term is not
descriptive of the nature of the
appointments given to him. Rather, it
is used to denote the manner in which
said appointments were made, that is,
done by the President of the
Pamantasan in the meantime, while
the Board of Regents, which is
originally vested by the University
Charter
with
the
power
of
appointment, is unable to act. X x x.
Thus, the term ad interim
appointment, as used in letters of
appointment signed by the President,
means a permanent appointment
made by the President in the
meantime that Congress is in recess.
It does not mean a temporary
appointment that can be withdrawn or
revoked at any time.
The term,
although not found in the text of the
Constitution, has acquired a definite
legal
meaning
under
Philippine
jurisprudence. The Court had again
occasion to explain the nature of an ad
interim appointment in the more
recent case of Marohombsar v. Court

of Appeals (326 SCRA 62 [2000]),


where the Court stated:
We have already mentioned that an
ad interim appointment is not
descriptive of the nature of the
appointment, that is, it is not
indicative of whether the appointment
is temporary or in an acting capacity,
rather it denotes the manner in which
the appointment was made. In the
instant
case,
the
appointment
extended to private respondent by
then MSU President Alonto, Jr. was
issued without condition nor limitation
as to tenure. The permanent status of
private respondents appointment as
Executive Assistant II was recognized
and attested to by the Civil Service
Commission Regional Office No. 12.
Petitioners submission that private
respondents ad interim appointment
is synonymous with a temporary
appointment which could be validly
terminated at any time is clearly
untenable. Ad interim appointments
are permanent appointment but their
terms are only until the Board
disapproves them.
An ad interim appointee who
has qualified and assumed office
becomes
at
that
moment
a
government employee and therefore
part of the civil service. He enjoys the
constitutional protection that [n]o
officer or employee in the civil service
shall be removed or suspended except
for cause provided by law. (Section
2[3], Article IX-B of the Constitution)
Thus, an ad interim appointment
becomes complete and irrevocable
once the appointee has qualified into
office. The withdrawal or revocation of
an ad interim appointment is possible
only if it is communicated to the
appointee before the moment he
qualifies, and any withdrawal or
revocation thereafter is tantamount to
removal from office (See concurring
opinion of Justice Cesar Bengzon in
Erana v. Vergel de Dios, 85 Phil. 17
[1949]).
Once an appointee has
qualified, he acquires a legal right to
the office which is protected not only
by
statute
but
also
by
the

Constitution. He can only be removed


for cause, after notice and hearing,
consistent with the requirements of
due process.
An ad interim appointment can be
terminated for two causes specified in
the Constitution. The first cause is the
disapproval
of
his
ad
interim
appointment by the Commission on
Appointments. The second cause is
the adjournment of Congress without
the Commission on Appointments
acting on his appointment. These two
causes are resolutory conditions
expressly imposed by the Constitution
on all ad interim appointments. These
resolutory conditions constitute, in
effect, a Sword of Damocles over the
heads of ad interim appointees. No
one, however, can complain because it
is the Constitution itself that places
the Sword of Damocles over the heads
of the ad interim appointees.
While an ad interim appointment is
permanent and irrevocable except as
provided by law, an appointment or
designation in a temporary or acting
capacity can be withdrawn or revoked
at the pleasure of the appointing
power (Binamira v. Garrucho, 188
SCRA
154
[1990];
Santiago
v.
Commission on Audit, 199 SCRA 125
[1991]; Sevilla v. Court of Appeals, 209
SCRA 637 [1992]). A temporary or
acting appointee does not enjoy any
security of tenure, no matter how
briefly. This is the kind of appointment
that the Constitution prohibits the
President from making to the three
independent
constitutional
commissions, including the COMELEC.
Thus, in Brillantes v. Yorac (192 SCRA
358 [1990]), this Court struck down as
unconstitutional the designation by
then President Corazon Aquino of
Associate Commissioner Haydee Yorac
as
Acting
Chairperson
of
the
COMELEC. This Court ruled that:
A designation as Acting Chairman is
by
its
very
terms
essentially
temporary and therefore revocable at
will. No cause need be established to
justify its revocation. Assuming its

validity, the designation of the


respondent as Acting Chairman of the
Commission on Elections may be
withdrawn by the President of the
Philippines at any time and for
whatever reason she sees fit. It is
doubtful if the respondent, having
accepted such designation, will not be
estopped
from
challenging
its
withdrawal.
The Constitution provides for many
safeguards to the independence of the
Commission on Elections, foremost
among which is the security of tenure
of its members. That guarantee is not
available to the respondent as Acting
Chairman of the Commission on
Elections by designation of the
President of the Philippines.
Earlier, in Nacionalista Party v.
Bautista (85 Phil. 101 [1949]), a case
decided under the 1935 Constitution,
which did not have a provision
prohibiting
temporary
or
acting
appointments to the COMELEC, this
Court
nevertheless
declared
unconstitutional the designation of the
Solicitor General as acting member of
the COMELEC. This Court ruled that
the
designation
of
an
acting
Commissioner would undermine the
independence of the COMELEC and
hence violate the Constitution. We
declared then: It would be more in
keeping with the intent, purpose and
aim of the framers of the Constitution
to appoint a permanent Commissioner
than to designate one to act
temporarily.
In
the
instant
case,
the
President
did
in
fact
appoint
permanent Commissioners to fill the
vacancies in the COMELEC, subject
only
to
confirmation
by
the
Commission
on
Appointments.
Benipayo, Borra and Tuason were
extended permanent appointments
during the recess of Congress. They
were not appointed or designated in a
temporary or acting capacity, unlike
Commissioner
Haydee
Yorac
in
Brillantes v. Yorac and Solicitor
General Felix Bautista in Nacionalista

Party v. Bautista.
The ad interim
appointments of Benipayo, Borra and
Tuason are expressly allowed by the
Constitution which authorizes the
President, during the recess of
Congress, to make appointments that
take effect immediately.

three constitutional commissions. In


his concurring opinion in Guevarra v.
Inocentes (16 SCRA 379 [1966]),
decided under the 1935 Constitution,
Justice
Roberto
Concepcion,
Jr.
explained the rationale behind ad
interim appointments in this manner:

While the Constitution mandates that


the COMELEC shall be independent,
this provision should be harmonized
with the Presidents power to extend
ad interim appointments. To hold that
the independence of the COMELEC
requires
the
Commission
on
Appointments to first confirm ad
interim
appointees
before
the
appointees can assume office will
negate the Presidents power to make
ad interim appointments.
This is
contrary to the rule on statutory
construction to give meaning and
effect to every provision of the law. It
will also run counter to the clear intent
of the framers of the Constitution.

Now, why is the lifetime of ad interim


appointments so limited? Because, if
they expired before the session of
Congress, the evil sought to be
avoided interruption in the discharge
of essential functions may take
place. Because the same evil would
result if the appointments ceased to
be effective during the session of
Congress and before its adjournment.
Upon the other hand, once Congress
has
adjourned,
the
evil
aforementioned
may
easily
be
conjured by the issuance of other ad
interim
appointments
or
reappointments.

The original draft of Section 16, Article


VII of the Constitution on the
nomination of officers subject to
confirmation by the Commission on
Appointments did not provide for ad
interim appointments.
The original
intention of the framers of the
Constitution was to do away with ad
interim appointments because the
plan was for Congress to remain in
session throughout the year except for
a brief 30-day compulsory recess.
However, because of the need to
avoid
disruptions
in
essential
government services, the framers of
the Constitution thought it wise to
reinstate the provisions of the 1935
Constitution
on
ad
interim
appointments. X x x.

Indeed, the timely application of the


last sentence of Section 16, Article VII
of the Constitution barely avoided the
interruption of essential government
services in the May 2001 national
elections. Following the decision of
this Court in Gaminde v. Commission
on Appointments (347 SCRA 655
[2000]), promulgated on December
13, 2000, the terms of office of
constitutional officers first appointed
under the Constitution would have to
be counted starting February 2, 1987,
the date of ratification of the
Constitution, regardless of the date of
their actual appointment.
By this
reckoning, the terms of office of three
Commissioners of the COMELEC,
including the Chairman, would end on
February 2, 2001 (See Section 1[2],
Article IX-C of the Constitution).

Xxx
Xxx
Clearly, the reinstatement in the
present Constitution of the ad interim
appointing power of the President was
for
the
purpose
of
avoiding
interruptions in vital government
services that otherwise would result
from
prolonged
vacancies
in
government offices, including the

During an election year, Congress


normally goes on voluntary recess
between
February
and
June
considering that many of the members
of the House of Representatives and
the Senate run for re-election.
In
2001,
the
Eleventh
Congress

adjourned from January 9, 2001 to


June 3, 2001. Concededly, there was
no more time for Benipayo, Borra and
Tuason, who were originally extended
ad interim appointments only on
March 22, 2001, to be confirmed by
the Commission on Appointments
before the May 14, 2001 elections.
If Benipayo, Borra and Tuason were
not extended ad interim appointments
to fill up the three vacancies in the
COMELEC, there would only have been
one division functioning in the
COMELEC instead of two during the
May 2001 elections. Considering that
the Constitution requires that all x x x
election cases shall be heard and
decided in division, the remaining
one
division
would
have
been
swamped
with
election
cases.
Moreover, since under the Constitution
motions for reconsideration shall be
decided by the Commission en banc,
the mere absence of one of the four
remaining members would have
prevented a quorum, a less than ideal
situation
considering
that
the
Commissioners are expected to travel
around the country before, during and
after the elections. There was a great
probability that disruptions in the
conduct of the May 2001 elections
could occur because of the three
vacancies in the COMELEC.
The
successful conduct of the May 2001
national elections, right after the
tumultuous EDSA II and EDSA III
events, was certainly essential in
safeguarding and strengthening our
democracy.
Evidently,
the
exercise
by
the
President in the instant case of her
constitutional power to make ad
interim appointments prevented the
occurrence of the very evil sought to
be avoided by the second paragraph
of Section 16, Article VII of the
Constitution. This power to make ad
interim appointments is lodged in the
President to be exercised by her in her
sound judgment. Under the second
paragraph of Section 16, Article VII of
the Constitution, the President can
choose either of two modes in

appointing officials who are subject to


confirmation by the Commission on
Appointments. First, while Congress is
in
session,
the
President
may
nominate the prospective appointee,
and
pending
consent
of
the
Commission on Appointments, the
nominee cannot qualify and assume
office. Second, during the recess of
Congress, the President may extend
an ad interim appointment which
allows the appointee to immediately
qualify and assume office.
Whether the President chooses to
nominate the prospective appointee or
extend an ad interim appointment is a
matter within the prerogative of the
President because the Constitution
grants her that power.
This Court
cannot inquire into the propriety of the
choice made by the President in the
exercise of her constitutional power,
absent grave abuse of discretion
amounting to lack or excess of
jurisdiction on her part, which has not
been shown in the instant case.
The issuance by Presidents of ad
interim appointments to the COMELEC
is a long-standing practice. X x x
The Presidents power to extend ad
interim appointments may indeed
briefly put the appointee at the mercy
of both the appointing and confirming
powers. This situation, however, in
only for a short period from the time
of issuance of the ad interim
appointment until the Commission on
Appointments gives or withholds its
consent.
The Constitution itself
sanctions this situation, as a trade-off
against the evil of disruptions in vital
government services. This is also part
of the check-and-balance under the
separation of powers, as a trade-off
against the evil of granting the
President absolute and sole power to
appoint. The Constitution has wisely
subjected the Presidents appointing
power to the checking power of the
legislature.
This situation, however, does not
compromise the independence of the

COMELEC as a constitutional body.


The vacancies in the COMELEC are
precisely staggered to insure that the
majority
of
its
members
hold
confirmed appointments, and no one
President will appoint all the COMELEC
members.
X x x.
The special
constitutional safeguards that insure
the independence of the COMELEC
remain in place (See Sections, 3, 4, 5
and 6, Article IX-A of the Constitution).

interim appointment can no longer be


renewed because this will violate
Section 1 (2), Article IX-C of the
Constitution
which
prohibits
reappointments.
Petitioner asserts
that this is particularly true to
permanent appointees who have
assumed office, which is the situation
of Benipayo, Borra and Tuason if their
ad interim appointments are deemed
permanent in character.

In fine, we rule that the ad interim


appointments
extended
by
the
President to Benipayo, Borra and
Tuason, as COMELEC Chairman and
Commissioners, respectively, do not
constitute
temporary
or
acting
appointments prohibited by Section 1
(2), Article IX-C of the Constitution.
(Matibag v. Benipayo, 380 SCRA
49, April 2, 2002, En Banc
[Carpio])

There is no dispute that an ad


interim appointee disapproved by the
Commission on Appointments can no
longer
be
extended
a
new
appointment.
The disapproval is a
final decision of the Commission on
Appointments in the exercise of its
checking power on the appointing
authority of the President.
The
disapproval is a decision on the
merits, being a refusal by the
Commission on Appointments to give
its consent after deliberating on the
qualifications of the appointee. Since
the Constitution does not provide for
any appeal from such decision, the
disapproval is final and binding on the
appointee as well as on the appointing
power. In this instance, the President
can no longer renew the appointment
not because of the constitutional
prohibition
on
appointment,
but
because of a final decision by the
Commission on Appointments to
withhold
its
consent
to
the
appointment.

90.
Does the renewal of ad interim
appointments violate the prohibition
on reappointment under Section 1(2),
Article IX-C of the 1987 Constitution?
Held:
Petitioner also argues
that assuming the first ad interim
appointment and the first assumption
of office of Benipayo, Borra and Tuason
are constitutional, the renewal of their
ad interim appointments and their
subsequent assumption of office to the
same positions violate the prohibition
on reappointment under Section 1 (2),
Article IX-C of the Constitution, which
provides as follows:
The
Chairman
and
the
Commissioners shall be appointed by
the President with the consent of the
Commission on Appointments for a
term
of
seven
years
without
reappointment.
Of
those
first
appointed, three Members shall hold
office for seven years, two Members
for five years, and the last Members
for
three
years,
without
reappointment.
Petitioner theorizes that once an ad
interim appointee is by-passed by the
Commission on Appointments, his ad

An ad interim appointment that


is by-passed because of lack of time or
failure
of
the
Commission
on
Appointments to organize is another
matter. A by-passed appointment is
one that has not been finally acted
upon on the merits by the Commission
on Appointments at the close of the
session of Congress. There is no final
decision by the Commission on
Appointments to give or withhold its
consent to the appointment as
required by the Constitution. Absent
such decision, the President is free to
renew the ad interim appointment of a
by-passed
appointee.
This
is
recognized in Section 17 of the Rules

of the Commission on Appointments x


x x. Hence, under the Rules of the
Commission on Appointments, a bypassed
appointment
can
be
considered again if the President
renew the appointment.
It
is
well-settled
in
this
jurisdiction that the President can
renew the ad interim appointments of
by-passed appointees. Justice Roberto
Concepcion, Jr. lucidly explained in his
concurring opinion in Guevarra v.
Inocentes (Supra, note 34) why bypassed ad interim appointees could be
extended new appointments, thus:
In short, an ad interim appointment
ceases
to
be
effective
upon
disapproval
by
the
Commission,
because the incumbent can not
continue holding office over the
positive objection of the Commission.
It ceases, also, upon the next
adjournment of the Congress, simply
because the President may then issue
new appointments not because of
implied disapproval of the Commission
deduced from its intention during the
session of Congress, for, under the
Constitution, the Commission may
affect
adversely
the
interim
appointments only by action, never by
omission.
If the adjournment of
Congress were an implied disapproval
of ad interim appointments made prior
thereto, then the President could no
longer appoint those so by-passed by
the Commission. But, the fact is that
the President may reappoint them,
thus clearly indicating that the reason
for said termination of the ad interim
appointments is not the disapproval
thereof allegedly inferred from said
omission of the Commission, but the
circumstance
that
upon
said
adjournment of the Congress, the
President is free to make ad interim
appointments or reappointments.
Guevarra was decided under the 1935
Constitution from where the second
paragraph of Section 16, Article VII of
the present Constitution on ad interim
appointments was lifted verbatim.
The jurisprudence under the 1935

Constitution governing ad interim


appointments by the President is
doubtless applicable to the present
Constitution. The established practice
under the present Constitution is that
the
President
can
renew
the
appointments of by-passed ad interim
appointees. This is a continuation of
the well-recognized practice under the
1935 Constitution, interrupted only by
the 1973 Constitution which did not
provide
for
a
Commission
on
Appointments
but
vested
sole
appointing power in the President.
The
prohibition
on
reappointment in Section 1 (2), Article
IX-C of the Constitution applies neither
to disapproval nor by-passed ad
interim appointments. A disapproved
ad interim appointment cannot be
revived by another
ad interim
appointment because the disapproval
is final under Section 16, Article VII of
the Constitution, and not because a
reappointment is prohibited under
Section 1 (2), Article IX-C of the
Constitution. A by-passed ad interim
appointment cannot be revived by a
new ad interim appointment because
there is no final disapproval under
Section 16, Article VII of the
Constitution,
and
such
new
appointment will not result in the
appointee serving beyond the fixed
term of seven years.
Section 1 (2), Article IX-C of the
Constitution provides that [t]he
Chairman and the Commissioners shall
be appointed x x x for a term of seven
years without reappointment. There
are four situations where this provision
will apply. The first situation is where
an ad interim appointee to the
COMELEC, after confirmation by the
Commission on Appointments, serves
his full seven-year term. Such person
cannot
be
reappointed
to
the
COMELEC, whether as a member or as
a chairman, because he will then be
actually serving more than seven
years. The second situation is where
the appointee, after confirmation,
serves a part of his term and then
resigns before his seven-year term of

office ends. Such person cannot be


reappointed, whether as a member or
as a chair, to a vacancy arising from
retirement because a reappointment
will result in the appointee also
serving more than seven years. The
third situation is where the appointee
is confirmed to serve the unexpired
term of someone who died or
resigned,
and
the
appointee
completes the unexpired term. Such
person
cannot
be
reappointed,
whether as a member or chair, to a
vacancy arising
from
retirement
because a reappointment will result in
the appointee also serving more than
seven years.
The fourth situation is where the
appointee has previously served a
term of less than seven years, and a
vacancy
arises
from
death
or
resignation. Even if it will not result in
his serving more than seven years, a
reappointment of such person to serve
an unexpired term is also prohibited
because his situation will be similar to
those appointed under the second
sentence of Section 1 (2), Article IX-C
of the Constitution.
This provision
refers to the first appointees under the
Constitution whose terms of office are
less than seven years, but are barred
from ever being reappointed under
any situation. Not one of these four
situations applies to the case of
Benipayo, Borra and Tuason.
The framers of the Constitution
made it quite clear that any person
who has served any term of office as
COMELEC member whether for a full
term of seven years, a truncated term
of five or three years, or even an
unexpired term for any length of time
can no longer be reappointed to the
COMELEC. X x x
Xxx
In Visarra v. Miraflor (8 SCRA 1
[1963]), Justice Angelo Bautista, in his
concurring
opinion,
quoted
Nacionalista v. De Vera (85 Phil. 126
[1949]) that a [r]eappointment is not
prohibited when a Commissioner has

held, office only for, say, three or six


years, provided his term will not
exceed nine years in all. This was the
interpretation despite the express
provision in the 1935 Constitution that
a COMELEC member shall hold office
for a term of nine years and may not
be reappointed.
To foreclose this interpretation,
the phrase without reappointment
appears twice in Section 1 (2), Article
IX-C of the present Constitution. The
first phrase prohibits reappointment of
any person previously appointed for a
term of seven years.
The second
phrase prohibits reappointment of any
person previously appointed for a term
of five or three years pursuant to the
first set of appointees under the
Constitution. In either case, it does
not matter if the person previously
appointed completes his term of office
for the intention is to prohibit any
reappointment of any kind.
However,
an
ad
interim
appointment that has lapsed by
inaction of the Commission on
Appointments does not constitute a
term of office. The period from the
time the ad interim appointment is
made to the time it lapses is neither a
fixed term nor an unexpired term. To
hold otherwise would mean that the
President by his unilateral action could
start and complete the running of a
term of office in the COMELEC without
the consent of the Commission on
Appointments.
This interpretation
renders inutile the confirming power of
the Commission on Appointments.
The
phrase
without
reappointment applies only to one
who has been appointed by the
President and confirmed by the
Commission
on
Appointments,
whether or not such person completes
his term of office. There must be a
confirmation by the Commission on
Appointments
of
the
previous
appointment before the prohibition on
reappointment can apply.
To hold
otherwise will lead to absurdities and

negate the Presidents power to make


ad interim appointments.
In the great majority of cases,
the Commission on Appointments
usually fails to act, for lack of time, on
the ad interim appointments first
issued to appointees.
If such ad
interim appointments can no longer be
renewed, the President will certainly
hesitate
to
make
ad
interim
appointments because most of her
appointees
will
effectively
be
disapproved by mere inaction of the
Commission on Appointments. This
will nullify the constitutional power of
the President to make ad interim
appointments, a power intended to
avoid disruptions in vital government
services. This Court cannot subscribe
to a proposition that will wreak havoc
on vital government services.
The
prohibition
on
reappointment is common to the three
constitutional commissions.
The
framers of the present Constitution
prohibited reappointments for two
reasons.
The first is to prevent a
second appointment for those who
have been previously appointed and
confirmed even if they served for less
than seven years. The second is to
insure that the members of the three
constitutional commissions do not
serve beyond the fixed term of seven
years. X x x.
Xxx
Plainly,
the
prohibition
on
reappointment is intended to insure
that there will be no reappointment of
any kind. On the other hand, the
prohibition on temporary or acting
appointments is intended to prevent
any circumvention of the prohibition
on reappointment that may result in
an appointees total term of office
exceeding seven years.
The evils
sought to be avoided by the twin
prohibitions are very specific
reappointment of any kind and
exceeding ones term in office beyond
the maximum period of seven years.

Not contented with these ironclad twin


prohibitions, the framers of the
Constitution tightened even further
the screws on those who might wish to
extend their terms of office. Thus, the
word designated was inserted to
plug any loophole that might be
exploited
by
violators
of
the
Constitution x x x.
The ad interim appointments
and
subsequent
renewals
of
appointments of Benipayo, Borra and
Tuason do not violate the prohibition
on reappointments because there
were no previous appointments that
were confirmed by the Commission on
Appointments.
A reappointment
presupposes a previous confirmed
appointment. The same ad interim
appointments
and
renewal
of
appointments will also not breach the
seven-year term limit because all the
appointments
and
renewals
of
appointments of Benipayo, Borra and
Tuason are for a fixed term expiring on
February 2, 2008. Any delay in their
confirmation will not extend the expiry
date of their terms of office.
Consequently, there is no danger
whatsoever that the renewal of the ad
interim appointments of these three
respondents will result in any of the
evils intended to be exorcised by the
twin prohibitions in the Constitution.
The continuing renewal of the ad
interim appointment of these three
respondents, for so long as their terms
of office expire on February 2, 2008,
does not violate the prohibition on
reappointments in Section 1 (2),
Article IX-C of the Constitution.
(Matibag v. Benipayo, 380 SCRA
49, April 2, 2002, En Banc
[Carpio])

91.
Ma. Evelyn S. Abeja was a
municipal mayor.
She ran for
reelection but lost.
Before she
vacated her office, though, she
extended permanent appointments to
fourteen new employees of the
municipal government. The incoming
mayor, upon assuming office, recalled
said appointments contending that
these were midnight appointments
and, therefore, prohibited under Sec.

15, Art. VII of the 1987 Constitution.


Should the act of the new mayor of
recalling said appointments on the
aforestated ground be sustained?
Held: The records reveal that when
the petitioner brought the matter of
recalling the appointments of the
fourteen (14) private respondents
before the CSC, the only reason he
cited to justify his action was that
these were midnight appointments
that are forbidden under Article VII,
Section 15 of the Constitution.
However, the CSC ruled, and correctly
so, that the said prohibition applies
only to presidential appointments. In
truth and in fact, there is no law that
prohibits local elective officials from
making appointments during the last
days of his or her tenure. (De Rama
v. Court of Appeals (353 SCRA 94,
Feb. 28, 2001, En Banc [YnaresSantiago])

92.
Distinguish
the
Presidents
power to call out the armed forces as
their Commander-in-Chief in order to
prevent or suppress lawless violence,
invasion or rebellion, from his power to
proclaim martial and suspend the
privilege of the writ of habeas corpus.
Explain why the former is not subject
to judicial review while the latter two
are.
Held:
There is a clear textual
commitment under the Constitution to
bestow
on
the
President
full
discretionary power to call out the
armed forces and to determine the
necessity for the exercise of such
power. Section 18, Article VII of the
Constitution, which embodies the
powers
of
the
President
as
Commander-in-Chief, provides in part:
The
President
shall
be
the
Commander-in-Chief of all armed
forces of the Philippines and whenever
it becomes necessary, he may call out
such armed forces to prevent or
suppress lawless violence, invasion or
rebellion.
In case of invasion or
rebellion, when the public safety
requires it, he may, for a period not
exceeding sixty days, suspend the
privilege of the writ of habeas corpus,
or place the Philippines or any part
thereof under martial law.
The full discretionary power of the
President to determine the factual
basis for the
exercise of the calling
out power is also implied and further

reinforced in the rest of Section 18,


Article VII x x x.
Under
the
foregoing
provisions,
Congress
may
revoke
such
proclamations (of martial law) or
suspension (of the privilege of the writ
of habeas corpus) and the Court may
review the sufficiency of the factual
basis thereof. However, there is no
such equivalent provision dealing with
the revocation or review of the
President's action to call out the
armed forces. The distinction places
the calling out power in a different
category from the power to declare
martial law and the power to suspend
the privilege of the writ of habeas
corpus, otherwise, the framers of the
Constitution
would
have
simply
lumped together the three powers and
provided for their revocation and
review without any qualification.
Expressio unios est exclusio alterius.
X x x.
That the intent of the
Constitution is exactly what its letter
says, i.e., that the power to call is fully
discretionary to the President, is
extant in the deliberation of the
Constitutional Commission x x x.
The reason for the difference in the
treatment of the aforementioned
powers highlights the intent to grant
the President the widest leeway and
broadest discretion in using the power
to call out because it is considered as
the lesser and more benign power
compared to the power to suspend the
privilege of the writ of habeas corpus
and the power to impose martial law,
both of which involve the curtailment
and suppression of certain basic civil
rights and individual freedoms, and
thus necessitating safeguards by
Congress and review by this Court.
Moreover, under Section 18, Article VII
of the Constitution, in the exercise of
the power to suspend the privilege of
the writ of habeas corpus or to impose
martial law, two conditions must
concur: (1) there must be an actual
invasion or rebellion and, (2) public
safety must require it.
These
conditions are not required in the case
of the power to call out the armed
forces.
The only criterion is that
"whenever it becomes necessary," the
President may call the armed forces
"to prevent or suppress lawless
violence, invasion or rebellion." The
implication is that the President is
given full discretion and wide latitude
in the exercise of the power to call as
compared to the two other powers.

If the petitioner fails, by way of proof,


to support the assertion that the
President acted without factual basis,
then this Court cannot undertake an
independent investigation beyond the
pleadings. The factual necessity of
calling out the armed forces is not
easily quantifiable and cannot be
objectively established since matters
considered for satisfying the same is a
combination of several factors which
are not always accessible to the
courts. Besides the absence of textual
standards that the court may use to
judge necessity, information necessary
to arrive at such judgment might also
prove unmanageable for the courts.
Certain pertinent information might be
difficult to verify, or wholly unavailable
to the courts. In many instances, the
evidence upon which the President
might decide that there is a need to
call out the armed forces may be of a
nature not constituting technical proof.
On the other hand, the President as
Commander-in-Chief
has
a
vast
intelligence
network
to
gather
information, some of which may be
classified as highly confidential or
affecting the security of the state. In
the exercise of the power to call, onthe-spot
decisions
may
be
imperatively necessary in emergency
situations to avert great loss of human
lives and mass destruction of property.
Indeed, the decision to call out the
military to prevent or suppress lawless
violence must be done swiftly and
decisively if it were to have any effect
at all.
Such a scenario is not
farfetched when we consider the
present situation in Mindanao, where
the insurgency problem could spill
over the other parts of the country.
The determination of the necessity for
the calling out power if subjected to
unfettered judicial scrutiny could be a
veritable prescription for disaster, as
such
power
may
be
unduly
straitjacketed by an injunction or a
temporary restraining order every
time it is exercised.
Thus, it is the unclouded intent of the
Constitution
to
vest
upon
the
President, as Commander-in-Chief of
the Armed Forces, full discretion to call
forth the military when in his judgment
it is necessary to do so in order to
prevent or suppress lawless violence,
invasion or rebellion.
Unless the
petitioner can show that the exercise
of such discretion was gravely abused,
the President's exercise of judgment
deserves to be accorded respect from
this Court. (Integrated Bar of the

Philippines v. Hon. Ronaldo B.


Zamora, G.R. No. 141284, Aug. 15,
2000, En Banc [Kapunan])
93.
By issuing a TRO on the date
convicted rapist Leo Echegaray is to
be executed by lethal injection, the
Supreme Court was criticized on the
ground,
among
others,
that
it
encroached on the power of the
President to grant reprieve under
Section
19,
Article
VII,
1987
Constitution. Justify the SC's act.
Held: Section 19, Article VII of the
1987 Constitution is simply the source
of power of the President to grant
reprieves, commutations, and pardons
and remit fines and forfeitures after
conviction by final judgment.
This
provision,
however,
cannot
be
interpreted as denying the power of
courts to control the enforcement of
their decisions after the finality. In
truth, an accused who has been
convicted by final judgment still
possesses collateral rights and these
rights can be claimed in the
appropriate courts. For instance, a
death convict who becomes insane
after his final conviction cannot be
executed while in a state of insanity
(See Article 79 of the Revised Penal
Code).
The suspension of such a
death sentence is undisputably an
exercise of judicial power. It is not
usurpation of the presidential power of
reprieve though its effect is the same
the temporary suspension of the
execution of the death convict. In the
same vein, it cannot be denied that
Congress can at any time amend R.A.
No. 7659 by reducing the penalty of
death to life imprisonment. The effect
of such an amendment is like that of
commutation of sentence. But by no
stretch of the imagination can the
exercise by Congress of its plenary
power to amend laws be considered as
a violation of the Presidents power to
commute
final
sentences
of
conviction.
The powers of the
Executive, the Legislative and the
Judiciary to save the life of a death
convict do not exclude each other for
the simple reason that there is no
higher right than the right to life.
(Echegaray v. Secretary of Justice,
301 SCRA 96, Jan. 19, 1999, En
Banc [Puno])
94.
Discuss
the
nature
of
a
conditional pardon.
Is its grant or
revocation by the President subject to
judicial review?

Held: A conditional pardon is in the


nature of a contract between the
sovereign
power
or
the
Chief
Executive and the convicted criminal
to the effect that the former will
release the latter subject to the
condition that if he does not comply
with the terms of the pardon, he will
be recommitted to prison to serve the
unexpired portion of the sentence or
an additional one (Alvarez v. Director
of Prisons, 80 Phil. 50).
By the
pardonees consent to the terms
stipulated in this contract, the
pardonee has thereby placed himself
under the supervision of the Chief
Executive or his delegate who is dutybound to see to it that the pardonee
complies
with
the
terms
and
conditions of the pardon.
Under
Section
64(i)
of
the
Revised
Administrative
Code,
the
Chief
Executive is authorized to order the
arrest and re-incarceration of any such
person who, in his judgment, shall fail
to comply with the condition, or
conditions of his pardon, parole, or
suspension of sentence. It is now a
well-entrenched rule in this jurisdiction
that this exercise of presidential
judgment is beyond judicial scrutiny.
The determination of the violation of
the
conditional
pardon
rests
exclusively in the sound judgment of
the Chief Executive, and the pardonee,
having consented to place his liberty
on conditional pardon upon the
judgment of the power that has
granted it, cannot invoke the aid of the
courts,
however
erroneous
the
findings may be upon which his
recommitment was ordered.
It matters not that the pardonee has
allegedly been acquitted in two of the
three criminal cases filed against him
subsequent to his conditional pardon,
and that the third remains pending for
thirteen (13) years in apparent
violation of his right to a speedy trial.
Ultimately, solely vested in the Chief
Executive, who in the first place was
the exclusive author of the conditional
pardon and of its revocation, is the
corollary prerogative to reinstate the
pardon if in his own judgment, the
acquittal of the pardonee from the
subsequent charges filed against him,
warrants the same. Courts have no
authority to interfere with the grant by
the President of a pardon to a
convicted criminal. It has been our
fortified ruling that a final judicial
pronouncement as to the guilt of a
pardonee is not a requirement for the
President to determine whether or not

there has been a breach of the terms


of a conditional pardon.
There is
likewise nil a basis for the courts to
effectuate the reinstatement of a
conditional pardon revoked by the
President in the exercise of powers
undisputably solely and absolutely in
his office.
(In Re: Wilfredo
Sumulong Torres, 251 SCRA 709,
Dec. 29, 1995 [Hermosisima])
95.
Who has the power to ratify a
treaty?
Held: In our jurisdiction, the power to
ratify is vested in the President and
not, as commonly believed, in the
legislature. The role of the Senate is
limited only to giving or withholding its
consent, or concurrence, to the
ratification.
(BAYAN
[Bagong
Alyansang
Makabayan]
v.
Executive
Secretary
Ronaldo
Zamora, G.R. No. 138570, Oct. 10,
2000, En Banc [Buena])
96.
What
is
the
power
of
impoundment of the President? What
are its principal sources?
Held:
Impoundment refers to the
refusal of the President, for whatever
reason, to spend funds made available
by Congress. It is the failure to spend
or obligate budget authority of any
type.
Proponents of impoundment have
invoked at least three principal
sources of the authority of the
President. Foremost is the authority to
impound given to him either expressly
or impliedly by Congress. Second is
the executive power drawn from the
Presidents role as Commander-inChief. Third is the Faithful Execution
Clause.
The proponents insist that a faithful
execution of the laws requires that the
President desist from implementing
the law if doing so would prejudice
public interest. An example given is
when through efficient and prudent
management of a project, substantial
savings are made. In such a case, it is
sheer folly to expect the President to
spend the entire amount budgeted in
the law. (PHILCONSA v. Enriquez,
235 SCRA 506, Aug. 9, 1994
[Quiason])
97.
Distinguish
the
Presidents
power of general supervision over
local governments from his control
power.

Held: On many occasions in


the past, this Court has had the
opportunity to distinguish the power of
supervision from the power of control.
In Taule v. Santos (200 SCRA 512
[1991]), we held that the Chief
Executive wielded no more authority
than that of checking whether a local
government or the officers thereof
perform their duties as provided by
statutory enactments.
He cannot
interfere with local governments
provided that the same or its officers
act within the scope of their authority.
Supervisory power, when contrasted
with control, is the power of mere
oversight over an inferior body; it does
not include any restraining authority
over such body (Ibid.).
Officers in
control lay down the rules in the doing
of an act. If they are not followed, it is
discretionary on his part to order the
act undone or redone by his
subordinate or he may even decide to
do it himself. Supervision does not
cover such authority.
Supervising
officers merely see to it that the rules
are followed, but he himself does not
lay down such rules, nor does he have
the discretion to modify or replace
them. If the rules are not observed,
he may order the work done or redone to conform to the prescribed
rules. He cannot prescribe his own
manner for the doing of the act (Drilon
v. Lim, supra, 142). (Bito-Onon v.
Fernandez, 350 SCRA 732, Jan. 31,
2001, 3rd Div. [Gonzaga-Reyes])
98.
Is
the
absence
of
a
recommendation of the Secretary of
Justice to the President fatal to the
appointment
of
respondent
as
prosecutor?
Held: This question would x x x pivot
on the proper understanding of the
provision of the Revised Administrative
Code of 1987 (Book IV, Title III,
Chapter II, Section 9) to the effect that

All provincial and city prosecutors and


their assistants shall be appointed by
the
President
upon
the
recommendation of the Secretary.

Petitioners
contend
that
an
appointment of a provincial prosecutor
mandatorily
requires
a
prior
recommendation of the Secretary of
Justice
endorsing
the
intended
appointment citing, by analogy, the
case of San Juan v. CSC (196 SCRA 69)
x x x.
When the Constitution or the
law clothes the President with the
power to appoint a subordinate officer,
such conferment must be understood
as necessarily carrying with it an
ample discretion of whom to appoint.
It should be here pertinent to state
that the President is the head of
government whose authority includes
the power of control over all
executive departments, bureaus and
offices. Control means the authority
of an empowered officer to alter or
modify, or even nullify or set aside,
what a subordinate officer has done in
the performance of his duties, as well
as to substitute the judgment of the
latter, as and when the former deems
it to be appropriate. Expressed in
another way, the President has the
power to assume directly the functions
of an executive department, bureau
and office.
It can accordingly be
inferred therefrom that the President
can interfere in the exercise of
discretion of officials under him or
altogether
ignore
their
recommendations.
It is the considered view of the
Court, given the above disquisition,
that
the
phrase
upon
recommendation of the Secretary,
found in Section 9, Chapter II, Title III,
Book IV, of the Revised Administrative
Code, should be interpreted, as it is
normally so understood, to be a mere
advise, exhortation or indorsement,
which is essentially persuasive in
character
and
not
binding
or
obligatory upon the party to whom it is
made. The recommendation is here
nothing really more than advisory in
nature. The President, being the head
of the Executive Department, could
very well disregard or do away with
the action of the departments,
bureaus or offices even in the exercise
of discretionary authority, and in so
opting, he cannot be said as having
acted beyond the scope of his
authority.
The doctrine in San Juan, relied
upon by petitioners, is tangential.
While the tenor of the legal provision
in Executive Order No. 112 has some
similarity with the provision in the

1987 Administrative Code in question,


it is to be pointed out, however, that
San Juan (196 SCRA 69), in construing
the law, has distinctively given stress
to the constitutional mandate on local
autonomy; x x x. The Court there has
explained that the President merely
exercises general supervision over
local government units and local
officials
(Section
4,
Article
X,
Constitution);
hence,
in
the
appointment of a Provincial Budget
Officer, the executive department,
through the Secretary of Budget and
Management, indeed had to share the
questioned power with the local
government.
In
the
instant
case,
the
recommendation of the Secretary of
Justice and the appointment of the
President are acts of the Executive
Department itself, and there is no
sharing of power to speak of, the latter
being deemed for all intents and
purposes
as
being
merely
an
extension of the personality of the
President. (Bermudez v. Executive
Secretary Ruben Torres, G.R. No.
131429, Aug. 4, 1999, 3rd Div.
[Vitug])
99.
Discuss
the
three
distinct
powers of the President under Section
18, Art. VII of the 1987 Constitution.
Are they subject to judicial review, or
are they political questions?
Ans.: There are three distinct
powers of the President under Sec. 18,
Art. VII of the Constitution, to wit: 1)
her calling out power, as Commanderin-Chief of the Armed Forces; 2) her
martial law power; and 3) her power to
suspend the privilege of the writ of
habeas corpus.
Her martial law power and her
power to suspend the privilege of the
writ of habeas corpus are subject to
judicial review as expressly provided
under Sec. 18, Art. VII of the 1987
Constitution because these two are
the greater powers, compared with
her calling out power, as they involve
the curtailment and suppression of
certain basic civil rights and individual
freedoms (IBP v. Zamora, G.R. No.
141284, Aug. 15, 2000, En Banc
[Kapunan]).
Her calling out power is a
political question and not subject to
judicial power as this is the lesser and

more benign of the three powers


under Sec. 18, Art. VII of the 1987
Constitution (IBP v. Zamora, G.R. No.
141284, Aug. 15, 2000, En Banc
[Kapunan]). It is a question in regard
to which full discretionary authority
has
been
delegated
by
the
Constitution to the President, as their
Commander-in-Chief, to call out the
armed forces whenever she deems it
necessary in order to prevent or
suppress lawless violence, invasion, or
rebellion. To subject such calling out
power to unfettered judicial scrutiny
could be a veritable prescription for
disaster as such power may be unduly
straitjacketed by an injunction or a
TRO every time it is exercised.
Unless it can be shown that the
exercise of such discretion to call out
the armed forces was gravely abused,
the Presidents exercise of judgment
deserves to be accorded respect from
the Court. And the burden to show
that the President gravely abused her
discretion in calling out the armed
forces to prevent or suppress lawless
violence, invasion, or rebellion, lies

The Judicial Department


100. What are the requisites before
the Court can exercise the power of
judicial review?
Held: 1. The time-tested standards
for the exercise of judicial review are:
(1) the existence of an appropriate
case; (2) an interest personal and
substantial by the party raising the
constitutional question; (3) the plea
that the function be exercised at the
earliest opportunity; and (4) the
necessity that the constitutional
question be passed upon in order to
decide the case (Separate Opinion,
Kapunan, J., in Isagani Cruz v.
Secretary of Environment and
Natural Resources, et al., G.R. No.
135385, Dec. 6, 2000, En Banc).
2. When questions of constitutional
significance are raised, the Court can
exercise its power of judicial review
only if the following requisites are
complied with, namely:
(1) the
existence of an actual and appropriate
case; (2) a personal and substantial
interest of the party raising the
constitutional
question;
(3)
the
exercise of judicial review is pleaded
at the earliest opportunity; and (4) the
constitutional question is the lis mota
of the case. (Integrated Bar of the

Philippines v. Hon. Ronaldo B.


Zamora, G.R. No. 141284, Aug. 15,
2000, En Banc [Kapunan])
101. What is an "actual case or
controversy"?
Held: An "actual case or controversy"
means an existing case or controversy
which is both ripe for resolution and
susceptible of judicial determination,
and that which is not conjectural or
anticipatory, or that which seeks to
resolve
hypothetical
or
feigned
constitutional problems.
A petition
raising a constitutional question does
not present an "actual controversy,"
unless it alleges a legal right or power.
Moreover, it must show that a conflict
of rights exists, for inherent in the
term "controversy" is the presence of
opposing
views
or
contentions.
Otherwise, the Court will be forced to
resolve issues which remain unfocused
because they lack such concreteness
provided when a question emerges
precisely framed from a clash of
adversary arguments exploring every
aspect of a multi-faceted situation
embracing conflicting and demanding
interests. The controversy must also
be justiciable; that is, it must be
susceptible of judicial determination.
(Integrated Bar of the Philippines
v. Hon. Ronaldo B. Zamora, G.R.
No. 141284, Aug. 15, 2000, En
Banc [Kapunan])
102. Petitioners Isagani Cruz and
Cesar Europa brought a suit for
prohibition and mandamus as citizens
and
taxpayers,
assailing
the
constitutionality of certain provisions
of Republic Act No. 8371, otherwise
known as the Indigenous Peoples
Rights Act of 1997 (IPRA), and its
Implementing Rules and Regulations.
A preliminary issue resolved by the SC
was whether the petition presents an
actual controversy.
Held: Courts can only decide actual
controversies,
not
hypothetical
questions or cases.
The threshold
issue, therefore, is whether an
"appropriate case" exists for the
exercise of judicial review in the
present case.
Xxx
In the case at bar, there exists a live
controversy involving a clash of legal
rights. A law has been enacted, and
the
Implementing
Rules
and
Regulations approved.
Money has
been
appropriated
and
the

government agencies concerned have


been directed to implement the
statute.
It cannot be successfully
maintained that we should await the
adverse consequences of the law in
order to consider the controversy
actual and ripe for judicial resolution.
It is precisely the contention of the
petitioners that the law, on its face,
constitutes
an
unconstitutional
abdication of State ownership over
lands of the public domain and other
natural resources.
Moreover, when
the State machinery is set into motion
to
implement
an
alleged
unconstitutional statute, this Court
possesses sufficient authority to
resolve and prevent imminent injury
and violation of the constitutional
process.
(Separate
Opinion,
Kapunan, J., in Isagani Cruz v.
Secretary of Environment and
Natural Resources, et al., G.R. No.
135385, Dec. 6, 2000, En Banc)
103. What is the meaning of "legal
standing" or locus standi?
Held: "Legal standing" or locus standi
has been defined as a personal and
substantial interest in the case such
that the party has sustained or will
sustain direct injury as a result of the
governmental act that is being
challenged.
The term "interest"
means a material interest, an interest
in issue affected by the decree, as
distinguished from mere interest in the
question involved, or a mere incidental
interest. The gist of the question of
standing is whether a party alleges
"such personal stake in the outcome of
the controversy as to assure that
concrete adverseness which sharpens
the presentation of issues upon which
the court depends for illumination of
difficult
constitutional
questions."
(Integrated Bar of the Philippines
v. Hon. Ronaldo B. Zamora, G.R.
No. 141284, Aug. 15, 2000)
In addition to the existence of an
actual case or controversy, a person
who assails the validity of a statute
must have a personal and substantial
interest in the case, such that, he has
sustained, or will sustain, a direct
injury as a result of its enforcement.
Evidently, the rights asserted by
petitioners as citizens and taxpayers
are held in common by all the citizens,
the violation of which may result only
in a "generalized grievance". Yet, in a
sense, all citizen's and taxpayer's suits
are
efforts
to
air
generalized
grievances about the conduct of
government and the allocation of

power.
(Separate
Opinion,
Kapunan, J., in Isagani Cruz v.
Secretary of Environment and
Natural Resources, et al., G.R. No.
135385, Dec. 6, 2000, En Banc)
104. Asserting itself as the official
organization of Filipino lawyers tasked
with the bounden duty to uphold the
rule of law and the Constitution, the
Integrated Bar of the Philippines (IBP)
filed a petition before the SC
questioning the validity of the order of
the
President
commanding
the
deployment and utilization of the
Philippine Marines to assist the
Philippine National Police (PNP) in law
enforcement by joining the latter in
visibility patrols around the metropolis.
The Solicitor General questioned the
legal standing of the IBP to file the
petition? Resolve.
Held: In the case at bar, the IBP
primarily anchors its standing on its
alleged responsibility to uphold the
rule of law and the Constitution. Apart
from this declaration, however, the IBP
asserts no other basis in support of its
locus standi. The mere invocation by
the IBP of its duty to preserve the rule
of law and nothing more, while
undoubtedly true, is not sufficient to
clothe it with standing in this case.
This is too general an interest which is
shared by other groups and the whole
citizenry.
Based on the standards
above-stated, the IBP has failed to
present a specific and substantial
interest in the resolution of the case.
Its fundamental purpose which, under
Section 2, Rule 139-A of the Rules of
Court, is to elevate the standards of
the law profession and to improve the
administration of justice is alien to,
and cannot be affected by the
deployment of the Marines. x x x
Moreover, the IBP x x x has not shown
any specific injury which it has
suffered or may suffer by virtue of the
questioned governmental act. Indeed,
none of its members, whom the IBP
purportedly represents, has sustained
any form of injury as a result of the
operation of the joint visibility patrols.
Neither is it alleged that any of its
members has been arrested or that
their civil liberties have been violated
by the deployment of the Marines.
What the IBP projects as injurious is
the supposed "militarization" of law
enforcement which might threaten
Philippine democratic institutions and
may cause more harm than good in
the long run.
Not only is the
presumed "injury" not personal in
character, it is likewise too vague,

highly speculative and uncertain to


satisfy the requirement of standing.
Since petitioner has not successfully
established a direct and personal
injury as a consequence of the
questioned act, it does not possess the
personality to assail the validity of the
deployment of the Marines.
This
Court, however, does not categorically
rule that the IBP has absolutely no
standing to raise constitutional issues
now or in the future. The IBP must, by
way of allegations and proof, satisfy
this Court that it has sufficient stake to
obtain judicial resolution of the
controversy. (Integrated Bar of the
Philippines v. Hon. Ronaldo B.
Zamora, G.R. No. 141284, Aug. 15,
2000, En Banc [Kapunan])
105. Considering the lack of requisite
standing of the IBP to file the petition
questioning the validity of the order of
the President to deploy and utilize the
Philippine Marines to assist the PNP in
law enforcement, may the Court still
properly take cognizance of the case?
Held: Having stated the foregoing, it
must be emphasized that this Court
has the discretion to take cognizance
of a suit which does not satisfy the
requirement of legal standing when
paramount interest is involved. In not
a few cases, the Court has adopted a
liberal attitude on the locus standi of a
petitioner where the petitioner is able
to craft an issue of transcendental
significance to the people. Thus, when
the issues raised are of paramount
importance to the public, the Court
may brush aside technicalities of
procedure. In this case, a reading of
the petition shows that the IBP has
advanced constitutional issues which
deserve the attention of this Court in
view of their seriousness, novelty and
weight as precedents.
Moreover,
because peace and order are under
constant threat and lawless violence
occurs
in
increasing
tempo,
undoubtedly
aggravated
by
the
Mindanao insurgency problem, the
legal controversy raised in the petition
almost certainly will not go away. It
will stare us in the face again. It,
therefore, behooves the Court to relax
the rules on standing and to resolve
the issue now, rather than later.
(Integrated Bar of the Philippines
v. Hon. Ronaldo B. Zamora, G.R.
No. 141284, Aug. 15, 2000)
106. What are the requisites for the
proper exercise of the power of judicial
review? Illustrative case.

We are not persuaded.


Held: Respondents assert that
the petition fails to satisfy all the four
requisites before this Court may
exercise its power of judicial review in
constitutional cases. Out of respect
for the acts of the Executive
department, which is co-equal with
this Court, respondents urge this Court
to
refrain
from
reviewing
the
constitutionality of the ad interim
appointments issued by the President
to Benipayo, Borra and Tuason unless
all the four requisites are present. X x
x
Respondents argue that the
second, third and fourth requisites are
absent in this case.
Respondents
maintain that petitioner does not have
a personal and substantial interest in
the case because she has not
sustained a direct injury as a result of
the ad interim appointments of
Benipayo, Borra and Tuason and their
assumption of office.
Respondents
point out that petitioner does not
claim to be lawfully entitled to any of
the positions assumed by Benipayo,
Borra or Tuason.
Neither does
petitioner claim to be directly injured
by the appointments of these three
respondents.
Respondents also contend that
petitioner failed to question the
constitutionality of the ad interim
appointments
at
the
earliest
opportunity.
Petitioner filed the
petition only on August 3, 2001
despite the fact that the ad interim
appointments of Benipayo, Borra and
Tuason were issued as early as March
22, 2001. Moreover, the petition was
filed after the third time that these
three respondents were issued ad
interim appointments.
Respondents insist that the real
issue in this case is the legality of
petitioners reassignment from the EID
to
the
Law
Department.
Consequently, the constitutionality of
the ad interim appointments is not the
lis mota of this case.

Benipayo reassigned petitioner


from the EID, where she was Acting
Director, to the Law Department,
where she was placed on detail.
Respondents
claim
that
the
reassignment was pursuant to x x x
Benipayos authority as Chairman of
the Commission on Elections, and as
the Commissions Chief Executive
Officer.
Evidently, respondents
anchor the legality of petitioners
reassignment on Benipayos authority
as Chairman of the COMELEC. The
real issue then turns on whether or not
Benipayo is the lawful Chairman of the
COMELEC. Even if petitioner is only an
Acting director of the EID, her
reassignment is without legal basis if
Benipayo is not the lawful COMELEC
Chairman, an office created by the
Constitution.
On the other hand, if Benipayo
is the lawful COMELEC Chairman
because
he
assumed
office
in
accordance with the Constitution, then
petitioners reassignment is legal and
she has no cause to complain provided
the reassignment is in accordance
with the Civil Service Law. Clearly,
petitioner has a personal and material
stake in the resolution of the
constitutionality
of
Benipayos
assumption of office.
Petitioners
personal and substantial injury, if
Benipayo is not the lawful COMELEC
Chairman, clothes her with the
requisite locus standi to raise the
constitutional issue in this petition.
Respondents harp on petitioners
belated act of questioning the
constitutionality of the ad interim
appointments of Benipayo, Borra and
Tuason.
Petitioner filed the instant
petition only on August 3, 2001, when
the first ad interim appointments were
issued as early as March 22, 2001.
However, it is not the date of filing of
the petition that determines whether
the constitutional issue was raised at
the earliest opportunity. The earliest
opportunity to raise a constitutional
issue is to raise it in the pleadings

before a competent court that can


resolve the same, such that, if it is
not raised in the pleadings, it cannot
be considered on appeal. (Joaquin G.
Bernas, The 1987 Constitution of the
Republic
of
the
Philippines:
A
Commentary, p. 858 [1996], citing
People v. Vera, 65 Phil. 56 [1937]).
Petitioner
questioned
the
constitutionality of the ad interim
appointments of Benipayo, Borra and
Tuason when she filed her petition
before this Court, which is the earliest
opportunity
for
pleading
the
constitutional
issue
before
a
competent body.
Furthermore, this
Court may determine, in the exercise
of sound discretion, the time when a
constitutional issue may be passed
upon (Ibid., citing Sotto v. Commission
on Elections, 76 Phil. 516 [1946]).
There is no doubt petitioner raised the
constitutional issue on time.
Moreover,
the
legality
of
petitioners reassignment hinges on
the constitutionality of Benipayos ad
interim appointment and assumption
of office. Unless the constitutionality
of Benipayos ad interim appointment
and assumption of office is resolved,
the
legality
of
petitioners
reassignment from the EID to the Law
Department cannot be determined.
Clearly, the lis mota of this case is the
very constitutional issue raised by
petitioner.
In any event, the issue raised by
petitioner is of paramount importance
to the public.
The legality of the
directives and decisions made by the
COMELEC in the conduct of the May
14, 2001 national elections may be
put in doubt if the constitutional issue
raised by petitioner is left unresolved.
In keeping with this Courts duty to
determine whether other agencies of
government have remained within the
limits of the Constitution and have not
abused the discretion given them, this
Court
may
even
brush
aside
technicalities of procedure and resolve
any constitutional issue raised (Ople v.
Torres, 293 SCRA 1412 [1998]; others
omitted).
Here the petitioner has

complied with all the requisite


technicalities.
Moreover,
public
interest requires the resolution of the
constitutional
issue
raised
by
petitioner. (Matibag v. Benipayo,
380 SCRA 49, April 2, 2002, En
Banc [Carpio])
107. What is the meaning of
justiciable controversy as requisite
for the proper exercise of the power of
judicial review? Illustrative case.
Held: From a reading of the
records it appears to us that the
petition was prematurely filed. Under
the undisputed facts there is as yet no
justiciable controversy for the court to
resolve and the petition should have
been dismissed by the appellate court
on this ground.
We gather from the allegations
of the petition and that of the
petitioners memorandum that the
alleged application for certificate of
ancestral land claim (CALC) filed by
the heirs of Carantes under the
assailed DENR special orders has not
been granted nor the CALC applied
for, issued.
The DENR is still
processing the application of the heirs
of Carantes for a certificate of
ancestral land claim, which the DENR
may or may not grant. It is evident
that the adverse legal interests
involved in this case are the
competing claims of the petitioners
and that of the heirs of Carantes to
possess a common portion of a piece
of land. As the undisputed facts stand
there is no justiciable controversy
between the petitioners and the
respondents as there is no actual or
imminent violation of the petitioners
asserted right to possess the land by
reason of the implementation of the
questioned administrative issuance.
A justiciable controversy has been
defined as, a definite and concrete
dispute touching on the legal relations
of parties having adverse legal
interests (Sinco, Philippine Political
Law, 1962 ed., quoting from the U.S.
Declaratory Judgment Act of 1934, p.

360) which may be resolved by a court


of law through the application of a law
(Macasiano
v.
National
Housing
Authority, 224 SCRA 238 [1993];
Bernas, The Constitution of the
Republic
of
the
Philippines:
A
Commentary, Vol. II, 1988 ed., pp.
274-275).
Courts have no judicial
power to review cases involving
political questions and as a rule, will
desist from taking cognizance of
speculative or hypothetical cases,
advisory opinions and in cases that
has become moot (Cruz, Philippine
Political Law, 1998 ed., p. 257-259).
Subject
to
certain
well-defined
exceptions (Solicitor-General v. MMA,
December 11, 1991, 204 SCRA 837;
Dumlao v. Comelec, 95 SCRA 392
[1980]) courts will not touch an issue
involving the validity of a law unless
there has been a governmental act
accomplished or performed that has a
direct adverse effect on the legal right
of the person contesting its validity
(Tan v. Macapagal, 43 SCRA 678
[1972]).
In the case of PACU v.
Secretary of Education (97 Phil. 806
[1955]) the petition contesting the
validity of a regulation issued by the
Secretary of Education requiring
private schools to secure a permit to
operate was dismissed on the ground
that all the petitioners have permits
and are actually operating under the
same. The petitioners questioned the
regulation because of the possibility
that the permit might be denied them
in the future. This Court held that
there was no justiciable controversy
because the petitioners suffered no
wrong by the implementation of the
questioned regulation and therefore,
they are not entitled to relief. A mere
apprehension that the Secretary of
Education will withdraw the permit
does not amount to justiciable
controversy.
The
questioned
regulation in the PACU case may be
questioned by a private school whose
permit to operate has been revoked or
one whose application therefore has
been denied (Bernas, supra.).
This Court cannot rule on the
basis of petitioners speculation that

the DENR will approve the application


of the heirs of Carantes. There must
be an actual governmental act which
directly causes or will imminently
cause injury to the alleged legal right
of the petitioner to possess the land
before the jurisdiction of this Court
may be invoked. There is no showing
that the petitioners were being evicted
from the land by the heirs of Carantes
under orders from the DENR.
The
petitioners allegation that certain
documents from the DENR were shown
to them by the heirs of Carantes to
justify eviction is vague, and it would
appear that the petitioners did not
verify if indeed the respondent DENR
or
its
officers
authorized
the
attempted eviction. Suffice it to say
that by the petitioners own admission
that
the
respondents
are
still
processing and have not approved the
application of the heirs of Carantes,
the petitioners alleged right to possess
the land is not violated nor is in
imminent danger of being violated, as
the DENR may or may not approve
Carantes application. Until such time,
the petitioners are simply speculating
that they might be evicted from the
premises at some future time.
Borrowing from the pronouncements
of this Court in the PACU case, They
(the petitioners) have suffered no
wrong under the terms of the law
and, naturally need no relief in the
form they now seek to obtain. (PACU,
supra, at p. 810) If indeed the heirs of
Carantes are trying to enter the land
and
disturbing
the
petitioners
possession thereof even without prior
approval by the DENR of the claim of
the heirs of Carantes, the case is
simply one of forcible entry. (Cutaran
v. DENR, 350 SCRA 697, Jan. 31,
2001, 3rd Div. [Gonzaga-Reyes])
108. Should the Court still resolve
the case despite that the issue has
already become moot and academic?
Exception.
Held:
Neither do we agree that
merely because a plebiscite had
already been held in the case of the
proposed
Barangay
Napico,
the
petition of the Municipality of Cainta
has already been rendered moot and

academic. The issue raised by the


Municipality of Cainta in its petition
before the COMELEC against the
holding of the plebiscite for the
creation of Barangay Napico are still
pending determination before the
Antipolo Regional Trial Court.
In Tan v. Commission on Elections
(G.R. No. 73155, 142 SCRA 727, 741742 [1986]), we struck down the moot
and academic argument as follows
Considering that the legality of the
plebiscite itself is challenged for noncompliance
with
constitutional
requisites, the fact that such plebiscite
had been held and a new province
proclaimed and its officials appointed,
the case before Us cannot truly be
viewed as already moot and academic.
Continuation of the existence of this
newly proclaimed province which
petitioners strongly profess to have
been illegally born, deserves to be
inquired into by this Tribunal so that, if
indeed, illegality attaches to its
creation, the commission of that error
should not provide the very excuse for
perpetration of such wrong. For this
Court to yield to the respondents
urging that, as there has been fait
accompli, then this Court should
passively accept and accede to the
prevailing situation is an unacceptable
suggestion. Dismissal of the instant
petition, as respondents so propose is
a proposition fraught with mischief.
Respondents submission will create a
dangerous precedent.
Should this
Court decline now to perform its duty
of interpreting and indicating what the
law is and should be, this might tempt
again those who strut about in the
corridors of power to recklessly and
with ulterior motives, create, merge,
divide and/or alter the boundaries of
political subdivisions, either brazenly
or stealthily, confident that this Court
will abstain from entertaining future
challenges to their acts if they
manage to bring about a fait
accompli.
(City of Pasig v. COMELEC, 314
SCRA 179, Sept. 10, 1999, En Banc
[Ynares-Santiago])
109. On May 1, 2001, President
Macapagal-Arroyo, faced by an angry
and
violent
mob
armed
with
explosives, firearms, bladed weapons,
clubs, stones and other deadly
weapons assaulting and attempting
to break into Malacanang, issued
Proclamation No. 38 declaring that

there was a state of rebellion in the


National Capital Region. She likewise
issued General Order No. 1 directing
the Armed Forces of the Philippines
and the Philippine National Police to
suppress the rebellion in the National
Capital Region. Warrantless arrests of
several alleged leaders and promoters
of the rebellion were thereafter
effected.
Hence, several petitions
were filed before the SC assailing the
declaration of State of Rebellion by
President Gloria Macapagal-Arroyo and
the warrantless arrests allegedly
effected by virtue thereof.
Held:
All the foregoing petitions
assail the declaration of state of
rebellion
by
President
Gloria
Macapagal-Arroyo and the warrantless
arrests allegedly effected by virtue
thereof, as having no basis both in fact
and in law. Significantly, on May 6,
2001,
President
Macapagal-Arroyo
ordered the lifting of the declaration of
a state of rebellion in Metro Manila.
Accordingly, the instant petitions have
been rendered moot and academic.
As to petitioners claim that the
proclamation of a state of rebellion
is being used by the authorities to
justify
warrantless
arrests,
the
Secretary of Justice denies that it has
issued a particular order to arrest
specific persons in connection with the
rebellion. He states that what is
extant are general instructions to law
enforcement officers and military
agencies to implement Proclamation
No. 38. x x x With this declaration,
petitioners apprehensions as to
warrantless arrests should be laid to
rest. (Lacson v. Perez, 357 SCRA
756, May 10, 2001, En Banc
[Melo])
110. When is an action considered
moot? May the court still resolve
the case once it has become moot and
academic?
Held: 1. It is alleged by respondent
that, with respect to the PCCR
[Preparatory
Commission
on
Constitutional Reform], this case has
become moot and academic.
We
agree.

An action is considered moot


when it no longer presents a
justiciable controversy because the
issues
involved
have
become
academic or dead. Under E.O. No. 43,
the PCCR was instructed to complete
its task on or before June 30, 1999.
However, on February 19, 1999, the
President issued Executive Order No.
70 (E.O. No. 70), which extended the
time frame for the completion of the
commissions work x x x. The PCCR
submitted its recommendations to the
President on December 20, 1999 and
was dissolved by the President on the
same day. It had likewise spent the
funds allocated to it. Thus, the PCCR
has ceased to exist, having lost its
raison dtre.
Subsequent events
have overtaken the petition and the
Court has nothing left to resolve.
The staleness of the issue
before us is made more manifest by
the impossibility of granting the relief
prayed for by petitioner. Basically,
petitioner asks this Court to enjoin the
PCCR from acting as such. Clearly,
prohibition is an inappropriate remedy
since the body sought to be enjoined
no longer exists. It is well-established
that prohibition is a preventive remedy
and does not lie to restrain an act that
is already fait accompli. At this point,
any ruling regarding the PCCR would
simply be in the nature of an advisory
opinion, which is definitely beyond the
permissible scope of judicial power.
(Gonzales v. Narvasa, 337 SCRA
733, Aug. 14, 2000, En Banc
[Gonzaga-Reyes])
2. The petition which was filed
by private respondents before the trial
court sought the issuance of a writ of
mandamus, to command petitioners to
admit them for enrolment. Taking into
account the admission of private
respondents that they have finished
their Nursing course at the Lanting
College of Nursing even before the
promulgation
of
the
questioned
decision, this case has clearly been
overtaken by events and should
therefore be dismissed. However, the
case
of
Eastern
Broadcasting
Corporation (DYRE) v. Dans, etc., et
al., G.R. No. 59329, July 19, 1985, 137
SCRA 628 is the authority for the view
that "even if a case were moot and
academic,
a
statement
of
the
governing principle is appropriate in
the resolution of dismissal for the
guidance not only of the parties but of
others similarly situated. We shall
adhere to this view and proceed to
dwell on the merits of this petition.

(University of San Agustin, Inc. v.


Court of Appeals, 230 SCRA 761,
770, March 7, 1994 [Nocon])
111. In connection with the May 11,
1998 elections, the COMELEC issued a
resolution prohibiting the conduct of
exit polls on the ground, among
others, that it might cause disorder
and
confusion
considering
the
randomness of selecting interviewees,
which further makes the exit polls
unreliable. The constitutionality of this
resolution was challenged by ABS-CBN
Broadcasting Corporation as violative
of freedom of expression.
The
Solicitor General contends that the
petition has been rendered moot and
academic because the May 11, 1998
election has already been held and
done with and, therefore, there is no
longer any actual controversy to be
resolved. Resolve.
Held: While the assailed Resolution
referred specifically to the May 11,
1998 election, its implications on the
peoples fundamental freedom of
expression
transcend
the
past
election.
The holding of periodic
elections is a basic feature of our
democratic government. By its very
nature, exit polling is tied up with
elections. To set aside the resolution
of the issue now will only postpone a
task that could well crop up again in
future elections.
In any event, in Salonga v. Cruz Pano
(134 SCRA 438, 463, Feb. 18, 1985),
the Court had occasion to reiterate
that it also has the duty to formulate
guiding and controlling constitutional
principles, precepts, doctrines, or
rules. It has the symbolic function of
educating bench and bar on the extent
of protection given by constitutional
guarantees. Since the fundamental
freedoms of speech and of the press
are being invoked here, we have
resolved to settle, for the guidance of
posterity,
whether
they
likewise
protect the holding of exit polls and
the dissemination of data derived
therefrom. (ABS-CBN Broadcasting
Corporation v. COMELEC, G.R. No.
133486, Jan. 28, 2000, En Banc
[Panganiban])
112. Discuss
the
nature
of
a
taxpayers suit.
When may it be
allowed?
Held: 1. Petitioner and respondents
agree that to constitute a taxpayer's
suit, two requisites must be met,
namely,
that
public
funds
are

disbursed by a political subdivision or


instrumentality and in doing so, a law
is violated or some irregularity is
committed, and that the petitioner is
directly affected by the alleged ultra
vires act. The same pronouncement
was made in Kilosbayan, Inc. v.
Guingona, Jr., (232 SCRA 110 [1994],
where the Court also reiterated its
liberal stance in entertaining so-called
taxpayer's suits, especially when
important issues are involved.
A
closer examination of the facts of this
case would readily demonstrate that
petitioner's standing should not even
be made an issue here, "since
standing is a concept in constitutional
law and here no constitutional
question is actually involved."
In the case at bar, disbursement
of public funds was only made in 1975
when the Province bought the lands
from Ortigas at P110.00 per square
meter in line with the objectives of P.D.
674. Petitioner never referred to such
purchase as an illegal disbursement of
public funds but focused on the
alleged fraudulent reconveyance of
said property to Ortigas because the
price paid was lower than the
prevailing market value of neighboring
lots. The first requirement, therefore,
which would make this petition a
taxpayer's suit is absent. The only
remaining justification for petitioner to
be allowed to pursue this action is
whether it is, or would be, directly
affected by the act complained of. As
we stated in Kilosbayan, Inc. v. Morato
(supra.),
"Standing is a special concern in
constitutional law because in some
cases suits are brought not by parties
who have been personally injured by
the operation of a law or by official
action taken, but by concerned
citizens, taxpayers or voters who
actually sue in the public interest.
Hence the question in standing is
whether such parties have 'alleged
such a personal stake in the outcome
of the controversy as to assure that
concrete adverseness which sharpens
the presentation of issues upon which
the court so largely depends for
illumination of difficult constitutional
questions.' (Citing Baker v. Carr, 369
U.S. 186, 7l. Ed. 2d 633 [1962])"
Undeniably, as a taxpayer, petitioner
would somehow be adversely affected
by an illegal use of public money.
When, however, no such unlawful
spending has been shown, as in the
case at bar, petitioner, even as a

taxpayer,
cannot
question
the
transaction validly executed by and
between the Province and Ortigas for
the simple reason that it is not privy to
said contract.
In other words,
petitioner has absolutely no cause of
action, and consequently no locus
standi, in the instant case. (The AntiGraft League of the Philippines,
Inc. v. San Juan, 260 SCRA 250,
253-255, Aug. 1, 1996, En Banc
[Romero])
2. A taxpayer is deemed to have the
standing to raise a constitutional issue
when it is established that public funds
have been disbursed in alleged
contravention of the law or the
Constitution.
Thus, a taxpayers
action is properly brought only when
there is an exercise by Congress of its
taxing or spending power (Flast v.
Cohen, 392 US 83, 20 L Ed 2d 947, 88
S Ct 1942). This was our ruling in a
recent
case
wherein
petitioners
Telecommunications and Broadcast
Attorneys of the Philippines (TELEBAP)
and GMA Network, Inc. questioned the
validity of Section 92 of B.P. Blg. 881
(otherwise known as the Omnibus
Election Code) requiring radio and
television stations to give free air time
to the Commission on Elections during
the
campaign
period
(Telecommunications and Broadcast
Attorneys of the Philippines, Inc. v.
Commission on Elections, 289 SCRA
337 [1998]).
The Court held that
petitioner TELEBAP did not have any
interest as a taxpayer since the
assailed law did not involve the taxing
or spending power of Congress.
Many other rulings have premised the
grant or denial of standing to
taxpayers upon whether or not the
case involved a disbursement of public
funds by the legislature. In Sanidad v.
Commission on Elections (73 SCRA
333 [1976]), the petitioners therein
were allowed to bring a taxpayers suit
to
question
several
presidential
decrees
promulgated
by
then
President Marcos in his legislative
capacity
calling
for
a
national
referendum, with the Court explaining
that
X x x [i]t is now an ancient rule that
the valid source of a statute
Presidential Decrees are of such
nature may be contested by one who
will sustain a direct injury as a result of
its enforcement. At the instance of
taxpayers, laws providing for the
disbursement of public funds may be
enjoined, upon the theory that the

expenditure of public funds by an


officer of the State for the purpose of
executing an unconstitutional act
constitutes a misapplication of such
funds.
The breadth of Presidential
Decree
No.
991
carries
an
appropriation of Five Million Pesos for
the effective implementation of its
purposes.
Presidential Decree No.
1031 appropriates the sum of Eight
Million Pesos to carry out its
provisions.
The interest of the
aforenamed petitioners as taxpayers
in the lawful expenditure of these
amounts of public money sufficiently
clothes them with that personality to
litigate the validity of the Decrees
appropriating said funds x x x.
In still another case, the Court held
that petitioners the Philippine
Constitution Association, Inc., a nonprofit civic organization had standing
as
taxpayers
to
question
the
constitutionality of Republic Act No.
3836 insofar as it provides for
retirement gratuity and commutation
of vacation and sick leaves to
Senators and Representatives and to
the elective officials of both houses of
Congress
(Philippine
Constitution
Association, Inc. v. Gimenez, 15 SCRA
479 [1965]).
And in Pascual v.
Secretary of Public Works (110 Phil.
331 [1960]), the Court allowed
petitioner to maintain a taxpayers suit
assailing the constitutional soundness
of Republic Act No. 920 appropriating
P85,000 for the construction, repair
and improvement of feeder roads
within private property.
All these
cases involved the disbursement of
public funds by means of a law.
Meanwhile,
in
Bugnay
Construction
and
Development
Corporation v. Laron (176 SCRA 251
[1989]), the Court declared that the
trial court was wrong in allowing
respondent Ravanzo to bring an action
for injunction in his capacity as a
taxpayer in order to question the
legality of the contract of lease
covering the public market entered
into between the City of Dagupan and
petitioner. The Court declared that
Ravanzo did not possess the requisite
standing to bring such taxpayers suit
since [o]n its face, and there is no
evidence to the contrary, the lease
contract
entered
into
between
petitioner and the City shows that no
public funds have been or will be used
in the construction of the market
building.

Coming now to the instant case, it is


readily apparent that there is no
exercise by Congress of its taxing or
spending power.
The PCCR was
created by the President by virtue of
E.O. No. 43, as amended by E.O. No.
70. Under Section 7 of E.O. No. 43,
the
amount
of
P3
million
is
"appropriated" for its operational
expenses "to be sourced from the
funds of the Office of the President. x
x x. The appropriations for the PCCR
were authorized by the President, not
by Congress. In fact, there was no
appropriation at all. In a strict sense,
appropriation has been defied as
nothing more than the legislative
authorization
prescribed
by
the
Constitution that money may be paid
out
of
the
Treasury,
while
appropriation made by law refers to
the act of the legislature setting apart
or assigning to a particular use a
certain sum to be used in the payment
of debt or dues from the State to its
creditors. The funds used for the
PCCR were taken from funds intended
for the Office of the President, in the
exercise of the Chief Executives
power to transfer funds pursuant to
Section 25 (5) of Article VI of the
Constitution.
In the final analysis, it must be
stressed that the Court retains the
power to decide whether or not it will
entertain a taxpayers suit. In the
case at bar, there being no exercise by
Congress of its taxing or spending
power, petitioner cannot be allowed to
question the creation of the PCCR in
his capacity as a taxpayer, but rather,
he must establish that he has a
personal and substantial interest in
the case and that he has sustained or
will sustain direct injury as a result of
its enforcement.
In other words,
petitioner must show that he is a real
party in interest that he will stand to
be benefited or injured by the
judgment or that he will be entitled to
the avails of the suit. Nowhere in his
pleadings does petitioner presume to
make
such
a
representation.
(Gonzales v. Narvasa, 337 SCRA
733, Aug. 14, 2000, En Banc
[Gonzaga-Reyes])
113. What
controversy?
questions?

is
a
justiciable
What are political

Held:
As a general proposition, a
controversy is justiciable if it refers to
a matter which is appropriate for court
review. It pertains to issues which are
inherently
susceptible
of
being

decided on grounds recognized by law.


Nevertheless, the Court does not
automatically assume jurisdiction over
actual constitutional cases brought
before it even in instances that are
ripe for resolution. One class of cases
wherein the Court hesitates to rule on
are "political questions." The reason is
that political questions are concerned
with issues dependent upon the
wisdom, not the legality, of a
particular act or measure being
assailed.
Moreover, the political
question being a function of the
separation of powers, the courts will
not normally interfere with the
workings of another co-equal branch
unless the case shows a clear need for
the courts to step in to uphold the law
and the Constitution.
As Tanada v. Angara (103 Phil. 1051
[1957]) puts it, political questions refer
"to those questions which, under the
Constitution, are to be decided by the
people in their sovereign capacity, or
in regard to which full discretionary
authority has been delegated to the
legislative or executive branch of
government."
Thus, if an issue is
clearly identified by the text of the
Constitution
as
matters
for
discretionary action by a particular
branch of government or to the people
themselves then it is held to be a
political question.
In the classic
formulation of Justice Brennan in Baker
v. Carr (369 U.S. 186, 82 S Ct. 691, 7
L. Ed. 663, 678 [1962]), "[p]rominent
on the surface of any case held to
involve a political question is found a
textually demonstrable constitutional
commitment of the issue to a
coordinate political department; or a
lack of judicially discoverable and
manageable standards for resolving it;
or the impossibility of deciding without
an initial policy determination of a kind
clearly for nonjudicial discretion; or the
impossibility of a court's undertaking
independent
resolution
without
expressing lack of the respect due
coordinate branches of government;
or an unusual need for unquestioning
adherence to a political decision
already made; or the potentiality of
embarrassment
from
multifarious
pronouncements
by
various
departments on the one question."
The 1987 Constitution expands the
concept of judicial review by providing
that "(T)he Judicial power shall be
vested in one Supreme Court and in
such lower courts as may be
established by law.
Judicial power
includes the duty of the courts of

justice to settle actual controversies


involving rights which are legally
demandable and enforceable, and to
determine whether or not there has
been a grave abuse of discretion
amounting to lack or excess of
jurisdiction on the part of any branch
or instrumentality of the Government."
(Article VIII, Sec. 1 of the 1987
Constitution) Under this definition, the
Court cannot agree x x x that the issue
involved is a political question beyond
the jurisdiction of this Court to review.
When the grant of power is qualified,
conditional or subject to limitations,
the issue of whether the prescribed
qualifications or conditions have been
met or the limitations respected, is
justiciable - the problem being one of
legality or validity, not its wisdom.
Moreover, the jurisdiction to delimit
constitutional boundaries has been
given to this Court. When political
questions
are
involved,
the
Constitution limits the determination
as to whether or not there has been a
grave abuse of discretion amounting
to lack or excess of jurisdiction on the
part of the official whose action is
being questioned.
By grave abuse of discretion is meant
simply
capricious
or
whimsical
exercise of judgment that is patent
and gross as to amount to an evasion
of positive duty or a virtual refusal to
perform a duty enjoined by law, or to
act at all in contemplation of law, as
where the power is exercised in an
arbitrary and despotic manner by
reason of passion or hostility. Under
this definition, a court is without power
to directly decide matters over which
full discretionary authority has been
delegated. But while this Court has no
power to substitute its judgment for
that of Congress or of the President, it
may look into the question of whether
such exercise has been made in grave
abuse of discretion. A showing that
plenary power is granted either
department of government may not
be an obstacle to judicial inquiry, for
the improvident exercise or abuse
thereof may give rise to justiciable
controversy. (Integrated Bar of the
Philippines v. Hon. Ronaldo B.
Zamora, G.R. No. 141284, Aug. 15,
2000, En Banc [Kapunan])
114. Is
the
legitimacy
of
the
assumption to the Presidency of
President Gloria Macapagal Arroyo a
political question and, therefore, not
subject to judicial review? Distinguish
EDSA People Power I from EDSA
People Power II.

Held: Respondents rely on the case


of Lawyers League for a Better
Philippines and/or Oliver A. Lozano v.
President Corazon C. Aquino, et al. and
related cases to support their thesis
that since the cases at bar involve the
legitimacy of the government of
respondent Arroyo, ergo, they present
a political question. A more cerebral
reading of the cited cases will show
that they are inapplicable. In the cited
cases, we held that the government of
former President Aquino was the result
of a successful revolution by the
sovereign people, albeit a peaceful
one.
No less than the Freedom
Constitution declared that the Aquino
government was installed through a
direct exercise of the power of the
Filipino people in defiance of the
provisions of the 1973 Constitution, as
amended. It is familiar learning that
the legitimacy of a government sired
by a successful revolution by people
power is beyond judicial scrutiny for
that government automatically orbits
out of the constitutional loop.
In
checkered contrast, the government of
respondent Arroyo is not revolutionary
in character. The oath that she took at
the EDSA Shrine is the oath under the
1987 Constitution. In her oath, she
categorically swore to preserve and
defend the 1987 Constitution. Indeed,
she has stressed that she is
discharging the powers of the
presidency under the authority of the
1987 Constitution.
In fine, the legal distinction between
EDSA People Power I and EDSA People
Power II is clear. EDSA I involves the
exercise of the people power of
revolution which overthrows the whole
government. EDSA II is an exercise of
people power of freedom of speech
and freedom of assembly to petition
the government for redress of
grievances which only affected the
office of the President. EDSA I is extra
constitutional and the legitimacy of
the new government that resulted
from it cannot be the subject of
judicial review, but EDSA II is intra
constitutional and the resignation of
the sitting President that it caused and
the succession of the Vice President as
President are subject to judicial review.
EDSA I presented a political question;
EDSA II involves legal questions. X x x

Needless to state, the cases at bar


pose legal and not political questions.
The principal issues for resolution
require the proper interpretation of
certain provisions in the 1987
Constitution, notably Section 1 of
Article II, and Section 8 of Article VII,
and the allocation of governmental
powers under Section 11 of Article VII.
The issues likewise call for a ruling on
the scope of presidential immunity
from suit.
They also involve the
correct calibration of the right of
petitioner against prejudicial publicity.
As early as the 1803 case of Marbury
v. Madison (1 Cranch [5 US] 137, L Ed
60 [1803]), the doctrine has been laid
down that it is emphatically the
province and duty of the judicial
department to say what the law is x x
x. Thus, respondents invocation of
the doctrine of political question is but
a foray in the dark.
(Joseph E.
Estrada v. Aniano Desierto, G.R.
Nos. 146710-15, March 2, 2001,
En Banc [Puno])
115. Is the Presidents power to call
out the armed forces as their
Commander-in-Chief
in
order
to
prevent or suppress lawless violence,
invasion or rebellion subject to judicial
review, or is it a political question?
Clarify.
Held: When the President calls the
armed forces to prevent or suppress
lawless violence, invasion or rebellion,
he
necessarily
exercises
a
discretionary power solely vested in
his wisdom. This is clear from the
intent of the framers and from the text
of the Constitution itself. The Court,
thus, cannot be called upon to
overrule the President's wisdom or
substitute its own. However, this does
not prevent an examination of
whether such power was exercised
within permissible constitutional limits
or whether it was exercised in a
manner constituting grave abuse of
discretion.
In
view
of
the
constitutional intent to give the
President full discretionary power to
determine the necessity of calling out
the armed forces, it is incumbent upon
the petitioner to show that the
President's decision is totally bereft of
factual basis.
The present petition
fails to discharge such heavy burden
as there is no evidence to support the
assertion
that
there
exists
no
justification for calling out the armed
forces. There is, likewise, no evidence
to support the proposition that grave
abuse was committed because the
power to call was exercised in such a

manner as to violate the constitutional


provision on civilian supremacy over
the military. In the performance of this
Court's duty of "purposeful hesitation"
before declaring an act of another
branch as unconstitutional, only where
such grave abuse of discretion is
clearly shown shall the Court interfere
with the President's judgment.
To
doubt is to sustain. (Integrated Bar
of the Philippines v. Hon. Ronaldo
B. Zamora, G.R. No. 141284, Aug.
15, 2000, En Banc [Kapunan])

those
who
participated
in
its
discussion (Art. VIII, Sec. 4[2],
Constitution)
(Drilon v. Lim, 235
SCRA 135, 139-140, Aug. 4, 1994,
En Banc [Cruz])

116. Do
lower
courts
have
jurisdiction
to
consider
the
constitutionality of a law? If so, how
should they act in the exercise of this
jurisdiction?

X x x, the following
considered en banc cases:

Held: We stress at the outset that the


lower court had jurisdiction to consider
the constitutionality of Section 187,
this authority being embraced in the
general definition of the judicial power
to determine what are the valid and
binding laws by the criterion of their
conformity to the fundamental law.
Specifically, BP 129 vests in the
regional trial courts jurisdiction over
all civil cases in which the subject of
the litigation is incapable of pecuniary
estimation (Sec. 19[1]), even as the
accused in a criminal action has the
right to question in his defense the
constitutionality of a law he is charged
with violating and of the proceedings
taken against him, particularly as they
contravene
the
Bill
of
Rights.
Moreover, Article VIII, Section 5(2), of
the Constitution vests in the Supreme
Court appellate jurisdiction over final
judgments and orders of lower courts
in
all
cases
in
which
the
constitutionality or validity of any
treaty, international or executive
agreement, law, presidential decree,
proclamation,
order,
instruction,
ordinance, or regulation is in question.
In the exercise of this jurisdiction,
lower courts are advised to act with
the utmost circumspection, bearing in
mind
the
consequences
of
a
declaration of unconstitutionality upon
the stability of laws, no less than on
the doctrine of separation of powers.
As the questioned act is usually the
handiwork of the legislative or the
executive departments, or both, it will
be prudent for such courts, if only out
of a becoming modesty, to defer to
the higher judgment of this Court in
the consideration of its validity, which
is better determined after a thorough
deliberation by a collegiate body and
with the concurrence of the majority of

117. What cases are to be heard by


the Supreme Court en banc?
Held: Under Supreme Court Circular
No. 2-89, dated February 7, 1989, as
amended by the Resolution of
November 18, 1993:

1)

2)
3)
4)
5)

6)

7)

8)

9)

are

Cases
in
which
the
constitutionality or validity of any
treaty, international or executive
agreement, law, executive order, or
presidential
decree,
proclamation,
order,
instruction,
ordinance,
or
regulation is in question;
Criminal cases in which the
appealed decision imposes the death
penalty;
Cases
raising
novel
questions of law;
Cases
affecting
ambassadors, other public ministers
and consuls;
Cases involving decisions,
resolutions or orders of the Civil
Service Commission, Commission on
Elections, and Commission on Audit;
Cases where the penalty to
be imposed is the dismissal of a judge,
officer or employee of the judiciary,
disbarment of a lawyer, or either the
suspension of any of them for a period
of more than one (1) year or a fine
exceeding P10,000.00 or both;
Cases where a doctrine or
principle laid down by the court en
banc or in division may be modified or
reversed;
Cases assigned to a division
which in the opinion of at least three
(3) members thereof merit the
attention of the court en banc and are
acceptable to a majority of the actual
membership of the court en banc; and
All other cases as the court
en banc by a majority of its actual
membership may deem of sufficient
importance to merit its attention.
(Firestone Ceramics, Inc. v. Court
of Appeals, 334 SCRA 465, 471472, June 28, 2000, En Banc
[Purisima])
118. What is fiscal autonomy? The
fiscal autonomy clause?
Held:
As envisioned in the
Constitution, the fiscal autonomy

enjoyed by the Judiciary, the Civil


Service Commission, the Commission
on Audit, the Commission on Elections,
and the Office of the Ombudsman
contemplates a guarantee of full
flexibility to allocate and utilize their
resources with the wisdom and
dispatch that their needs require. It
recognizes the power and authority to
levy, assess and collect fees, fix rates
of compensation not exceeding the
highest rates authorized by law for
compensation and pay plans of the
government and allocate and disburse
such sums as may be provided by law
or prescribed by them in the course of
the discharge of their functions.
Fiscal autonomy means freedom from
outside control.
The Judiciary, the
Constitutional Commissions, and the
Ombudsman
must
have
the
independence and flexibility needed in
the discharge of their constitutional
duties. The imposition of restrictions
and constraints on the manner the
independent
constitutional
offices
allocate
and
utilize
the
funds
appropriated for their operations is
anathema to fiscal autonomy and
violative not only of the express
mandate of the Constitution but
especially as regards the Supreme
Court, of the independence and
separation of powers upon which the
entire fabric of our constitutional
system is based.
(Bengzon v.
Drilon, 208 SCRA 133, April 15,
1992, En Banc [Gutierrez])
119. May the Ombudsman validly
entertain criminal charges against a
judge of the regional trial court in
connection with his handling of cases
before the court.
Held: Petitioner criticizes the
jurisprudence (Maceda v. Vasquez, 221
SCRA 464 [1993] and Dolalas v. Office
of the Ombudsman-Mindanao, 265
SCRA 818 [1996]) cited by the Office
of the Ombudsman as erroneous and
not applicable to his complaint. He
insists that since his complaint
involved a criminal charge against a
judge, it was within the authority of
the Ombudsman not the Supreme
Court to resolve whether a crime was
committed and the judge prosecuted
therefor.
The petition can not succeed.
Xxx
We agree with the Solicitor General
that the Ombudsman committed no

grave abuse of discretion warranting


the writs prayed for. The issues have
been settled in the case of In Re:
Joaquin Borromeo (241 SCRA 408, 460
[1995]). There, we laid down the rule
that before a civil or criminal action
against a judge for a violation of Arts.
204 and 205 (knowingly rendering an
unjust judgment or order) can be
entertained, there must first be a
final
and
authoritative
judicial
declaration that the decision or order
in question is indeed unjust. The
pronouncement may result from
either:
(a)

an action of certiorari or
prohibition
in
a
higher
court
impugning
the
validity
of
the
judgment; or
(b)
an administrative proceeding
in the Supreme Court against the
judge precisely for promulgating an
unjust judgment or order.
Likewise, the determination of whether
a judge has maliciously delayed the
disposition of the case is also an
exclusive judicial function (In Re:
Borromeo, supra, at 461).
To repeat, no other entity or official of
the government, not the prosecution
or investigation service of any other
branch, not any functionary thereof,
has competence to review a judicial
order or decision whether final and
executory or not and pronounce it
erroneous so as to lay the basis for a
criminal or administrative complaint
for rendering an unjust judgment or
order. That prerogative belongs to the
courts alone.
This having been said, we find
that the Ombudsman acted in
accordance with law and jurisprudence
when he referred the cases against
Judge Pelayo to the Supreme Court for
appropriate action.
(De Vera v.
Pelayo, 335 SCRA 281, July 6,
2000, 1st Div. [Pardo])
120. Discuss
the
validity
Memorandum Decisions.

of

Held: 1. The constitutional mandate


that no decision shall be rendered by
any court without expressing therein
clearly and distinctly the facts and the
law on which it is based does not
preclude the validity of "memorandum
decisions" which adopt by reference
the findings of fact and conclusions of
law contained in the decisions of
inferior tribunals. X x x

Hence, even in this jurisdiction,


incorporation by reference is allowed if
only to avoid the cumbersome
reproduction of the decision of the
lower courts, or portions thereof, in
the decisions of the higher court
(Francisco v. Permskul, 173SCRA 324,
333). This is particularly true when
the decision sought to be incorporated
is a lengthy and thorough discussion
of the facts and conclusions arrived at
x x x.
(Oil and Natural Gas
Commission v. Court of Appeals,
293 SCRA 26, July 23, 1998
[Martinez])
2. We have sustained decisions
of lower courts as having substantially
or sufficiently complied with the
constitutional
injunction
notwithstanding the laconic and terse
manner in which they were written
and even if there [was left] much to
be desired in terms of [their] clarity,
coherence
and
comprehensibility
provided that they eventually set out
the facts and the law on which they
were based, as when they stated the
legal qualifications of the offense
constituted by the facts proved, the
modifying
circumstances,
the
participation of the accused, the
penalty imposed and the civil liability;
or discussed the facts comprising the
elements of the offense that was
charged in the information, and
accordingly rendered a verdict and
imposed the corresponding penalty; or
quoted the facts narrated in the
prosecutions memorandum but made
their own findings and assessment of
evidence, before finally agreeing with
the prosecutions evaluation of the
case.
We have also sanctioned the use of
memorandum decisions (In Francisco
v. Permskul, 173 SCRA 324, 333
[1989], the Court described [t]he
distinctive features of a memorandum
decision are, first, it is rendered by an
appellate
court,
second,
it
incorporates by reference the findings
of fact or the conclusions of law
contained in the decision, order, or
ruling under review. Most likely, the
purpose is to affirm the decision,
although it is not impossible that the
approval of the findings of facts by the
lower court may lead to a different
conclusion of law by the higher court.
At any rate, the reason for allowing
the incorporation by reference is
evidently to avoid the cumbersome
reproduction of the decision of the
lower court, or portions thereof, in the
decision of the higher court. The idea

is to avoid having to repeat in the


body of the latter decision the findings
or conclusions of the lower court since
they are being approved or adopted
anyway.), a specie of succinctly
written decisions by appellate courts
in accordance with the provisions of
Section 40, B.P. Blg. 129 on the
grounds of expediency, practicality,
convenience and docket status of our
courts. We have also declared that
memorandum decisions comply with
the constitutional mandate.
In
Francisco
v.
Permskul,
however, we laid the conditions for the
validity of memorandum decisions,
thus:
The memorandum decision, to be
valid, cannot incorporate the findings
of fact and the conclusions of law of
the lower court only by remote
reference, which is to say that the
challenged decision is not easily and
immediately available to the person
reading the memorandum decision.
For the incorporation by reference to
be allowed, it must provide for direct
access to the facts and the law being
adopted, which must be contained in a
statement attached to the said
decision.
In other words, the
memorandum
decision
authorized
under Section 40 of B.P. Blg. 129
should actually embody the findings of
fact and conclusions of law of the
lower court in an annex attached to
and made an indispensable part of the
decision.
It is expected that this requirement
will allay the suspicion that no study
was made of the decision of the lower
court and that its decision was merely
affirmed without a prior examination
of the facts and the law on which it is
based. The proximity at least of the
annexed statement should suggest
that such examination has been
undertaken.
It is, of course, also
understood that the decision being
adopted should, to begin with, comply
with Article VIII, Section 14 as no
amount of incorporation or adoption
will rectify its violation.
The
Court
finds
necessary
to
emphasize that the memorandum
decision should be sparingly used lest
it become an additive excuse for
judicial sloth.
It is an additional
condition for the validity of this kind of
decision may be resorted to only in
cases where the facts are in the main
accepted by both parties and easily
determinable by the judge and there

are
no
doctrinal
complications
involved that will require an extended
discussion of the laws involved. The
memorandum
decision
may
be
employed in simple litigations only,
such as ordinary collection cases,
where
the
appeal is obviously
groundless and deserves no more than
the time needed to dismiss it.
Xxx
Henceforth,
all
memorandum
decisions shall comply with the
requirements herein set forth as to the
form prescribed and the occasions
when they may be rendered. Any
deviation will summon the strict
enforcement of Article VIII, Section 14
of the Constitution and strike down the
flawed
judgment
as
a
lawless
disobedience.
Tested against these standards,
we find that the RTC decision at bar
miserably failed to meet them and,
therefore,
fell
short
of
the
constitutional injunction.
The RTC
decision is brief indeed, but it is
starkly
hallow,
otiosely
written,
vacuous in its content and trite in its
form.
It achieved nothing and
attempted at nothing, not even at a
simple summation of facts which could
easily be done. Its inadequacy speaks
for itself.
We cannot even consider or
affirm said RTC decision as a
memorandum decision because it
failed to comply with the measures of
validity laid down in Francisco v.
Permskul. It merely affirmed in toto
the MeTC decision without saying
more.
A decision or resolution,
especially one resolving an appeal,
should directly meet the issues for
resolution; otherwise, the appeal
would be pointless (See ABD Overseas
Manpower Corporation v. NLRC, 286
SCRA 454, 464 [1998]).
We therefore reiterate our
admonition
in
Nicos
Industrial
Corporation v. Court of Appeals (206
SCRA 127, 134 [1992]), in that while
we conceded that brevity in the
writing of decisions is an admirable
trait, it should not and cannot be
substituted for substance; and again
in Francisco v. Permskul, where we
cautioned that expediency alone, no
matter how compelling, cannot excuse
non-compliance with the constitutional
requirements.

This is not to discourage the


lower courts to write abbreviated and
concise decisions, but never at the
expense of scholarly analysis, and
more significantly, of justice and fair
play, lest the fears expressed by
Justice Feria as the ponente in Romero
v. Court of Appeals come true, i.e., if
an appellate court failed to provide the
appeal the attention it rightfully
deserved, said court deprived the
appellant of due process since he was
accorded a fair opportunity to be
heard by a fair and responsible
magistrate. This situation becomes
more ominous in criminal cases, as in
this case, where not only property
rights are at stake but also the liberty
if not the life of a human being.
Faithful adherence to the requirements
of Section 14, Article VIII of the
Constitution
is
indisputably
a
paramount component of due process
and fair play. It is likewise demanded
by the due process clause of the
Constitution.
The parties to a
litigation should be informed of how it
was decided, with an explanation of
the factual and legal reasons that led
to the conclusions of the court. The
court cannot simply say that judgment
is rendered in favor of X and against Y
and just leave it at that without any
justification whatsoever for its action.
The losing party is entitled to know
why he lost, so he may appeal to the
higher court, if permitted, should he
believe that the decision should be
reversed. A decision that does not
clearly and distinctly state the facts
and the law on which it is based
leaves the parties in the dark as to
how it was reached and is precisely
prejudicial to the losing party, who is
unable to pinpoint the possible errors
of the court for review by a higher
tribunal.
More than that, the
requirement is an assurance to the
parties that, in reaching judgment, the
judge did so through the processes of
legal reasoning.
It is, thus, a
safeguard against the impetuosity of
the judge, preventing him from
deciding ipse dixit.
Vouchsafed
neither the sword nor the purse by the
Constitution but nonetheless vested
with the sovereign prerogative of
passing judgment on the life, liberty or
property of his fellowmen, the judge
must ultimately depend on the power
of
reason
for
sustained
public
confidence in the justness of his
decision.
Thus the Court has struck down as
void, decisions of lower courts and

even of the Court of Appeals whose


careless disregard of the constitutional
behest exposed their sometimes
cavalier attitude not only to their
magisterial
responsibilities
but
likewise to their avowed fealty to the
Constitution.
Thus, we nullified or deemed to have
failed to comply with Section 14,
Article VIII of the Constitution, a
decision, resolution or order which:
contained no analysis of the evidence
of the parties nor reference to any
legal basis in reaching its conclusions;
contained nothing more than a
summary of the testimonies of the
witnesses of both parties; convicted
the accused of libel but failed to cite
any legal authority or principle to
support conclusions that the letter in
question was libelous; consisted
merely of one (1) paragraph with
mostly sweeping generalizations and
failed to support its conclusion of
parricide; consisted of five (5) pages,
three (3) pages of which were
quotations from the labor arbiters
decision including the dispositive
portion and barely a page (two [2]
short paragraphs of two [2] sentences
each) of its own discussion or
reasonings; was merely based on the
findings of another court sans
transcript of stenographic notes, or
failed to explain the factual and legal
bases for the award of moral
damages.
In the same vein do we strike down as
a nullity the RTC decision in question.
(Yao v. Court of Appeals, 344
SCRA 202, Oct. 24, 2000, 1st Div.
[Davide])
121. Does the period for decision
making under Section 15, Article VIII,
1987 Constitution, apply to the
Sandiganbayan? Explain.
Held: The above provision does not
apply to the Sandiganbayan.
The
provision refers to regular courts of
lower collegiate level that in the
present hierarchy applies only to the
Court of Appeals.
The Sandiganbayan is a special
court of the same level as the Court of
Appeals and possessing all the
inherent powers of a court of justice,
with functions of a trial court.
Thus, the Sandiganbayan is not
a regular court but a special one. The
Sandiganbayan
was
originally
empowered to promulgate its own

rules of procedure.
However, on
March 30, 1995, Congress repealed
the
Sandiganbayans
power
to
promulgate its own rules of procedure
and instead prescribed that the Rules
of Court promulgated by the Supreme
Court shall apply to all cases and
proceedings
filed
with
the
Sandiganbayan.
Special courts are judicial tribunals
exercising limited jurisdiction over
particular or specialized categories of
actions. They are the Court of Tax
Appeals, the Sandiganbayan, and the
Sharia Courts. (Supra, Note 23, at p.
8)
Under Article VIII, Section 5[5]
of the Constitution Rules of procedure
of special courts and quasi-judicial
bodies shall remain effective unless
disapproved by the Supreme Court.
In
his
report,
the
Court
Administrator
would
distinguish
between
cases
which
the
Sandiganbayan has cognizance of in
its original jurisdiction, and cases
which fall within the appellate
jurisdiction of the Sandiganbayan.
The Court Administrator posits that
since in the first class of cases, the
Sandiganbayan acts more as a trial
court, then for that classification of
cases,
the
three
[3]
month
reglementary period applies. For the
second
class
of
cases,
the
Sandiganbayan has the twelve-month
reglementary period for collegiate
courts. We do not agree.
The
law
creating
the
Sandiganbayan, P.D. No. 1606 is clear
on this issue. It provides:
Sec.
6.
Maximum
period
for
termination of cases As far as
practicable, the trial of cases before
the Sandiganbayan once commenced
shall be continuous until terminated
and the judgment shall be rendered
within three [3] months from the date
the case was submitted for decision.
On September 18, 1984, the
Sandiganbayan promulgated its own
rules, thus:
Sec. 3. Maximum Period to Decide
Cases The judgment or final order of
a division of the Sandiganbayan shall
be rendered within three [3] months
from the date the case was submitted
for decision.

Given the clarity of the rule that does


not distinguish, we hold that the three
[3] month period, not the twelve [12]
month period, to decide cases applies
to the Sandiganbayan. Furthermore,
the Sandiganbayan presently sitting in
five [5] divisions, functions as a trial
court. The term trial is used in its
broad sense, meaning, it allows
introduction of evidence by the parties
in the cases before it.
The
Sandiganbayan, in original cases
within its jurisdiction, conducts trials,
has the discretion to weigh the
evidence of the parties, admit the
evidence it regards as credible and
reject that which they consider
perjurious or fabricated.
(Re:
Problem of Delays in Cases Before
the Sandiganbayan, A.M. No. 008-05-SC, Nov. 28, 2001, En Banc
[Pardo])
CONSTITUTIONAL LAW
122. What is the effect of declaration
of unconstitutionality of a law?
Illustrative case.
Held: Respondents are seeking
a reconsideration of the Courts 25
January 2000 decision, wherein we
declared section 8 of Republic Act No.
8551 (RA 8551) to be violative of
petitioners constitutionally mandated
right to security of tenure.
As a
consequence of our ruling, we held
that
petitioners
removal
as
commissioners of the National Police
Commission (NAPOLCOM) and the
appointment of new Commissioners in
their stead were nullities and ordered
the reinstatement of petitioners and
the payment of full backwages to be
computed from the date they were
removed from office.
Xxx
An unconstitutional act is not a law; it
confers no rights, imposes no duties,
and affords no protection (Fernandez
v. Cuerva, 21 SCRA 1095 [1967]).
Therefore,
the
unavoidable
consequence
of
the
Courts
declaration that section 8 of RA 8551
violates the fundamental law is that all
acts done pursuant to such provision
shall be null and void, including the
removal of petitioners and Adiong
from their positions in the NAPOLCOM
and
the
appointment
of
new
commissioners in their stead. When a
regular government employee is

illegally dismissed, his position does


not become vacant and the new
appointment made in order to replace
him is null and void ab initio (Aquino v.
Civil Service Commission, 208 SCRA
240 [1992]).
Rudimentary is the
precept that there can be no valid
appointment to a non-vacant position
(Garces v. Court of Appeals, 259 SCRA
99 [1996]).
Accordingly, Adiongs
appointment on 11 March 1998 for a
term of two years, pursuant to section
8 of RA 8551, is null and void.
However, he should now be permitted
to enjoy the remainder of his term
under RA 6975. Therefore, based on
our
foregoing
disquisition,
there
should no longer be any doubt as to
the proper execution of our 25 January
2000 decision all the Commissioners
appointed under RA 8551 should be
removed from office, in order to give
way
to
the
reinstatement
of
petitioners and respondent Adiong.
(Canonizado v. Aguirre, 351 SCRA
659, Feb. 15, 2001, En Banc
[Gonzaga-Reyes])

123. Discuss
the
Void
for
Vagueness Doctrine, and why is it
repugnant
to
the
Constitution.
Distinguish a perfectly vague act
from legislation couched in imprecise
language.
Held: 1. Due process requires that
the terms of a penal statute must be
sufficiently explicit to inform those
who are subject to it what conduct on
their part will render them liable to its
penalties
(Connally
v.
General
Construction Co., 269 US 385, 70 L Ed
322 46 S Ct 126 [1926]). A criminal
statute that fails to give a person of
ordinary intelligence fair notice that
his contemplated conduct is forbidden
by the statute, or is so indefinite that
it encourages arbitrary and erratic
arrests and convictions, is void for
vagueness (Colautti v. Franklin, 439 US
379, 58 L Ed 2d 596, 99 S Ct 675
[1979]). The constitutional vice in a
vague or indefinite statute is the
injustice to the accused in placing him
on trial for an offense, the nature of
which he is given no fair warning

(American Communications Asso. v.


Douds, 339 US 382, 94 L Ed 925, 70 S
Ct 674 [1950])
We reiterated these principles in
People v. Nazario (165 SCRA 186
[1988]):
As a rule, a statute or act may be said
to
be
vague
when
it
lacks
comprehensible standards that men
of
common
intelligence
must
necessarily guess at its meaning and
differ as to its application.
It is
repugnant to the Constitution in two
respects: (1) it violates due process for
failure to accord persons, especially
the parties targeted by it, fair notice of
the conduct to avoid; and (2) it leaves
law enforcers unbridled discretion in
carrying out its provisions and become
an arbitrary flexing of the Government
muscle.
We added, however, that:
X x x the act must be utterly vague on
its face, that is to say, it cannot be
clarified by either a saving clause or
by construction. Thus, in Coates v.
City of Cincinnati, the U.S. Supreme
Court struck down an ordinance that
had made it illegal for three or more
persons to assemble on any sidewalk
and there conduct themselves in a
manner annoying to persons passing
by. Clearly, the ordinance imposed
no standard at all because one may
never know in advance what annoys
some people but does not annoy
others.
Coates highlights what has been
referred to as a perfectly vague act
whose obscurity is evident on its face.
It is to be distinguished, however, from
legislation
coached
in
imprecise
language but which nonetheless
specifies
a
standard
though
defectively phrased in which case, it
may
be
saved
by
proper
construction. X x x (People v. Dela
Piedra, 350 SCRA 163, Jan. 24,
2001, 1st Div. [Kapunan])

2.
The doctrine has been
formulated in various ways, but is
commonly stated to the effect that a
statute establishing a criminal offense
must define the offense with sufficient
definiteness that persons of ordinary
intelligence can understand what
conduct is prohibited by the statute. It
can only be invoked against that
specie of legislation that is utterly
vague on its face, i.e., that which
cannot be clarified either by a saving
clause or by construction.
A statute or act may be said to
be
vague
when
it
lacks
comprehensible standards that men of
common intelligence must necessarily
guess at its meaning and differ in its
application.
In such instance, the
statute
is
repugnant
to
the
Constitution in two (2) respects it
violated due process for failure to
accord persons, especially the parties
targeted by it, fair notice of what
conduct to avoid; and, it leaves law
enforcers unbridled discretion in
carrying out its provisions and
becomes an arbitrary flexing of the
Government muscle (See People v.
Nazario, No. L-44143, 31 August 1988,
165 SCRA 186, 195-196).
But the
doctrine does not apply as against
legislations that are merely couched in
imprecise
language
but
which
nonetheless specify a standard though
defectively phrased; or to those that
are apparently ambiguous yet fairly
applicable
to
certain
types
of
activities. The first may be saved by
proper
construction,
while
no
challenge may be mounted as against
the second whenever directed against
such activities (Ibid.)
With more
reason, the doctrine cannot be
invoked where the assailed statute is
clear and free from ambiguity, as in
this case.
The test in determining whether
a criminal statute is void for
uncertainty is whether the language
conveys a sufficiently definite warning
as to the proscribed conduct when
measured by common understanding
and practice (State v. Hill, 189 Kan

403, 369 P2d 365, 91 ALR 2d 750). It


must be stressed, however, that the
vagueness doctrine merely requires
a reasonable degree of certainty for
the statute to be upheld not absolute
precision or mathematical exactitude,
as petitioner seems to suggest.
Flexibility, rather than meticulous
specificity, is permissible as long as
the metes and bounds of the statute
are clearly delineated. An act will not
be held invalid merely because it
might have been more explicit in its
wordings or detailed in its provisions,
especially where, because of the
nature of the act, it would be
impossible to provide all the details in
advance as in all other statutes.
(Joseph
Ejercito
Estrada
v.
Sandiganbayan [Third Division],
G.R. No. 148560, Nov. 19, 2001,
En Banc [Bellosillo])
124. Does Article 13 (b) of the Labor
Code
defining
recruitment
and
placement violate the due process
clause?
Held: In support of her submission
that Article 13 (b) is void for
vagueness, appellant invokes People v.
Panis (142 SCRA 664 [1986]), where
this Court x x x criticized the
definition
of
recruitment
and
placement x x x.
Appellant further argues that the acts
that constitute recruitment and
placement suffer from overbreadth
since by merely referring a person
for employment, a person may be
convicted of illegal recruitment.
These
contentions
sustained.

cannot

be

Appellants reliance on People v. Panis


is misplaced. The issue in Panis was
whether, under the proviso of Article
13(b), the crime of illegal recruitment
could be committed only whenever
two or more persons are in any
manner promised or offered any
employment for a fee. The Court
held in the negative x x x.

X x x The Court, in Panis, merely


bemoaned the lack of records that
would help shed light on the meaning
of the proviso. The absence of such
records notwithstanding, the Court
was able to arrive at a reasonable
interpretation of the proviso by
applying principles in criminal law and
drawing from the language and intent
of the law itself.
Section 13(b),
therefore, is not a perfectly vague
act whose obscurity is evident on its
face. If at all, the proviso therein is
merely couched in imprecise language
that
was
salvaged
by
proper
construction.
It is not void for
vagueness.
Xxx
That Section 13(b) encompasses what
appellant apparently considers as
customary and harmless acts such as
labor
or
employment
referral
(referring
an
applicant,
for
employment
to
a
prospective
employer) does not render the law
overbroad.
Evidently,
appellant
misapprehends
concept
of
overbreadth.
A statute may be said to be overbroad
where it operates to inhibit the
exercise
of
individual
freedoms
affirmatively
guaranteed
by
the
Constitution, such as the freedom of
speech or religion. A generally worded
statute, when construed to punish
conduct
which
cannot
be
constitutionally
punished
is
unconstitutionally vague to the extent
that it fails to give adequate warning
of
the
boundary
between
the
constitutionally permissible and the
constitutionally
impermissible
applications of the statute (Wright v.
Georgia, 373 US 284, 10 L Ed 2d 349,
83 S Ct 1240 [1963]).
In Blo Umpar Adiong v. Commission on
Elections (207 SCRA 712 [1992]), for
instance, we struck down as void for
overbreadth provisions prohibiting the
posting of election propaganda in any
place including private vehicles
other than in the common poster

areas sanctioned by the COMELEC.


We held that the challenged provisions
not only deprived the owner of the
vehicle the use of his property but also
deprived the citizen of his right to free
speech
and
information.
The
prohibition in Adiong, therefore, was
so broad that it covered even
constitutionally guaranteed rights and,
hence, void for overbreadth. In the
present case, however, appellant did
not even specify what constitutionally
protected freedoms are embraced by
the definition of recruitment and
placement that would render the
same
constitutionally
overbroad.
(People v. Dela Piedra, 350 SCRA
163, Jan. 24, 2001, 1st Div.
[Kapunan])
125. Is
the
Plunder
Law
unconstitutional for being vague?
Held:
As it is written, the
Plunder Law contains ascertainable
standards
and
well-defined
parameters which would enable the
accused to determine the nature of his
violation.
Section 2 is sufficiently
explicit in its description of the acts,
conduct and conditions required or
forbidden,
and
prescribes
the
elements of the crime with reasonable
certainty and particularity. X x x
As long as the law affords some
comprehensible guide or rule that
would inform those who are subject to
it what conduct would render them
liable to its penalties, its validity would
be sustained.
It must sufficiently
guide the judge in its application; the
counsel, in defending one charged
with
its
violation;
and
more
importantly,
the
accused,
in
identifying the realm of the proscribed
conduct. Indeed, it can be understood
with little difficulty that what the
assailed statute punishes is the act of
a public officer in amassing or
accumulating ill-gotten wealth of at
least P50,000,000.00 through a series
or combination of acts enumerated in
Sec. 1, par. (d), of the Plunder Law.

In
fact,
the
amended
Information itself closely tracks the
language of the law, indicating with
reasonable certainty the various
elements of the offense which
petitioner
is
alleged
to
have
committed x x x.
We discern nothing in the
foregoing that is vague or ambiguous
as there is obviously none that will
confuse petitioner in his defense.
Although subject to proof, these
factual assertions clearly show that
the elements of the crime are easily
understood and provide adequate
contrast between the innocent and the
prohibited
acts.
Upon
such
unequivocal assertions, petitioner is
completely
informed
of
the
accusations against him as to enable
him to prepare for an intelligent
defense.
Petitioner, however, bewails the
failure of the law to provide for the
statutory definition of the terms
combination and series in the key
phrase a combination or series of
overt or criminal acts found in Sec. 1,
par. (d), and Sec. 2, and the word
pattern in Sec. 4. These omissions,
according to petitioner, render the
Plunder Law unconstitutional for being
impermissibly vague and overbroad
and deny him the right to be informed
of the nature and cause of the
accusation
against
him,
hence,
violative of his fundamental right to
due process.
The rationalization seems to us
to be pure sophistry. A statute is not
rendered uncertain and void merely
because general terms are used
therein, or because of the employment
of terms without defining them (82
C.J.S. 68, P. 113; People v. Ring, 70
P.2d 281, 26 Cal. App. 2d Supp. 768);
much less do we have to define every
word we use. Besides, there is no
positive constitutional or statutory
command requiring the legislature to
define each and every word in an
enactment. Congress is not restricted
in the form of expression of its will,

and its inability to so define the words


employed in a statute will not
necessarily result in the vagueness or
ambiguity of the law so long as the
legislative will is clear, or at least, can
be gathered from the whole act, which
is distinctly expressed in the Plunder
Law.
Moreover, it is a well-settled
principle of legal hermeneutics that
words of a statute will be interpreted
in their natural, plain and ordinary
acceptation and signification (Mustang
Lumber, Inc. v. Court of Appeals, G.R.
No. 104988, 18 June 1965, 257 SCRA
430, 448), unless it is evident that the
legislature intended a technical or
special legal meaning to those words
(PLDT v. Eastern Telecommunications
Phil., Inc., G.R. No. 943774, 27 August
1992, 213 SCRA 16, 26). The intention
of the lawmakers who are, ordinarily,
untrained
philologists
and
lexicographers to use statutory
phraseology in such a manner is
always presumed.
Thus, Websters
New Collegiate Dictionary contains the
following
commonly
accepted
definition of the words combination
and series.
Combination the result or product of
combining; the act or process of
combining. To combine is to bring into
such close relationship as to obscure
individual characters.
Series a number of things or events
of the same class coming one after
another in spatial and temporal
succession.
That Congress intended the
words combination and series to
be understood in their popular
meanings is pristinely evident from
the legislative deliberations on the bill
which eventually became RA 7080 or
the Plunder Law x x x.
Xxx
Thus when the Plunder Law
speaks of combination, it is referring
to at least two (2) acts falling under

different categories or enumeration


provided in Sec. 1, par. (d), e.g., raids
on the public treasury in Sec. 1, par.
(d), subpar. (1), and fraudulent
conveyance of assets belonging to the
National Government under Sec. 1,
par. (d), subpar. (3).
On the other hand, to constitute
a series there must be two (2) or
more overt or criminal acts falling
under
the
same
category
of
enumeration found in Sec. 1, par. (d),
say, misappropriation, malversation
and raids on the public treasury, all of
which fall under Sec. 1, par. (d),
subpar. (1). Verily, had the legislature
intended a technical or distinctive
meaning
for
combination
and
series, it would have taken greater
pains in specifically providing for it in
the law.
As for pattern, we agree with
the
observations
of
the
Sandiganbayan that this term is
sufficiently defined in Sec. 4, in
relation to Sec. 1, par. (d), and Sec. 2
x x x under Sec. 1 (d) of the law, a
pattern consists of at least a
combination or series of overt or
criminal
acts
enumerated
in
subsections (1) to (6) of Sec. 1
(d). Secondly, pursuant to Sec. 2 of
the law, the pattern of overt or
criminal acts is directed towards a
common purpose or goal which is
to enable the public officer to
amass, accumulate or acquire illgotten wealth. And thirdly, there
must either be an overall unlawful
scheme or conspiracy to achieve
said common goal.
As commonly
understood, the term overall unlawful
scheme indicates a general plan of
action or method which the principal
accused and public officer and others
conniving with him follow to achieve
the aforesaid common goal. In the
alternative, if there is no such overall
scheme or where the schemes or
methods used by multiple accused
vary, the overt or criminal acts must
form part of a conspiracy to attain a
common goal.

Xxx
Hence,
it
cannot
plausibly
be
contended that the law does not give
a fair warning and sufficient notice of
what it seeks to penalize. Under the
circumstances, petitioners reliance on
the void-for-vagueness doctrine is
manifestly misplaced.
Xxx
Moreover, we agree with, hence
we adopt, the observations of Mr.
Justice Vicente V. Mendoza during the
deliberations of the Court that the
allegations that the Plunder Law is
vague and overbroad do not justify a
facial review of its validity
The void-for-vagueness doctrine states
that a statute which either forbids or
requires the doing of an act in terms
so vague that men of common
intelligence must necessarily guess at
its meaning and differ as to its
application violates the first essential
of due process of law. (Connally v.
General Constr. Co., 269 U.S. 385,
391, 70 L. Ed. 328 [1926] cited in
Ermita-Malate
Hotel
and
Motel
Operators Assn. v. City Mayor, 20
SCRA 849, 867 [1967])
The
overbreadth doctrine, on the other
hand, decrees that a governmental
purpose may not be achieved by
means which sweep unnecessarily
broadly and thereby invade the area of
protected
freedoms.
(NAACP
v.
Alabama, 377 U.S. 288, 307, 12, 2 L.
Ed 325, 338 [1958]; Shelton v. Tucker,
364 U.S. 479, 5 L. Ed. 2d 231 [1960])
A facial challenge is allowed to be
made to a vague statute and to one
which is overbroad because of
possible
chilling
effect
upon
protected speech. The theory is that
[w]hen statutes regulate or proscribe
speech and no readily apparent
construction suggests itself as a
vehicle for rehabilitating the statutes
in
a
single
prosecution,
the
transcendent value to all society of
constitutionally protected expression

is deemed to justify allowing attacks


on overly broad statutes with no
requirement that the person making
the attack demonstrate that his own
conduct could not be regulated by a
statute drawn with narrow specificity.
(Gooding v. Wilson, 405 U.S. 518, 521,
31 L. Ed. 2d 408, 413 [1972] [internal
quotation marks omitted])
The
possible harm to society in permitting
some unprotected speed to go
unpunished is outweighed by the
possibility that the protected speech
of others may be deterred and
perceived grievances left to fester
because of possible inhibitory effects
of overly broad statutes.
This rationale does not apply to penal
statutes.
Criminal statutes have
general in terrorem effect resulting
from their very existence, and, if facial
challenge is allowed for this reason
alone, the State may well be
prevented from enacting laws against
socially harmful conduct. In the area
of criminal law, the law cannot take
chances as in the area of free speech.
The overbreadth and vagueness
doctrine then have special application
only to free speech cases. They are
inapt for testing the validity of penal
statutes. As the U.S. Supreme Court
put it, in an opinion by Chief Justice
Rehnquist, we have not recognized
an overbreadth doctrine outside the
limited
context
of
the
First
Amendment.
In Broadwick v.
Oklahoma (413 U.S. 601, 612-613, 37
L Ed. 2d 830, 840-841 [1973]), the
Court ruled that claims of facial
overbreadth have been entertained in
cases involving statutes which, by
their terms, seek to regulate only
spoken words and, again, that
overbreadth claims, if entertained at
all, have been curtailed when invoked
against ordinary criminal laws that are
sought to be applied to protected
conduct. For this reason, it has been
held that a facial challenge to a
legislative act is the most difficult
challenge to mount successfully, since
the challenger must establish that no
set of circumstances exists under

which the Act would be valid. (United


States v. Salerno, supra.) As for the
vagueness doctrine, it is said that a
litigant may challenge a statute on its
face only if it is vague in all its
possible applications. A plaintiff who
engages in some conduct that is
clearly proscribed cannot complain of
the vagueness of the law as applied to
the conduct of others. (Village of
Hoffman Estates v. Flipside, Hoffman
Estates, Inc., 455 U.S. 489, 494-95, 71
L Ed. 2d 362, 369 [1982])
In sum, the doctrines of strict scrutiny,
overbreadth, and vagueness are
analytical tools developed for testing
on their faces statutes in free
speech cases or, as they are called in
American law, First Amendment cases.
They cannot be made to do service
when what is involved is a criminal
statute. With respect to such statute,
the established rule is that one to
whom application of a statute is
constitutional will not be heard to
attack the statute on the ground that
impliedly it might also be taken as
applying to other persons or other
situations in which its application
might be unconstitutional. (United
States v. Raines, 362 U.S. 17, 21, 4 L.
Ed. 2d 524, 529 [1960].
The
paradigmatic
case
is
Yazoo
&
Mississippi Valley RR. v. Jackson
Vinegar Co., 226 U.S. 217, 57 l. Ed.
193 [1912]) As has been pointed out,
vagueness challenges in the First
Amendment context, like overbreadth
challenges typically produce facial
invalidation, while statutes found to be
vague as a matter of due process
typically are invalidated [only] as
applied to a particular defendant. (G.
Gunther & K. Sullivan, Constitutional
Law 1299 [2001])
Consequently,
there is no basis for petitioners claim
that this Court review the Anti-Plunder
Law on its face and in its entirety.
Indeed, on its face invalidation of
statutes results in striking them down
entirely on the ground that they might
be applied to parties not before the
Court
whose
activities
are
constitutionally protected
(Id. at

1328). It constitutes a departure from


the case and controversy requirement
of the Constitution and permits
decisions to be made without concrete
factual settings and in sterile abstract
contexts
(Constitution,
Art.
VIII,
Sections 1 and 5. Compare Angara v.
Electoral Commission, 63 Phil. 139,
158 [1936]). But, as the U.S. Supreme
Court pointed out in Younger v. Harris
(401 U.S. 37, 52-53, 27 L. Ed. 2d 669,
680 [1971]; others omitted.)
[T]he task of analyzing a proposed
statute, pinpointing its deficiencies,
and requiring correction of these
deficiencies before the statute is put
into effect, is rarely if ever an
appropriate task for the judiciary. The
combination
of
the
relative
remoteness of the controversy, the
impact on the legislative process of
the relief sought, and above all the
speculative and amorphous nature of
the required line-by-line analysis of
detailed statutes, x x x ordinarily
results in a kind of case that is wholly
unsatisfactory
for
deciding
constitutional questions, whichever
way they might be decided.
For these reasons, on its face
invalidation of statutes has been
described
as
manifestly
strong
medicine, to be employed sparingly
and only as a last resort, (Broadwick
v. Oklahoma, 413 U.S. at 613, 37
L.Ed.2d at 841; National Endowment
for the Arts v. Finley, 524 U.S. 569,
580
[1998])
and
is
generally
disfavored (FW/PBS, Inc. v. City of
Dallas, 493 U.S. 223, 107 L.Ed.2d 603
[1990];
Cruz
v.
Secretary
of
Environment and Natural Resources,
G.R. No. 135385, 6 December 2000
[Mendoza, J., Separate Opinion]). In
determining the constitutionality of a
statute, therefore, its provisions which
are alleged to have been violated in a
case must be examined in the light of
the conduct with which the defendant
is charged (United States v. National
Dairy Prod. Corp., 372 U.S. 29, 32-33,
9 L.Ed.2d 561, 565-6 [1963])

In light of the foregoing


disquisition, it is evident that the
purported ambiguity of the Plunder
Law, so tenaciously claimed and
argued at length by petitioner, is more
imagined than real. Ambiguity, where
none exists, cannot be created by
dissecting parts and words in the
statute to furnish support to critics
who cavil at the want of scientific
precision in the law. Every provision of
the law should be construed in relation
and with reference to every other part.
To be sure, it will take more than
nitpicking to overturn the wellentrenched
presumption
of
constitutionality and validity of the
Plunder Law.
A fortiori, petitioner
cannot feign ignorance of what the
Plunder Law is all about. Being one of
the Senators who voted for its
passage, petitioner must be aware
that
the
law
was
extensively
deliberated upon by the Senate and its
appropriate committees by reason of
which
he
even
registered
his
affirmative vote with full knowledge of
its legal implications and sound
constitutional anchorage.
(Joseph
Ejercito Estrada v. Sandiganbayan
[Third Division], G.R. No. 148560,
Nov.
19,
2001,
En
Banc
[Bellosillo])
A. THE INHERENT POWERS OF
THE STATE
Police Power
126. Define Police Power and clarify
its scope.
Held: 1. Police power is an inherent
attribute of sovereignty. It has been
defined as the power vested by the
Constitution in the legislature to make,
ordain, and establish all manner of
wholesome and reasonable laws,
statutes and ordinances, either with
penalties or without, not repugnant to
the Constitution, as they shall judge to
be for the good and welfare of the
commonwealth, and for the subjects of
the same. The power is plenary and
its scope is vast and pervasive,
reaching and justifying measures for
public health, public safety, public
morals, and the general welfare.

It bears stressing that police power is


lodged primarily in the National
Legislature. It cannot be exercised by
any group or body of individuals not
possessing legislative power.
The
National Legislature, however, may
delegate this power to the President
and administrative boards as well as
the lawmaking bodies of municipal
corporations or local government
units. Once delegated, the agents can
exercise only such legislative powers
as are conferred on them by the
national
lawmaking
body.
(Metropolitan
Manila
Development Authority v. Bel-Air
Village Association, Inc., 328 SCRA
836, 843-844, March 27, 2000, 1st
Div. [Puno])
2. The scope of police power has been
held to be so comprehensive as to
encompass
almost
all
matters
affecting the health, safety, peace,
order,
morals,
comfort
and
convenience of the community. Police
power is essentially regulatory in
nature and the power to issue licenses
or grant business permits, if exercised
for a regulatory and not revenueraising purpose, is within the ambit of
this power.
Xxx
[T]he issuance of business licenses
and permits by a municipality or city is
essentially regulatory in nature. The
authority, which devolved upon local
government units to issue or grant
such licenses or permits, is essentially
in the exercise of the police power of
the State within the contemplation of
the general welfare clause of the Local
Government
Code.
(Acebedo
Optical Company, Inc. v. Court of
Appeals, 329 SCRA 314, March 31,
2000, En Banc [Purisima])
127. Discuss the nature of the
authority of local government units to
issue or grant licenses or permits.
Held:
The issuance of business
licenses and permits by a municipality
or city is essentially regulatory in
nature. The authority, which devolved
upon local government units to issue
or grant such licenses or permits, is
essentially in the exercise of the police
power of the State within the
contemplation of the general welfare
clause of the Local Government Code.
(Acebedo Optical Company, Inc. v.
Court of Appeals, 329 SCRA 314,
March
31,
2000,
En
Banc
[Purisima])

128. How should laws that grant the


right to exercise a part of the police
power of the State be construed?
Held: Lest the idea gets lost in the
shoals of our subconsciousness, let us
not forget that PAGCOR is engaged in
business affected with public interest.
The phrase affected with public
interest means that an industry is
subject to control for the public good
(Nebbia v. New York, 291 U.S. 502); it
has been considered as the equivalent
of subject to the exercise of the
police power. (Bernas, The 1987
Constitution of the Republic of the
Philippines, A Commentary, 1996 ed.,
p. 1053)
Perforce, a legislative
franchise to operate jai-alai is imbued
with public interest and involves an
exercise of police power. The familiar
rule is that laws which grant the right
to exercise a part of the police power
of the state are to be construed
strictly and any doubt must be
resolved against the grant (People v.
Chicago, 103 N.E. 609; Slaughter v.
OBerry, 35 S.E. 241, 48 L.R.A. 442).
The legislature is regarded as the
guardian of society, and therefore is
not presumed to disable itself or
abandon the discharge of its duty.
Thus, courts do not assume that the
legislature intended to part away with
its power to regulate public morals
(Stone v. Mississippi, 101 U.S. 814).
The presumption is influenced by
constitutional
considerations.
Constitutions are widely understood to
withhold
from
legislatures
any
authority to bargain away their police
power
(Sutherland
Statutory
Construction, Vol. 3, 5th ed., p. 244)
for the power to protect the public
interest is beyond abnegation.
It is stressed that the case at
bar does not involve a franchise to
operate a public utility (such as water,
transportation,
communication
or
electricity) the operation of which
undoubtedly redounds to the benefit
of the general public. What is claimed
is an alleged legislative grant of a
gambling franchise a franchise to
operate jai-alai.
A statute which
legalizes a gambling activity or
business should be strictly construed
and every reasonable doubt must be
resolved to limit the powers and rights
claimed under its authority (Aicardi v.
Alabama, 22 L.Ed. 215; West Indies,
Inc. v. First National Bank, 214 P.2d
144). (Del Mar v. Philippine
Amusement
and
Gaming

Corporation, 346 SCRA 485, Nov.


29, 2000, En Banc [Puno])
129. Discuss why rates to be charged
by public utilities like MERALCO are
subject to State regulation.
Held: The regulation of rates to be
charged by public utilities is founded
upon the police power of the State and
statutes prescribing rules for the
control and regulations of public
utilities are a valid exercise thereof.
When private property is used for a
public purpose and is affected with
public interest, it ceases to be juris
privati only and becomes subject to
regulation.
The regulation is to
promote
the
common
good.
Submission to regulation may be
withdrawn
by
the
owner
by
discontinuing use; but as long as the
use of the property is continued, the
same is subject to public regulation
(Munn v. People of the State of Illinois,
94 U.S. 113, 126 [1877]).
In regulating rates charged by
public utilities, the State protects the
public against arbitrary and excessive
rates while maintaining the efficiency
and quality of services rendered.
However, the power to regulate rates
does not give the State the right to
prescribe rates which are so low as to
deprive the public utility of a
reasonable return on investment.
Thus, the rates prescribed by the
State must be one that yields a
fair return on the public utility
upon the value of the property
performing the service and one
that is reasonable to the public
for the service rendered (IV A.F.
Agbayani,
Commentaries
and
Jurisprudence on the Commercial Laws
of the Philippines 500 [1993]). The
fixing of just and reasonable rates
involves a balancing of the investor
and the consumer interests (Federal
Power Commission v. Hope Natural
Gas Co., 320 U.S. 591). (Republic of
the Philippines v. Manila Electric
Company, G.R. No. 141314, Nov.
15, 2002, 3rd Div. [Puno])

130. What powers of the State are


involved in the implementation of the
Comprehensive Agrarian Reform Law
(CARL)? Discuss.
Held:
The implementation of the
CARL is an exercise of the States
police power and the power of
eminent domain. To the extent that
the CARL prescribes retention limits to
the landowners, there is an exercise of
police power for the regulation of
private property in accordance with
the Constitution (Association of Small
Landowners in the Philippines v.
Secretary of Agrarian Reform, 175
SCRA 343, 373-374 [1989]).
But
where, to carry out such regulation,
the owners are deprived of lands they
own in excess of the maximum area
allowed, there is also a taking under
the power of eminent domain. The
taking contemplated is not a mere
limitation of the use of the land. What
is required is the surrender of the title
to and physical possession of the said
excess and all beneficial rights
accruing to the owner in favor of the
farmer beneficiary (Id.). The Bill of
Rights provides that [n]o person shall
be deprived of life, liberty or property
without due process of law. (Section
1, Article III, 1987 Constitution) The
CARL was not intended to take away
property without due process of law
(Development Bank of the Philippines
v. Court of Appeals, 262 SCRA 245,
253 [1996]).
The exercise of the
power of eminent domain requires
that due process be observed in the
taking of private property. (Roxas &
Co., Inc. v. Court of Appeals, 321
SCRA 106, Dec. 17, 1999, En Banc
[Puno])
131. Does Article 263(g) of the Labor
Code (vesting upon the Secretary of
Labor the discretion to determine what
industries are indispensable to the
national
interest
and
thereafter,
assume jurisdiction over disputes in
said industries) violate the workers
constitutional right to strike?
Held: Said article does not interfere
with the workers right to strike but
merely regulates it, when in the
exercise of such right, national
interests will be affected. The rights
granted by the Constitution are not
absolute.
They are still subject to
control and limitation to ensure that
they are not exercised arbitrarily. The
interests of both the employers and
the employees are intended to be

protected and not one of them is given


undue preference.
The Labor Code vests upon the
Secretary of Labor the discretion to
determine
what
industries
are
indispensable to national interest.
Thus, upon the determination of the
Secretary of Labor that such industry
is indispensable to the national
interest, it will assume jurisdiction
over the labor dispute of said industry.
The assumption of jurisdiction is in the
nature of police power measure. This
is done for the promotion of the
common good considering that a
prolonged strike or lockout can be
inimical to the national economy. The
Secretary of Labor acts to maintain
industrial
peace.
Thus,
his
certification for compulsory arbitration
is not intended to impede the workers
right to strike but to obtain a speedy
settlement of the dispute. (Philtread
Workers
Union
[PTWU]
v.
Confesor, 269 SCRA 393, March
12, 1997)
132. May solicitation for religious
purposes be subject to proper
regulation by the State in the exercise
of police power?
Held: The constitutional inhibition of
legislation on the subject of religion
has a double aspect.
On the one
hand, it forestalls compulsion by law of
the acceptance of any creed or the
practice of any form of worship.
Freedom of conscience and freedom to
adhere to such religious organization
or form of worship as the individual
may choose cannot be restricted by
law. On the other hand, it safeguards
the free exercise of the chosen form of
religion.
Thus, the Constitution
embraces two concepts, that is,
freedom to believe and freedom to
act. The first is absolute but, in the
nature of things, the second cannot
be.
Conduct remains subject to
regulation for the protection of society.
The freedom to act must have
appropriate definitions to preserve the
enforcement of that protection.
In
every case, the power to regulate
must be so exercised, in attaining a
permissible end, as not to unduly
infringe on the protected freedom.
Whence, even the exercise of religion
may be regulated, at some slight
inconvenience, in order that the State
may protect its citizens from injury.
Without doubt, a State may protect its
citizens from fraudulent solicitation by
requiring a stranger in the community,

before permitting him publicly to


solicit funds for any purpose, to
establish his identity and his authority
to act for the cause which he purports
to represent. The State is likewise free
to regulate the time and manner of
solicitation generally, in the interest of
public safety, peace, comfort, or
convenience.
It does not follow, therefore, from the
constitutional guarantees of the free
exercise of religion that everything
which may be so called can be
tolerated. It has been said that a law
advancing a legitimate governmental
interest is not necessarily invalid as
one interfering with the free exercise
of religion merely because it also
incidentally has a detrimental effect
on the adherents of one or more
religion. Thus, the general regulation,
in the public interest, of solicitation,
which does not involve any religious
test and does not unreasonably
obstruct or delay the collection of
funds, is not open to any constitutional
objection, even though the collection
be for a religious purpose.
Such
regulation would not constitute a
prohibited previous restraint on the
free exercise of religion or interpose
an inadmissible obstacle to its
exercise.
Even with numerous regulative laws in
existence, it is surprising how many
operations are carried on by persons
and associations who, secreting their
activities
under
the
guise
of
benevolent purposes, succeed in
cheating and defrauding a generous
public.
It is in fact amazing how
profitable the fraudulent schemes and
practices
are
to
people
who
manipulate them.
The State has
authority under the exercise of its
police power to determine whether or
not there shall be restrictions on
soliciting by unscrupulous persons or
for unworthy causes or for fraudulent
purposes.
That
solicitation
of
contributions under the guise of
charitable and benevolent purposes is
grossly abused is a matter of common
knowledge. Certainly the solicitation
of contributions in good faith for
worthy purposes should not be denied,
but somewhere should be lodged the
power to determine within reasonable
limits the worthy from the unworthy.
The
objectionable
practices
of
unscrupulous persons are prejudicial
to worthy and proper charities which
naturally suffer when the confidence
of the public in campaigns for the
raising of money for charity is

lessened
or
destroyed.
Some
regulation of public solicitation is,
therefore, in the public interest.
To conclude, solicitation for religious
purposes may be subject to proper
regulation by the State in the exercise
of police power. (Centeno v. VillalonPornillos, 236 SCRA 197, Sept. 1,
1994 [Regalado])
The Power of Eminent Domain
132.

What is Eminent Domain?

Held: 1. Eminent domain is the right


or power of a sovereign state to
appropriate
private
property
to
particular uses to promote public
welfare.
It is an indispensable
attribute of sovereignty; a power
grounded in the primary duty of
government to serve the common
need and advance the general
welfare. Thus, the right of eminent
domain
appertains
to
every
independent government without the
necessity
for
constitutional
recognition. The provisions found in
modern constitutions of civilized
countries relating to the taking of
property for the public use do not by
implication grant the power to the
government, but limit a power which
would otherwise be without limit.
Thus, our own Constitution provides
that [p]rivate property shall not be
taken for public use without just
compensation. (Art. III, Sec. 9).
Furthermore, the due process and
equal
protection
clauses
(1987
Constitution, Art. III, Sec. 1) act as
additional safeguards against the
arbitrary exercise of this governmental
power.
Since the exercise of the power of
eminent domain affects an individuals
right
to
private
property,
a
constitutionally-protected
right
necessary for the preservation and
enhancement of personal dignity and
intimately connected with the rights to
life and liberty, the need for its
circumspect operation cannot be
overemphasized. In City of Manila v.
Chinese Community of Manila we said
(40 Phil. 349 [1919):
The exercise of the right of eminent
domain, whether directly by the State,
or by its authorized agents, is
necessarily in derogation of private
rights, and the rule in that case is that
the
authority
must
be
strictly
construed. No species of property is
held by individuals with greater

tenacity, and none is guarded by the


Constitution and the laws more
sedulously, than the right to the
freehold of inhabitants.
When the
legislature interferes with that right,
and, for greater public purposes,
appropriates the land of ah individual
without his consent, the plain meaning
of the law should not be enlarged by
doubt[ful] interpretation. (Bensley v.
Mountainlake Water Co., 13 Cal., 306
and cases cited [73 Am. Dec., 576])
The statutory power of taking property
from the owner without his consent is
one of the most delicate exercise of
governmental authority. It is to be
watched
with
jealous
scrutiny.
Important as the power may be to the
government, the inviolable sanctity
which all free constitutions attach to
the right of property of the citizens,
constrains the strict observance of the
substantial provisions of the law which
are prescribed as modes of the
exercise of the power, and to protect it
from abuse x x x.
The power of eminent domain is
essentially legislative in nature. It is
firmly settled, however, that such
power may be validly delegated to
local government units, other public
entities and public utilities, although
the scope of this delegated legislative
power is necessarily narrower than
that of the delegating authority and
may only be exercised in strict
compliance with the terms of the
delegating law. (Heirs of Alberto
Suguitan v. City of Mandaluyong,
328 SCRA 137, 144-146, March 14,
2000, 3rd Div. [Gonzaga-Reyes])
2. Eminent domain is a fundamental
State power that is inseparable from
sovereignty. It is governments right
to appropriate, in the nature of a
compulsory sale to the State, private
property for public use or purpose.
Inherently possessed by the national
legislature, the power of eminent
domain may be validly delegated to
local
governments,
other
public
entities and public utilities. For the
taking of private property by the
government to be valid, the taking
must be for public purpose and there
must be just compensation. (Moday
v. Court of Appeals, 268 SCRA
586, February 20, 1997)
133. State some limitations on the
exercise of the power of Eminent
Domain.

Held: The limitations on the power of


eminent domain are that the use must
be public, compensation must be
made and due process of law must be
observed. The Supreme Court, taking
cognizance of such issues as the
adequacy of compensation, necessity
of the taking and the public use
character or the purpose of the taking,
has ruled that the necessity of
exercising eminent domain must be
genuine and of a public character.
Government may not capriciously
choose what private property should
be taken.
(Moday v. Court of
Appeals, 268 SCRA 586, February
20, 1997)
134. Discuss the expanded notion of
public
use
in
eminent
domain
proceedings.
Held:
The City of Manila, acting
through its legislative branch, has the
express power to acquire private lands
in the city and subdivide these lands
into home lots for sale to bona fide
tenants or occupants thereof, and to
laborers and low-salaried employees
of the city.
That only a few could actually benefit
from the expropriation of the property
does not diminish its public character.
It is simply not possible to provide all
at once land and shelter for all who
need them.
Corollary to the expanded notion of
public use, expropriation is not
anymore confined to vast tracts of
land and landed estates.
It is
therefore of no moment that the land
sought to be expropriated in this case
is less than half a hectare only.
Through the years, the public use
requirement in eminent domain has
evolved into a flexible concept,
influenced by changing conditions.
Public use now includes the broader
notion of indirect public benefit or
advantage, including in particular,
urban land reform and housing.
(Filstream
International
Incorporated v. CA, 284 SCRA 716,
Jan. 23, 1998 [Francisco])
135. The constitutionality of Sec. 92
of B.P. Blg. 881 (requiring radio and
television
station
owners
and
operators to give to the Comelec radio
and television time free of charge) was
challenged on the ground, among
others, that it violated the due process
clause and the eminent domain
provision of the Constitution by taking

airtime from radio and television


broadcasting
stations
without
payment
of
just
compensation.
Petitioners claim that the primary
source of revenue of radio and
television stations is the sale of
airtime to advertisers and that to
require these stations to provide free
airtime is to authorize a taking which
is not a de minimis temporary
limitation or restraint upon the use of
private property. Will you sustain the
challenge?
Held: All broadcasting, whether by
radio or by television stations, is
licensed by the government. Airwave
frequencies have to be allocated as
there are more individuals who want
to
broadcast
than
there
are
frequencies to assign. A franchise is
thus a privilege subject, among other
things, to amendment by Congress in
accordance with the constitutional
provision that any such franchise or
right granted x x x shall be subject to
amendment, alteration or repeal by
the Congress when the common good
so requires. (Art. XII, Sec. 11)
Indeed, provisions for Comelec Time
have been made by amendment of the
franchises of radio and television
broadcast stations and such provisions
have not been thought of as taking
property without just compensation.
Art. XII, Sec. 11 of the Constitution
authorizes
the
amendment
of
franchises for the common good.
What
better
measure
can
be
conceived for the common good than
one for free airtime for the benefit not
only of candidates but even more of
the public, particularly the voters, so
that they will be fully informed of the
issues in an election? [I]t is the right
of the viewers and listeners, not the
right of the broadcasters, which is
paramount.
Nor indeed can there be any
constitutional
objection
to
the
requirement that broadcast stations
give free airtime. Even in the United
States, there are responsible scholars
who believe that government controls
on
broadcast
media
can
constitutionally be instituted to ensure
diversity of views and attention to
public affairs to further the system of
free expression.
For this purpose,
broadcast stations may be required to
give free airtime to candidates in an
election.
In
truth,
radio
and
television
broadcasting companies, which are

given franchises, do not own the


airwaves and frequencies through
which they transmit broadcast signals
and images. They are merely given
the temporary privilege of using them.
Since a franchise is a mere privilege,
the exercise of the privilege may
reasonably be burdened with the
performance by the grantee of some
form of public service.
In the granting of the privilege to
operate
broadcast
stations
and
thereafter supervising radio and
television stations, the State spends
considerable public funds in licensing
and supervising such stations.
It
would be strange if it cannot even
require the licensees to render public
service by giving free airtime.
The claim that petitioner would be
losing P52,380,000.00 in unrealized
revenue from advertising is based on
the assumption that airtime is
finished product which, it is said,
become the property of the company,
like oil produced from refining or
similar
natural
resources
after
undergoing a process for their
production.
As held in Red Lion
Broadcasting Co. v. F.C.C. (395 U.S. at
394, 23 L. Ed. 2d at 391, quoting 47
U.S.C. Sec. 301), which upheld the
right of a party personally attacked to
reply, licenses to broadcast do not
confer
ownership
of
designated
frequencies, but only the temporary
privilege
of
using
them.
Consequently, a license permits
broadcasting, but the licensee has no
constitutional right to be the one who
holds the license or to monopolize a
radio frequency to the exclusion of his
fellow citizens. There is nothing in the
First Amendment which prevents the
government from requiring a licensee
to share his frequency with others and
to conduct himself as a proxy or
fiduciary with obligations to present
those views and voices which are
representative of his community and
which would otherwise, by necessity,
be barred from the airwaves.
As
radio and television broadcast stations
do not own the airwaves, no private
property is taken by the requirement
that they provide airtime to the
Comelec.
(TELEBAP,
Inc.
v.
COMELEC, 289 SCRA 337, April 21,
1998 [Mendoza])
136. May eminent domain be barred
by "res judicata" or "law of the case"?
Held: The principle of res judicata,
which finds application in generally all

cases and proceedings, cannot bar the


right of the State or its agents to
expropriate private property. The very
nature of eminent domain, as an
inherent power of the State, dictates
that the right to exercise the power be
absolute and unfettered even by a
prior judgment or res judicata. The
scope of eminent domain is plenary
and, like police power, can reach
every form of property which the State
might need for public use.
All
separate interests of individuals in
property are held of the government
under this tacit agreement or implied
reservation.
Notwithstanding the
grant to individuals, the eminent
domain, the highest and most exact
idea of property, remains in the
government, or in the aggregate body
of the people in their sovereign
capacity; and they have the right to
resume the possession of the property
whenever the public interest requires
it. Thus, the State or its authorized
agent cannot be forever barred from
exercising said right by reason alone
of previous non-compliance with any
legal requirement.
While the principle of res judicata does
not denigrate the right of the State to
exercise eminent domain, it does
apply to specific issues decided in a
previous case. For example, a final
judgment dismissing an expropriation
suit on the ground that there was no
prior offer precludes another suit
raising the same issue; it cannot,
however, bar the State or its agent
from thereafter complying with this
requirement, as prescribed by law, and
subsequently exercising its power of
eminent domain over the same
property.
(Municipality
of
Paranaque
v.
V.M.
Realty
Corporation, 292 SCRA 678, July
20, 1998 [Panganiban])
137. Discuss how expropriation may
be initiated, and the two stages in
expropriation.
Held: Expropriation may be initiated
by court action or by legislation. In
both instances, just compensation is
determined by the courts (EPZA v.
Dulay, 149 SCRA 305 [1987]).
The expropriation of lands consists of
two stages.
As explained in
Municipality of Binan v. Garcia (180
SCRA 576, 583-584 [1989], reiterated
in National Power Corp. v. Jocson, 206
SCRA 520 [1992]):

The first is concerned with the


determination of the authority of the
plaintiff to
exercise the power of
eminent domain and the propriety of
its exercise in the context of the facts
involved in the suit. It ends with an
order, if not dismissal of the action, "of
condemnation declaring that the
plaintiff has a lawful right to take the
property sought to be condemned, for
the public use or purpose declared in
the complaint, upon the payment of
just compensation to be determined
as of the date of the filing of the
complaint" x x x.
The second phase of the eminent
domain action is concerned with the
determination by the court of "the just
compensation for the property sought
to be taken." This is done by the court
with the assistance of not more than
three (3) commissioners x x x.
It is only upon the completion of these
two stages that expropriation is said to
have been completed. Moreover, it is
only
upon
payment
of
just
compensation that title over the
property passes to the government.
Therefore,
until
the
action
for
expropriation has been completed and
terminated,
ownership
over
the
property being expropriated remains
with
the
registered
owner.
Consequently, the latter can exercise
all rights pertaining to an owner,
including the right to dispose of his
property, subject to the power of the
State ultimately to acquire it through
expropriation. (Republic v. Salem
Investment Corporation, et. al.,
G.R. No. 137569, June 23, 2000,
2nd Div. [Mendoza])
138. Does the two (2) stages in
expropriation apply only to judicial,
and not to legislative, expropriation?
Held: The De la Ramas are mistaken
in arguing that the two stages of
expropriation x x x only apply to
judicial, and not to legislative,
expropriation. Although Congress has
the power to determine what land to
take, it can not do so arbitrarily.
Judicial determination of the propriety
of the exercise of the power, for
instance, in view of allegations of
partiality and prejudice by those
adversely affected, and the just
compensation for the subject property
is provided in our constitutional
system.
We see no point in distinguishing
between
judicial
and
legislative
expropriation as far as the two stages

mentioned above are concerned. Both


involve these stages and in both the
process is not completed until
payment of just compensation is
made.
The Court of Appeals was
correct in saying that B.P. Blg. 340 did
not effectively expropriate the land of
the De la Ramas. As a matter of fact,
it
merely
commenced
the
expropriation of the subject property.
Xxx
The De la Ramas make much of the
fact that ownership of the land was
transferred
to
the
government
because the equitable and the
beneficial title was already acquired
by it in 1983, leaving them with only
the naked title. However, as this Court
held
in
Association
of
Small
Landowners in the Phil., Inc. v.
Secretary of Agrarian Reform (175
SCRA 343, 389 [1989]):
The recognized rule, indeed, is that
title to the property expropriated shall
pass
from
the
owner
to
the
expropriator only upon full payment of
the just compensation. Jurisprudence
on this settled principle is consistent
both here and in other democratic
jurisdictions. X x x
(Republic v. Salem Investment
Corporation, et. al., G.R. No.
137569, June 23, 2000, 2nd Div.
[Mendoza])
139. Is prior unsuccessful negotiation
a condition precedent for the exercise
of eminent domain?
Held:
Citing Iron and Steel
Authority v. Court of Appeals (249
SCRA 538, October 25, 1995),
petitioner insists that before eminent
domain may be exercised by the state,
there must be a showing of prior
unsuccessful negotiation with the
owner of the property to be
expropriated.
This contention is not correct.
As pointed out by the Solicitor General
the current effective law on delegated
authority to exercise the power of
eminent domain is found in Section
12,
Book
III
of
the
Revised
Administrative Code, which provides:
SEC. 12. Power of Eminent Domain
The President shall determine when it
is necessary or advantageous to
exercise the power of eminent domain
in behalf of the National Government,
and direct the Solicitor General,
whenever he deems the action

advisable, to institute expropriation


proceedings in the proper court.
The foregoing provision does
not
require
prior
unsuccessful
negotiation as a condition precedent
for the exercise of eminent domain. In
Iron and Steel Authority v. Court of
Appeals, the President chose to
prescribe
this
condition
as
an
additional requirement instead. In the
instant case, however, no such
voluntary restriction was imposed.
(SMI Development Corporation v.
Republic, 323 SCRA 862, Jan. 28,
2000, 3rd Div. [Panganiban])

140. Discuss the nature of the right


of eminent domain and the limitations
thereof.
Held:
The right of eminent
domain is usually understood to be an
ultimate right of the sovereign power
to appropriate any property within its
territorial sovereignty for a public
purpose (Bernas, 1987 Edition, p. 276,
quoting Justice Story in Charles River
Bridge
v.
Warren
Bridge).
Fundamental to the independent
existence of a State, it requires no
recognition by the Constitution, whose
provisions are taken as being merely
confirmatory of its presence and as
being regulatory, at most, in the due
exercise of the power. In the hands of
the legislature, the power is inherent,
its scope matching that of taxation,
even that of police power itself, in
many respects. It reaches to every
form of property the State needs for
public use and, as an old case so puts
it, all separate interests of individuals
in property are held under a tacit
agreement or implied reservation
vesting upon the sovereign the right to
resume the possession of the property
whenever the public interest so
requires it (US v. Certain Lands in
Highlands [DY NY] 48 F Supp 306).
The ubiquitous character of
eminent domain is manifest in the
nature
of
the
expropriation
proceedings.
Expropriation
proceedings are not adversarial in the
conventional
sense,
for
the

condemning authority is not required


to assert any conflicting interest in the
property. Thus, by filing the action,
the condemnor in effect merely serves
notice that it is taking title and
possession of the property, and the
defendant asserts title or interest in
the property, not to prove a right to
possession, but to prove a right to
compensation for the taking (US v.
Certain Lands in Highlands [DY NY] 48
F Supp 306; San Bernardino Valley
Municipal Water District v. Gage Canal
Co. [4th Dist] Cal App 2d 206, 37 Cal
Rptr 856).
Obviously, however, the power
is not without its limits: first, the
taking must be for public use, and
second, that just compensation must
be given to the private owner of the
property (Sena v. Manila Railroad Co.,
42 Phil. 102). These twin proscriptions
have their origin in the recognition of
the necessity for achieving balance
between the State interests, on the
one hand, and private rights, upon the
other hand, by effectively restraining
the former and affording protection to
the latter (Visayan Refining Co. v.
Camus, 40 Phil. 550). In determining
public use, two approaches are
utilized

the
first
is
public
employment or the actual use by the
public, and the second is public
advantage
or
benefit
(Thornton
Development authority v. Upah [DC
Colo] 640 F Supp 1071). It is also
useful to view the matter as being
subject to constant growth, which is to
say that as society advances, its
demands upon the individual so
increases, and each demand is a new
use to which the resources of the
individual may be devoted (Visayan
Refining, supra). (Republic of the
Philippines v. The Hon. Court of
Appeals, G.R. No. 146587, July 2,
2002, 1st Div. [Vitug])
141. What is the meaning of public
use in eminent domain proceedings?
Illustrative case.
Held:
This Court holds that
respondent
(Philippine
Export
Processing Zone) has the legal

authority to expropriate the subject


Lot 1406-B and that the same was for
a valid public purpose. In Sumulong v.
Guerrero (154 SCRA 461, 467-468
[1987]), this Court has ruled that,
The public use requirement for a
valid exercise of the power of eminent
domain is a flexible and evolving
concept
influenced
by
changing
conditions.
In this jurisdiction, the
statutory and judicial trend has been
summarized as follows:
This Court has ruled that the taking to
be valid must be for public use. There
was a time when it was felt that a
literal meaning should be attached to
such a requirement. Whatever project
is undertaken must be for the public to
enjoy, as in the case of streets or
parks. Otherwise, expropriation is not
allowable. It is not anymore. As long
as the purpose of the taking is public,
then the power of eminent domain
comes into play . . . It is accurate to
state then that at present whatever
may be beneficially employed for the
general
welfare
satisfies
the
requirement of public use. (Heirs of
Juancho Ardona v. Reyes, 125 SCRA
220 [1983] at 234-235 quoting E.
Fernando, the Constitution of the
Philippines 523-4 [2nd Ed. 1977])
The term public use has acquired a
more comprehensive coverage. To the
literal import of the term signifying
strict use or employment by the public
has been added the broader notion of
indirect public benefit or advantage.
In Manosca v. Court of Appeals,
this Court has also held that what
ultimately emerged is a concept of
public use which is just as broad as
public welfare. (252 SCRA 412, 422
[1996], quoting Joaquin Bernas, The
Constitution of the Republic of the
Philippines, Vol. 1, 1987 ed., p. 282)
Respondent PEZA expropriated
the subject parcel of land pursuant to
Proclamation No. 1980 x x x issued by
former President Ferdinand Marcos.
Meanwhile, the power of eminent
domain of respondent is contained in
its original charter, Presidential Decree
No. 66 x x x.
Accordingly, subject Lot 1406-B
was expropriated for the construction
. . . of terminal facilities, structures
and approaches thereto.
The
authority is broad enough to give the
respondent substantial leeway in
deciding for what public use the

expropriated
property
would
be
utilized.
Pursuant to this broad
authority, respondent leased a portion
of the lot to commercial banks while
the rest was made a transportation
terminal. Said public purposes were
even reaffirmed by Republic Act No.
7916, a law amending respondent
PEZAs original charter x x x.
In Manila Railroad Co. v. Mitchel
(50 Phil. 832, 837-838 [1927]), this
Court has ruled that in the exercise of
eminent domain, only as much land
can be taken as is necessary for the
legitimate
purpose
of
the
condemnation. The term necessary,
in this connection, does not mean
absolutely indispensable but requires
only a reasonable necessity of the
taking for the stated purpose, growth
and future needs of the enterprise.
The respondent cannot attain a selfsustaining and viable ECOZONE if
inevitable needs in the expansion in
the surrounding areas are hampered
by the mere refusal of the private
landowners
to
part
with
their
properties. The purpose of creating an
ECOZONE and other facilities is better
served if respondent directly owns the
areas subject of the expansion
program.
X x x. The expropriation of Lot
1406-B for the purpose of being leased
to banks and for the construction of a
terminal has the purpose of making
banking and transportation facilities
easily accessible to the persons
working at the industries located in
PEZA. The expropriation of adjacent
areas therefore comes as a matter of
necessity to bring life to the purpose
of the law. In such a manner, PEZAs
goal of being a major force in the
economic development of the country
would be realized. Furthermore, this
Court has already ruled that:
X x x [T]he Legislature may directly
determine
the
necessity
for
appropriating private property for a
particular improvement for public use,
and it may select the exact location of
the improvement. In such a case, it is
well-settled that the utility of the
proposed improvement, the existence
of the public necessity for its
construction,
the
expediency
of
constructing it, the suitableness of the
location selected, are all questions
exclusively for the legislature to
determine, and the courts have no
power to interfere or to substitute
their own views for those of the
representatives of the people.

In the absence of some constitutional


or statutory provisions to the contrary,
the necessity and expediency of
exercising the right of eminent domain
are questions essentially political and
not judicial in their character. (City of
Manila v. Chinese Community of
Manila, 40 Phil. 349 [1919])
Inasmuch as both Presidential Decree
No. 66 and Republic Act No. 7916,
bestow respondent with authority to
develop terminal facilities and banking
centers, this Court will not question
the respondents lease of certain
portions of the expropriated lot to
banks, as well as the construction of
terminal facilities.
Petitioner
contends
that
respondent
is
bound
by
the
representations of its Chief Civil
Engineer when the latter testified
before the trial court that the lot was
to be devoted for the construction of
government offices. Anent this issue,
suffice it to say that PEZA can vary the
purpose for which a condemned lot
will be devoted to, provided that the
same is for public use.
Petitioner
cannot impose or dictate on the
respondent what facilities to establish
for as long as the same are for public
purpose. (Estate of Salud Jimenez
v. PEZA, 349 SCRA 240, Jan. 16,
2001, 2nd Div. [De Leon])
142. Discuss the meaning of just
compensation in eminent domain
proceedings.
Does it include the
payment of interest and, if so, how
is it to be computed?
Held:
1. The constitutional
limitation of just compensation is
considered to be the sum equivalent
to the market value of the property,
broadly described to be the price fixed
by the seller in open market in the
usual and ordinary course of legal
action and competition or the fair
value of the property as between one
who receives, and one who desires to
sell, it fixed at the time of the actual
taking by the government (Manila
Railway Co. v. Fabie, 17 Phil. 206).
Thus, if property is taken for public use
before compensation is deposited with
the court having jurisdiction over the
case, the final compensation must

include interests on its just value to be


computed from the time the property
is
taken
to
the
time
when
compensation is actually paid or
deposited with the court (Philippine
Railway Co. v. Solon, 13 Phil. 34). In
fine, between the taking of the
property and the actual payment,
legal interests accrue in order to place
the owner in a position as good as (but
not better than) the position he was in
before
the
taking
occurred
(Commissioner of Public Highways v.
Burgos, 96 SCRA 831). (Republic of
the Philippines v. The Hon. Court
of Appeals, G.R. No. 146587, July
2, 2002, 1st Div. [Vitug])
2.
We have ruled that the
concept
of
just
compensation
embraces not only the correct
determination of the amount to be
paid to the owners of the land, but
also the payment of the land within a
reasonable time from its taking.
Without
prompt
payment,
compensation cannot be considered
just inasmuch as the property owner
is made to suffer the consequences of
being immediately deprived of his land
while being made to wait for a decade
or more before actually receiving the
amount necessary to cope with his
loss (Land Bank of the Philippines v.
Court of Appeals, 258 SCRA 404, 408409 [1996] quoting Municipality of
Makati v. Court of Appeals, 190 SCRA
207, 213 [1990]). Payment of just
compensation should follow as a
matter of right immediately after the
order of expropriation is issued. Any
delay in payment must be counted
from said order. However, the delay to
constitute a violation of due process
must
be
unreasonable
and
inexcusable; it must be deliberately
done by a party in order to defeat the
ends of justice.
We find that respondent capriciously
evaded its duty of giving what is due
to petitioner. In the case at bar, the
expropriation order was issued by the
trial court in 1991. The compromise
agreement between the parties was
approved by the trial court in 1993.
However, from 1993 up to the present,
respondent has failed in its obligation
to pay petitioner to the prejudice of
the latter. Respondent cause damage
to petitioner in making the latter to
expect that it had a good title to the
property to be swapped with Lot 1406B; and meanwhile, respondent has

been reaping benefits from the lease


or rental income of the said
expropriated lot. We cannot tolerate
this oppressive exercise of the power
of eminent domain by respondent. As
we have ruled in Cosculluela v. Court
of Appeals (164 SCRA 393, 401
[1988]):
In the present case, the irrigation
project was completed and has been
in operation since 1976. The project is
benefiting the farmers specifically and
the community in general. Obviously,
petitioners land cannot be returned to
him. However, it is high time that the
petitioner be paid what has been due
him eleven years ago. It is arbitrary
and capricious for a government
agency
to
initiate
expropriation
proceedings,
seize
a
persons
property, allow the judgment of the
court to become final and executory
and then refuse to pay on the ground
that there was no appropriations for
the property earlier taken and
profitably used. We condemn in the
strongest possible terms the cavalier
attitude of government officials who
adopt
such
a
despotic
and
irresponsible stance.
(Estate of Salud Jimenez v. PEZA,
349 SCRA 240, Jan. 16, 2001, 2nd
Div. [De Leon])
143.
When may the property owner
be entitled to the return of the
expropriated property in eminent
domain cases?
Held: 1. In insisting on the
return of the expropriated property,
respondents would exhort on the
pronouncement
in
Provincial
Government of Sorsogon v. Vda. De
Villaroya (153 SCRA 291) where the
unpaid landowners were allowed the
alternative remedy of recovery of the
property there in question. It might be
borne in mind that the case involved
the
municipal
government
of
Sorsogon, to which the power of
eminent domain is not inherent, but
merely delegated and of limited
application. The grant of the power of
eminent domain to local governments
under Republic Act No. 7160 (See
Local Government Code of 1991)
cannot be understood as being the
pervasive and all-encompassing power
vested in the legislative branch of
government. For local governments to

be able to wield the power, it must, by


enabling law, be delegated to it by the
national legislature, but even then,
this delegated power of eminent
domain is not, strictly speaking, a
power of eminent, but only of inferior,
domain or only as broad or confined as
the real authority would want it to be
(City of Manila v. Chinese Cemetery of
Manila, 40 Phil. 349).
Thus, in Valdehueza v. Republic
(17 SCRA 107) where the private
landowners had remained unpaid ten
years after the termination of the
expropriation proceedings, this Court
ruled
The points in dispute are whether
such payment can still be made and, if
so, in what amount. Said lots have
been the subject of expropriation
proceedings. By final and executory
judgment in said proceedings, they
were condemned for public use, as
part of an airport, and ordered sold to
the government. X x x It follows that
both by virtue of the judgment, long
final, in the expropriation suit, as well
as the annotations upon their title
certificates, plaintiffs are not entitled
to
recover
possession
of
their
expropriated lots which are still
devoted to the public use for which
they were expropriated but only to
demand the fair market value of the
same.
Said relief may be granted under
plaintiffs prayer for: such other
remedies, which may be deemed just
and equitable under the premises.
(At p. 112)
The Court proceeded to reiterate its
pronouncement in Alfonso v. Pasay
City (106 Phil. 1017) where the
recovery of possession of property
taken for public use prayed for by the
unpaid landowner was denied even
while
no
requisite
expropriation
proceedings were first instituted. The
landowner was merely given the relief
of recovering compensation for his
property computed at its market value

at the time it was taken


appropriated by the State.

and

The judgment rendered by the


Bulacan RTC in 1979 on the
expropriation proceedings provides
not only for the payment of just
compensation to herein respondents
but likewise adjudges the property
condemned in favor of petitioner over
which parties, as well as their privies,
are bound (Mines v. Canal Authority of
the State [Fla] 467 So2d 989, 10 FLW
230). Petitioner has occupied, utilized
and, for all intents and purposes,
exercised dominion over the property
pursuant to the judgment.
The
exercise of such rights vested to it as
the condemnee indeed has amounted
to at least a partial compliance or
satisfaction of the 1979 judgment,
thereby preempting any claim of bar
by prescription on grounds of nonexecution. In arguing for the return of
their property on the basis of nonpayment, respondents ignore the fact
that the right of the expropriatory
authority is far from that of an unpaid
seller in ordinary sales, to which the
remedy of rescission might perhaps
apply.
An in rem proceeding,
condemnation acts upon the property
(Cadorette v. US CCA [Mass] 988 F2d
215).
After condemnation, the
paramount title is in the public under a
new and independent title (Ibid.); thus,
by giving notice to all claimants to a
disputed
title,
condemnation
proceedings provide a judicial process
for securing better title against all the
world than may be obtained by
voluntary
conveyance
(Ibid.).
(Republic of the Philippines v. The
Hon. Court of Appeals, G.R. No.
146587, July 2, 2002, 1st Div.
[Vitug])
2.
Though the respondent has
committed a misdeed to petitioner, we
cannot,
however,
grant
the
petitioners prayer for the return of the
expropriated Lot No. 1406-B.
The
Order of expropriation dated July 11,
1991, has long become final and
executory. Petitioner cited Provincial
Government of Sorsogon v. Rosa E.
Vda. De Villaroya (153 SCRA 291, 302
[1987]) to support its contention that

it is entitled to a return of the lot


where this Court ruled that under
ordinary circumstances, immediate
return to the owners of the unpaid
property is the obvious remedy.
However, the said statement was not
the ruling in that case. As in other
cases where there was no prompt
payment by the government, this
Court declared in Sorsogon that the
Provincial Government of Sorsogon is
expected to immediately pay as
directed. Should any further delay be
encountered, the trial court is directed
to seize any patrimonial property or
cash savings of the province in the
amount necessary to implement this
decision. However, this Court also
stressed and declared in that case that
in cases where land is taken for
public use, public interest, however,
must be considered.
(Estate of
Salud Jimenez v. PEZA, 349 SCRA
240, Jan. 16, 2001, 2nd Div. [De
Leon])
The Power of Taxation
144. Can taxes be subject to offsetting or compensation?
Held:
Taxes cannot be subject to
compensation for the simple reason
that the government and the taxpayer
are not creditors and debtors of each
other. There is a material distinction
between a tax and debt. Debts are
due to the Government in its corporate
capacity, while taxes are due to the
Government in its sovereign capacity.
It must be noted that a distinguishing
feature of a tax is that it is compulsory
rather than a matter of bargain.
Hence, a tax does not depend upon
the consent of the taxpayer. If any
taxpayer can defer the payment of
taxes by raising the defense that it still
has a pending claim for refund or
credit, this would adversely affect the
government revenue system.
A
taxpayer cannot refuse to pay his
taxes when they fall due simply
because he has a claim against the
government or that the collection of a
tax is contingent on the result of the
lawsuit
it
filed
against
the
government.
(Philex
Mining
Corporation v. Commissioner of
Internal Revenue, 294 SCRA 687,
Aug. 28, 1998 [Romero])
145. Under Article VI, Section 28,
paragraph 3 of the 1987 Constitution,
"[C]haritable institutions, churches
and
parsonages
or
convents
appurtenant thereto, mosques, non-

profit cemeteries, and all lands,


buildings, and improvements, actually,
directly and exclusively used for
religious, charitable or educational
purposes shall be exempt from
taxation."
YMCA claims that the
income earned by its building leased
to private entities and that of its
parking space is likewise covered by
said exemption. Resolve.
Held:
The debates, interpellations
and expressions of opinion of the
framers of the Constitution reveal their
intent that which, in turn, may have
guided the people in ratifying the
Charter.
Such intent must be
effectuated.
Accordingly, Justice Hilario G. Davide,
Jr.,
a
former
constitutional
commissioner, who is now a member
of this Court, stressed during the
Concom debates that "x x x what is
exempted is not the institution itself x
x x; those exempted from real estate
taxes are lands, buildings and
improvements actually, directly and
exclusively
used
for
religious,
charitable or educational purposes.
Father Joaquin G. Bernas, an eminent
authority on the Constitution and also
a member of the Concom, adhered to
the same view that the exemption
created by said provision pertained
only to property taxes.
In his treatise on taxation, Mr. Justice
Jose C. Vitug concurs, stating that
"[t]he tax exemption covers property
taxes
only."
(Commissioner
of
Internal Revenue v. CA, 298 SCRA
83, Oct. 14, 1998 [Panganiban])
146. Under Article XIV, Section 4,
paragraph 3 of the 1987 Constitution,
"[A]ll revenues and assets of nonstock,
non-profit
educational
institutions used actually, directly, and
exclusively for educational purposes
shall be exempt from taxes and
duties." YMCA alleged that it "is a nonprofit educational institution whose
revenues and assets are used actually,
directly and exclusively for educational
purposes so it is exempt from taxes on
its properties and income."
Held:
We reiterate that private
respondent is exempt from the
payment of property tax, but not
income tax on the rentals from its
property. The bare allegation alone
that it is a non-stock, non-profit
educational institution is insufficient to
justify its exemption from the payment
of income tax.

[L]aws allowing tax exemption are


construed strictissimi juris. Hence, for
the YMCA to be granted the exemption
it claims under the abovecited
provision,
it
must
prove
with
substantial evidence that (1) it falls
under the classification non-stock,
non-profit educational institution; and
(2) the income it seeks to be
exempted from taxation is used
actually, directly, and exclusively for
educational purposes. However, the
Court notes that not a scintilla of
evidence was submitted by private
respondent to prove that it met the
said requisites. (Commissioner of
Internal Revenue v. CA, 298 SCRA
83, Oct. 14, 1998 [Panganiban])
147. Is the YMCA an educational
institution within the purview of Article
XIV, Section 4, par. 3 of the
Constitution?
Held: We rule that it is not. The term
"educational institution" or "institution
of learning" has acquired a well-known
technical meaning, of which the
members
of
the
Constitutional
Commission are deemed cognizant.
Under the Education Act of 1982, such
term refers to schools. The school
system is synonymous with formal
education, which "refers to the
hierarchically
structured
and
chronologically
graded
learnings
organized and provided by the formal
school
system
and
for
which
certification is required in order for the
learner to progress through the grades
or move to the higher levels." The
Court has examined the "Amended
Articles of Incorporation" and "ByLaws" of the YMCA, but found nothing
in them that even hints that it is a
school or an educational institution.
Furthermore, under the Education Act
of 1982, even non-formal education is
understood to be school-based and
"private auspices such as foundations
and civic-spirited organizations" are
ruled out. It is settled that the term
"educational institution," when used in
laws granting tax exemptions, refers
to a "x x x school seminary, college or
educational establishment x x x." (84
CJS 566)
Therefore, the private
respondent cannot be deemed one of
the educational institutions covered by
the constitutional provision under
consideration.
(Commissioner of
Internal Revenue v. CA, 298 SCRA
83, Oct. 14, 1998 [Panganiban])

148. May the PCGG validly commit to


exempt from all forms of taxes the
properties to be retained by the
Marcos heirs in a Compromise
Agreement between the former and
the latter?
Held: The power to tax and to grant
exemptions is vested in the Congress
and, to a certain extent, in the local
legislative bodies.
Section 28(4),
Article
VI
of
the
Constitution,
specifically provides: No law granting
any tax exemption shall be passed
without the concurrence of a majority
of all the members of the Congress.
The PCGG has absolutely no power to
grant tax exemptions, even under the
cover of its authority to compromise
ill-gotten wealth cases.
Even granting that Congress enacts a
law exempting the Marcoses from
paying taxes on their properties, such
law will definitely not pass the test of
the equal protection clause under the
Bill of Rights. Any special grant of tax
exemption in favor only of the Marcos
heirs will constitute class legislation.
It will also violate the constitutional
rule that taxation shall be uniform
and equitable. (Chavez v. PCGG,
299 SCRA 744, Dec. 9, 1998
[Panganiban])
149. Discuss
treaties?

the

purpose

of

tax

Held: The RP-US Tax Treaty is just


one of a number of bilateral treaties
which the Philippines has entered into
for the avoidance of double taxation.
The purpose of these international
agreements is to reconcile the national
fiscal legislations of the contracting
parties in order to help the taxpayer
avoid simultaneous taxation in two
different jurisdictions. More precisely,
the tax conventions are drafted with a
view towards the elimination of
international juridical double taxation
x x x. (Commissioner of Internal
Revenue v. S.C. Johnson and Son,
Inc., 309 SCRA 87, 101-102, June
25, 1999, 3rd Div. [GonzagaReyes])
150. What is "international juridical
double taxation"?
Held: It is defined as the imposition
of comparable taxes in two or more
states on the same taxpayer in
respect of the same subject matter
and
for
identical
periods.
(Commissioner
of
Internal
Revenue v. S.C. Johnson and Son,

Inc., 309 SCRA 87, 102, June 25,


1999)
151. What is the rationale for doing
away
with
international
juridical
double taxation?
What are the
methods resorted to by tax treaties to
eliminate double taxation?
Held:
The apparent rationale for
doing away with double taxation is to
encourage the free flow of goods and
services and the movement of capital,
technology and persons between
countries, conditions deemed vital in
creating
robust
and
dynamic
economies. Foreign investments will
only thrive in a fairly predictable and
reasonable international investment
climate and the protection against
double taxation is crucial in creating
such a climate.
Double taxation usually takes place
when a person is resident of a
contracting state and derives income
from, or owns capital in, the other
contracting state and both states
impose tax on that income or capital.
In order to eliminate double taxation,
a tax treaty resorts to several
methods.
First, it sets out the
respective rights to tax of the state of
source or situs and of the state of
residence with regard to certain
classes of income or capital. In some
cases, an exclusive right to tax is
conferred on one of the contracting
states; however, for other items of
income or capital, both states are
given the right to tax, although the
amount of tax that may be imposed by
the state of source is limited.
The second method for the elimination
of double taxation applies whenever
the state of source is given a full or
limited right to tax together with the
state of residence. In this case, the
treaties make it incumbent upon the
state of residence to allow relief in
order to avoid double taxation. There
are two methods of relief - the
exemption method and the credit
method. In the exemption method,
the income or capital which is taxable
in the state of source or situs is
exempted in the state of residence,
although in some instances it may be
taken into account in determining the
rate of tax applicable to the taxpayer's
remaining income or capital. On the
other hand, in the credit method,
although the income or capital which
is taxed in the state of source is still
taxable in the state of residence, the
tax paid in the former is credited

against the tax levied in the latter.


The basic difference between the two
methods is that in the exemption
method, the focus is on the income or
capital itself, whereas the credit
method focuses upon the tax.
(Commissioner
of
Internal
Revenue v. S.C. Johnson and Son,
Inc., 309 SCRA 87, 102-103, June
25, 1999)
152. What is the rationale for
reducing the tax rate in negotiating
tax treaties?
Held: In negotiating tax treaties, the
underlying rationale for reducing the
tax rate is that the Philippines will give
up a part of the tax in the expectation
that the tax given up for this particular
investment is not taxed by the other
country. (Commissioner of Internal
Revenue v. S.C. Johnson and Son,
Inc., 309 SCRA 87, 103, June 25,
1999)
B. THE BILL OF RIGHTS
The Due Process Clause
153. Discuss the Due Process Clause.
Distinguish substantive due process
from procedural due process.
Held: Section 1 of the Bill of Rights
lays down what is known as the "due
process clause" of the Constitution.
In order to fall within the aegis of this
provision, two conditions must concur,
namely, that there is a deprivation and
that such deprivation is done without
proper observance of due process.
When one speaks of due process of
law, however, a distinction must be
made between matters of procedure
and matters of substance. In essence,
procedural due process "refers to the
method or manner by which the law is
enforced," while substantive due
process "requires that the law itself,
not merely the procedures by which
the law would be enforced, is fair,
reasonable, and just."
(Corona v.
United Harbor Pilots Association
of the Phils., 283 SCRA 31, Dec.
12, 1997 [Romero])
154. Respondents
United
Harbor
Pilots Association of the Philippines
argue that due process was not
observed in the adoption of PPA-AO
No. 04-92 which provides that: (a)ll
existing regular appointments which
have been previously issued by the
Bureau of Customs or the PPA shall

remain valid up to 31 December 1992


only, and (a)ll appointments to
harbor pilot positions in all pilotage
districts shall, henceforth, be only for
a term of one (1) year from date of
effectivity subject to renewal or
cancellation by the Philippine Ports
Authority after conduct of a rigid
evaluation of performance, allegedly
because no hearing was conducted
whereby
relevant
government
agencies and the harbor pilots
themselves could ventilate their views.
They also contended that the sole and
exclusive right to the exercise of
harbor pilotage by pilots has become
vested and can only be withdrawn or
shortened
by
observing
the
constitutional mandate of due process
of law.
Held: They are obviously referring to
the
procedural
aspect
of
the
enactment. Fortunately, the Court has
maintained a clear position in this
regard, a stance it has stressed in the
recent case of Lumiqued v. Hon.
Exevea (G.R. No. 117565, November
18, 1997), where it declared that (a)s
long as a party was given the
opportunity to defend his interests in
due course, he cannot be said to have
been denied due process of law, for
this opportunity to be heard is the
very
essence
of
due
process.
Moreover, this constitutional mandate
is deemed satisfied if a person is
granted an opportunity to seek
reconsideration of the action or ruling
complained of.
In the case at bar, respondents
questioned PPA-AO No. 04-92 no less
than four times before the matter was
finally elevated to this Tribunal. Their
arguments on this score, however,
failed to persuade. X x x
Neither does the fact that the pilots
themselves were not consulted in any
way taint the validity of the
administrative order.
As a general
rule, notice and hearing, as the
fundamental
requirements
of
procedural due process, are essential
only when an administrative body
exercises its quasi-judicial function. In
the performance of its executive or
legislative functions, such as issuing
rules
and
regulations,
an
administrative body need not comply
with the requirements of notice and
hearing.
Upon the other hand, it is also
contended that the sole and exclusive
right to the exercise of harbor pilotage

by
pilots
is
a
settled
issue.
Respondents aver that said right has
become vested and can only be
withdrawn
or
shortened
by
observing the constitutional mandate
of due process of law. Their argument
has thus shifted from the procedural to
one of substance. It is here where
PPA-AO No. 04-92 fails to meet the
condition set by the organic law.
Pilotage, just like other professions,
may be practiced only by duly licensed
individuals. Licensure is the granting
of license especially to practice a
profession. It is also the system of
granting licenses (as for professional
practice)
in
accordance
with
established standards. A license is a
right or permission granted by some
competent authority to carry on a
business or do an act which, without
such license, would be illegal.
Before harbor pilots can earn a license
to practice their profession, they
literally have to pass through the
proverbial eye of a needle by taking,
not one but five examinations, each
followed by actual training and
practice. X x x
Their license is granted in the form of
an appointment which allows them to
engage in pilotage until they retire at
the age of 70 years. This is a vested
right. Under the terms of PPA-AO No.
04-92,
[a]ll
existing
regular
appointments
which
have
been
previously issued by the Bureau of
Customs or the PPA shall remain valid
up to 31 December 1992 only, and
(a)ll appointments to harbor pilot
positions in all pilotage districts shall,
henceforth, be only for a term of one
(1) year from date of effectivity
subject to renewal or cancellation by
the Authority after conduct of a rigid
evaluation of performance.
It is readily apparent that PPA-AO No.
04-92 unduly restricts the right of
harbor pilots to enjoy their profession
before their compulsory retirement. In
the past, they enjoyed a measure of
security knowing that after passing
five examinations and undergoing
years of on-the-job training, they
would have a license which they could
use until their retirement, unless
sooner revoked by the PPA for mental
or physical unfitness. Under the new
issuance, they have to contend with
an annual cancellation of their license
which can be temporary or permanent
depending on the outcome of their
performance evaluation.
Veteran

pilots and neophytes alike are


suddenly confronted with one-year
terms which ipso facto expire at the
end of that period. Renewal of their
license is now dependent on a rigid
evaluation of performance which is
conducted only after the license has
already been cancelled. Hence, the
use of the term renewal. It is this
pre-evaluation
cancellation
which
primarily makes PPA-AO No. 04-92
unreasonable
and
constitutionally
infirm.
In a real sense, it is a
deprivation of property without due
process of law. (Corona v. United
Harbor Pilots Association of the
Phils., 283 SCRA 31, December 12,
1997 [Romero])
155. Does the due process clause
encompass the right to be assisted by
counsel during an administrative
inquiry?
Held:
The right to counsel, which
cannot be waived unless the waiver is
in writing and in the presence of
counsel, is a right afforded a suspect
or an accused during custodial
investigation. It is not an absolute
right and may, thus, be invoked or
rejected in a criminal proceeding and,
with more reason, in an administrative
inquiry. In the case at bar, petitioners
invoke the right of an accused in
criminal
proceedings
to
have
competent and independent counsel
of his own choice.
Lumiqued,
however, was not accused of any
crime in the proceedings below. The
investigation
conducted
by
the
committee x x x was for the sole
purpose of determining if he could be
held administratively liable under the
law for the complaints filed against
him. x x x
As such, the hearing
conducted
by
the
investigating
committee was not part of a criminal
prosecution. X x x
While investigations conducted by an
administrative body may at times be
akin to a criminal proceeding, the fact
remains that under existing laws, a
party in an administrative inquiry may
or may not be assisted by counsel,
irrespective of the nature of the
charges and of the respondent's
capacity to represent himself, and no
duty rests on such a body to furnish
the person being investigated with
counsel.
In
an
administrative
proceeding x x x a respondent x x x
has the option of engaging the
services of counsel or not. x x x Thus,
the right to counsel is not imperative
in
administrative
investigations

because such inquiries are conducted


merely to determine whether there are
facts that merit disciplinary measures
against erring public officers and
employees, with the purpose of
maintaining the dignity of government
service.
The
right
to
counsel
is
not
indispensable to due process unless
required by the Constitution or the
law. X x x. (Lumiqued v. Exevea,
282 SCRA 125, Nov. 18, 1997
[Romero])
156. Does an extraditee have the
right to notice and hearing during the
evaluation stage of an extradition
proceeding?
Held: Considering that in the case at
bar, the extradition proceeding is only
at its evaluation stage, the nature of
the right being claimed by the private
respondent is nebulous and the
degree of prejudice he will allegedly
suffer is weak, we accord greater
weight to the interests espoused by
the government thru the petitioner
Secretary of Justice. X x x
In tilting the balance in favor of the
interests of the State, the Court
stresses that it is not ruling that the
private respondent has no right to due
process at all throughout the length
and breadth of the extradition
proceedings. Procedural due process
requires a determination of what
process is due, when it is due, and the
degree of what is due.
Stated
otherwise, a prior determination
should be made as to whether
procedural protections are at all due
and when they are due, which in turn
depends on the extent to which an
individual will be "condemned to suffer
grievous loss."
We have explained
why an extraditee has no right to
notice
and hearing during
the
evaluation stage of the extradition
process. As aforesaid, P.D. No. 1069
which
implements
the
RP-US
Extradition
Treaty
affords
an
extraditee sufficient opportunity to
meet the evidence against him once
the petition is filed in court. The time
for the extraditee to know the basis of
the request for his extradition is
merely moved to the filing in court of
the formal petition for extradition. The
extraditee's
right
to
know
is
momentarily withheld during the
evaluation stage of the extradition
process to accommodate the more
compelling interest of the State to
prevent
escape
of
potential

extraditees which can be precipitated


by premature information of the basis
of the request for his extradition. No
less compelling at that stage of the
extradition proceedings is the need to
be more deferential to the judgment of
a co-equal branch of the government,
the Executive, which has been
endowed by our Constitution with
greater power over matters involving
our foreign relations.
Needless to
state, this balance of interests is not a
static but a moving balance which can
be adjusted as the extradition process
moves from the administrative stage
to the judicial stage and to the
execution stage depending on factors
that will come into play. In sum, we
rule that the temporary hold on
private respondent's privilege of
notice and hearing is a soft restraint
on his right to due process which will
not deprive him of fundamental
fairness should he decide to resist the
request for his extradition to the
United States. There is no denial of
due process as long as fundamental
fairness
is
assured
a
party.
(Secretary of Justice v. Hon. Ralph
C. Lantion, G.R. No. 139465, Oct.
17, 2000, En Banc [Puno])
157. Will Mark Jimenezs detention
prior to the conclusion of the
extradition proceedings not amount to
a violation of his right to due process?
Held: Contrary to his contention, his
detention prior to the conclusion of the
extradition proceedings does not
amount to a violation of his right to
due process. We iterate the familiar
doctrine that the essence of due
process is the opportunity to be heard
(Garcia v. NLRC, GR No. 110494,
November 18, 1996; Paat v. Court of
Appeals, January 10, 1997) but, at the
same time, point out that the doctrine
does not always call for a prior
opportunity to be heard (See Central
Bank of the Philippines v. Court of
Appeals, 220 SCRA 536, March 20,
1993).
Where the circumstances
such as those present in an extradition
case call for it, a subsequent
opportunity to be heard is enough
(Ibid. See also Busuego v. Court of
Appeals, 304 SCRA 473, March 11,
1999).
In the present case,
respondent
will
be
given
full
opportunity to be heard subsequently,
when the extradition court hears the
Petition for Extradition. Hence, there
is no violation of his right to due
process and fundamental fairness.

Contrary to the contention of


Jimenez, we find no arbitrariness,
either, in the immediate deprivation of
his liberty prior to his being heard.
That his arrest and detention will not
be arbitrary is sufficiently ensured by
(1) the DOJs filing in court the Petition
with its supporting documents after a
determination that the extradition
request meets the requirements of the
law and the relevant treaty; (2) the
extradition judges independent prima
facie determination that his arrest will
best serve the ends of justice before
the issuance of a warrant for his
arrest; and (3) his opportunity, once
he is under the courts custody, to
apply for bail as an exception to the
no-initial-bail rule.
It is also worth noting that
before the US government requested
the
extradition
of
respondent,
proceedings
had
already
been
conducted in that country.
But
because he left the jurisdiction of the
requesting
state
before
those
proceedings could be completed, it
was hindered from continuing with the
due processes prescribed under its
laws. His invocation of due process
now had thus become hollow.
He
already had that opportunity in the
requesting state; yet, instead of taking
it, he ran away.
In this light, would it be proper
and just for the government to
increase the risk of violating its treaty
obligations
in
order
to
accord
Respondent Jimenez his personal
liberty in the span of time that it takes
to resolve the Petition for Extradition?
His supposed immediate deprivation
of liberty without due process that he
had previously shunned pales against
the governments interest in fulfilling
its Extradition Treaty obligations and in
cooperating with the world community
in the suppression of crime. Indeed,
[c]onstitutional liberties do not exist
in a vacuum; the due process rights
accorded to individuals must be
carefully balanced against exigent and
palpable
government
interest.
(Coquia, On the Implementation of
the US-RP Extradition Treaty, supra;
citing Kelso v. US Department of State,
13 F Supp. 291 [DDC 1998])
Too, we cannot allow our
country to be a haven for fugitives,
cowards and weaklings who, instead of
facing the consequences of their
actions, choose to run and hide.
Hence, it would not be good policy to

increase the risk of violating our treaty


obligations if, through overprotection
or excessively liberal treatment,
persons sought to be extradited are
able to evade arrest or escape from
our custody. In the absence of any
provision in the Constitution, the law
or the treaty expressly guaranteeing
the right to bail in extradition
proceedings, adopting the practice of
not granting them bail, as a general
rule, would be a step towards
deterring fugitives from coming to the
Philippines to hide from or evade their
prosecutors.
The denial of bail as a matter of
course in extradition cases falls into
place with and gives life to Article 14
(It states: If the person sought
consents in writing to surrender to the
Requesting State, the Requested State
may
surrender
the
person
as
expeditiously as possible without
further proceedings.) of the Treaty,
since this practice would encourage
the accused to voluntarily surrender to
the requesting state to cut short their
detention here.
Likewise, their
detention pending the resolution of
extradition proceedings would fall into
place with the emphasis of the
Extradition Law on the summary
nature of extradition cases and the
need for their speedy disposition.
(Government of the United States
of America v. Hon. Guillermo
Purganan, G.R. No. 148571, Sept.
24, 2002, En Banc [Panganiban])

158. Is respondent in an Extradition


Proceeding entitled to notice and
hearing before the issuance of a
warrant of arrest?
Held: Both parties cite Section 6 of
PD 1069 in support of their arguments.
Xxx
Does this provision sanction RTC
Judge Purganans act of immediately
setting for hearing the issuance of a
warrant of arrest?
We rule in the
negative.
1.
On
Extradition Law

the

Basis

of

the

It is significant to note that


Section 6 of PD 1069, our Extradition
Law, uses the word immediate to
qualify the arrest of the accused. This
qualification
would
be
rendered
nugatory by setting for hearing the
issuance of the arrest warrant.
Hearing entails sending notices to the
opposing parties, receiving facts and
arguments from them, and giving
them time to prepare and present
such facts and arguments.
Arrest
subsequent to a hearing can no longer
be considered immediate. The law
could not have intended the word as a
mere superfluity but, on the whole, as
a means of impairing a sense of
urgency
and
swiftness
in
the
determination of whether a warrant of
arrest should be issued.
By using the phrase if it
appears, the law further conveys that
accuracy is not as important as speed
at such early stage. The trial court is
not expected to make an exhaustive
determination to ferret out the true
and actual situation, immediately
upon the filing of the petition. From
the knowledge and the material then
available to it, the court is expected
merely to get a good first impression
a prima facie finding sufficient to
make a speedy initial determination as
regards the arrest and detention of the
accused.
Xxx
We stress that the prima facie
existence of probable cause for
hearing the petition and, a priori, for
issuing an arrest warrant was already
evident from the Petition itself and its
supporting documents. Hence, after
having already determined therefrom
that a prima facie finding did exist,
respondent judge gravely abused his
discretion when he set the matter for
hearing upon motion of Jimenez.
Moreover, the law specifies that
the court sets a hearing upon receipt
of the answer or upon failure of the
accused to answer after receiving the
summons.
In connection with the
matter of immediate arrest, however,
the word hearing is notably absent
from the provision. Evidently, had the
holding of a hearing at that stage
been intended, the law could have
easily so provided.
It also bears
emphasizing at this point that
extradition proceedings are summary
(See Sec. 9, PD 1069) in nature.
Hence, the silence of the Law and the

Treaty leans to the more reasonable


interpretation
that
there
is
no
intention to punctuate with a hearing
every little step in the entire
proceedings.
Xxx
Verily x x x sending to persons
sought to be extradited a notice of the
request for their arrest and setting it
for hearing at some future date would
give them ample opportunity to
prepare and execute an escape.
Neither the Treaty nor the Law could
have intended that consequence, for
the very purpose of both would have
been defeated by the escape of the
accused from the requested state.
2

On the Basis of the Constitution


Even Section 2 of Article III of our
Constitution, which is invoked by
Jimenez, does not require a notice or a
hearing before the issuance of a
warrant of arrest. X x x
To determine probable cause for the
issuance of arrest warrants, the
Constitution itself requires only the
examination

under
oath
or
affirmation of complainants and the
witnesses they may produce. There is
no requirement to notify and hear the
accused before the issuance of
warrants of arrest.
In Ho v. People (280 SCRA 365,
October 9, 1997) and in all the cases
cited therein, never was a judge
required to go to the extent of
conducting a hearing just for the
purpose of personally determining
probable cause for the issuance of a
warrant of arrest. All we required was
that the judge must have sufficient
supporting documents upon which to
make his independent judgment, or at
the very least, upon which to verify
the findings of the prosecutor as to the
existence of probable cause.
In Webb v. De Leon (247 SCRA 652,
680, per Puno, J.), the Court
categorically stated that a judge was
not supposed to conduct a hearing
before issuing a warrant of arrest x x
x.
At most, in cases of clear insufficiency
of evidence on record, judges merely
further examine complainants and
their witnesses (Ibid; citing Allado v.
Diokno, 233 SCRA 192, May 5, 1994).
In the present case, validating the act
of respondent judge and instituting the

practice of hearing the accused and


his witnesses at this early stage would
be discordant with the rationale for
the entire system. If the accused were
allowed to be heard and necessarily to
present evidence during the prima
facie determination for the issuance of
a warrant of arrest, what would stop
him from presenting his entire
plethora of defenses at this stage if
he so desires in his effort to negate a
prima facie finding? Such a procedure
could convert the determination of a
prima facie case into a full-blown trial
of the entire proceedings and possibly
make
trial
of
the
main
case
superfluous.
This scenario is also
anathema to the summary nature of
extraditions. (Government of the
United States of America v. Hon.
Guillermo Purganan, G.R. No.
148571, Sept. 24, 2002, En Banc
[Panganiban])
The Equal Protection Clause
159. Explain and discuss the equal
protection of the law clause.
Held: 1. The equal protection of the
law is embraced in the concept of due
process, as every unfair discrimination
offends the requirements of justice
and fair play. It has nonetheless been
embodied in a separate clause in
Article III, Sec. 1, of the Constitution to
provide for a more specific guaranty
against any form of undue favoritism
or hostility from the government.
Arbitrariness in general may be
challenged on the basis of the due
process clause. But if the particular
act
assailed
partakes
of
an
unwarranted partiality or prejudice,
the sharper weapon to cut it down is
the equal protection clause.
According to a long line of decisions,
equal protection simply requires that
all persons or things similarly situated
should be treated alike, both as to
rights conferred and responsibilities
imposed. Similar subjects, in other
words,
should
not
be
treated
differently, so as to give undue favor
to some and unjustly discriminate
against others.
The equal protection clause does not
require the universal application of the
laws on all persons or things without
distinction.
This might in fact
sometimes
result
in
unequal
protection, as where, for example, a
law prohibiting mature books to all
persons, regardless of age, would
benefit the morals of the youth but

violate the liberty of adults. What the


clause requires is equality among
equals as determined according to a
valid classification. By classification is
meant the grouping of persons or
things similar to each other in certain
particulars and different from all
others in these same particulars.
(Philippine Judges Association v.
Prado, 227 SCRA 703, 711-712,
Nov. 11, 1993, En Banc [Cruz])
2. The equal protection clause exists
to prevent undue favor or privilege. It
is intended to eliminate discrimination
and oppression based on inequality.
Recognizing the existence of real
difference among men, the equal
protection clause does not demand
absolute equality. It merely requires
that all persons shall be treated alike,
under
like
circumstances
and
conditions both as to the privileges
conferred and liabilities enforced.
Thus, the equal protection clause does
not absolutely forbid classifications x x
x. If the classification is based on real
and
substantial
differences;
is
germane to the purpose of the law;
applies to all members of the same
class; and applies to current as well as
future conditions, the classification
may not be impugned as violating the
Constitution's
equal
protection
guarantee. A distinction based on real
and reasonable considerations related
to a proper legislative purpose x x x is
neither unreasonable, capricious nor
unfounded.
(Himagan v. People,
237 SCRA 538, Oct. 7, 1994, En
Banc [Kapunan])
160. Congress enacted R.A. No. 8189
which provides, in Section 44 thereof,
that "No Election Officer shall hold
office
in
a
particular
city
or
municipality for more than four (4)
years. Any election officer who, either
at the time of the approval of this Act
or subsequent thereto, has served for
at least four (4) years in a particular
city or municipality shall automatically
be reassigned by the Commission to a
new station outside the original
congressional district."
Petitioners,
who are City and Municipal Election
Officers, theorize that Section 44 of RA
8189 is violative of the "equal
protection clause" of the 1987
Constitution because it singles out the
City and Municipal Election Officers of
the COMELEC as prohibited from
holding office in the same city or
municipality for more than four (4)
years. They maintain that there is no
substantial distinction between them
and other COMELEC officials, and

therefore,
there
is
no
valid
classification to justify the objective of
the provision of law under attack.
Resolve.
Held: The Court is not persuaded by
petitioners' arguments.
The "equal
protection clause" of the 1987
Constitution
permits
a
valid
classification under the following
conditions:
1)

The classification must rest


on substantial distinction;
2)
The classification must be
germane to the purpose of the law;
3)
The classification must not
be limited to existing conditions only;
and
4)
The classification must apply
equally to all members of the same
class.
After a careful study, the ineluctable
conclusion is that the classification
under Section 44 of RA 8189 satisfies
the aforestated requirements.
The singling out of election officers in
order to "ensure the impartiality of
election officials by preventing them
from developing familiarity with the
people of their place of assignment"
does not violate the equal protection
clause of the Constitution.
In Lutz v. Araneta (98 Phil. 148, 153
[1955]), it was held that "the
legislature is not required by the
Constitution to adhere to a policy of
'all or none'".
This is so for
underinclusiveness is not an argument
against a valid classification. It may
be true that all other officers of
COMELEC referred to by petitioners
are exposed to the same evils sought
to be addressed by the statute.
However, in this case, it can be
discerned that the legislature thought
the noble purpose of the law would be
sufficiently served by breaking an
important link in the chain of
corruption than by breaking up each
and every link thereof. Verily, under
Section 3(n) of RA 8189, election
officers are the highest officials or
authorized representatives of the
COMELEC in a city or municipality. It is
safe to say that without the complicity
of such officials, large-scale anomalies
in the registration of voters can hardly
be carried out. (Agripino A. De
Guzman, Jr., et al. v. COMELEC
(G.R. No. 129118, July 19, 2000,
en Banc [Purisima])

161.
Appellant, who was charged
with Illegal Recruitment in the RTC of
Zamboanga City, invokes the equal
protection clause in her defense. She
points out that although the evidence
purportedly
shows
that
Jasmine
Alejandro handed out application
forms and even received Lourdes
Modestos payment, appellant was the
only
one
criminally
charged.
Alejandro,
on
the
other
hand,
remained scot-free.
From this,
appellant
concludes
that
the
prosecution discriminated against her
on grounds of regional origins.
Appellant is a Cebuana while Alejandro
is a Zamboanguena, and the alleged
crime took place in Zamboanga City.
Held: The argument has no merit.
The prosecution of one guilty while
others
equally
guilty
are
not
prosecuted, however, is not, by itself,
a denial of the equal protection of the
laws (Application of Finn, 356 P.2d 685
[1960]).
Where the official action
purports to be in conformity to the
statutory classification, an erroneous
or mistaken performance of the
statutory duty, although a violation of
the statute, is not without more a
denial of the equal protection of the
laws (Snowden v. Hughes, 321 US 1,
88 L Ed 497, 64 S Ct 397 [1943]). The
unlawful administration by officers of a
statute fair on its face, resulting in its
unequal application to those who are
entitled to be treated alike, is not a
denial of equal protection, unless
there is shown to be present in it an
element of intentional or purposeful
discrimination. This may appear on
the face of the action taken with
respect to a particular class or person,
or it may only be shown by extrinsic
evidence showing a discriminatory
design over another not to be inferred
from the action itself.
But a
discriminatory
purpose
is
not
presumed, there must be a showing of
clear and intentional discrimination.
(Ibid.) Appellant has failed to show
that, in charging appellant in court,
that there was a clear and intentional

discrimination on the part of the


prosecuting officials.
The discretion of who to prosecute
depends on the prosecutions sound
assessment whether the evidence
before it can justify a reasonable belief
that a person has committed an
offense (Tan, Jr. v. Sandiganbayan
[Third Division], 292 SCRA 452
[1998]). The presumption is that the
prosecuting
officers
regularly
performed their duties (Rules of Court,
Rule 131, Sec. 5 [m]), and this
presumption can be overcome only by
proof to the contrary, not by mere
speculation. Indeed, appellant has not
presented any evidence to overcome
this presumption. The mere allegation
that appellant, a Cebuana, was
charged with the commission of a
crime, while a Zamboanguena, the
guilty party in appellants eyes, was
not, is insufficient to support a
conclusion
that
the
prosecution
officers
denied
appellant
equal
protection of the laws.
There
is
also
common
sense
practicality in sustaining appellants
prosecution.
While all persons accused of crime are
to be treated on a basis of equality
before the law, it does not follow that
they are to be protected in the
commission of crime.
It would be
unconscionable,
for
instance,
to
excuse a defendant guilty of murder
because others have murdered with
impunity.
The remedy for unequal
enforcement of the law in such
instances does not lie in the
exoneration of the guilty at the
expense of society x x x. Protection of
the law will be extended to all persons
equally in the pursuit of their lawful
occupations, but no person has the
right to demand protection of the law
in the commission of a crime (People
v. Montgomery, 117 P.2d 437 [1941]).
Likewise,
[i]f the failure of prosecutors to
enforce the criminal laws as to some

persons should be converted into a


defense for others charged with crime,
the result would be that the trial of the
district attorney for nonfeasance
would become an issue in the trial of
many persons charged with heinous
crimes and the enforcement of law
would suffer a complete breakdown
(State v. Hicks, 325 P.2d 794 [1958]).
(People v. Dela Piedra, 350 SCRA
163, Jan. 24, 2001, 1st Div.
[Kapunan])

162. Are
there
substantial
distinctions between print media and
broadcast media to justify the
requirement for the latter to give free
airtime to be used by the Comelec to
inform the public of qualifications and
program of government of candidates
and political parties during the
campaign period? Discuss.
Held: There are important differences
in the characteristics of the two media
which
justify
their
differential
treatment for free speech purposes.
Because of the physical limitations of
the
broadcast
spectrum,
the
government
must,
of
necessity,
allocate broadcast frequencies to
those wishing to use them. There is
no similar justification for government
allocation and regulation of the print
media.
In the allocation of limited resources,
relevant conditions may validly be
imposed on the grantees or licensees.
The reason for this is that the
government spends public funds for
the allocation and regulation of the
broadcast industry, which it does not
do in the case of print media. To
require radio and television broadcast
industry to provide free airtime for the
Comelec Time is a fair exchange for
what the industry gets.
From another point of view, the SC has
also held that because of the unique
and pervasive influence of the
broadcast media, [n]ecessarily x x x
the freedom of television and radio
broadcasting is somewhat lesser in
scope than the freedom accorded to
newspaper
and
print
media.
(TELEBAP, Inc. v. COMELEC, 289
SCRA
337,
April
21,
1998
[Mendoza])
163. Does the death penalty law
(R.A. No. 7659) violate the equal
protection clause considering that, in

effect, it punishes only people who are


poor, uneducated, and jobless?
Held:
R.A. No. 7659 specifically
provides that [T]he death penalty
shall be imposed if the crime of rape is
committed x x x when the victim is a
religious or a child below seven (7)
years old.
Apparently, the death
penalty law makes no distinction. It
applies to all persons and to all classes
of persons rich or poor, educated or
uneducated, religious or non-religious.
No particular person or classes of
persons are identified by the law
against whom the death penalty shall
be exclusively imposed.
The law
punishes with death a person who
shall commit rape against a child
below seven years of age. Thus, the
perpetration of rape against a 5-year
old girl does not absolve or exempt an
accused from the imposition of the
death penalty by the fact that he is
poor, uneducated, jobless, and lacks
catechetical instruction.
To hold
otherwise will not eliminate but
promote inequalities.
In Cecilleville Realty and Service
Corporation v. CA, 278 SCRA 819
[1997]),
the
SC
clarified
that
compassion for the poor is an
imperative of every humane society
but only when the recipient is not a
rascal
claiming
an
undeserved
privilege. (People v. Jimmy Mijano
y Tamora, G.R. No. 129112, July
23, 1999, En Banc [Per Curiam])
164. The
International
School
Alliance
of
Educators
(ISAE)
questioned
the
point-of-hire
classification
employed
by
International School, Inc. to justify
distinction in salary rates between
foreign-hires and local-hires, i.e.,
salary rates of foreign-hires are higher
by 25% than their local counterparts,
as discriminatory and, therefore,
violates the equal protection clause.
The International School contended
that this is necessary in order to entice
foreign-hires to leave their domicile
and work here. Resolve.
Held:
That public policy abhors
inequality
and
discrimination
is
beyond contention. Our Constitution
and laws reflect the policy against
these evils. X x x
International law, which springs from
general principles of law, likewise
proscribes discrimination x x x. The
Universal
Declaration
of
Human
Rights, the International Covenant on

Economic, Social and Cultural Rights,


the International Convention on the
Elimination of All Forms of Racial
Discrimination, the Convention against
Discrimination
in
Education,
the
Convention (No. 111) Concerning
Discrimination
in
Respect
of
Employment and Occupation - all
embody the general principle against
discrimination, the very antithesis of
fairness and justice. The Philippines,
through
its
Constitution,
has
incorporated this principle as part of
its national laws.
[I]t would be an affront to both the
spirit and letter of these provisions if
the State, in spite of its primordial
obligation to promote and ensure
equal
employment
opportunities,
closes its eyes to unequal and
discriminatory terms and conditions of
employment x x x.
Discrimination, particularly in terms of
wages, is frowned upon by the Labor
Code.
Article 135, for example,
prohibits and penalizes the payment of
lesser compensation to a female
employee as against a male employee
for work of equal value. Article 248
declares it an unfair labor practice for
an employer to discriminate in regards
to wages in order to encourage or
discourage membership in any labor
organization. X x x
The foregoing provisions impregnably
institutionalize in this jurisdiction the
long honored legal truism of "Equal
pay for equal work."
Persons who
work
with
substantially
equal
qualifications,
skill,
effort
and
responsibility,
under
similar
conditions, should be paid similar
salaries.
This rule applies to the
School (International School, Inc.), its
"international
character"
notwithstanding.
The School contends that petitioner
has not adduced evidence that localhires perform work equal to that of
foreign-hires.
The Court finds this
argument a little cavalier.
If an
employer accords employees the
same
position
and
rank,
the
presumption is that these employees
perform equal work. This presumption
is borne by logic and human
experience. If the employer pays one
employee less than the rest, it is not
for that employee to explain why he
receives less or why the others receive
more. That would be adding insult to
injury.
The
employer
has
discriminated against that employee;

it is for the employer to explain why


the employee is treated unfairly.
The employer in this case failed to
discharge this burden. There is no
evidence
here
that foreign-hires
perform 25% more efficiently or
effectively than the local-hires. Both
groups have similar functions and
responsibilities, which they perform
under similar working conditions.
The School cannot invoke the need to
entice foreign-hires to leave their
domicile to rationalize the distinction
in salary rates without violating the
principle of equal work for equal pay.
Xxx
While we recognize the need of the
School to attract foreign-hires, salaries
should not be used as an enticement
to the prejudice of local-hires. The
local-hires perform the same services
as foreign-hires and they ought to be
paid the same salaries as the latter.
For the same reason, the "dislocation
factor" and the foreign-hires' limited
tenure also cannot serve as valid
bases for the distinction in salary
rates.
The dislocation factor and
limited tenure affecting foreign-hires
are adequately compensated by
certain benefits accorded them which
are not enjoyed by local-hires, such as
housing,
transportation,
shipping
costs, taxes and home leave travel
allowances.
The Constitution enjoins the State to
"protect the rights of workers and
promote their welfare", "to afford labor
full protection." The State, therefore,
has the right and duty to regulate the
relations between labor and capital.
These relations are not merely
contractual but are so impressed with
public interest that labor contracts,
collective
bargaining
agreements
included, must yield to the common
good. Should such contracts contain
stipulations that are contrary to public
policy, courts will not hesitate to strike
down these stipulations.
In this case, we find the point-of-hire
classification employed by respondent
School to justify the distinction in the
salary rates of foreign-hires and localhires to be an invalid classification.
There is no reasonable distinction
between the services rendered by
foreign-hires and local-hires.
The
practice of the School of according
higher
salaries
to
foreign-hires
contravenes
public
policy
and,

certainly, does not deserve the


sympathy
of
this
Court.
(International School Alliance of
Educators (ISAE) v. Hon. Leonardo
A. Quisumbing, G.R. No. 128845,
June 1, 2000, 1st Div. [Kapunan])
165. Accused-appellant Romeo G.
Jalosjos filed a motion before the Court
asking that he be allowed to fully
discharge
the
duties
of
a
Congressman, including attendance at
legislative sessions and committee
meetings despite his having been
convicted in the first instance of a
non-bailable offense. Does being an
elective official result in a substantial
distinction
that
allows
different
treatment? Is being a Congressman a
substantial
differentiation
which
removes the accused-appellant as a
prisoner from the same class as all
persons validly confined under law?
Held: In the ultimate analysis, the
issue before us boils down to a
question
of
constitutional
equal
protection.
Xxx
The performance of legitimate and
even essential duties by public officers
has never been an excuse to free a
person validly in prison. The duties
imposed by the mandate of the
people are multifarious.
The
accused-appellant asserts that the
duty to legislate ranks highest in the
hierarchy of government.
The
accused-appellant is only one of 250
members
of
the
House
of
Representatives, not to mention the
24 members of the Senate, charged
with
the
duties
of
legislation.
Congress continues to function well in
the physical absence of one or a few
of its members. Depending on the
exigency of Government that has to
be addressed, the President or the
Supreme Court can also be deemed
the highest for that particular duty.
The importance of a function depends
on the need for its exercise. The duty
of a mother to nurse her infant is most
compelling under the law of nature. A
doctor with unique skills has the duty
to save the lives of those with a
particular affliction.
An elective
governor has to serve provincial
constituents.
A police officer must
maintain peace and order. Never had
the call of a particular duty lifted a
prisoner into a different classification
from those others who are validly
restrained by law.

A strict scrutiny of classifications is


essential lest wittingly or otherwise,
insidious discriminations are made in
favor of or against groups or types of
individuals.
The Court cannot validate badges of
inequality. The necessities imposed by
public welfare may justify exercise of
government authority to regulate even
if
thereby
certain
groups
may
plausibly assert that their interests are
disregarded.
We, therefore, find that election to the
position of Congressman is not a
reasonable classification in criminal
law enforcement. The functions and
duties of the office are not substantial
distinctions which lift him from the
class of prisoners interrupted in their
freedom and restricted in liberty of
movement.
Lawful arrest and
confinement are germane to the
purposes of the law and apply to all
those belonging to the same class.
Xxx
It can be seen from the foregoing that
incarceration, by its nature, changes
an individuals status in society. Prison
officials have the difficult and often
thankless job of preserving the
security in a potentially explosive
setting, as well as of attempting to
provide rehabilitation that prepare
inmates for re-entry into the social
mainstream. Necessarily, both these
demands require the curtailment and
elimination of certain rights.
Premises
considered,
we
are
constrained to rule against the
accused-appellants claim that reelection to public office gives priority
to any other right or interest, including
the police power of the State.
(People v. Jalosjos, 324 SCRA 689,
Feb. 3, 2000, En Banc [YnaresSantiago])
The Right against Unreasonable
Searches and Seizures
166. Discuss
the
constitutional
requirement that a judge, in issuing a
warrant of arrest, must determine
probable
cause
personally.
Distinguish determination of probable
cause
by
the
prosecutor
and
determination of probable cause by
the judge.
Held: It must be stressed that the
1987 Constitution requires the judge

to
determine
probable
cause
personally, a requirement which
does not appear in the corresponding
provisions
of
our
previous
constitutions. This emphasis evinces
the intent of the framers to place a
greater degree of responsibility upon
trial judges than that imposed under
previous Constitutions.
In Soliven v. Makasiar, this Court
pronounced:
What the Constitution underscores is
the
exclusive
and
personal
responsibility of the issuing judge to
satisfy himself of the existence of
probable cause. In satisfying himself
of the existence of probable cause for
the issuance of a warrant of arrest, the
judge is not required to personally
examine the complainant and his
witnesses.
Following established
doctrine and procedure, he shall: (1)
personally evaluate the report and the
supporting documents submitted by
the fiscal regarding the existence of
probable cause and, on the basis
thereof, issue a warrant of arrest; or
(2) if in the basis thereof he finds no
probable cause, he may disregard the
fiscals
report
and
require
the
submission of supporting affidavits of
witnesses to aid him in arriving at a
conclusion as to the existence of
probable cause.
Ho v. People (Ibid.) summarizes
existing jurisprudence on the matter
as follows:
Lest we be too repetitive, we only
wish to emphasize three vital matters
once more: First, as held in Inting, the
determination of probable cause by
the prosecutor is for a purpose
different from that which is to be
made by the judge. Whether there is
reasonable ground to believe that the
accused is guilty of the offense
charged and should be held for trial is
what the prosecutor passes upon. The
judge, on the other hand, determines
whether a warrant of arrest should be
issued against the accused, i.e.,
whether there is a necessity for
placing him under immediate custody
in order not to frustrate the ends of
justice. Thus, even if both should base
their findings on one and the same
proceeding or evidence, there should
be no confusion as to their distinct
objectives.
Second, since their objectives are
different, the judge cannot rely solely
on the report of the prosecutor in

finding probable cause to justify the


issuance of a warrant of arrest.
Obviously and understandably, the
contents of the prosecutors report will
support his own conclusion that there
is reason to charge the accused for an
offense and hold him for trial.
However, the judge must decide
independently. Hence, he must have
supporting evidence, other than the
prosecutors bare report, upon which
to legally sustain his own findings on
the existence (or nonexistence) of
probable cause to issue an arrest
order.
This
responsibility
of
determining
personally
and
independently
the
existence
or
nonexistence of probable cause is
lodged in him by no less than the most
basic law of the land. Parenthetically,
the prosecutor could ease the burden
of the judge and speed up the
litigation process by forwarding to the
latter not only the information and his
bare resolution finding probable cause,
but also so much of the records and
the evidence on hand as to enable the
His Honor to make his personal and
separate judicial finding on whether to
issue a warrant of arrest.
Lastly, it is not required that the
complete or entire records of the case
during the preliminary investigation be
submitted to and examined by the
judge. We do not intend to unduly
burden trial courts by obliging them to
examine the complete records of
every case all the time simply for the
purpose of ordering the arrest of an
accused. What is required, rather, is
that the judge must have sufficient
supporting documents (such as the
complaint,
affidavits,
counteraffidavits,
sworn
statements
of
witnesses or transcript of stenographic
notes, if any) upon which to make his
independent judgment or, at the very
least, upon which to verify the findings
of the prosecutor as to the existence
of probable cause. The point is: he
cannot rely solely and entirely on the
prosecutors
recommendation,
as
Respondent Court did in this case.
Although the prosecutor enjoys the
legal presumption of regularity in the
performance of his official duties and
functions, which in turn gives his
report the presumption of accuracy,
the
Constitution,
we
repeat,
commands the judge to personally
determine probable cause in the
issuance of warrants of arrest. This
Court has consistently held that a
judge fails in his bounden duty if he
relies merely on the certification or the

report of the investigating officer.


(Citations omitted)
In the case at bench, respondent
admits that he issued the questioned
warrant as there was no reason for
(him) to doubt the validity of the
certification made by the Assistant
Prosecutor
that
a
preliminary
investigation was conducted and that
probable cause was found to exist as
against
those
charged
in
the
information filed. The statement is
an admission that respondent relied
solely
and
completely
on
the
certification made by the fiscal that
probable cause exists as against those
charged in the information and issued
the challenged warrant of arrest on
the sole basis of the prosecutors
findings and recommendations.
He
adopted
the
judgment
of
the
prosecutor regarding the existence of
probable cause as his own. (Abdula
v. Guiani, 326 SCRA 1, Feb. 18,
2000, 3rd Div. [Gonzaga-Reyes])
167. Accused-appellant assails the
validity of his arrest and his
subsequent convictions for the two
crimes. Both the trial court and the
Court of Appeals found that the arrest
and subsequent seizure were legal.
Held: A review of the records at bar
shows no reason to depart therefrom.
The constitutional proscription,
that no person shall be arrested
without any warrant of arrest having
been issued prior thereto, is not a
hard-and-fast rule. X x x (Citations
omitted)
In the cases at bar, the police
saw the gun tucked in appellants
waist when he stood up. The gun was
plainly visible.
No search was
conducted as none was necessary.
Accused-appellant could not show any
license for the firearm, whether at the
time of his arrest or thereafter. Thus,
he was in effect committing a crime in
the presence of the police officers. No
warrant of arrest was necessary in
such a situation, it being one of the
recognized exceptions under the
Rules.

As a consequence of appellants
valid warrantless arrest, he may be
lawfully
searched
for
dangerous
weapons or anything which may be
used as proof of the commission of an
offense, without a search warrant, as
provided in Rule 126, Section 12. This
is a valid search incidental to a lawful
arrest. The subsequent discovery in
his car of drug paraphernalia and the
crystalline substance, which, was later
identified as shabu, though in a
distant place from where the illegal
possession of firearm was committed,
cannot be said to have been made
during an illegal search. As such, the
seized items do not fall within the
exclusionary clause x x x. Hence, not
being fruits of the poisonous tree x x x
the objects found at the scene of the
crime, such as the firearm, the shabu
and the drug paraphernalia, can be
used as evidence against appellant.
Besides, it has been held that drugs
discovered as a result of a consented
search is admissible in evidence.
(Citations omitted.) (People v. Go,
354 SCRA 338, Mar. 14, 2001, 1st
Div. [Ynares-Santiago])
168. In an application for search
warrant,
the
application
was
accompanied by a sketch of the
compound at 516 San Jose de la
Montana St., Mabolo, Cebu City,
indicating the 2-storey residential
house of private respondent with a
large X enclosed in a square. Within
the same compound are residences of
other people, workshops, offices,
factories and warehouse. The search
warrant issued, however, merely
indicated
the
address
of
the
compound which is 516 San Jose de la
Montana St., Mabolo, Cebu City. Did
this
satisfy
the
constitutional
requirement under Section 2, Article III
that the place to be searched must be
particularly described?
Held: This Court has held that the
applicant should particularly describe
the place to be searched and the
person or things to be seized,
wherever and whenever it is feasible.
In the present case, it must be noted
that the application for a search
warrant was accompanied by a sketch
of the compound at 516 San Jose de la
Montana St., Mabolo, Cebu City. The
sketch
indicated
the
2-storey
residential
house
of
private
respondent with a large "X" enclosed
in a square.
Within the same
compound are residences of other
people, workshops, offices, factories

and warehouse. With this sketch as


the guide, it could have been very
easy to describe the residential house
of private respondent with sufficient
particularity so as to segregate it from
the other buildings or structures inside
the same compound. But the search
warrant merely indicated the address
of the compound which is 516 San
Jose de la Montana St., Mabolo, Cebu
City. This description of the place to
be searched is too general and does
not pinpoint the specific house of
private respondent.
Thus, the
inadequacy of the description of the
residence
of
private
respondent
sought
to
be
searched
has
characterized the questioned search
warrant as a general warrant, which is
violative
of
the
constitutional
requirement.
(People v. Estrada,
296 SCRA 383, 400, [Martinez])
169. Can the place to be searched,
as set out in the warrant, be amplified
or modified by the officers own
personal knowledge of the premises,
or the evidence they adduce in
support of their application for the
warrant?
Held: Such a change is proscribed by
the Constitution which requires inter
alia the search warrant to particularly
describe the place to be searched as
well as the persons or things to be
seized. It would concede to police
officers the power of choosing the
place to be searched, even if it not be
that delineated in the warrant.
It
would open wide the door to abuse of
the search process, and grant to
officers executing a search warrant
that discretion which the Constitution
has precisely removed from them.
The particularization of the description
of the place to be searched may
properly be done only by the Judge,
and only in the warrant itself; it cannot
be left to the discretion of the police
officers conducting the search.
It is neither fair nor licit to allow police
officers to search a place different
from that stated in the warrant on the
claim that the place actually searched
although not that specified in the
warrant is exactly what they had in
view when they applied for the
warrant and had demarcated in their
supporting evidence. What is material
in determining the validity of a search
is the place stated in the warrant
itself, not what applicants had in their
thoughts, or had represented in the
proofs they submitted to the court
issuing the warrant.
(People v.

Court of Appeals, 291 SCRA 400,


June 26, 1998 [Narvasa])
170. What is search incidental to a
lawful arrest? Discuss.
Held:
While a contemporaneous
search of a person arrested may be
effected
to
discover
dangerous
weapons or proofs or implements used
in the commission of the crime and
which search may extend to the area
within his immediate control where he
might gain possession of a weapon or
evidence he can destroy, a valid arrest
must precede the search. The process
cannot be reversed.
In a search incidental to a lawful
arrest, as the precedent arrest
determines
the
validity
of
the
incidental search, the legality of the
arrest is questioned in a large majority
of these cases, e.g., whether an arrest
was merely used as a pretext for
conducting a search. In this instance,
the law requires that there be first a
lawful arrest before a search can be
made the process cannot be
reversed. (Malacat v. Court of Appeals,
283 SCRA 159, 175 [1997])
(People v. Chua Ho San, 308 SCRA
432, June 17, 1999, En Banc
[Davide, Jr., C.J.])
171. What is the plain view
doctrine?
What are its requisites?
Discuss.
Held: 1. Objects falling in plain view
of an officer who has a right to be in
the position to have that view are
subject to seizure even without a
search warrant and may be introduced
in evidence. The plain view doctrine
applies when the following requisites
concur: (a) the law enforcement officer
in search of the evidence has a prior
justification for an intrusion or is in a
position from which he can view a
particular area; (b) the discovery of
the evidence in plain view is
inadvertent; (c) it is immediately
apparent to the officer that the item
he observes may be evidence of a
crime,
contraband
or
otherwise
subject
to
seizure.
The
law
enforcement officer must lawfully

make an initial intrusion or properly be


in a position from which he can
particularly view the area.
In the
course of such lawful intrusion, he
came inadvertently across a piece of
evidence incriminating the accused.
The object must be open to eye and
hand and its discovery inadvertent.
It is clear that an object is in plain view
if the object itself is plainly exposed to
sight. The difficulty arises when the
object is inside a closed container.
Where the object seized was inside a
closed package, the object itself is not
in plain view and therefore cannot be
seized without a warrant. However, if
the package proclaims its contents,
whether
by
its
distinctive
configuration, its transparency, or if its
contents are obvious to an observer,
then the contents are in plain view
and may be seized. In other words, if
the package is such that an
experienced observer could infer from
its appearance that it contains the
prohibited article, then the article is
deemed in plain view. It must be
immediately apparent to the police
that the items that they observe may
be evidence of a crime, contraband or
otherwise subject to seizure. (People
v. Doria, 301 SCRA 668, Jan. 22,
1999, En Banc [Puno, J.])
2.
For the doctrine to apply, the
following elements must be present:
a)

a prior valid intrusion based


on the valid warrantless arrest in
which the police are legally present in
the pursuit of their official duties;
b)
the
evidence
was
inadvertently discovered by the police
who have the right to be where they
are; and
c)
the
evidence
must
be
immediately apparent; and
d)
plain view justified mere
seizure of evidence without further
search.
In the instant case, recall that PO2
Balut testified that they first located
the marijuana plants before appellant
was arrested without a warrant.
Hence, there was no valid warrantless
arrest which preceded the search of
appellants premises.
Note further
that the police team was dispatched to
appellants kaingin precisely to search
for and uproot the prohibited flora.
The seizure of evidence in plain view
applies only where the police officer is
not searching for evidence against the

accused, but inadvertently comes


across
an
incriminating
object.
Clearly, their discovery of the cannabis
plants was not inadvertent. We also
note the testimony of SPO2 Tipay that
upon arriving at the area, they first
had to look around the area before
they could spot the illegal plants.
Patently, the seized marijuana plants
were not immediately apparent and
further search was needed. In sum,
the marijuana plants in question were
not in plain view or open to eye and
hand.
The plain view doctrine,
thus, cannot be made to apply.
Nor can we sustain the trial courts
conclusion that just because the
marijuana plants were found in an
unfenced lot, appellant could not
invoke the protection afforded by the
Charter
against
unreasonable
searches by agents of the State. The
right against unreasonable searches
and seizures is the immunity of ones
person, which includes his residence,
his papers, and other possessions.
The guarantee refers to the right of
personal security of the individual. X
x x, what is sought to be protected
against the States unlawful intrusion
are persons, not places. To conclude
otherwise would not only mean
swimming against the stream, it would
also lead to the absurd logic that for a
person
to
be
immune
against
unreasonable searches and seizures,
he must be in his home or office,
within a fenced yard or a private
place. The Bill of Rights belongs as
much to the person in the street as to
the individual in the sanctuary of his
bedroom. (People v. Abe Valdez,
G.R. No. 129296, Sept. 25, 2000,
En Banc [Quisumbing])
3. Considering its factual milieu, this
case falls squarely under the plain
view doctrine. X x x.
When Spencer wrenched himself free
from the grasp of PO2 Gaviola, he
instinctively ran towards the house of
appellant. The members of the buybust team were justified in running
after him and entering the house
without a search warrant for they were
hot in the heels of a fleeing criminal.
Once inside the house, the police
officers
cornered
Spencer
and
recovered the buy-bust money from
him. They also caught appellant in
flagrante
delicto
repacking
the
marijuana bricks which were in full
view on top of a table. X x x.

Hence, appellants subsequent arrest


was likewise lawful, coming as it is
within the purview of Section 5(a) of
Rule 113 of the 1985 Rules on Criminal
Procedure x x x.
Section 5(a) is commonly referred to
as the rule on in flagrante delicto
arrests.
Here two elements must
concur: (1) the person to be arrested
must execute an overt act indicating
that he has just committed, is actually
committing, or is attempting to
commit a crime; and (2) such overt act
is done in the presence or within the
view of the arresting officer. Thus,
when appellant was seen repacking
the marijuana, the police officers were
not only authorized but also dutybound to arrest him even without a
warrant. (People v. Elamparo, 329
SCRA 404, 414-415, March 31,
2000, 2nd Div. [Quisumbing])
172. What
search?

is

stop-and-frisk

Held: 1. In the landmark case of


Terry v. Ohio (20 L Ed 2d 889; 88 S Ct
1868, 392 US 1, 900, June 10, 1968), a
stop-and-frisk was defined as the
vernacular designation of the right of a
police officer to stop a citizen on the
street, interrogate him, and pat him
for weapon(s):
x x x (W)here a police officer
observes an unusual conduct which
leads him reasonably to conclude in
light of his experience that criminal
activity may be afoot and that the
persons with whom he is dealing may
be armed and presently dangerous,
where in the course of investigating
this behavior he identified himself as a
policeman and make reasonable
inquiries, and where nothing in the
initial stages of the encounter serves
to dispel his reasonable fear for his
own or others safety, he is entitled for
the protection of himself or others in
the area to conduct a carefully limited
search of the outer clothing of such
persons in an attempt to discover
weapons which might be used to
assault him.
Such a search is a
reasonable search under the Fourth
Amendment, and any weapon seized
may properly be introduced in
evidence against the person from
whom they were taken. (Herrera, A
Handbook on Arrest, Search and
Seizure and Custodial Investigation,
1995 ed., p. 185; and Terry v. Ohio,
supra, p. 911)

In allowing such a search, the United


States Supreme Court held that the
interest of effective crime prevention
and detection allows a police officer to
approach a person, in appropriate
circumstances
and
manner,
for
purposes of investigating possible
criminal behavior even though there is
insufficient probable cause to make an
actual arrest.
In admitting in evidence two guns
seized during the stop-and-frisk, the
US Supreme Court held that what
justified the limited search was the
more immediate interest of the police
officer in taking steps to assure
himself that the person with whom he
was dealing was not armed with a
weapon that could unexpectedly and
fatally be used against him.
It did not, however, abandon the rule
that the police must, whenever
practicable, obtain advance judicial
approval of searches and seizures
through
the
warrant
procedure,
excused
only
by
exigent
circumstances. (Manalili v. CA, 280
SCRA
400,
Oct.
9,
1997
[Panganiban])
2. We now proceed to the justification
for and allowable scope of a stopand-frisk as a limited protective
search of outer clothing for weapons,
as laid down in Terry, thus:
We merely hold today that where a
police
officer
observes
unusual
conduct which leads him reasonably to
conclude in light of his experience that
criminal activity may be afoot and that
the persons with whom he is dealing
may
be
armed
and
presently
dangerous, where in the course of
investigating
this
behavior
he
identifies himself as a policeman and
makes reasonable inquiries, and where
nothing in the initial stages of the
encounter serves to dispel his
reasonable fear for his own or others
safety, he is entitled for the protection
of himself and others in the area to
conduct a carefully limited search of
the outer clothing of such persons in
an attempt to discover weapons which
might be used to assault him. Such a
search is a reasonable search under
the Fourth Amendment (Terry, at 911.
In fact, the Court noted that the sole
justification for a stop-and-frisk was
the protection of the police officer and
others nearby; while the scope of the
search conducted in the case was
limited to patting down the outer
clothing
of
petitioner
and
his

companions, the police officer did not


place his hands in their pockets nor
under the outer surface of their
garments until he had felt weapons,
and then he merely reached for and
removed the guns.
This did not
constitute a general exploratory
search, Id.)
Other notable points of Terry are that
while probable cause is not required to
conduct
a
stop-and-frisk,
it
nevertheless
holds
that
mere
suspicion or a hunch will not validate a
stop-and-frisk.
A genuine reason
must exist, in light of the police
officers experience and surrounding
conditions, to warrant the belief that
the person detained has weapons
concealed about him. Finally, a stopand-frisk serves a two-fold interest:
(1) the general interest of effective
crime prevention and detection, which
underlies the recognition that a police
officer
may,
under
appropriate
circumstances and in an appropriate
manner, approach a person for
purposes of investigating possible
criminal
behavior
even
without
probable cause; and (2) the more
pressing interest of safety and selfpreservation which permit the police
officer to take steps to assure himself
that the person with whom he deals is
not armed with a deadly weapon that
could unexpectedly and fatally be
used against the police officer.
(Malacat v. Court of Appeals, 283
SCRA 159, Dec. 12, 1997 [Davide])
173. Are searches
valid? Discuss.

at

checkpoints

Held: Accused-appellants assail the


manner by which the checkpoint in
question was conducted.
They
contend that the checkpoint manned
by elements of the Makati Police
should have been announced. They
also complain of its having been
conducted in an
arbitrary and
discriminatory manner.
We take judicial notice of the
existence of the COMELEC resolution
imposing a gun ban during the
election period issued pursuant to
Section 52(c) in relation to Section
26(q) of the Omnibus Election Code
(Batas Pambansa Blg. 881).
The
national and local elections in 1995
were held on 8 May, the second
Monday of the month. The incident,
which happened on 5 April 1995, was
well within the election period.

This Court has ruled that not all


checkpoints are illegal. Those which
are warranted by the exigencies of
public order and are conducted in a
way least intrusive to motorists are
allowed.
For, admittedly, routine
checkpoints do intrude, to a certain
extent, on motorists right to free
passage without interruption, but it
cannot be denied that, as a rule, it
involves only a brief detention of
travelers during which the vehicles
occupants are required to answer a
brief question or two. For as long as
the vehicle is neither searched nor its
occupants subjected to a body search,
and the inspection of the vehicle is
limited to a visual search, said routine
checks cannot be regarded as
violative of an individuals right
against unreasonable search. In fact,
these routine checks, when conducted
in a fixed area, are even less intrusive.
The
checkpoint
herein
conducted was in pursuance of the
gun ban enforced by the COMELEC.
The COMELEC would be hard put to
implement the ban if its deputized
agents were limited to a visual search
of pedestrians. It would also defeat
the purpose for which such ban was
instituted. Those who intend to bring
a gun during said period would know
that they only need a car to be able to
easily perpetrate their malicious
designs.
The facts adduced do not
constitute a ground for a violation of
the constitutional rights of the
accused against illegal search and
seizure. PO3 Suba admitted that they
were merely stopping cars they
deemed suspicious, such as those
whose windows are heavily tinted just
to see if the passengers thereof were
carrying guns. At best they would
merely direct their flashlights inside
the cars they would stop, without
opening the cars doors or subjecting
its passengers to a body search.
There is nothing discriminatory in this
as this is what the situation demands.
We see no need for checkpoints
to be announced x x x. Not only would
it be impractical, it would also
forewarn those who intend to violate
the ban.
Even so, badges of
legitimacy of checkpoints may still be
inferred from their fixed location and
the regularized manner in which they
are operated.
(People v. Usana,
323 SCRA 754, Jan. 28, 2000, 1st
Div. [Davide, CJ])

174. Do the ordinary rights against


unreasonable searches and seizures
apply to searches conducted at the
airport pursuant to routine airport
security procedures?
Held:
Persons may lose the
protection of the search and seizure
clause by exposure of their persons or
property to the public in a manner
reflecting
a
lack
of
subjective
expectation
of
privacy,
which
expectation society is prepared to
recognize as reasonable.
Such
recognition is implicit in airport
security procedures. With increased
concern over airplane hijacking and
terrorism has come increased security
at the nations airports. Passengers
attempting to board an aircraft
routinely
pass
through
metal
detectors; their carry-on baggage as
well as checked luggage are routinely
subjected to x-ray scans.
Should
these
procedures
suggest
the
presence
of
suspicious
objects,
physical searches are conducted to
determine what the objects are. There
is little question that such searches
are reasonable, given their minimal
intrusiveness, the gravity of the safety
interests involved, and the reduced
privacy expectations associated with
airline travel. Indeed, travelers are
often notified through airport public
address systems, signs, and notices in
their airline tickets that they are
subject to search and, if any prohibited
materials or substances are found,
such would be subject to seizure.
These
announcements
place
passengers on notice that ordinary
constitutional
protections
against
warrantless searches and seizures do
not
apply
to
routine
airport
procedures.
The packs of methamphetamine
hydrochloride
having
thus
been
obtained through a valid warrantless
search, they are admissible in
evidence
against
the
accusedappellant herein.
Corollarily, her
subsequent arrest, although likewise
without warrant, was justified since it
was effected upon the discovery and
recovery of shabu in her person in
flagrante delicto. (People v. Leila
Johnson, G.R. No. 138881, Dec.
18, 2000, 2nd Div. [Mendoza])
175. May
the
constitutional
protection
against
unreasonable
searches and seizures be extended to
acts committed by private individuals?

Held: As held in People v. Marti (193


SCRA 57 [1991]), the constitutional
protection
against
unreasonable
searches and seizures refers to the
immunity of one's person from
interference by government and it
cannot be extended to acts committed
by private individuals so as to bring it
within the ambit of alleged unlawful
intrusion. (People v. Mendoza, 301
SCRA 66, Jan. 18, 1999, 1st Div.
[Melo])
176. Should the seized drugs which
are pharmaceutically correct but not
properly documented subject of an
illegal search because the applicant
failed to allege in the application for
search warrant that the subject drugs
for which she was applying for search
warrant were either fake, misbranded,
adulterated, or unregistered, be
returned to the owner?
Held: With the State's obligation to
protect and promote the right to
health of the people and instill health
consciousness among them (Article II,
Section 15, 1987 Constitution), in
order to develop a healthy and alert
citizenry (Article XIV, Section 19[1]), it
became
mandatory
for
the
government to supervise and control
the proliferation of drugs in the
market. The constitutional mandate
that "the State shall adopt an
integrated
and
comprehensive
approach to health development
which shall endeavor to make
essential goods, health and other
social services available to all people
at affordable cost" (Article XIII, Section
11) cannot be neglected. This is why
"the State shall establish and maintain
an effective food and drug regulatory
system." (Article XIII, Section 12) The
BFAD is the government agency
vested by law to make a mandatory
and authoritative determination of the
true therapeutic effect of drugs
because it involves technical skill
which is within its special competence.
The health of the citizenry should
never be compromised.
To the
layman, medicine is a cure that may
lead to better health.
If the seized 52 boxes of drugs are
pharmaceutically correct but not
properly documented, they should be
promptly disposed of in the manner
provided by law in order to ensure that
the same do not fall into the wrong
hands who might use the drugs
underground.
Private respondent
cannot rely on the statement of the
trial court that the applicant "failed to

allege in the application for search


warrant that the subject drugs for
which she was applying for search
warrant were either fake, misbranded,
adulterated, or unregistered" in order
to obtain the return of the drugs. The
policy of the law enunciated in R.A. No.
8203 is to protect the consumers as
well as the licensed businessmen.
Foremost among these consumers is
the government itself which procures
medicines and distributes them to the
local communities through direct
assistance to the local health centers
or through outreach and charity
programs.
Only with the proper
government sanctions can medicines
and drugs circulate the market. We
cannot afford to take any risk, for the
life and health of the citizenry are as
precious as the existence of the State.
(People
v. Judge
Estrella T.
Estrada, G.R No. 124461, June 26,
2000, Spcl. 2nd Div. [YnaresSantiago])
177. Do Regional Trial Courts have
competence to pass upon the validity
or regularity of seizure and forfeiture
proceedings conducted by the Bureau
of Customs and to enjoin or otherwise
interfere with these proceedings?
Held: In Jao v. Court of Appeals (249
SCRA 35, 42-43 [1995]), this Court,
reiterating its rulings x x x said:
There is no question that Regional Trial
Courts are devoid of any competence
to pass upon the validity or regularity
of seizure and forfeiture proceedings
conducted by the Bureau of Customs
and to enjoin or otherwise interfere
with these proceedings. The Collector
of Customs sitting in seizure and
forfeiture proceedings has exclusive
jurisdiction to hear and determine all
questions touching on the seizure and
forfeiture of dutiable goods.
The
Regional Trial Courts are precluded
from assuming cognizance over such
matters even through petitions of
certiorari, prohibition or mandamus.
It is likewise well-settled that the
provisions of the Tariff and Customs
Code and that of Republic Act No.
1125, as amended, otherwise known
as An Act Creating the Court of Tax
Appeals, specify the proper fora and
procedure for the ventilation of any
legal objections or issues raised
concerning these proceedings. Thus,
actions of the Collector of Customs are
appealable to the Commissioner of
Customs, whose decision, in turn, is
subject to the exclusive appellate

jurisdiction of the Court of Tax Appeals


and from there to the Court of
Appeals.
The rule that Regional Trial Courts
have no review powers over such
proceedings is anchored upon the
policy of placing no unnecessary
hindrance on the governments drive,
not only to prevent smuggling and
other frauds upon Customs, but more
importantly, to render effective and
efficient the collection of import and
export duties due the State, which
enables the government to carry out
the functions it has been instituted to
perform.
Even if the seizure by the Collector of
Customs were illegal, x x x we have
said that such act does not deprive
the Bureau of Customs of jurisdiction
thereon.
Respondents cite the statement of the
Court of Appeals that regular courts
still retain jurisdiction where, as in
this case, for lack of probable cause,
there is serious doubt as to the
propriety of placing the articles under
Customs
jurisdiction
through
seizure/forfeiture proceedings. They
overlook the fact, however, that under
the law, the question of whether
probable cause exists for the seizure
of the subject sacks of rice is not for
the Regional Trial Court to determine.
The customs authorities do not have
to prove to the satisfaction of the
court that the articles on board a
vessel were imported from abroad or
are intended to be shipped abroad
before they may exercise the power to
effect customs searches, seizures, or
arrests provided by law and continue
with the administrative hearings. As
the Court held in Ponce Enrile v.
Vinuya (37 SCRA 381, 388-389 [1971],
reiterated in Jao v. Court of Appeals,
supra and Mison v. Natividad, 213
SCRA 734 [1992]):
The governmental agency concerned,
the Bureau of Customs, is vested with
exclusive authority.
Even if it be
assumed that in the exercise of such
exclusive competence a taint of
illegality may be correctly imputed,
the most that can be said is that under
certain circumstances the grave abuse
of discretion conferred may oust it of
such jurisdiction. It does not mean
however that correspondingly a court
of first instance is vested with
competence when clearly in the light
of the above decisions the law has not
seen fit to do so. The proceeding
before the Collector of Customs is not

final.
An appeal lies to the
Commissioner
of
Customs
and
thereafter to the Court of Tax Appeals.
It may even reach this Court through
the appropriate petition for review.
The proper ventilation of the legal
issues raised is thus indicated.
Certainly a court of first instance is not
therein included.
It is devoid of
jurisdiction.
(Bureau of Customs v. Ogario, 329
SCRA 289, 296-298, March 30,
2000, 2nd Div. [Mendoza])
178. Discuss the nature of an in
flagrante delicto warrantless arrest.
Illustrative case.
Held: In the case at bar, the
court a quo anchored its judgment of
conviction on a finding that the
warrantless
arrest
of
accusedappellants, and the subsequent search
conducted by the peace officers, are
valid
because
accused-appellants
were caught in flagrante delicto in
possession of prohibited drugs. This
brings us to the issue of whether or
not the warrantless arrest, search and
seizure in the present case fall within
the recognized exceptions to the
warrant requirement.
In People v. Chua Ho San, the
Court held that in cases of in flagrante
delicto arrests, a peace officer or a
private person may, without a warrant,
arrest a person when, in his presence,
the person to be arrested has
committed, is actually committing, or
is attempting to commit an offense.
The arresting office, therefore, must
have personal knowledge of such fact
or, as a recent case law adverts to,
personal knowledge of facts or
circumstances convincingly indicative
or constitutive of probable cause. As
discussed in People v. Doria, probable
cause means an actual belief or
reasonable grounds of suspicion. The
grounds of suspicion are reasonable
when, in the absence of actual belief
of the arresting officers, the suspicion
that the person to be arrested is
probably guilty of committing the
offense, is based on actual facts, i.e.,
supported
by
circumstances
sufficiently strong in themselves to
create the probable cause of guilt of
the person to be arrested.
A
reasonable suspicion therefore must
be founded on probable cause,
coupled with good faith on the part of
the peace officers making the arrest.

As applied to in flagrante delicto


arrests, it is settled that reliable
information alone, absent any overt
act indicative of a felonious enterprise
in the presence and within the view of
the arresting officers, are not sufficient
to constitute probable cause that
would justify an in flagrante delicto
arrest. Thus, in People v. Aminnudin
(163 SCRA 402, 409-410 [1988]), it
was held that the accused-appellant
was not, at the moment of his arrest,
committing a crime nor was it shown
that he was about to do so or that he
had just done so. What he was doing
was descending the gangplank of the
M/V Wilcon 9 and there was no
outward indication that called for his
arrest. To all appearances, he was like
any
of
the
other
passengers
innocently disembarking from the
vessel. It was only when the informer
pointed to him as the carrier of the
marijuana that he suddenly became
suspect
and
so
subject
to
apprehension.
Likewise, in People v. Mengote
(210 SCRA 174, 179-180 [1992]), the
Court did not consider eyes . . .
darting from side to side . . . [while]
holding . . . [ones] abdomen, in a
crowded street at 11:30 in the
morning,
as
overt
acts
and
circumstances sufficient to arouse
suspicion and indicative of probable
cause. According to the Court, [b]y
no stretch of the imagination could it
have been inferred from these acts
that an offense had just been
committed, or was actually being
committed, or was at least being
attempted in [the arresting officers]
presence.
So also, in People v.
Encinada (280 SCRA 72, 86-87
[1997]), the Court ruled that no
probable cause is gleanable from the
act of riding a motorela while holding
two plastic baby chairs.
Then, too, in Malacat v. Court of
Appeals (283 SCRA 159 [1997]), the
trial court concluded that petitioner
was attempting to commit a crime as
he was standing at the corner of
Plaza Miranda and Quezon Boulevard
with his eyes moving very fast and
looking at every person that come
(sic) nearer (sic) to them.
In
declaring
the
warrantless
arrest
therein illegal, the Court said:
Here, there could have been no valid
in flagrante delicto arrest preceding
the search in light of the lack of
personal knowledge on the part of Yu,
the arresting officer, or an overt

physical act, on the part of petitioner,


indicating that a crime had just been
committed, was being committed or
was going to be committed. (Id., at
175)
It went on to state that
Second,
there
was
nothing
in
petitioners behavior or conduct which
could have reasonably elicited even
mere suspicion other than that his
eyes were moving very fast an
observation
which
leaves
us
incredulous
since
Yu
and
his
teammates
were
nowhere
near
petitioner and it was already 6:60
p.m.,
thus
presumably
dusk.
Petitioner and his companions were
merely standing at the corner and
were not creating any commotion or
trouble . . .
Third, there was at all no ground,
probable or otherwise, to believe that
petitioner was armed with a deadly
weapon. None was visible to Yu, for as
he admitted, the alleged grenade was
discovered
inside
the
front
waistline of petitioner, and from all
indications as to the distance between
Yu and petitioner, any telltale bulge,
assuming that petitioner was indeed
hiding a grenade, could not have been
visible to Yu. (Id., at 178).
Clearly, to constitute a valid in
flagrante delicto arrest, two requisites
must concur: (1) the person to be
arrested must execute an overt act
indicating that he has just committed,
is
actually
committing,
or
is
attempting to commit a crime; and (2)
such overt act is done in the presence
or within the view of the arresting
officer (Concurring Opinion of Justice
Artemio V. Panganiban in People v.
Doria, 301 SCRA 668, 720 [1999]).
In the case at bar, accused-appellants
manifested no outward indication that
would justify their arrest. In holding a
bag on board a trisikad, accusedappellants could not be said to be
committing, attempting to commit or
have committed a crime. It matters
not that accused-appellant Molina
responded Boss, if possible we will
settle this to the request of SPO1
Pamplona to open the bag.
Such
response which allegedly reinforced
the suspicion of the arresting officers

that
accused-appellants
were
committing a crime, is an equivocal
statement which standing alone will
not constitute probable cause to effect
an in flagrante delicto arrest. Note
that were it not for SPO1 Marino
Paguidopon (who did not participate in
the arrest but merely pointed accusedappellants to the arresting officers),
accused-appellants could not be
subject of any suspicion, reasonable or
otherwise.
While SPO1 Paguidopon claimed that
he and his informer conducted a
surveillance
of
accused-appellant
Mula, SPO1 Paguidopon, however,
admitted that he only learned Mulas
name and address after the arrest.
What is more, it is doubtful if SPO1
Paguidopon
indeed
recognized
accused-appellant Mula. It is worthy
to note that, before the arrest, he was
able to see Mula in person only once,
pinpointed to him by his informer
while they were on the side of the
road. These circumstances could not
have afforded SPO1 Paguidopon a
closer look at accused-appellant Mula,
considering that the latter was then
driving a motorcycle when SPO1
Paguidopon caught a glimpse of him.
With respect to accused-appellant
Molina, SPO1 Paguidopon admitted
that he had never seen him before the
arrest.
This belies the claim of SPO1
Pamplona that he knew the name of
accused-appellants even before the
arrest x x x.
The aforesaid testimony of SPO1
Pamplona,
therefore,
is
entirely
baseless. SPO1 Pamplona could not
have learned the name of accusedappellants from SPO1 Paguidopon
because Paguidopon himself, who
allegedly conducted the surveillance,
was not even aware of accusedappellants name and address prior to
the arrest.
Evidently, SPO1 Paguidopon, who
acted as informer of the arresting
officers, more so the arresting officers

themselves, could not have been


certain of accused-appellants identity,
and were, from all indications, merely
fishing for evidence at the time of the
arrest.
Compared to People v. Encinada, the
arresting officer in the said case knew
appellant Encinada even before the
arrest because of the latters illegal
gambling activities, thus, lending at
least a semblance of validity on the
arrest effected by the peace officers.
Nevertheless, the Court declared in
said case that the warrantless arrest
and the consequent search were
illegal,
holding
that
[t]he
prosecutions evidence did not show
any suspicious behavior when the
appellant disembarked from the ship
or while he rode the motorela. No act
or fact demonstrating a felonious
enterprise could be ascribed to
appellant
under
such
bare
circumstances. (People v. Encinada,
supra.)
Moreover, it could not be said that
accused-appellants waived their right
against unreasonable searches and
seizure. Implied acquiescence to the
search, if there was any, could not
have been more than mere passive
conformity given under intimidating or
coercive circumstances and is thus
considered no consent at all within the
purview
of
the
constitutional
guarantee (Id., at 91; citing Aniag v.
Commission on Elections, 237 SCRA
424, 436-437 [1994]).
Withal, the Court holds that the arrest
of accused-appellants does not fall
under the exceptions allowed by the
rules. Hence, the search conducted
on their person was likewise illegal.
Consequently, the marijuana seized by
the peace officers could not be
admitted
as
evidence
against
accused-appellants, and the Court is
thus, left with no choice but to find in
favor of accused-appellants. (People
v. Molina, 352 SCRA 174, Feb. 19,
2001, En Banc [Ynares-Santiago])
The Privacy of Communications
and Correspondence

179. Private respondent Rafael S.


Ortanez filed with the Regional Trial
Court of Quezon City a complaint for
annulment of marriage with damages
against petitioner Teresita SalcedoOrtanez, on grounds of lack of
marriage license and/or psychological
incapacity of the petitioner. Among
the exhibits offered by private
respondent were three (3) cassette
tapes
of
alleged
telephone
conversations between petitioner and
unidentified persons. The trial court
issued the assailed order admitting all
of the evidence offered by private
respondent, including tape recordings
of
telephone
conversations
of
petitioner with unidentified persons.
These tape recordings were made and
obtained when private respondent
allowed his friends from the military to
wire tap his home telephone. Did the
trial court act properly when it
admitted in evidence said tape
recordings?
Held: Republic Act No. 4200 entitled
"An Act to Prohibit and Penalize Wire
Tapping and Other Related Violations
of the Privacy of Communication, and
For Other Purposes" expressly makes
such tape recordings inadmissible in
evidence. x x x.
Clearly, respondent trial court and
Court of Appeals failed to consider the
afore-quoted provisions of the law in
admitting in evidence the cassette
tapes in question.
Absent a clear
showing that both parties to the
telephone conversations allowed the
recording
of
the
same,
the
inadmissibility of the subject tapes is
mandatory under Rep. Act No. 4200.
Additionally, it should be mentioned
that the above-mentioned Republic
Act in Section 2 thereof imposes a
penalty of imprisonment of not less
than six (6) months and up to six (6)
years for violation of said Act.
(Salcedo-Ortanez
v.
Court
of
Appeals, 235 SCRA 111, Aug. 4,
1994 [Padilla])
The Right to Privacy
180. Is there a constitutional right to
privacy?
Held: The essence of privacy is the
right to be let alone. In the 1965
case of Griswold v. Connecticut (381
U.S. 479, 14 l. ed. 2D 510 [1965]), the
United States Supreme Court gave
more substance to the right of privacy

when it ruled that the right has a


constitutional foundation. It held that
there is a right of privacy which can be
found within the penumbras of the
First, Third, Fourth, Fifth and Ninth
Amendments. In the 1968 case of
Morfe v. Mutuc (22 SCRA 424, 444445), we adopted the Griswold ruling
that there is a constitutional right to
privacy.
The SC clarified that the right of
privacy is recognized and enshrined in
several provisions of our Constitution.
It is expressly recognized in Section
3(1) of the Bill of Rights. Other facets
of the right to privacy are protected in
various provisions of the Bill of Rights,
i.e., Secs. 1, 2, 6, 8, and 17. (Ople v.
Torres, G.R. No. 127685, July 23,
1998 [Puno])
135. Identify the zones of privacy
recognized and protected in our laws.
Held: The Civil Code provides that
[e]very person shall respect the
dignity, personality, privacy and peace
of mind of his neighbors and other
persons and punishes as actionable
torts several acts by a person of
meddling and prying into the privacy
of another.
It also holds a public
officer or employee or any private
individual liable for damages for any
violation of the rights and liberties of
another person, and recognizes the
privacy of letters and other private
communications. The Revised Penal
Code makes a crime the violation of
secrets by an officer, the revelation of
trade and industrial secrets, and
trespass to dwelling.
Invasion of
privacy is an offense in special laws
like the Anti-Wiretapping Law (R.A.
4200), the Secrecy of Bank Deposits
(R.A. 1405) and the Intellectual
Property Code (R.A. 8293). The Rules
of Court on privileged communication
likewise recognize the privacy of
certain information (Section 24, Rule
130[c], Revised Rules on Evidence).
(Ople v. Torres, G.R. No. 127685,
July 23, 1998 [Puno])
180. Discuss
why
Administrative
Order No. 308 (issued by the President
prescribing for a National ID system
for all citizens to facilitate business
transactions
with
government
agencies engaged in the delivery of
basic services and social security
provisions)
should
be
declared
unconstitutional.
Held: We prescind from the premise
that the right to privacy is a

fundamental right guaranteed by the


Constitution, hence, it is the burden of
government to show that A.O. No. 308
is justified by some compelling state
interest and that it is narrowly drawn.
A.O. No. 308 is predicated on two
considerations: (1) the need to provide
our citizens and foreigners with the
facility
to
conveniently
transact
business with basic service and social
security
providers
and
other
government instrumentalities and (2)
the need to reduce, if not totally
eradicate, fraudulent transactions and
misrepresentations
by
persons
seeking basic services. It is debatable
whether
these
interests
are
compelling enough to warrant the
issuance of A.O. No. 308. But what is
not arguable is the broadness, the
vagueness, the overbreadth of A.O.
No. 308 which if implemented will put
our peoples right to privacy in clear
and present danger.
The heart of A.O. No. 308 lies in its
Section 4 which provides for a
Population Reference Number (PRN) as
a common reference number to
establish a linkage among concerned
agencies
through
the
use
of
Biometrics
Technology
and
computer application designs.
It is noteworthy that A.O. No. 308 does
not state what specific biological
characteristics and what particular
biometrics technology shall be used to
identify people who will seek its
coverage. Considering the banquet of
options available to the implementors
of A.O. No. 308, the fear that it
threatens the right to privacy of our
people is not groundless.
A.O. No. 308 should also raise our
antennas for a further look will show
that it does not state whether
encoding of data is limited to
biological
information
alone
for
identification purposes.
X x x.
Clearly, the indefiniteness of A.O. No.
308 can give the government the
roving authority to store and retrieve
information for a purpose other than
the identification of the individual
through his PRN.
The potential for misuse of the data to
be gathered under A.O. No. 308
cannot be underplayed x x x. The
more frequent the use of the PRN, the
better the chance of building a huge
and formidable information base
through the electronic linkage of the
files. The data may be gathered for
gainful
and
useful
government

purposes; but the existence of this


vast reservoir of personal information
constitutes a covert invitation to
misuse, a temptation that may be too
great for some of our authorities to
resist.
It is plain and we hold that A.O. No.
308 falls short of assuring that
personal information which will be
gathered about our people will only be
processed for unequivocally specified
purposes.
The lack of proper
safeguards in this regard of A.O. No.
308 may interfere with the individuals
liberty of abode and travel by enabling
authorities
to
track
down
his
movement; it may also enable
unscrupulous
persons
to
access
confidential
information
and
circumvent the right against selfincrimination; it may pave the way for
fishing expeditions by government
authorities and evade the right against
unreasonable searches and seizures.
The possibilities of abuse and misuse
of the PRN, biometrics and computer
technology are accentuated when we
consider that the individual lacks
control over what can be read or
placed on his ID, much less verify the
correctness of the data encoded. They
threaten the very abuses that the Bill
of Rights seeks to prevent.
The ability of a sophisticated data
center to generate a comprehensive
cradle-to-grave
dossier
on
an
individual and transmit it over a
national network is one of the most
graphic threats of the computer
revolution. The computer is capable
of producing a comprehensive dossier
on individuals out of information given
at different times and for varied
purposes. X x x. Retrieval of stored
data is simple. When information of a
privileged character finds its way into
the computer, it can be extracted
together with other data on the
subject.
Once
extracted,
the
information is putty in the hands of
any person.
The end of privacy
begins.
[T]he Court will not be true to its role
as the ultimate guardian of the
peoples liberty if it would not
immediately smother the sparks that
endanger their rights but would rather
wait for the fire that could consume
them.
[A]nd we now hold that when the
integrity of a fundamental right is at
stake, this Court will give the
challenged law, administrative order,

rule or regulation a stricter scrutiny. It


will not do for the authorities to invoke
the presumption of regularity in the
performance of official duties. Nor is it
enough for the authorities to prove
that their act is not irrational for a
basic right can be diminished, if not
defeated, even when the government
does not act irrationally. They must
satisfactorily show the presence of
compelling state interest and that the
law, rule, or regulation is narrowly
drawn to preclude abuses.
This
approach is demanded by the 1987
Constitution whose entire matrix is
designed to protect human rights and
to prevent authoritarianism. In case of
doubt, the least we can do is to lean
towards the stance that will not put in
danger the rights protected by the
Constitution.
The right to privacy is one of the most
threatened rights of man living in a
mass society. The threats emanate
from various sources governments,
journalists,
employers,
social
scientists, etc. In the case at bar, the
threat comes from the executive
branch of government which by
issuing A.O. No. 308 pressures the
people to surrender their privacy by
giving information about themselves
on the pretext that it will facilitate
delivery of basic services. Given the
record-keeping power of the computer,
only the indifferent will fail to perceive
the danger that A.O. No. 308 gives the
government the power to compile a
devastating
dossier
against
unsuspecting citizens. X x x [W]e
close with the statement that the right
to privacy was not engraved in our
Constitution for flattery.
(Ople v.
Torres, G.R. No. 127685, July 23,
1998 [Puno])
181. Should in camera inspection of
bank accounts be allowed? If in the
affirmative, under what circumstances
should it be allowed?
Held: The issue is whether petitioner
may be cited for indirect contempt for
her failure to produce the documents
requested by the Ombudsman. And
whether the order of the Ombudsman
to have an in camera inspection of the
questioned account is allowed as an
exception to the law on secrecy of
bank deposits (R.A. No. 1405).
An examination of the secrecy of bank
deposits law (R.A. No. 1405) would
reveal the following exceptions:

1)
2)
3)
4)
5)

Where
the
depositor
consents in writing;
Impeachment cases;
By court order in bribery or
dereliction of duty cases against public
officials;
Deposit
is
subject
of
litigation;
Sec. 8, R.A. No. 3019, in
cases of unexplained wealth as held in
the case of PNB v. Gancayco (122 Phil.
503, 508 [1965]).
The order of the Ombudsman to
produce for in camera inspection the
subject accounts with the Union Bank
of the Philippines, Julia Vargas Branch,
is based on a pending investigation at
the Office of the Ombudsman against
Amado Lagdameo, et. al. for violation
of R.A. No. 3019, Sec. 3 (e) and (g)
relative
to
the
Joint
Venture
Agreement between the Public Estates
Authority and AMARI.
We rule that before an in camera
inspection may be allowed, there must
be a pending case before a court of
competent jurisdiction. Further, the
account must be clearly identified, the
inspection limited to the subject
matter of the pending case before the
court of competent jurisdiction. The
bank personnel and the account
holder must be notified to be present
during the inspection, and such
inspection may cover only the account
identified in the pending case.
In Union Bank of the Philippines v.
Court of Appeals, we held that
Section 2 of the Law on Secrecy of
Bank Deposits, as amended, declares
bank deposits to be absolutely
confidential except:

In an examination made in
the course of a special or general
examination of a bank that is
specifically
authorized
by
the
Monetary Board after being satisfied
that there is reasonable ground to
believe that a bank fraud or serious
irregularity has been or is being
committed and that it is necessary to
look into the deposit to establish such
fraud or irregularity,
2)
In an examination made by
an independent auditor hired by the
bank to conduct its regular audit
provided that the examination is for
audit purposes only and the results
thereof shall be for the exclusive use
of the bank,
3)
Upon written permission of
the depositor,
4)
In cases of impeachment,

5)

Upon order of a competent


court in cases of bribery or dereliction
of duty of public officials, or
6)
In cases where the money
deposited or invested is the subject
matter of the litigation.
In the case at bar, there is yet no
pending litigation before any court of
competent authority. What is existing
is an investigation by the Office of the
Ombudsman. In short, what the Office
of the Ombudsman would wish to do is
to fish for additional evidence to
formally charge Amado Lagdameo, et.
al., with the Sandiganbayan. Clearly,
there was no pending case in court
which would warrant the opening of
the bank account for inspection.
Zones of privacy are recognized and
protected in our laws. The Civil Code
provides that [e]very person shall
respect
the
dignity,
personality,
privacy and peace of mind of his
neighbors and other persons and
punishes as actionable torts several
acts for meddling and prying into the
privacy of another. It also holds public
officer or employee or any private
individual liable for damages for any
violation of the rights and liberties of
another person, and recognizes the
privacy of letters and other private
communications. The Revised Penal
Code makes a crime of the violation of
secrets by an officer, revelation of
trade and industrial secrets, and
trespass to dwelling.
Invasion of
privacy is an offense in special laws
like the anti-Wiretapping Law, the
Secrecy of Bank Deposits Act, and the
Intellectual Property Code. (Lourdes
T. Marquez v. Hon. Aniano A.
Desierto, G.R. No. 135882, June
27, 2001, En Banc [Pardo])

1)

Freedom of Expression
182. Distinguish
content-based
restrictions on free speech from
content-neutral
restrictions,
and
give example of each.
Held: Content-based restrictions are
imposed because of the content of the
speech and are, therefore, subject to
the clear-and-present danger test. For
example, a rule such as that involved
in Sanidad v. Comelec (181 SCRA 529
[1990]),
prohibiting
columnists,
commentators, and announcers from
campaigning either for or against an
issue in a plebiscite must have
compelling reason to support it, or it
will not pass muster under strict

scrutiny.
These restrictions are
censorial and therefore they bear a
heavy presumption of constitutional
invalidity. In addition, they will be
tested for possible overbreadth and
vagueness.
Content-neutral restrictions, on the
other hand, like Sec. 11(b) of R.A. No.
6646, which prohibits the sale or
donation of print space and air time to
political
candidates
during
the
campaign period, are not concerned
with the content of the speech. These
regulations need only a substantial
governmental interest to support
them.
A deferential standard of
review will suffice to test their validity.
The clear-and-present danger rule is
inappropriate as a test for determining
the constitutional validity of laws, like
Sec. 11(b) of R.A. No. 6646, which are
not concerned with the content of
political ads but only with their
incidents.
To apply the clear-andpresent danger test to such regulatory
measures would be like using a
sledgehammer to drive a nail when a
regular hammer is all that is needed.
The test for this difference in the level
of justification for the restriction of
speech
is
that
content-based
restrictions distort public debate, have
improper motivation, and are usually
imposed because of fear of how
people will react to a particular
speech.
No such reasons underlie
content-neutral
regulations,
like
regulation of time, place and manner
of holding public assemblies under B.P.
Blg. 880, the Public Assembly Act of
1985. (Osmena v. COMELEC, 288
SCRA
447,
March
31,
1998
[Mendoza])
183. Does the conduct of exit poll by
ABS CBN present a clear and present
danger of destroying the credibility
and integrity of the electoral process
as it has the tendency to sow
confusion considering the randomness
of
selecting
interviewees,
which
further makes the exit poll highly
unreliable, to justify the promulgation
of a Comelec resolution prohibiting the
same?
Held:
Such arguments are purely
speculative and clearly untenable.
First, by the very nature of a survey,
the interviewees or participants are
selected at random, so that the results
will
as
much
as
possible
be
representative or reflective of the
general sentiment or view of the
community or group polled. Second,

the survey result is not meant to


replace or be at par with the official
Comelec count. It consists merely of
the opinion of the polling group as to
who the electorate in general has
probably voted for, based on the
limited data gathered from polled
individuals. Finally, not at stake are
the credibility and the integrity of the
elections, which are exercises that are
separate and independent from the
exit polls.
The holding and the
reporting of the results of exit polls
cannot undermine those of the
elections, since the former is only part
of the latter. If at all, the outcome of
one can only be indicative of the other.
The COMELECs concern with the
possible noncommunicative effect of
exit polls disorder and confusion in
the voting centers does not justify a
total ban on them. Undoubtedly, the
assailed Comelec Resolution is too
broad, since its application is without
qualification as to whether the polling
is disruptive or not.
There is no
showing, however, that exit polls or
the means to interview voters cause
chaos in voting centers. Neither has
any evidence been presented proving
that the presence of exit poll reporters
near an election precinct tends to
create disorder or confuse the voters.
Moreover, the prohibition incidentally
prevents the collection of exit poll data
and their use for any purpose. The
valuable information and ideas that
could be derived from them, based on
the voters answers to the survey
questions will forever remain unknown
and unexplored. Unless the ban is
restrained, candidates, researchers,
social scientists and the electorate in
general would be deprived of studies
on the impact of current events and of
election-day and other factors on
voters choices.
The absolute ban imposed by the
Comelec
cannot,
therefore,
be
justified. It does not leave open any
alternative channel of communication
to gather the type of information
obtained through exit polling. On the
other hand, there are other valid and
reasonable ways and means to
achieve the Comelec end of avoiding
or minimizing disorder and confusion
that may be brought about by exit
surveys.
With
foregoing
premises,
it
is
concluded that the interest of the
state
in reducing
disruption
is
outweighed by the drastic abridgment

of the constitutionally guaranteed


rights of the media and the electorate.
Quite
the
contrary,
instead
of
disrupting elections, exit polls
properly conducted and publicized
can be vital tools for the holding of
honest, orderly, peaceful and credible
elections; and for the elimination of
election-fixing,
fraud
and
other
electoral
ills.
(ABS-CBN
Broadcasting
Corporation
v.
COMELEC, G.R. No. 133486, Jan.
28, 2000, En Banc [Panganiban])
184. Section 5.4 of R.A. No. 9006
(Fair Election Act) which provides:
Surveys affecting national candidates
shall not be published fifteen (15) days
before an election and surveys
affecting local candidates shall not be
published seven (7) days before an
election.
The Social Weather
Stations, Inc. (SWS), a private nonstock,
non-profit
social
research
institution conducting surveys in
various
fields;
and
Kamahalan
Publishing Corporation, publisher of
the Manila Standard, a newspaper of
general circulation, which features
newsworthy items of information
including election surveys, challenged
the constitutionality of aforesaid
provision as it constitutes a prior
restraint on the exercise of freedom of
speech without any clear and present
danger to justify such restraint.
Should the challenge be sustained?

Nor may it be argued that because of


Art. IX-C, Sec. 4 of the Constitution,
which gives the Comelec supervisory
power to regulate the enjoyment or
utilization
of
franchise
for
the
operation of media of communication,
no presumption of invalidity attaches
to a measure like Sec. 5.4. For as we
have pointed out in sustaining the ban
on media political advertisements, the
grant of power to the Comelec under
Art. IX-C, Sec. 4 is limited to ensuring
equal opportunity, time, space, and
the right to reply as well as uniform
and reasonable rates of charges for
the use of such media facilities for
public information campaigns and
forums among candidates.
Xxx
Nor can the ban on election surveys
be justified on the ground that there
are other countries x x x which
similarly impose restrictions on the
publication of election surveys.
At
best this survey is inconclusive. It is
noteworthy that in the United States
no restriction on the publication of
election survey results exists.
It
cannot be argued that this is because
the United States is a mature
democracy. Neither are there laws
imposing an embargo on survey
results, even for a limited period, in
other countries. X x x.

Held: For reason hereunder given, we


hold that Section 5.4 of R.A. No. 9006
constitutes
an
unconstitutional
abridgment of freedom of speech,
expression, and the press.

What test should then be employed to


determine the constitutional validity of
Section 5.4?
The United States
Supreme Court x x x held in United
States v. O Brien:

To be sure, Section 5.4 lays a prior


restraint on freedom of speech,
expression,
and
the
press
by
prohibiting the publication of election
survey results affecting candidates
within the prescribed periods of fifteen
(15) days immediately preceding a
national election and seven (7) days
before a local election. Because of the
preferred status of the constitutional
rights of speech, expression, and the
press, such a measure is vitiated by a
weighty presumption of invalidity.
Indeed, any system of prior restraints
of expression comes to this Court
bearing a heavy presumption against
its constitutional validity x x x. The
Government thus carries a heavy
burden of showing justification for the
enforcement of such restraint. There
is thus a reversal of the normal
presumption of validity that inheres in
every legislation.

[A]
government
regulation
is
sufficiently justified (1) if it is within
the constitutional power of the
government; (2) if it furthers an
important or substantial governmental
interest; (3) if the governmental
interest
is
unrelated
to
the
suppression of free expression; and (4)
if the incidental restriction on alleged
First Amendment freedoms (of speech,
expression and press) is no greater
than is essential to the furtherance of
that interest (391 U.S. 367, 20 L. Ed.
2d 692, 680 [1968] [bracketed
numbers added]).
This is so far the most influential test
for distinguishing content-based from
content-neutral regulations and is said
to have become canonical in the
review of such laws. It is noteworthy
that the O Brien test has been applied
by this Court in at least two cases

(Adiong v. Comelec, 207 SCRA 712


[1992]; Osmena v. Comelec, supra.).
Under this test, even if a law furthers
an
important
or
substantial
governmental interest, it should be
invalidated if such governmental
interest is not unrelated to the
suppression of free expression.
Moreover, even if the purpose is
unrelated to the suppression of free
speech, the law should nevertheless
be invalidated if the restriction on
freedom of expression is greater than
is
necessary
to
achieve
the
governmental purpose in question.
Our inquiry should accordingly focus
on these two considerations as applied
to Sec. 5.4.
First. Sec. 5.4 fails to meet criterion
(3) of the O Brien test because the
causal connection of expression to the
asserted governmental interest makes
such interest not unrelated to the
suppression of free expression. By
prohibiting the publication of election
survey
results
because
of
the
possibility that such publication might
undermine the integrity of the
election, Sec. 5.4 actually suppresses
a whole class of expression, while
allowing the expression of opinion
concerning the same subject matter
by newspaper columnists, radio and
TV commentators, armchair theorists,
and other opinion makers. In effect,
Sec. 5.4 shows a bias for a particular
subject matter, if not viewpoint, by
preferring
personal
opinion
to
statistical results. The constitutional
guarantee of freedom of expression
means that the government has no
power to restrict expression because
of its message, its ideas, its subject
matter, or its contents.
The
inhibition of speech should be upheld
only if the expression falls within one
of the few unprotected categories
dealt with in Chaplinsky v. New
Hampshire (315 U.S. 568, 571-572, 86
L. Ed. 1031, 1035 [1942]), thus:
There are certain well-defined and
narrowly limited classes of speech, the
prevention and punishment of which
have never been thought to raise any
Constitutional problem. These include
the lewd and obscene, the profane,
the libelous, and the insulting or
fighting words those which by their
very utterance inflict injury or tend to
incite an immediate breach of the
peace.
[S]uch utterances are no
essential part of any exposition of
ideas, and are of such slight social

value as a step to truth that any


benefit that may be derived from them
is clearly outweighed by the social
interest in order and morality.
Nor is there justification for the prior
restraint which Sec. 5.4 lays on
protected speech.
In Near v.
Minnesota (283 U.S. 697, 715-716, 75
l. Ed. 1357, 1367 [1931]), it was held:
[T]he protection even as to previous
restraint is not absolutely unlimited.
But the limitation has been recognized
only in exceptional cases x x x. No
one would question but that a
government might prevent actual
obstruction to its recruiting service or
the publication of the sailing dates of
transports or the number and location
of troops. On similar grounds, the
primary requirements of decency may
be
enforced
against
obscene
publications.
The security of the
community life may be protected
against incitements to acts of violence
and the overthrow by force of orderly
government x x x.
Thus, x x x the prohibition imposed by
Sec. 5.4 cannot be justified on the
ground that it is only for a limited
period and is only incidental.
The
prohibition may be for a limited time,
but the curtailment of the right of
expression is direct, absolute, and
substantial.
It constitutes a total
suppression of a category of speech
and is not made less so because it is
only for a period of fifteen (15) days
immediately before a national election
and seven (7) days immediately
before a local election.
This sufficiently distinguishes Sec. 5.4
from R.A. No. 6646, Sec. 11(b), which
this Court found to be valid in National
Press Club v. Comelec (supra.), and
Osmena v. Comelec (supra.). For the
ban imposed by R.A. No. 6646, Sec.
11(b) is not only authorized by a
specific constitutional provision (Art.
IX-C, Sec. 4), but it also provided an
alternative so that, as this Court
pointed out in Osmena, there was
actually no ban but only a substitution
of media advertisements by the
Comelec space, and Comelec hour.
Second.
Even if the governmental
interest sought to be promoted is
unrelated to the suppression of speech
and the resulting restriction of free
expression is only incidental, Sec. 5.4
nonetheless fails to meet criterion (4)
of the O Brien test, namely, that the
restriction be not greater than is

necessary to further the governmental


interest. As already stated, Sec. 5.4.
aims at the prevention of last-minute
pressure on voters, the creation of
bandwagon effect, junking of weak
or losing candidates, and resort to
the form of election cheating called
dagdag-bawas.
Praiseworthy as
these aims of the regulation might be,
they cannot be attained at the
sacrifice of the fundamental right of
expression, when such aim can be
more narrowly pursued by punishing
unlawful acts, rather than speech
because of apprehension that such
speech creates the danger of such
evils. Thus, under the Administrative
Code of 1987 (Bk. V, Tit. I, Subtit. C,
Ch 1, Sec. 3[1]), the Comelec is given
the power:
To stop any illegal activity, or
confiscate, tear down, and stop any
unlawful, libelous, misleading or false
election propaganda, after due notice
and hearing.
This is surely a less restrictive means
than the prohibition contained in Sec.
5.4. Pursuant to this power of the
Comelec, it can confiscate bogus
survey results calculated to mislead
voters.
Candidates can have their
own surveys conducted. No right of
reply can be invoked by others. No
principle of equality is involved. It is a
free market to which each candidate
brings his ideas. As for the purpose of
the law to prevent bandwagon effects,
it is doubtful whether the Government
can deal with this natural-enough
tendency of some voters.
Some
voters want to be identified with the
winners. Some are susceptible to
the herd mentality.
Can these be
legitimately prohibited by suppressing
the publication of survey results which
are a form of expression? It has been
held
that
[mere]
legislative
preferences or beliefs respecting
matters of public convenience may
well support regulation directed at
other personal activities, but be
insufficient
to
justify
such
as
diminishes the exercise of rights so
vital to the maintenance of democratic
institutions.
To summarize then, we hold that Sec.
5.4. is invalid because (1) it imposes a
prior restraint on the freedom of
expression, (2) it is a direct and total
suppression
of
a
category
of
expression
even
though
such
suppression is only for a limited
period, and (3) the governmental
interest sought to be promoted can be

achieved by means other than the


suppression of freedom of expression.
(Social Weather Stations, Inc., v.
COMELEC, G.R. No. 147571, May 5,
2001, En Banc [Mendoza])
185. Discuss the "doctrine of fair
comment" as a valid defense in an
action for libel or slander.
Held: Fair commentaries on matters
of public interest are privileged and
constitute a valid defense in an action
for libel or slander. The doctrine of fair
comment means that while in general
every discreditable imputation publicly
made is deemed false, because every
man is presumed innocent until his
guilt is judicially proved, and every
false imputation is deemed malicious,
nevertheless, when the discreditable
imputation is directed against a public
person in his public capacity, it is not
necessarily actionable. In order that
such discreditable imputation to a
public official may be actionable, it
must either be a false allegation of
fact or a comment based on a false
supposition. If the comment is an
expression of opinion, based on
established facts, then it is immaterial
that the opinion happens to be
mistaken, as long as it might
reasonably be inferred from the facts.
(Borjal v. CA, 301 SCRA 1, Jan. 14,
1999, 2nd Div. [Bellosillo])
186. What is the raison detre for
the New York Times v. Sullivan (376 US
254) holding that honest criticisms on
the conduct of public officials and
public figures are insulated from libel
judgments?
Held: The guarantees of freedom of
speech and press prohibit a public
official or public figure from recovering
damages for a defamatory falsehood
relating to his official conduct unless
he proves that the statement was
made with actual malice, i.e., with
knowledge that it was false or with
reckless disregard of whether it was
false or not.
The raison detre for the New York
Times doctrine was that to require
critics of official conduct to guarantee
the truth of all their factual assertions
on pain of libel judgments would lead
to self-censorship, since would-be
critics would be deterred from voicing
out their criticisms even if such were
believed to be true, or were in fact
true, because of doubt whether it
could be proved or because of fear of
the expense of having to prove it.

(Borjal v. CA, 301 SCRA 1, Jan. 14,


1999, 2nd Div. [Bellosillo])
187. Who is a public figure, and
therefore subject to public comment?
Held: [W]e deem private respondent
a public figure within the purview of
the New York Times ruling. At any
rate, we have also defined public
figure in Ayers Production Pty., Ltd. v.
Capulong (G.R. Nos. 82380 and 82398,
29 April 1988, 160 SCRA 861) as
X x x a person who, by his
accomplishments, fame, mode of
living, or by adopting a profession or
calling which gives the public a
legitimate interest in his doings, his
affairs and his character, has become
a public personage. He is, in other
words, a celebrity. Obviously, to be
included in this category are those
who have achieved some degree of
reputation by appearing before the
public, as in the case of an actor, a
professional baseball player, a pugilist,
or any other entertainer. The list is,
however, broader than this.
It
includes
public
officers,
famous
inventors and explorers, war heroes
and even ordinary soldiers, infant
prodigy, and no less a personage than
the Great Exalted Ruler of the lodge.
It includes, in short, anyone who has
arrived at a position where the public
attention is focused upon him as a
person.
The FNCLT (First National Conference
on Land Transportation) was an
undertaking
infused
with
public
interest. It was promoted as a joint
project of the government and the
private sector, and organized by top
government officials and prominent
businessmen.
For this reason, it
attracted media mileage and drew
public attention not only to the
conference
itself
but
to
the
personalities behind as well. As its
Executive Director and spokesman,
private
respondent
consequently
assumed the status of a public figure.
But
even
assuming
ex-gratia
argumenti that private respondent,
despite the position he occupied in the
FNCLT, would not qualify as a public
figure, it does not necessarily follow
that he could not validly be the
subject of a public comment even if he
was not a public official or at least a
public figure, for he could be, as long
as he was involved in a public issue. If
a matter is a subject of public or
general interest, it cannot suddenly

become less so merely because a


private individual is involved or
because in some sense the individual
did not voluntarily choose to become
involved. The publics primary interest
is in the event; the public focus is on
the conduct of the participant and the
content, effect and significance of the
conduct, not the participants prior
anonymity or notoriety.
(Borjal v.
CA, 301 SCRA 1, Jan. 14, 1999,
2nd Div. [Bellosillo])
188. The question for determination
in this case is the liability for libel of a
citizen who denounces a barangay
official for misconduct in office. The
Regional Trial Court of Manila x x x
found petitioner guilty x x x on the
ground that petitioner failed to prove
the truth of the charges and that he
was motivated by vengeance in
uttering the defamatory statement.
Held: The decision appealed from
should be reversed.
In denouncing the barangay chairman
in this case, petitioner and the other
residents of the Tondo Foreshore Area
were not only acting in their selfinterest
but
engaging
in
the
performance of a civic duty to see to it
that public duty is discharged faithfully
and well by those on whom such duty
is incumbent. The recognition of this
right and duty of every citizen in a
democracy is inconsistent with any
requirement placing on him the
burden of proving that he acted with
good motives and for justifiable ends.
For
that
matter,
even
if
the
defamatory statement is false, no
liability can attach if it relates to
official conduct, unless the public
official concerned proves that the
statement was made with actual
malice that is, with knowledge that it
was false or with reckless disregard of
whether it was false or not. This is the
gist of the ruling in the landmark case
of New York Times v. Sullivan (376 U.S.
254, 11 L. Ed. 2d [1964].
For a
fascinating account of this case, see
Anthony Lewis, Make No Law The
Sullivan
Case
and
the
First
Amendment [1991]), which this Court
has cited with approval in several of
its own decisions (Lopez v. Court of
Appeals, 145 Phil. 219 [1970], others
omitted). This is the rule of actual
malice. In this case, the prosecution
failed to prove not only that the
charges made by petitioner were false
but also that petitioner made them
with knowledge of their falsity or with

reckless disregard of whether they


were false or not.
A rule placing on the accused the
burden of showing the truth of
allegations of official misconduct
and/or good motives and justifiable
ends for making such allegations
would not only be contrary to Art. 361
of the Revised Penal Code. It would,
above
all,
infringe
on
the
constitutionally guaranteed freedom of
expression. Such a rule would deter
citizens from performing their duties
as members of a self-governing
community. Without free speech and
assembly, discussions of our most
abiding concerns as a nation would be
stifled. As Justice Brandies has said,
public discussion is a political duty
and the greatest menace to freedom
is an inert people. (Whitney v.
California, 247 U.S. 357, 375, 71 L. Ed.
1095, 1105 [1927] [concurring])
(Vasquez v. Court of Appeals, 314
SCRA 460, Sept. 15, 1999, En Banc
[Mendoza])

189. The Office of the Mayor of Las


Pinas refused to issue permit to
petitioners to hold rally a rally in front
of the Justice Hall of Las Pinas on the
ground that it was prohibited under
Supreme Court En Banc Resolution
dated July 7,1998 in A.M. No. 98-7-02SC, entitled, "Re: Guidelines on the
Conduct of Demonstrations, Pickets,
Rallies and Other Similar Gatherings in
the Vicinity of the Supreme Court and
All Other Courts."
Petitioners thus
initiated the instant proceedings.
They submit that the Supreme Court
gravely abused its discretion and/or
acted without or in excess of
jurisdiction in promulgating those
guidelines.
Held:
We shall first dwell on the
critical argument made by petitioners
that
the
rules
constitute
an
abridgment of the people's aggregate
rights of free speech, free expression,
peaceful assembly and petitioning
government for redress of grievances
citing Sec. 4, Article III of the 1987
Constitution that "no law shall be
passed abridging" them.
It is true that the safeguarding of the
people's freedom of expression to the
end that individuals may speak as
they think on matters vital to them
and that falsehoods may be exposed
through the processes of education
and discussion, is essential to free

government. But freedom of speech


and
expression
despite
its
indispensability has its limitations. It
has never been understood as the
absolute right to speak whenever,
however, and wherever one pleases,
for the manner, place, and time of
public
discussion
can
be
constitutionally controlled.
[T]he
better policy is not liberty untamed
but liberty regulated by law where
every
freedom
is
exercised
in
accordance with law and with due
regard for the rights of others.
Conventional wisdom tells us that the
realities of life in a complex society
preclude an absolutist interpretation of
freedom of expression where it does
not involve pure speech but speech
plus physical actions like picketing.
There are other significant societal
values that must be accommodated
and when they clash, they must all be
weighed with the promotion of the
general welfare of the people as the
ultimate objective. In balancing these
values, this Court has accorded
freedom of expression a preferred
position
in
light
of
its
more
comparative importance. Hence, our
rulings now musty in years hold that
only the narrowest time, place and
manner
regulations
that
are
specifically tailored to serve an
important governmental interest may
justify the application of the balancing
of interests test in derogation of the
people's right of free speech and
expression. Where said regulations do
not aim particularly at the evils within
the allowable areas of state control
but, on the contrary, sweep within
their ambit other activities as to
operate as an overhanging threat to
free discussion, or where upon their
face they are so vague, indefinite, or
inexact as to permit punishment of the
fair use of the right of free speech,
such regulations are void.
Prescinding from this premise, the
Court
reiterates
that
judicial
independence and the fair and orderly
administration of justice constitute
paramount governmental interests
that can justify the regulation of the
public's right of free speech and
peaceful assembly in the vicinity of
courthouses. In the case of In Re: Emil
P. Jurado, the Court pronounced in no
uncertain terms that:
"x x x freedom of expression needs on
occasion to be adjusted to and
accommodated with the requirements
of equally important public interests.

One of these fundamental public


interests is the maintenance of the
integrity and orderly functioning of the
administration of justice. There is no
antinomy between free expression and
the integrity of the system of
administering justice.
For the
protection
and
maintenance
of
freedom of expression itself can be
secured only within the context of a
functioning and orderly system of
dispensing justice, within the context,
in other words, of viable independent
institutions for delivery of justice
which are accepted by the general
community. x x x" (In Re: Emil P.
Jurado, 243 SCRA 299, 323-324
[1995])
It is sadly observed that judicial
independence
and
the
orderly
administration of justice have been
threatened not only by contemptuous
acts inside, but also by irascible
demonstrations
outside,
the
courthouses.
They wittingly or
unwittingly, spoil the ideal of sober,
non-partisan proceedings before a
cold and neutral judge. Even in the
United States, a prohibition against
picketing and demonstrating in or near
courthouses, has been ruled as valid
and constitutional notwithstanding its
limiting effect on the exercise by the
public of their liberties. X x x
The administration of justice must not
only be fair but must also appear to be
fair and it is the duty of this Court to
eliminate everything that will diminish
if not destroy this judicial desideratum.
To be sure, there will be grievances
against our justice system for there
can be no perfect system of justice but
these grievances must be ventilated
through appropriate petitions, motions
or other pleadings. Such a mode is in
keeping with the respect due to the
courts as vessels of justice and is
necessary if judges are to dispose
their business in a fair fashion. It is
the traditional conviction of every
civilized society that courts must be
insulated from every extraneous
influence in their decisions. The facts
of a case should be determined upon
evidence produced in court, and
should be uninfluenced by bias,
prejudice or sympathies. (In Re:
Petition
to
Annul
En
Banc
Resolution
A.M.
98-7-02-SC
Ricardo C. Valmonte and Union of
Lawyers
and
Advocates
for
Transparency
in
Government
[ULAT], G.R. No. 134621, Sept. 29,
1998)

190. Did the Supreme Court commit


an act of judicial legislation in
promulgating En Banc Resolution A.M.
98-7-02-SC, entitled, "Re: Guidelines
on the Conduct of Demonstrations,
Pickets, Rallies and Other Similar
Gatherings in the Vicinity of the
Supreme Court and All Other Courts?"
Held: Petitioners also claim that this
Court committed an act of judicial
legislation
in
promulgating
the
assailed resolution. They charge that
this Court amended provisions of
Batas Pambansa (B.P.) Blg. 880,
otherwise known as "the Public
Assembly Act," by converting the
sidewalks and streets within a radius
of two hundred (200) meters from
every courthouse from a public forum
place into a "no rally" zone. Thus,
they accuse this Court of x x x
violating the principle of separation of
powers.
We reject these low watts arguments.
Public places historically associated
with the free exercise of expressive
activities, such as streets, sidewalks,
and parks, are considered, without
more, to be public fora.
In other
words, it is not any law that can imbue
such places with the public nature
inherent in them. But even in such
public fora, it is settled jurisprudence
that the government may restrict
speech plus activities and enforce
reasonable time, place, and manner
regulations as long as the restrictions
are content-neutral, are narrowly
tailored
to
serve
a
significant
governmental interest, and leave open
ample
alternative
channels
of
communication.
Contrary therefore to petitioners
impression, B.P. Blg. 880 did not
establish
streets
and
sidewalks,
among other places, as public fora. A
close look at the law will reveal that it
in fact prescribes reasonable time,
place, and manner regulations. Thus,
it requires a written permit for the
holding of public assemblies in public
places subject, even, to the right of
the mayor to modify the place and
time of the public assembly, to impose
a rerouting of the parade or street
march, to limit the volume of loud
speakers or sound system and to
prescribe
other
appropriate
restrictions on the conduct of the
public assembly.
The existence of B.P. Blg. 880,
however, does not preclude this Court
from promulgating rules regulating

conduct of demonstrations in the


vicinity of courts to assure our people
of
an
impartial
and
orderly
administration of justice as mandated
by the Constitution. To insulate the
judiciary from mob pressure, friendly
or otherwise, and isolate it from public
hysteria, this Court merely moved
away the situs of mass actions within
a 200-meter radius from every
courthouse.
In fine, B.P. Blg. 880
imposes general restrictions to the
time, place and manner of conducting
concerted actions. On the other hand,
the resolution of this Court regulating
demonstrations
adds
specific
restrictions as they involve judicial
independence
and
the
orderly
administration of justice. There is thus
no discrepancy between the two sets
of regulatory measures. Simply put,
B.P. Blg. 880 and the assailed
resolution complement each other.
We so hold following the rule in legal
hermeneutics that an apparent conflict
between a court rule and a statutory
provision should be harmonized and
both should be given effect if possible.
(In Re: Petition to Annul En Banc
Resolution
A.M.
98-7-02-SC
Ricardo C. Valmonte and Union of
Lawyers
and
Advocates
for
Transparency
in
Government
[ULAT], G.R. No. 134621, Sept. 29,
1998)
191. Should live media coverage of
court proceedings be allowed?
Held: The propriety of granting or
denying permission to the media to
broadcast, record, or photograph court
proceedings involves weighing the
constitutional guarantees of freedom
of the press, the right of the public to
information and the right to public
trial, on the one hand, and on the
other hand, the due process rights of
the defendant and the inherent and
constitutional power of the courts to
control their proceedings in order to
permit
the
fair
and
impartial
administration of justice. Collaterally,
it also raises issues on the nature of
the media, particularly television and
its role in society, and of the impact of
new technologies on law.
The records of the Constitutional
Commission are bereft of discussion
regarding the subject of cameras in
the courtroom. Similarly, Philippine
courts have not had the opportunity to
rule on the question squarely.
While we take notice of the September
1990 report of the United States

Judicial Conference Ad Hoc Committee


on Cameras in the Courtroom, still the
current rule obtaining in the Federal
Courts of the United States prohibits
the presence of television cameras in
criminal trials. Rule 53 of the Federal
Rules of Criminal Procedure forbids the
taking of photographs during the
progress of judicial proceedings or
radio
broadcasting
of
such
proceedings from the courtroom. A
trial of any kind or in any court is a
matter of serious importance to all
concerned and should not be treated
as a means of entertainment. To so
treat it deprives the court of the
dignity which pertains to it and
departs from the orderly and serious
quest for truth for which our judicial
proceedings are formulated.
Courts do not discriminate against
radio
and
television
media
by
forbidding the
broadcasting or
televising of a trial while permitting
the newspaper reporter access to the
courtroom, since a television or news
reporter has the same privilege, as the
news reporter is not permitted to bring
his typewriter or printing press into
the courtroom.
In Estes v. Texas (381 U.S. 532), the
United States Supreme Court held that
television
coverage
of
judicial
proceedings involves an inherent
denial of due process rights of a
criminal defendant. Voting 5-4, the
Court through Mr. Justice Clark,
identified four (4) areas of potential
prejudice which might arise from the
impact of the cameras on the jury,
witnesses, the trial judge and the
defendant.
The decision in part
pertinently stated:
"Experience likewise has established
the prejudicial effect of telecasting on
witnesses.
Witnesses might be
frightened, play to the camera, or
become nervous. They are subject to
extraordinary out-of-court influences
which might affect their testimony.
Also, telecasting not only increases
the trial judge's responsibility to avoid
actual prejudice to the defendant; it
may
as
well
affect
his
own
performance.
Judges are human
beings also and are subject to the
same psychological reactions as
laymen.
For
the
defendant,
telecasting is a form of mental
harassment and subjects him to
excessive
public
exposure
and
distracts him from the effective
presentation of his defense.

"The television camera is a powerful


weapon
which
intentionally
or
inadvertently can destroy an accused
and his case in the eyes of the public."
Representatives of the press have no
special standing to apply for a writ of
mandate to compel a court to permit
them to attend a trial, since within the
courtroom a reporter's constitutional
rights are no greater than those of any
other member of the public. Massive
intrusion of representatives of the
news media into the trial itself can so
alter or destroy the constitutionally
necessary judicial atmosphere and
decorum that the requirements of
impartiality imposed by due process of
law are denied the defendant and a
defendant in a criminal proceeding
should not be forced to run a gauntlet
of reporters and photographers each
time he enters or leaves the
courtroom.
Considering the prejudice it poses to
the defendant's right to due process
as well as to the fair and orderly
administration
of
justice,
and
considering further that the freedom
of the press and the right of the
people to information may be served
and satisfied by less distracting,
degrading and prejudicial means, live
radio and television coverage of court
proceedings shall not be allowed.
Video footages of court hearings for
news purposes shall be restricted and
limited to shots of the courtroom, the
judicial officers, the parties and their
counsel
taken
prior
to
the
commencement
of
official
proceedings.
No video shots or
photographs shall be permitted during
the trial proper. (Supreme Court En
Banc Resolution Re: Live TV and
Radio Coverage of the Hearing of
President Corazon C. Aquino's
Libel Case, dated Oct. 22, 1991)
192. Should the Court allow live
media coverage of the anticipated trial
of the plunder and other criminal
cases filed against former President
Joseph
E.
Estrada
before
the
Sandiganbayan in order to assure the
public of full transparency in the
proceedings of an unprecedented case
in our history as requested by the
Kapisanan ng mga Brodkaster ng
Pilipinas?
Held: The propriety of granting or
denying the instant petition involve
the weighing out of the constitutional
guarantees of freedom of the press
and the right to public information, on

the one hand, and the fundamental


rights of the accused, on the other
hand, along with the constitutional
power of a court to control its
proceedings in ensuring a fair and
impartial trial.
When these rights race against one
another, jurisprudence tells us that the
right of the accused must be preferred
to win.
With the possibility of losing not only
the precious liberty but also the very
life of an accused, it behooves all to
make absolutely certain that an
accused receives a verdict solely on
the basis of a just and dispassionate
judgment, a verdict that would come
only after the presentation of credible
evidence testified to by unbiased
witnesses unswayed by any kind of
pressure, whether open or subtle, in
proceedings that are devoid of
histrionics that might detract from its
basic aim to ferret veritable facts free
from improper influence, and decreed
by a judge with an unprejudiced mind,
unbridled by running emotions or
passions.
Due process guarantees the accused a
presumption of innocence until the
contrary is proved in a trial that is not
lifted above its individual settings nor
made an object of publics attention
and where the conclusions reached
are induced not by any outside force
or influence but only by evidence and
argument given in open court, where
fitting dignity and calm ambiance is
demanded.
Witnesses and judges may very well
be men and women of fortitude, able
to thrive in hardy climate, with every
reason to presume firmness of mind
and resolute endurance, but it must
also be conceded that television can
work profound changes in the
behavior of the people it focuses on.
Even while it may be difficult to
quantify the influence, or pressure that
media can bring to bear on them
directly and through the shaping of
public opinion, it is a fact, nonetheless,
that, indeed, it does so in so many
ways and in varying degrees. The
conscious or unconscious effect that
such a coverage may have on the
testimony of witnesses and the
decision
of
judges
cannot
be
evaluated but, it can likewise be said,
it is not at all unlikely for a vote of
guilt or innocence to yield to it. It
might be farcical to build around them
an impregnable armor against the

influence of the most powerful media


of public opinion.
To say that actual prejudice should
first be present would leave to near
nirvana the subtle threats to justice
that a disturbance of the mind so
indispensable to the calm and
deliberate dispensation of justice can
create. The effect of television may
escape the ordinary means of proof,
but it is not far-fetched for it to
gradually erode our basal conception
of a trial such as we know it now.
An accused has a right to a public trial
but it is a right that belongs to him,
more than anyone else, where his life
or liberty can be held critically in
balance. A public trial aims to ensure
that he is fairly dealt with and would
not be unjustly condemned and that
his rights are not compromised in
secret conclaves of long ago. A public
trial is not synonymous with publicized
trial; it only implies that the court
doors must be open to those who wish
to come, sit in the available seats,
conduct themselves with decorum and
observe the trial process.
In the
constitutional sense, a courtroom
should have enough facilities for a
reasonable number of the public to
observe the proceedings, not too small
as to render the openness negligible
and not too large as to distract the
trial participants from their proper
functions, who shall then be totally
free to report what they have
observed during the proceedings.
The
courts
recognize
the
constitutionally embodied freedom of
the press and the right to public
information.
It also approves of
medias exalted power to provide the
most accurate and comprehensive
means of conveying the proceedings
to the public and in acquainting the
public with the judicial process in
action;
nevertheless,
within
the
courthouse,
the
overriding
consideration is still the paramount
right of the accused to due process
which must never be allowed to suffer
diminution
in
its
constitutional
proportions.
Justice Clark thusly
pronounced,
while
a
maximum
freedom must be allowed the press in
carrying out the important function of
informing the public in a democratic
society, its exercise must necessarily
be subject to the maintenance of
absolute fairness in the judicial
process.
Xxx

The Integrated Bar of the Philippines x


x x expressed its own concern on the
live television and radio coverage of
the criminal trials of Mr. Estrada; to
paraphrase: Live television and radio
coverage can negate the rule on
exclusion of witnesses during the
hearings intended to assure a fair trial;
at stake in the criminal trial is not only
the life and liberty of the accused but
the very credibility of the Philippine
criminal justice system, and live
television and radio coverage of the
trial could allow the hooting throng
to arrogate unto themselves the task
of judging the guilt of the accused,
such that the verdict of the court will
be acceptable only if popular; and live
television and radio coverage of the
trial will not subserve the ends of
justice but will only pander to the
desire
for
publicity
of
a
few
grandstanding lawyers.
Xxx
Unlike other government offices,
courts do not express the popular will
of the people in any sense which,
instead, are tasked to only adjudicate
controversies on the basis of what
alone is submitted before them. A
trial is not a free trade of ideas. Nor is
a competing market of thoughts the
known test of truth in a courtroom.
(Re: Request Radio-TV coverage of
the Trial in the Sandiganbayan of
the Plunder Cases against the
former
President
Joseph
E.
Estrada, A.M. No. 01-4-03-SC, June
29, 2001, En Banc [Vitug])
Freedom of Religion
193. Discuss why the Gerona ruling
(justifying the expulsion from public
schools of children of Jehovahs
Witnesses who refuse to salute the
flag and sing the national anthem
during flag ceremony as prescribed by
the Flag Salute Law) should be
abandoned.
Held:
Our task here is extremely
difficult, for the 30-year old decision of
this court in Gerona upholding the flag
salute
law
and
approving
the
expulsion of students who refuse to
obey it, is not lightly to be trifled with.
It is somewhat ironic however, that
after the Gerona ruling had received
legislative cachet by its incorporation
in the Administrative Code of 1987,
the present Court believes that the

time has come to reexamine it. The


idea that one may be compelled to
salute the flag, sing the national
anthem, and recite the patriotic
pledge, during a flag ceremony on
pain of being dismissed from ones job
or of being expelled from school, is
alien to the conscience of the present
generation of Filipinos who cut their
teeth on the Bill of Rights which
guarantees their rights to free speech
(The flag salute, singing the national
anthem and reciting the patriotic
pledge are all forms of utterances.)
and the free exercise of religious
profession and worship.
Religious freedom is a fundamental
right which is entitled to the highest
priority and the amplest protection
among human rights, for it involves
the relationship of man to his Creator
(Chief Justice Enrique M. Fernandos
separate opinion in German v.
Barangan, 135 SCRA 514, 530-531).
The right to religious profession and
worship has a two-fold aspect, viz.,
freedom to believe and freedom to act
on ones belief. The first is absolute as
long as the belief is confined within
the realm of thought. The second is
subject to regulation where the belief
is translated into external acts that
affect the public welfare (J. Cruz,
Constitutional Law, 1991 Ed., pp. 176177).
Petitioners stress x x x that while they
do not take part in the compulsory flag
ceremony, they do not engage in
external acts or behavior that would
offend their countrymen who believe
in expressing their love of country
through the observance of the flag
ceremony.
They quietly stand at
attention during the flag ceremony to
show their respect for the rights of
those who choose to participate in the
solemn proceedings. Since they do
not engage in disruptive behavior,
there is no warrant for their expulsion.
The sole justification for a prior
restraint or limitation on the exercise
of religious freedom (according to the
late Chief Justice Claudio Teehankee in
his dissenting opinion in German v.
Barangan, 135 SCRA 514, 517) is the
existence of a grave and present
danger of a character both grave and
imminent, of a serious evil to public
safety, public morals, public health or
any other legitimate public interest,
that the State has a right (and duty) to
prevent. Absent such a threat to
public safety, the expulsion of the

petitioners from the schools is not


justified.
The situation that the Court directly
predicted in Gerona that:
[T]he flag ceremony will become a
thing of the past or perhaps conducted
with very few participants, and the
time will come when we would have
citizens untaught and uninculcated in
and not imbued with reverence for the
flag and love of country, admiration
for national heroes, and patriotism a
pathetic, even tragic situation, and all
because a small portion of the school
population imposed its will, demanded
and was granted an exemption.
has not come to pass. We are not
persuaded that by exempting the
Jehovahs Witnesses from saluting the
flag, singing the national anthem and
reciting the patriotic pledge, this
religious group which admittedly
comprises a small portion of the
school population will shake up our
part of the globe and suddenly
produce a nation untaught and
uninculcated in and unimbued with
reverence for the flag, patriotism, love
of country and admiration for national
heroes. After all, what the petitioners
seek only is exemption from the flag
ceremony, not exclusion from the
public schools where they may study
the Constitution, the democratic way
of life and form of government, and
learn not only the arts, sciences,
Philippine history and culture but also
receive training for a vocation or
profession and be taught the virtues of
patriotism, respect for human rights,
appreciation for national heroes, the
rights and duties of citizenship, and
moral and spiritual values (Sec. 3[2],
Art. XIV, 1987 Constitution) as part of
the curricula. Expelling or banning the
petitioners from Philippine schools will
bring about the very situation that this
Court had feared in Gerona. Forcing a
small religious group, through the iron
hand of the law, to participate in a
ceremony that violates their religious
beliefs, will hardly be conducive to
love of country or respect for duly
constituted authorities.
As Mr. Justice Jackson remarked in
West Virginia v. Barnette, 319 U.S. 624
(1943):
x x x To believe that patriotism will
not flourish if patriotic ceremonies are
voluntary and spontaneous instead of
a compulsory routine is to make an
unflattering statement of the appeal of

our institutions to free minds. x x x


When they (diversity) are so harmless
to others or to the State as those we
deal with here, the price is not too
great. But freedom to differ is not
limited to things that do not matter
much. That would be a mere shadow
of freedom. The test of its substance
is the right to differ as to things that
touch the heart of the existing order.
Furthermore, let it be noted that
coerced unity and loyalty even to the
country, x x x assuming that such
unity and loyalty can be attained
through coercion is not a goal that is
constitutionally obtainable at the
expense of religious liberty.
A
desirable end cannot be promoted by
prohibited
means.
(Meyer
v.
Nebraska, 262 U.S. 390, 67 L. ed.
1042, 1046)
Moreover, the expulsion of members
of Jehovahs Witnesses from the
schools where they are enrolled will
violate their right as Philippine
citizens, under the 1987 Constitution,
to receive free education, for it is the
duty of the State to protect and
promote the right of all citizens to
quality education x x x and to make
such education accessible to all (Sec.
1, Art. XIV).
In Victoriano v. Elizalde Rope Workers
Union, 59 SCRA 54, 72-75, we upheld
the exemption of members of the
Iglesia Ni Cristo, from the coverage of
a closed shop agreement between
their employer and a union because it
would violate the teaching of their
church not to join any labor group:
x x x It is certain that not every
conscience can be accommodated by
all the laws of the land; but when
general laws conflict with scruples of
conscience, exemptions ought to be
granted unless some compelling state
interests intervenes. (Sherbert v.
Berner, 374 U.S. 398, 10 L. Ed. 2d 965,
970, 83 S. Ct. 1790).
We hold that a similar exemption may
be
accorded
to
the
Jehovahs
Witnesses
with
regard
to
the
observance of the flag ceremony out
of respect for their religious beliefs,
however bizarre those beliefs may
seem to others. Nevertheless, their
right not to participate in the flag
ceremony does not give them a right
to disrupt such patriotic exercises.
Paraphrasing the warning cited by this
Court in Non v. Dames II, 185 SCRA
523, 535, while the highest regard

must be afforded their right to the free


exercise of their religion, this should
not be taken to mean that school
authorities are powerless to discipline
them if they should commit breaches
of the peace by actions that offend the
sensibilities,
both
religious
and
patriotic, of other persons. If they
quietly stand at attention during the
flag ceremony while their classmates
and teachers salute the flag, sing the
national anthem and recite the
patriotic pledge, we do not see how
such conduct may possibly disturb the
peace, or pose a grave and present
danger of a serious evil to public
safety, public morals, public health or
any other legitimate public interest
that the State has a right (and duty) to
prevent.
(Ebralinag
v.
The
Division
Superintendent
of
Schools of Cebu, 219 SCRA 256,
269-273, March 1, 1993, En Banc
[Grino-Aquino])
194. A pre-taped TV program of the
Iglesia Ni Cristo (INC) was submitted
to the MTRCB for review. The latter
classified it as rated X because it
was shown to be attacking another
religion.
The INC protested by
claiming that its religious freedom is
per se beyond review by the MTRCB.
Should this contention be upheld?
Held: The right to religious profession
and worship has a two-fold aspect,
viz., freedom to believe and freedom
to act on one's belief. The first is
absolute as long as the belief is
confined within the realm of thought.
The second is subject to regulation
where the belief is translated into
external acts that affect the public
welfare.
The Iglesia Ni Cristo's postulate that
its religious freedom is per se beyond
review by the MTRCB should be
rejected. Its public broadcast on TV of
its religious programs brings it out of
the bosom of internal belief. Television
is a medium that reaches even the
eyes and ears of children.
The
exercise of religious freedom can be
regulated by the State when it will
bring about the clear and present
danger of a substantive evil which the
State is duty-bound to prevent, i.e.,
serious detriment to the more
overriding interest of public health,
public morals, or public welfare. A
laissez faire policy on the exercise of
religion can be seductive to the liberal
mind but history counsels the Court
against its blind adoption as religion is
and continues to be a volatile area of

concern in our society today. "For


sure, we shall continue to subject any
act pinching the space for the free
exercise of religion to a heightened
scrutiny but we shall not leave its
rational exercise to the irrationality of
man. For when religion divides and its
exercise destroys, the State should not
stand still." (Iglesia Ni Cristo v. CA,
259 SCRA 529, July 26, 1996
[Puno])
195. Did the MTRCB act correctly
when it rated X the Iglesia Ni Cristo's
pre-taped TV program simply because
it was found to be "attacking" another
religion?
Held: The MTRCB may disagree with
the criticisms of other religions by the
Iglesia Ni Cristo but that gives it no
excuse to interdict such criticisms,
however unclean they may be. Under
our constitutional scheme, it is not the
task of the State to favor any religion
by protecting it against an attack by
another religion. Religious dogma and
beliefs are often at war and to
preserve peace among their followers,
especially
the
fanatics,
the
establishment clause of freedom of
religion prohibits the State from
leaning towards any religion. Vis--vis
religious differences, the State enjoys
no banquet of options.
Neutrality
alone is its fixed and immovable
stance. In fine, the MTRCB cannot
squelch the speech of the INC simply
because it attacks another religion. In
a State where there ought to be no
difference between the appearance
and the reality of freedom of religion,
the remedy against bad theology is
better theology.
The bedrock of
freedom of religion is freedom of
thought and it is best served by
encouraging
the
marketplace
of
dueling ideas. When the luxury of
time permits, the marketplace of ideas
demands that speech should be met
by more speech for it is the spark of
opposite speech, the heat of colliding
ideas, that can fan the embers of
truth. (Iglesia Ni Cristo v. CA, 259
SCRA 529, July 26, 1996 [Puno])
196. Is
solicitation
for
the
construction of a church covered by
P.D.
No.
1564
and,
therefore,
punishable if done without the
necessary permit for solicitation from
the DSWD?
Held:
First.
Solicitation
of
contributions for the construction of a
church
is
not
solicitation
for
"charitable or public welfare purpose"

but for a religious purpose, and a


religious purpose is not necessarily a
charitable or public welfare purpose.
A fund campaign for the construction
or repair of a church is not like fund
drives for needy families or victims of
calamity or for the construction of a
civic center and the like.
Like
solicitation of subscription to religious
magazines,
it
is
part
of
the
propagation of religious faith or
evangelization. Such solicitation calls
upon the virtue of faith, not of charity,
save as those solicited for money or
aid may not belong to the same
religion as the solicitor.
Such
solicitation does not engage the
philanthropic as much as the religious
fervor of the person who is solicited
for contribution.
Second. The purpose of the Decree is
to protect the public against fraud in
view of the proliferation of fund
campaigns for charity and other civic
projects. On the other hand, since
religious fund drives are usually
conducted among those belonging to
the same religion, the need for public
protection
against
fraudulent
solicitations does not exist in as great
a degree as does the need for
protection with respect to solicitations
for charity or civic projects as to justify
state regulation.
Third. To require a government permit
before solicitation for religious purpose
may be allowed is to lay a prior
restraint on the free exercise of
religion.
Such restraint, if allowed,
may well justify requiring a permit
before a church can make Sunday
collections or enforce tithing. But in
American Bible Society v. City of
Manila (101 Phil. 386 [1957]), we
precisely held that an ordinance
requiring payment of a license fee
before one may engage in business
could not be applied to the appellant's
sale of bibles because that would
impose a condition on the exercise of
a constitutional right. It is for the
same reason that religious rallies are
exempted from the requirement of
prior permit for public assemblies and
other uses of public parks and streets
(B.P. Blg. 880, Sec. 3[a]). To read the
Decree, therefore, as including within
its reach solicitations for religious
purposes would be to construe it in a
manner that it violates the Free
Exercise of Religion Clause of the
Constitution x x x.
(Concurring
Opinion, Mendoza, V.V., J., in
Centeno v. Villalon-Pornillos, 236
SCRA 197, Sept. 1, 1994)

197. What is a purely ecclesiastical


affair to which the State can not
meddle?
Held: An ecclesiastical affair is one
that concerns doctrine, creed, or form
of worship of the church, or the
adoption and enforcement within a
religious association of needful laws
and regulations for the government of
the membership, and the power of
excluding from such associations
those
deemed
not
worthy
of
membership.
Based
on
this
definition, an ecclesiastical affair
involves the relationship between the
church and its members and relate to
matters of faith, religious doctrines,
worship and governance of the
congregation.
To be concrete,
examples
of
this
so-called
ecclesiastical affairs to which the State
cannot meddle are proceedings for
excommunication,
ordinations
of
religious ministers, administration of
sacraments and other activities with
attached
religious
significance.
(Pastor Dionisio V. Austria v.
NLRC, G.R. No. 124382, Aug. 16,
1999, 1st Div. [Kapunan])
198. Petitioner is a religious minister
of the Seventh Day Adventist (SDA).
He was dismissed because of alleged
misappropriation of denominational
funds, willful breach of trust, serious
misconduct,
gross
and
habitual
neglect of duties and commission of
an offense against the person of his
employers
duly
authorized
representative.
He filed an illegal
termination case against the SDA
before the labor arbiter. The SDA filed
a motion to dismiss invoking the
doctrine of separation of Church and
State. Should the motion be granted?
Held: Where what is involved is the
relationship of the church as an
employer and the minister as an
employee
and has no relation
whatsoever with the practice of faith,
worship or doctrines of the church,
i.e.,
the
minister
was
not
excommunicated or expelled from the
membership of the congregation but
was terminated from employment, it is
a purely secular affair. Consequently,
the suit may not be dismissed
invoking the doctrine of separation of
church and the state.
(Pastor
Dionisio V. Austria v. NLRC, G.R.
No. 124382, Aug. 16, 1999, 1st
Div. [Kapunan])

The Right of the People to


Information on Matters of Public
Concern
199. Discuss the scope of the right to
information on matters of public
concern.
Held: In Valmonte v. Belmonte, Jr.,
the Court emphasized that the
information sought must be matters
of public concern, access to which
may be limited by law. Similarly, the
state policy of full public disclosure
extends only to transactions involving
public interest and may also be
subject to reasonable conditions
prescribed by law.
As to the
meanings of the terms public
interest and public concern, the
Court, in Legaspi v. Civil Service
Commission, elucidated:
In determining whether or not a
particular information is of public
concern there is no rigid test which
can be applied. Public concern like
public interest is a term that eludes
exact definition. Both terms embrace
a broad spectrum of subjects which
the public may want to know, either
because these directly affect their
lives, or simply because such matters
naturally arouse the interest of an
ordinary citizen. In the final analysis,
it is for the courts to determine on a
case by case basis whether the matter
at issue is of interest or importance, as
it relates to or affects the public.
Considered a public concern in the
above-mentioned
case
was
the
legitimate concern of citizens to
ensure that government positions
requiring civil service eligibility are
occupied only by persons who are
eligibles. So was the need to give the
general public adequate notification of
various laws that regulate and affect
the actions and conduct of citizens, as
held in Tanada.
Likewise did the
public nature of the loanable funds of
the GSIS and the public office held by
the alleged borrowers (members of
the defunct Batasang Pambansa)
qualify the information sought in
Valmonte as matters of public interest
and concern. In Aquino-Sarmiento v.
Morato (203 SCRA 515, 522-23,
November 13, 1991), the Court also
held that official acts of public officers
done in pursuit of their official
functions are public in character;
hence, the records pertaining to such
official acts and decisions are within
the ambit of the constitutional right of
access to public records.

Under Republic Act No. 6713, public


officials and employees are mandated
to provide information on their
policies and procedures in clear and
understandable
language,
[and]
ensure openness of information, public
consultations and hearing whenever
appropriate x x x, except when
otherwise provided by law or when
required by the public interest. In
particular, the law mandates free
public access, at reasonable hours, to
the annual performance reports of
offices and agencies of government
and government-owned or controlled
corporations; and the statements of
assets,
liabilities
and
financial
disclosures of all public officials and
employees.
In general, writings coming into the
hands of public officers in connection
with their official functions must be
accessible to the public, consistent
with the policy of transparency of
governmental affairs. This principle is
aimed at affording the people an
opportunity to determine whether
those to whom they have entrusted
the affairs of the government are
honestly, faithfully and competently
performing their functions as public
servants. Undeniably, the essence of
democracy lies in the free-flow of
thought; but thoughts and ideas must
be well-informed so that the public
would gain a better perspective of
vital issues confronting them and,
thus, be able to criticize as well as
participate in the affairs of the
government
in
a
responsible,
reasonable and effective manner.
Certainly, it is by ensuring an
unfettered and uninhibited exchange
of ideas among a well-informed public
that a government remains responsive
to the changes desired by the people.
(Chavez v. PCGG, 299 SCRA 744,
Dec. 9, 1998, [Panganiban])
200. What
are
some
of
the
recognized restrictions to the right of
the people to information on matters
of public concern?
Held:
1)

National security matters


and intelligence information.
This
jurisdiction recognizes the common
law
holding
that
there
is
a
governmental privilege against public
disclosure with respect to state secrets
regarding military, diplomatic and
other
national
security
matters.

Likewise,
information
on
intergovernment exchanges prior to the
conclusion of treaties and executive
agreements may be subject to
reasonable safeguards for the sake of
national interest;
2)
Trade or industrial secrets
(pursuant to the Intellectual Property
Code [R.A. No. 8293, approved on June
6, 1997] and other related laws) and
banking transactions (pursuant to the
Secrecy of Bank Deposits Act [R.A. No.
1405, as amended]);
3)
Criminal matters, such as
those relating to the apprehension, the
prosecution and the detention of
criminals, which courts may not
inquire into prior to such arrest,
detention and prosecution;
4)
Other
confidential
information. The Ethical Standards Act
(R.A. No. 6713, enacted on February
20, 1989) further prohibits public
officials and employees from using or
divulging confidential or classified
information officially known to them
by reason of their office and not made
available to the public. (Sec. 7[c],
ibid.) Other acknowledged limitations
to
information
access
include
diplomatic correspondence, closed
door Cabinet meetings and executive
sessions of either house of Congress,
as well as the internal deliberations of
the Supreme Court.
(Chavez v. PCGG, 299 SCRA 744,
Dec. 9, 1998 [Panganiban])
201. Is the alleged ill-gotten
wealth of the Marcoses a matter
of public concern subject to this
right?
Held: With such pronouncements of
our government, whose authority
emanates from the people, there is no
doubt that the recovery of the
Marcoses' alleged ill-gotten wealth is a
matter of public concern and imbued
with public interest. We may also add
that ill-gotten wealth refers to assets
and properties purportedly acquired,
directly or indirectly, by former
President Marcos, his immediate
family, relatives and close associates
through or as a result of their improper
or illegal use of government funds or
properties; or their having taken
undue advantage of their public office;
or their use of powers, influences or
relationships, resulting in their unjust
enrichment
and
causing
grave
damage and prejudice to the Filipino
people and the Republic of the
Philippines. Clearly, the assets and
properties referred to supposedly
originated from the government itself.

To all intents and purposes, therefore,


they belong to the people. As such,
upon reconveyance they will be
returned to the public treasury,
subject only to the satisfaction of
positive claims of certain persons as
may be adjudged by competent
courts. Another declared overriding
consideration for the expeditious
recovery of ill-gotten wealth is that it
may be used for national economic
recovery.
We believe the foregoing disquisition
settles the question of whether
petitioner has a right to respondents'
disclosure of any agreement that may
be arrived at concerning the Marcoses
purported ill-gotten wealth. (Chavez
v. PCGG, 299 SCRA 744, Dec. 9,
1998 [Panganiban])
Freedom of Association
202. Does the right of civil servants
to organize include their right to
strike? Clarify.
Held: Specifically, the right of civil
servants to organize themselves was
positively recognized in Association of
Court of Appeals Employees (ACAE) v.
Ferrer-Calleja
(203
SCRA
596,
November 15, 1991). But, as in the
exercise of the rights of free
expression and of assembly, there are
standards for allowable limitations
such as the legitimacy of the purposes
of the association, the overriding
considerations of national security and
the
preservation
of
democratic
institutions (People v. Ferrer, 48 SCRA
382, December 27, 1972, per Castro,
J., where the Court, while upholding
the validity of the Anti-Subversion Act
which outlawed the Communist Party
of
the
Philippines
and
other
"subversive" organizations, clarified,
"Whatever interest in freedom of
speech and freedom of association is
infringed by the prohibition against
knowing
membership
in
the
Communist Party of the Philippines, is
so indirect and so insubstantial as to
be clearly and heavily outweighed by
the
overriding
considerations
of
national security and the preservation
of democratic institutions in this
country." It cautioned, though, that
"the
need
for
prudence
and
circumspection
[cannot
be
overemphasized]
in
[the
law's]
enforcement, operating as it does in
the sensitive area of freedom of
expression and belief.")

As regards the right to strike, the


Constitution itself qualifies its exercise
with the proviso "in accordance with
law." This is a clear manifestation that
the state may, by law, regulate the
use of this right, or even deny certain
sectors such right. Executive Order
No. 180 (Issued by former President
Corazon C. Aquino on June 1, 1987)
which provides guidelines for the
exercise of the right of government
workers to organize, for instance,
implicitly endorsed an earlier CSC
circular which "enjoins under pain of
administrative
sanctions,
all
government officers and employees
from staging strikes, demonstrations,
mass leaves, walkouts and other forms
of mass action which will result in
temporary stoppage or disruption of
public service" (CSC Memorandum
Circular No. 6, s. 1987, dated April 21,
1987) by stating that the Civil Service
law and rules governing concerted
activities
and
strikes
in
the
government service shall be observed.
It is also settled in jurisprudence that,
in general, workers in the public sector
do not enjoy the right to strike.
Alliance of Concerned Government
Workers v. Minister of Labor and
Employment (124 SCRA 1, August 3,
1983, also per Gutierrez, Jr., J.)
rationalized the proscription thus:
"The general rule in the past and up to
the present is that the 'terms and
conditions of employment in the
Government, including any political
subdivision or instrumentality thereof
are governed by law.' X x x. Since the
terms and conditions of government
employment are
fixed
by law,
government workers cannot use the
same weapons employed by the
workers in the private sector to secure
concessions from their employers.
The principle behind labor unionism in
private industry is that industrial
peace cannot be secured through
compulsion by law. Relations between
private
employers
and
their
employees rest on an essentially
voluntary basis.
Subject to the
minimum requirements of wage laws
and
other
labor
and
welfare
legislation, the terms and conditions of
employment in the unionized private
sector are settled through the process
of
collective
bargaining.
In
government employment, however, it
is the legislature and, where properly
given
delegated
power,
the
administrative heads of government
which fix the terms and conditions of
employment.
And this is effected

through statutes or administrative


circulars, rules, and regulations, not
through
collective
bargaining
agreements." (Ibid., p. 13)
After delving into the intent of the
framers of the Constitution, the Court
affirmed the above rule in Social
Security
System
Employees
Association (SSSEA) v. Court of
Appeals (175 SCRA 686, July 28, 1989)
and explained:
"Government
employees
may,
therefore, through their unions or
associations,
either
petition
the
Congress for the betterment of the
terms and conditions of employment
which are within the ambit of
legislation or negotiate with the
appropriate government agencies for
the improvement of those which are
not fixed by law. If there be any
unresolved grievances, the dispute
may be referred to the Public Sector
Labor-Management
Council
for
appropriate action. But employees in
the civil service may not resort to
strikes, walkouts and other temporary
work stoppages, like workers in the
private sector, to pressure the
Government to accede to their
demands. As now provided under Sec.
4, Rule III of the Rules and Regulations
to Govern the Exercise of the Right of
Government
Employees
to
SelfOrganization, which took effect after
the instant dispute arose, '[t]he terms
and conditions of employment in the
government, including any political
subdivision or instrumentality thereof
and government-owned and controlled
corporations with original charters are
governed by law and employees
therein shall not strike for the purpose
of securing changes [thereto].'' (Ibid.,
p. 698)
(Jacinto v. Court of Appeals, 281
SCRA 657, Nov. 14, 1997, En Banc
[Panganiban])
203. Petitioners
public
school
teachers walked out of their classes
and engaged in mass actions during
certain dates in September 1990
protesting
the
alleged
unlawful
withholding of their salaries and other
economic benefits. They also raised
national issues, such as the removal of
US bases and the repudiation of
foreign debts, in their mass actions.
They refused to return to work despite
orders to do so and subsequently were
found guilty of conduct prejudicial to
the best interests of the service for
having absented themselves without
proper authority, from their schools

during regular school days, and


penalized.
They denied that they
engaged in strike but claimed that
they
merely
exercised
a
constitutionally guaranteed right the
right to peaceably assemble and
petition the government for redress of
grievances - and, therefore, should not
have been penalized.
Should their
contention be upheld?
Held:
Petitioners, who are public
schoolteachers and thus government
employees, do not seek to establish
that they have a right to strike.
Rather, they tenaciously insist that
their absences during certain dates in
September 1990 were a valid exercise
of their constitutional right to engage
in peaceful assembly to petition the
government
for
a
redress
of
grievances.
They claim that their
gathering was not a strike, therefore,
their participation therein did not
constitute any offense.
MPSTA v.
Laguio (Supra, per Narvasa, J., now
CJ.) and ACT v. Carino (Ibid.), in which
this Court declared that "these 'mass
actions' were to all intents and
purposes a strike; they constituted a
concerted and unauthorized stoppage
of, or absence from, work which it was
the teachers' duty to perform,
undertaken for essentially economic
reasons,"
should
not
principally
resolve the present case, as the
underlying facts are allegedly not
identical.
Strike, as defined by law, means any
temporary stoppage of work done by
the concerted action of employees as
a result of an industrial or labor
dispute. A labor dispute includes any
controversy or matter concerning
terms and conditions of employment;
or the association or representation of
persons
in
negotiating,
fixing,
maintaining, changing or arranging
the
terms
and
conditions
of
employment, regardless of whether
the disputants stand in the proximate
relation of employers and employees.
With these premises, we now evaluate
the circumstances of the instant
petition.
It cannot be denied that the mass
action or assembly staged by the
petitioners resulted in the non-holding
of classes in several public schools
during the corresponding period.
Petitioners do not dispute that the
grievances for which they sought
redress concerned the alleged failure
of public authorities - essentially, their
"employers" - to fully and justly

implement certain laws and measures


intended to benefit them materially x
x x.
And probably to clothe their
action with permissible character (In
justifying
their
mass
actions,
petitioners liken their activity to the
pro-bases rally led by former President
Corazon C. Aquino on September 10,
1991, participated in, as well, by
public
school
teachers
who
consequently absented themselves
from their classes. No administrative
charges were allegedly instituted
against any of the participants.), they
also raised national issues such as the
removal of the U.S. bases and the
repudiation of foreign debt.
In
Balingasan v. Court of Appeals (G.R.
No. 124678, July 31, 1997, per
Regalado, J.), however, this Court said
that the fact that the conventional
term "strike" was not used by the
participants to describe their common
course of action was insignificant,
since the substance of the situation,
and not its appearance, was deemed
controlling.
Moreover, the petitioners here x x x
were not penalized for the exercise of
their right to assemble peacefully and
to petition the government for a
redress of grievances.
Rather, the
Civil Service Commission found them
guilty of conduct prejudicial to the
best interest of the service for having
absented themselves without proper
authority, from their schools during
regular school days, in order to
participate in the mass protest, their
absence ineluctably resulting in the
non-holding of classes and in the
deprivation of students of education,
for which they were responsible. Had
petitioners availed themselves of their
free time - recess, after classes,
weekends or holidays - to dramatize
their grievances and to dialogue with
the proper authorities within the
bounds of law, no one - not the DECS,
the CSC or even this Court - could
have held them liable for the valid
exercise
of
their
constitutionally
guaranteed rights.
As it was, the
temporary
stoppage
of
classes
resulting from their activity necessarily
disrupted public services, the very evil
sought to be forestalled by the
prohibition
against
strikes
by
government workers.
Their act by
their nature was enjoined by the Civil
Service law, rules and regulations, for
which they must, therefore, be made
answerable.
(Jacinto v. CA, 281
SCRA 657, Nov. 14, 1997, En Banc
[Panganiban])

The Non-Impairment Clause


204. Is the constitutional prohibition
against impairing contractual
obligations absolute?
Held: 1. Nor is there merit in the
claim
that
the
resolution
and
memorandum circular violate the
contract clause of the Bill of Rights.
The executive order creating the POEA
was enacted to further implement the
social justice provisions of the 1973
Constitution, which have been greatly
enhanced and expanded in the 1987
Constitution by placing them under a
separate Article (Article XIII).
The
Article on Social Justice was aptly
described as the "heart of the new
Charter" by the President of the 1986
Constitutional Commission, retired
Justice Cecilia Munoz Palma. Social
justice is identified with the broad
scope of the police power of the state
and requires the extensive use of such
power. X x x.
The constitutional prohibition against
impairing contractual obligations is not
absolute and is not to be read with
literal exactness. It is restricted to
contracts with respect to property or
some object of value and which confer
rights that may be asserted in a court
of justice; it has no application to
statutes relating to public subjects
within the domain of the general
legislative powers of the State and
involving the public rights and public
welfare of the entire community
affected by it. It does not prevent a
proper exercise by the State of its
police power by enacting regulations
reasonably necessary to secure the
health, safety, morals, comfort, or
general welfare of the community,
even though contracts may thereby be
affected, for such matters cannot be
placed by contract beyond the power
of the State to regulate and control
them.
Verily, the freedom to contract is not
absolute; all contracts and all rights
are subject to the police power of the
State and not only may regulations
which affect them be established by
the State, but all such regulations
must be subject to change from time
to time, as the general well-being of
the community may require, or as the
circumstances may change, or as
experience may demonstrate the
necessity. And under the Civil Code,
contracts of labor are explicitly subject

to the police power of the State


because they are not ordinary
contracts but are impressed with
public interest. Article 1700 thereof
expressly provides:
Art. 1700.
The relations between
capital and labor are not merely
contractual. They are so impressed
with
public
interest
that
labor
contracts must yield to the common
good. Therefore, such contracts are
subject to the special laws on labor
unions, collective bargaining, strikes
and lockouts, closed shop, wages,
working conditions, hours of labor and
similar subjects.
The
challenged
resolution
and
memorandum circular being valid
implementations of E.O. No. 797
(Creating the POEA), which was
enacted under the police power of the
State, they cannot be struck down on
the ground that they violate the
contract clause. To hold otherwise is
to alter long-established constitutional
doctrine and to subordinate the police
power to the contract clause. (The
Conference of Maritime Manning
Agencies, Inc. v. POEA, 243 SCRA
666, April 21, 1995 [Davide, Jr.])
2. Petitioners pray that the present
action should be barred, because
private respondents have voluntarily
executed quitclaims and releases and
received
their
separation
pay.
Petitioners claim that the present suit
is a "grave derogation of the
fundamental principle that obligations
arising from a valid contract have the
force of law between the parties and
must be complied with in good faith."
The Court disagrees.
Jurisprudence
holds that the constitutional guarantee
of non-impairment of contract is
subject to the police power of the
state and to reasonable legislative
regulations promoting health, morals,
safety and welfare. Not all quitclaims
are per se invalid or against public
policy, except (1) where there is clear
proof that the waiver was wangled
from an unsuspecting or gullible
person, or (2) where the terms of
settlement are unconscionable on
their face. In these cases, the law will
step in to annul the questionable
transactions.
Such quitclaim and
release agreements are regarded as
ineffective to bar the workers from
claiming the full measure of their legal
rights.

In the case at bar, the private


respondents agreed to the quitclaim
and release in consideration of their
separation pay.
Since they were
dismissed allegedly for business
losses, they are entitled to separation
pay under Article 283 of the Labor
Code. And since there was thus no
extra consideration for the private
respondents
to
give
up
their
employment,
such
undertakings
cannot be allowed to bar the action for
illegal dismissal.
(Bogo-Medellin
Sugarcane Planters Association,
Inc. v. NLRC, 296 SCRA 108, 124,
[Panganiban])
3.
Only slightly less abstract but
nonetheless
hypothetical
is
the
contention
of
CREBA
that
the
imposition of the VAT on the sales and
leases of real estate by virtue of
contracts
entered
prior
to
the
effectivity of the law would violate the
constitutional provision that "No law
impairing the obligation of contracts
shall be passed." It is enough to say
that the parties to a contract cannot,
through the exercise of prophetic
discernment, fetter the exercise of the
taxing power of the State. For not
only are existing laws read into
contracts in order to fix obligations as
between parties, but the reservation
of essential attributes of sovereign
power is also read into contracts as a
basic postulate of the legal order. The
policy of protecting contracts against
impairment
presupposes
the
maintenance of a government which
retains adequate authority to secure
the peace and good order of society.
In truth, the Contract Clause has never
been thought as a limitation on the
exercise of the State's power of
taxation save only where a tax
exemption has been granted for a
valid consideration. X x x. (Tolentino
v. Secretary of Finance, 235 SCRA
630, 685-686, Aug. 25, 1994, En
Banc [Mendoza])
4.
Since timber licenses are not
contracts, the non-impairment clause
x x x cannot be invoked.
X x x, even if it is to be assumed that
the same are contracts, the instant
case does not involve a law or even an
executive issuance declaring the
cancellation or modification of existing
timber licenses.
Hence, the nonimpairment clause cannot as yet be
invoked.
Nevertheless, granting
further that a law has actually been
passed mandating cancellations or

modifications, the same cannot still be


stigmatized as a violation of the nonimpairment clause. This is because by
its very nature and purpose, such a
law could have only been passed in
the exercise of the police power of the
state for the purpose of advancing the
right of the people to a balanced and
healthful ecology, promoting their
health and enhancing their general
welfare. X x x.

procedure, guidelines and duties


which
the
arresting,
detaining,
inviting, or investigating officer or his
companions must do and observe at
the time of making an arrest and again
at and during the time of the custodial
interrogation in accordance with the
Constitution,
jurisprudence
and
Republic Act No. 7438 (An Act Defining
Certain Rights of Person Arrested,
Detained
or
Under
Custodial
Investigation as well as the Duties of
the
Arresting,
Detaining,
and
Investigating Officers and Providing
Penalties for Violations Thereof). It is
high-time
to
educate
our
lawenforcement agencies who neglect
either by ignorance or indifference the
so-called Miranda rights which had
become insufficient and which the
Court must update in the light of new
legal developments:

In short, the non-impairment clause


must yield to the police power of the
state.
Finally, it is difficult to imagine x x x
how the non-impairment clause could
apply with respect to the prayer to
enjoin the respondent Secretary from
receiving,
accepting,
processing,
renewing or approving new timber
license for, save in cases of renewal,
no contract would have as yet existed
in the other instances. Moreover, with
respect to renewal, the holder is not
entitled to it as a matter of right.
(Oposa v. Factoran, Jr., 224 SCRA
792 [1993])
5. Anent petitioners' contention that
the forcible refund of incentive
benefits
is
an
unconstitutional
impairment of a contractual obligation,
suffice it to state that "[n]ot all
contracts
entered
into
by
the
government will operate as a waiver of
its non-suability; distinction must be
made between its sovereign and
proprietary acts. The acts involved in
this case are governmental. Besides,
the Court is in agreement with the
Solicitor General that the incentive
pay or benefit is in the nature of a
bonus which is not a demandable or
enforceable obligation. (Blaquera v.
Alcala, 295 SCRA 366, 446, Sept.
11, 1998, En Banc [Purisima])
The In-Custodial Investigation
Rights of an Accused Person
205. State the procedure, guidelines
and duties which the arresting,
detaining, inviting, or investigating
officer or his companions must do and
observe at the time of making an
arrest and again at and during the
time of the custodial interrogation.
Held: Lastly, considering the heavy
penalty of death and in order to
ensure that the evidence against an
accused were obtained through lawful
means, the Court, as guardian of the
rights of the people lays down the

1)

2)

3)

4)

5)

6)

The
person
arrested,
detained, invited or under custodial
investigation must be informed in a
language known to and understood by
him of the reason for the arrest and he
must be shown the warrant of arrest, if
any.
Every
other
warnings,
information or communication must be
in a language known to and
understood by said person;
He must be warned that he
has a right to remain silent and that
any statement he makes may be used
as evidence against him;
He must be informed that he
has the right to be assisted at all times
and have the presence of an
independent and competent lawyer,
preferably of his own choice;
He must be informed that if
he has no lawyer or cannot afford the
services of a lawyer, one will be
provided for him; and that a lawyer
may also be engaged by any person in
his behalf, or may be appointed by the
court upon petition of the person
arrested or one acting on his behalf;
That whether or not the
person arrested has a lawyer, he must
be
informed
that
no
custodial
investigation in any form shall be
conducted except in the presence of
his counsel of after a valid waiver has
been made;
The person arrested must be
informed that, at any time, he has the
right to communicate or confer by the
most expedient means - telephone,
radio, letter or messenger - with his
lawyer (either retained or appointed),
any member of his immediate family,
or any medical doctor, priest or
minister chosen by him or by any one
from his immediate family or by his

counsel, or be visited by/confer with


duly
accredited
national
or
international
non-government
organization.
It shall be the
responsibility of the officer to ensure
that this is accomplished;
7)
He must be informed that he
has the right to waive any of said
rights provided it is made voluntarily,
knowingly and intelligently and ensure
that he understood the same;
8)
In addition, if the person
arrested waives his right to a lawyer,
he must be informed that it must be
done in writing and in the presence of
counsel, otherwise, he must be
warned that the waiver is void even if
he insist on his waiver and chooses to
speak;
9)
That the person arrested
must be informed that he may indicate
in any manner at any time or stage of
the process that he does not wish to
be questioned with warning that once
he makes such indication, the police
may not interrogate him if the same
had not yet commenced, or the
interrogation must cease if it has
already begun;
10)
The person arrested must be
informed that his initial waiver of his
right to remain silent, the right to
counsel or any of his rights does not
bar him from invoking it at any time
during the process, regardless of
whether he may have answered some
questions
or
volunteered
some
statements;
11)
He must also be informed
that any statement or evidence, as the
case may be, obtained in violation of
any of
the
foregoing, whether
inculpatory or exculpatory, in whole or
in part, shall be admissible in
evidence.
(People v. Mahinay, 302 SCRA 455,
Feb. 1, 1999, En Banc [Per
Curiam])
206. Explain the kind of information
that is required to be given by law
enforcement officers to suspect during
custodial investigation.
Held: [I]t is settled that ones right to
be informed of the right to remain
silent and to counsel contemplates the
transmission
of
meaningful
information rather just the ceremonial
and perfunctory recitation of an
abstract constitutional principle. It is
not enough for the interrogator to
merely repeat to the person under
investigation the provisions of Section
12, Article III of the 1987 Constitution;
the former must also explain the
effects of such provision in practical

terms e.g., what the person under


investigation may or may not do and
in a language the subject fairly
understands. The right to be informed
carries with it a correlative obligation
on the part of the police investigator
to explain, and contemplates effective
communication which results in the
subjects understanding of what is
conveyed. Since it is comprehension
that is sought to be attained, the
degree of explanation required will
necessarily vary and depend on the
education, intelligence, and other
relevant personal circumstances of the
person undergoing investigation. In
further ensuring the right to counsel, it
is not enough that the subject is
informed of such right; he should also
be asked if he wants to avail of the
same and should be told that he could
ask for counsel if he so desired or that
one could be provided him at his
request. If he decides not to retain a
counsel of his choice or avail of one to
be provided for him and, therefore,
chooses to waive his right to counsel,
such waiver, to be valid and effective,
must still be made with the assistance
of counsel, who, under prevailing
jurisprudence, must be a lawyer.
(People v. Canoy, 328 SCRA 385,
March 17, 2000, 1st Div. [Davide,
CJ])
207. What is the meaning of
competent counsel under Section 12
of the Bill of Rights?
Held: The meaning of competent
counsel was explained in People v.
Deniega (251 SCRA 626, 637) as
follows:
x x x [T]he lawyer called to be
present during such investigation
should be as far as reasonably
possible, the choice of the individual
undergoing questioning. If the lawyer
were one furnished in the accuseds
behalf, it is important that he should
be competent and independent, i.e.,
that he is willing to fully safeguard the
constitutional rights of the accused, as
distinguished from one who would
merely
be
giving
a
routine,
peremptory and meaningless recital of
the individuals rights. In People v.
Basay (219 SCRA 404, 418), this Court
stressed that an accuseds right to be
informed of the right to remain silent
and to counsel contemplates the
transmission
of
meaningful
information rather than just the
ceremonial and perfunctory recitation
of an abstract constitutional principle.

Ideally therefore, a lawyer engaged


for an individual facing custodial
investigation (if the latter could not
afford one) should be engaged by the
accused (himself), or by the latters
relative or person authorized by him to
engage an attorney or by the court,
upon proper petition of the accused or
person authorized by the accused to
file such petition. Lawyers engaged
by the police, whatever testimonials
are given as proof of their probity and
supposed independence, are generally
suspect, as in many areas, the
relationship between lawyers and law
enforcement
authorities
can
be
symbiotic.
x x x The competent or independent
lawyer so engaged should be present
from the beginning to end, i.e., at all
stages of the interview, counseling or
advising caution reasonably at every
turn of the investigation, and stopping
the interrogation once in a while either
to give advice to the accused that he
may either continue, choose to remain
silent or terminate the interview.
(People v. Espiritu, 302 SCRA 533,
Feb.
2,
1999,
3rd
Div.
[Panganiban])
208. Can a PAO lawyer be considered
an independent counsel within the
contemplation of Section 12, Article III,
1987 Constitution?
Held: In People v. Oracoy, 224 SCRA
759 [1993]; People v. Bandula, 232
SCRA 566 [1994], the SC has held that
a PAO lawyer can be considered an
independent
counsel
within
the
contemplation of the Constitution
considering that he is not a special
counsel, public or private prosecutor,
counsel of the police, or a municipal
attorney whose interest is admittedly
adverse to that of the accusedappellant. Thus, the assistance of a
PAO lawyer satisfies the constitutional
requirement of a competent and
independent counsel for the accused.
(People v. Bacor, 306 SCRA 522,
April
30,
1999,
2nd
Div.
[Mendoza])
209. Is the confession of an accused
given
spontaneously,
freely
and
voluntarily to the Mayor admissible in
evidence, considering that the Mayor
has operational supervision and
control over the local police and may
arguably
be
deemed
a
law
enforcement officer?
Held: While it is true that a municipal
mayor has operational supervision

and control over the local police and


may arguably be deemed a law
enforcement officer for purposes of
applying Section 12(1) and (3) of
Article III of the Constitution, however,
appellants confession to the mayor
was not made in response to any
interrogation by the latter. In fact, the
mayor did not question the appellant
at all. No police authority ordered
appellant to talk to the mayor. It was
appellant himself who spontaneously,
freely and voluntarily sought the
mayor for a private meeting.
The
mayor did not know that appellant was
going to confess his guilt to him.
When appellant talked with the mayor
as a confidant and not as a law
enforcement officer, his uncounselled
confession to him did not violate his
constitutional rights. Thus, it has been
held that the constitutional procedures
on custodial investigation do not apply
to a spontaneous statement, not
elicited through questioning by the
authorities, but given in an ordinary
manner whereby appellant orally
admitted having committed the crime.
What the Constitution bars is the
compulsory disclosure of incriminating
facts or confessions. The rights under
Section 12 are guaranteed to preclude
the slightest use of coercion by the
State as would lead the accused to
admit something false, not to prevent
him from freely and voluntarily telling
the truth. (People v. Andan, 269
SCRA 95, March 3, 1997)
210. Are
confessions
made
in
response to questions by news
reporters admissible in evidence?
Answer: Yes. Confessions made in
response to questions by news
reporters, not by the police or any
other
investigating
officer,
are
admissible. In People v. Vizcarra, 115
SCRA 743, 752 [1982], where the
accused,
under
custody,
gave
spontaneous answers to a televised
interview by several press reporters in
the office of the chief of the CIS, it was
held that statements spontaneously
made by a suspect to news reporters
on a televised interview are deemed
voluntary and are admissible in
evidence. In People v. Andan, 269
SCRA 95, March 3, 1997, it was held
that appellants confessions to the
news reporters were given free from
any undue influence from the police
authorities. The news reporters acted
as
news
reporters
when
they
interviewed appellant. They were not
acting under the direction and control
of the police.
They did not force

appellant to grant them an interview


and reenact the commission of the
crime.
In fact, they asked his
permission before interviewing him.
The Supreme Court further ruled that
appellants verbal confessions to the
newsmen are not covered by Section
12(1) and (3) of Article III of the
Constitution and, therefore, admissible
in evidence.
211. Discuss why lower courts
should act with extreme caution in
admitting in evidence accuseds
videotaped media confessions.
Held: Apropos the court a quos
admission of accused-appellants
videotaped confession, we find such
admission proper. The interview was
recorded on video and it showed
accused-appellant unburdening his
guilt willingly, openly and publicly in
the presence of newsmen. Such
confession does not form part of
custodial investigation as it was not
given to police officers but to media
men in an attempt to elicit sympathy
and forgiveness from the public.
Besides, if he had indeed been forced
into confessing, he could have easily
sought succor from the newsmen who,
in all likelihood, would have been
sympathetic with him. X x x
X x x However, because of the
inherent danger in the use of
television as a medium for admitting
ones guilt, and the recurrence of this
phenomenon in several cases (People
v. Vizcarra, No. L-38859, 30 July 1982,
115 SCRA 743; others omitted), it is
prudent that trial courts are reminded
that extreme caution must be taken in
further admitting similar confessions.
For in all probability, the police, with
the connivance of unscrupulous media
practitioners,
may
attempt
to
legitimize
coerced
extrajudicial
confessions and place them beyond
the exclusionary rule by having an
accused
admit
an
offense
on
television. Such a situation would be
detrimental to the guaranteed rights
of the accused and thus imperil our
criminal justice system.
We do not suggest that videotaped
confessions given before media men
by an accused with the knowledge of
and in the presence of police officers

are impermissible. Indeed, the line


between proper and invalid police
techniques and conduct is a difficult
one to draw, particularly in cases such
as this where it is essential to make
sharp
judgments
in
determining
whether a confession was given under
coercive physical or psychological
atmosphere.
A word of caution then to lower courts:
we should never presume that all
media
confessions
described
as
voluntary have been freely given. This
type of confession always remains
suspect and therefore should be
thoroughly examined and scrutinized.
Detection of coerced confessions is
admittedly a difficult and arduous task
for the courts to make. It requires
persistence and determination in
separating polluted confessions from
untainted ones. We have a sworn duty
to be vigilant and protective of the
rights guaranteed by the Constitution.
(People v. Endino, 353 SCRA 307,
Feb.
20,
2001,
2nd
Div.
[Bellosillo])
212. Discuss the two kinds of
involuntary or coerced confessions
under Section 12, Article III of the
1987 Constitution. Illustrate how the
Court
should
appreciate
said
involuntary or coerced confessions.
Held:
There are two kinds of
involuntary or coerced confessions
treated in this constitutional provision:
(1) those which are the product of
third degree methods such as torture,
force, violence, threat, intimidation,
which are dealt with in paragraph 2 of
Section 12, and (2) those which are
given without the benefit of Miranda
warnings, which are the subject of
paragraph 1 of the same Section 12.
Accused-appellant claims that his
confession was obtained by force and
threat. Aside from this bare assertion,
he has shown no proof of the use of
force and violence on him. He did not
seek medical treatment nor even a
physical examination. His allegation
that the fact that he was made to sign
the confession five times is proof that
he refused to sign it.
Xxx
We discern no sign that the confession
was involuntarily executed from the
fact that it was signed by accusedappellant five times.

Xxx

There was thus only a perfunctory


reading of the Miranda rights to
accused-appellant without any effort
to find out from him whether he
wanted to have counsel and, if so,
whether he had his own counsel or he
wanted the police to appoint one for
him. This kind of giving of warnings,
in several decisions of this Court, has
been found to be merely ceremonial
and
inadequate
to
transmit
meaningful information to the suspect.
Especially in this case, care should
have been scrupulously observed by
the police investigator that accusedappellant was specifically asked these
questions considering that he only
finished the fourth grade of the
elementary school. X x x

Extrajudicial confessions are presumed


voluntary, and, in the absence of
conclusive evidence showing the
declarants consent in executing the
same
has
been
vitiated,
such
confession will be sustained.
Moreover, the confession contains
details that only the perpetrator of the
crime could have given. X x x. It has
been held that voluntariness of a
confession may be inferred from its
being replete with details which could
possibly be supplied only by the
accused, reflecting spontaneity and
coherence which cannot be said of a
mind on which violence and torture
have been applied. When the details
narrated in an extrajudicial confession
are such that they could not have
been concocted by one who did not
take part in the acts narrated, where
the claim of maltreatment in the
extraction of the confession is
unsubstantiated and where abundant
evidence exists showing that the
statement was voluntarily executed,
the confession is admissible against
the declarant. There is greater reason
for finding a confession to be
voluntary where it is corroborated by
evidence aliunde which dovetails with
the essential facts contained in such
confession.

Moreover, Article III, Section 12(1)


requires
that
counsel
assisting
suspects in custodial interrogations be
competent and independent. Here,
accused-appellant was assisted by
Atty. De los Reyes, who, though
presumably competent, cannot be
considered an independent counsel
as contemplated by the law for the
reason
that
he
was
station
commander of the WPD at the time he
assisted accused-appellant. X x x.
This is error. As observed in People v.
Bandula (232 SCRA 566 [1994]), the
independent counsel required by
Article III, Section 12(1) cannot be
special counsel, public or private
prosecutor, municipal attorney, or
counsel of the police whose interest is
admittedly adverse to the accused. In
this case, Atty. De los Reyes, as PC
Captain and Station Commander of
the WPD, was part of the police force
who could not be expected to have
effectively and scrupulously assisted
accused-appellant in the investigation.
To allow such a happenstance would
render illusory the protection given to
the
suspect
during
custodial
investigation.
(People v. Obrero,
332 SCRA 190, 220 208, May 17,
2000, 2nd Div. [Mendoza])

But what renders the confession of


accused-appellant inadmissible is the
fact that accused-appellant was not
given the Miranda warnings effectively.
Under
the
Constitution,
an
uncounseled statement, such as it is
called in the United States from which
Article III, Section 12(1) was derived, is
presumed
to
be
psychologically
coerced.
Swept into an unfamiliar
environment and surrounded by
intimidating figures typical of the
atmosphere of police interrogation,
the suspect really needs the guiding
hand of counsel.
Now, under the first paragraph of this
provision, it is required that the
suspect in custodial interrogation must
be given the following warnings: (1) he
must be informed of his right to
remain silent; (2) he must be warned
that anything he says can and will be
used against him; and (3) he must be
told that he has a right to counsel, and
that if he is indigent, a lawyer will be
appointed to represent him.
Xxx

213. What are the requirements for


an extra-judicial confession of an
accused to be admissible in evidence?
Held:
1.
In jurisprudence, no
confession can be admitted in
evidence unless it is given:
1)

Freely
and
voluntarily,
without compulsion, inducement or
trickery;
2)
Knowingly based on an
effective
communication
to
the

individual
under
custodial
investigation of his constitutional
rights; and
3)
Intelligently
with
full
appreciation of its importance and
comprehension of its consequences.

214. Is the choice of a lawyer by a


person under custodial investigation
who cannot afford the services of a
counsel exclusive as to preclude other
equally competent and independent
attorneys from handling his defense?

Once admitted, the confession must


inspire credibility or be one which the
normal experience of mankind can
accept as being within the realm of
probability.

Held: It must be remembered in this


regard that while the right to counsel
is immutable, the option to secure the
services of counsel de parte is not
absolute. Indeed

A confession meeting all the foregoing


requisites constitutes evidence of a
high order since it is supported by the
strong presumption that no person of
normal mind will knowingly, freely and
deliberately confess that he is the
perpetrator
of
a
crime
unless
prompted by truth and conscience.
When all these requirements are met
and the confession is admitted in
evidence, the burden of proof that it
was obtained by undue pressure,
threat or intimidation rests upon the
accused.
(People v. Fabro, 277
SCRA
19,
Aug.
11,
1997
[Panganiban])

The
phrase
competent
and
independent and preferably of his
own choice were explicit details
which
were
added
upon
the
persistence of human rights lawyers in
the 1986 Constitutional Commission
who pointed out cases where, during
the martial law period, the lawyers
made available to the detainee would
be one appointed by the military and
therefore beholden to the military.
(Citing I Record of the Constitutional
Commission 731-734; I Bernas, The
Constitution of the Republic of the
Philippines, 1987 1st ed., p. 347)
Xxx xxx xxx

2. Numerous decisions of this Court


rule
that
for
an
extrajudicial
confession to be admissible, it must
be: 1) voluntary; 2) made with the
assistance
of
competent
and
independent counsel; 3) express; and
4) in writing.
The mantle of protection afforded by
the
above-quoted
constitutional
provision covers the period from the
time a person is taken into custody for
the investigation of his possible
participation in the commission of a
crime or from the time he is singled
out as a suspect in the commission of
the offense although not yet in
custody.
The exclusionary rule is
premised on the presumption that the
defendant is thrust into an unfamiliar
atmosphere
running
through
menacing
police
interrogation
procedures where the potentiality for
compulsion, physical or psychological
is forcefully apparent.
However, the rule is not intended as a
deterrent to the accused from
confessing guilt if he voluntarily and
intelligently so desires but to protect
the accused from admitting what he is
coerced to admit although untrue.
(People v. Base, 329 SCRA 158,
169-171, March 30, 2000, 1st Div.
[Ynares-Santiago])

Withal, the word preferably under


Section 12(1), Article 3 of the 1987
Constitution does not convey the
message that the choice of a lawyer
by a person under investigation is
exclusive as to preclude other equally
competent and independent attorneys
from handling his defense. If the rule
were otherwise, then, the tempo of a
custodial investigation will be solely in
the hands of the accused who can
impede, nay, obstruct the progress of
the interrogation by simply selecting a
lawyer who for one reason or another,
is not available to protect his interest.
This absurd scenario could not have
been contemplated by the framers of
the charter.
While the initial choice in cases where
a person under custodial investigation
cannot afford the services of a lawyer
is naturally lodged in the police
investigators, the accused really has
the final choice as he may reject the
counsel chosen for him and ask for
another one. A lawyer provided by the
investigators is deemed engaged by
the accused where he never raised
any objection against the formers
appointment during the course of the
investigation
and
the
accused
thereafter subscribes to the veracity of
his statement before the swearing
officer.

Verily, to be an effective counsel [a]


lawyer need not challenge all the
questions being propounded to his
client. The presence of a lawyer is not
intended to stop an accused from
saying
anything
which
might
incriminate him but, rather, it was
adopted in our Constitution to
preclude the slightest coercion as
would lead the accused to admit
something false (People v. Layuso, 175
SCRA 47 [1989]).
The counsel,
however, should never prevent an
accused from freely and voluntarily
telling the truth. (People v. Base,
329 SCRA 158, 169-171, March 30,
2000, 1st Div. [Ynares-Santiago])

admitted that he was with the victim


on the evening of January 12, 1994,
the probable time of the commission
of the crime and that he carried her on
his shoulder but that he was too drunk
to remember what subsequently
happened. The arresting policeman
admitted that he did not inform the
appellant of his constitutional rights to
remain silent and to counsel. We note
that
the
alleged
admission
is
incriminating because it places the
accused in the company of the victim
at the time the crime was probably
committed.

215. Should courts be allowed to


distinguish
between
preliminary
questioning
and
custodial
investigation proper when applying
the exclusionary rule?

The accused was under arrest for the


rape and killing of x x x and any
statement allegedly made by him
pertaining to his possible complicity in
the crime without prior notification of
his constitutional rights is inadmissible
in evidence.
The
policemans
apparent attempt to circumvent the
rule by insisting that the admission
was made during an informal talk
prior to custodial investigation prior is
not tenable. The appellant was not
invited to the police station as part of
a general inquiry for any possible lead
to the perpetrators of the crime under
investigation. At the time the alleged
admission was made the appellant
was in custody and had been arrested
as the prime suspect in the rape and
killing of x x x. The exclusionary rule
presumes that the alleged admission
was coerced, the very evil the rule
stands to avoid. Supportive of such
presumption is the absence of a
written extra-judicial confession to
that effect and the appellants denial
in court of the alleged oral admission.
The alleged admission should be
struck down as inadmissible. (People
v. Bravo, 318 SCRA 812, Nov. 22,
1999, En Banc [Gonzaga-Reyes])

Held: The exclusionary rule sprang


from
a
recognition
that
police
interrogatory procedures lay fertile
grounds for coercion, physical and
psychological, of the suspect to admit
responsibility for the crime under
investigation. It was not intended as a
deterrent to the accused from
confessing guilt, if he voluntarily and
intelligently so desires but to protect
the accused from admitting what he is
coerced to admit although untrue.
Law
enforcement
agencies
are
required to effectively communicate
the rights of a person under
investigation and to insure that it is
fully understood. Any measure short
of this requirement is considered a
denial of such right. Courts are not
allowed
to
distinguish
between
preliminary questioning and custodial
investigation proper when applying
the exclusionary rule. Any information
or admission given by a person while
in custody which may appear harmless
or innocuous at the time without the
competent
assistance
of
an
independent counsel should be struck
down as inadmissible. It has been
held, however, that an admission
made to news reporters or to a
confidant of the accused is not
covered by the exclusionary rule.
The admission allegedly made by the
appellant is not in the form of a
written extra-judicial confession; the
admission was allegedly made to the
arresting officer during an informal
talk at the police station after his
arrest as a prime suspect in the rape
and killing of x x x. The arresting
policeman testified that the appellant

The exclusionary rule applies.

216. Explain the procedure for out-ofcourt identification of suspects and the
test to determine the admissibility of
such identification.
Held: 1. In People v. Teehankee, Jr.
(249 SCRA 54, October 6, 1995), the
Court x x x explained the procedure
for out-of-court identification and the
test to determine the admissibility of
such identification.
It listed the
following ways of identifying the
suspects
during
custodial
investigation: show-up, mug shots and
line-ups. The Court there ruled:

x x x. Out-of-court identification is
conducted by the police in various
ways. It is done thru show-ups where
the suspect alone is brought face to
face with the witness for identification.
It is done thru mug shots where
photographs are shown to the witness
to identify the suspect. It is also done
thru line ups where a witness
identifies the suspect from a group of
persons lined up for the purpose.
Since
corruption
of
out-of-court
identification
contaminates
the
integrity of in court identification
during the trial of the case, courts
have fashioned out rules to assure its
fairness and its compliance with the
requirements of constitutional due
process. In resolving the admissibility
of and relying on out-of- court
identification of suspects, courts have
adopted the totality of circumstances
test where they consider the following
factors,
viz:
(1)
the
witness
opportunity to view the criminal at the
time of the crime; (2) the witness
degree of attention at that time; (3)
the accuracy of any prior description
given by the witness; (4) the level of
certainty demonstrated by the witness
at the identification; (5) the length of
time between the crime and the
identification;
and
(6)
the
suggestiveness of the identification
procedure. (Ibid., p. 95) (People v.
Timon, 281 SCRA 577, Nov. 12,
1997 [Panganiban])
2. x x x. The totality test has been
fashioned precisely to assure fairness
as
well
as
compliance
with
constitutional requirements of due
process in regard to out-of-court
identification.
These cited factors
must be considered to prevent
contamination of the integrity of incourt identifications better. (People
v. Gamer, 326 SCRA 660, Feb. 29,
2000, 2nd Div. [Quisumbing])
217. Does
the
prohibition
for
custodial
investigation
conducted
without the assistance of counsel
extend to a person in a police line-up?
Consequently, is the identification by
private complainant of accused who
was not assisted by counsel during
police line-up admissible in evidence?
Held: The prohibition x x x does not
extend to a person in a police line-up
because that stage of an investigation
is not yet a part of custodial
investigation. It has been repeatedly
held that custodial investigation
commences when a person is taken
into custody and is singled out as a

suspect in the commission of the


crime under investigation and the
police officers begin to ask questions
on the suspects participation therein
and which tend to elicit an admission.
The stage of an investigation wherein
a person is asked to stand in a police
line-up has been held to be outside
the mantle of protection of the right to
counsel because it involves a general
inquiry into an unsolved crime and is
purely investigatory in nature. It has
also been held that an uncounseled
identification at the police line-up does
not preclude the admissibility of an incourt identification. The identification
made by the private complainant in
the police line-up pointing to Pavillare
as one of his abductors is admissible
in evidence although the accusedappellant was not assisted by counsel.
X x x (People v. Pavillare, 329
SCRA 684, 694-695, April 5, 2000,
En Banc [Per Curiam])
218. Petitioner in a case x x x posits
the theory that since he had no
counsel
during
the
custodial
investigation when his urine sample
was taken and chemically examined,
Exhibits L and M, x x x are also
inadmissible in evidence since his
urine sample was derived in effect
from an uncounselled extra-judicial
confession. Petitioner claims that the
taking of his urine sample allegedly
violates Article III, Section 2 of the
Constitution x x x.
Should his
contentions be upheld?
Held: We are not persuaded. The
right to counsel begins from the time a
person is taken into custody and
placed under investigation for the
commission of a crime, i.e., when the
investigating officer starts to ask
questions to elicit information and/or
confession or admissions from the
accused. Such right is guaranteed by
the Constitution and cannot be waived
except in writing and in the presence
of counsel.
However, what the
Constitution prohibits is the use of
physical or moral compulsion to extort
communication from the accused, but
not an inclusion of his body in
evidence, when it may be material. In
fact, an accused may validly be
compelled to be photographed or
measured, or his garments or shoes
removed or replaced, or to move his
body to enable the foregoing things to
be done, without running afoul of the
proscription
against
testimonial
compulsion. The situation in the case
at bar falls within the exemption under
the
freedom
from
testimonial

compulsion since what was sought to


be examined came from the body of
the accused. This was a mechanical
act the accused was made to undergo
which was not meant to unearth
undisclosed facts but to ascertain
physical attributes determinable by
simple observation. In fact, the record
shows that petitioner and his coaccused were not compelled to give
samples of their urine but they in fact
voluntarily gave the same when they
were requested to undergo a drug
test. (Gutang v. People, 335 SCRA
479, July 11, 2000, 2nd Div. [De
Leon])
The Right to Bail
219. In bail application where the
accused is charged with a capital
offense, will it be proper for the judge
to grant bail without conducting
hearing if the prosecutor interposes no
objection to such application? Why?
Held: Jurisprudence is replete with
decisions
compelling
judges
to
conduct the required hearings in bail
applications, in which the accused
stands charged with a capital offense.
The absence of objection from the
prosecution is never a basis for the
grant of bail in such cases, for the
judge has no right to presume that the
prosecutor knows what he is doing on
account of familiarity with the case.
"Said reasoning is tantamount to
ceding to the prosecutor the duty of
exercising
judicial
discretion
to
determine whether the guilt of the
accused is strong. Judicial discretion is
the domain of the judge before whom
the petition for provisional liberty will
be decided. The mandated duty to
exercise discretion has never been
reposed upon the prosecutor."
Imposed in Baylon v. Sison (243 SCRA
284, April 6, 1995) was this mandatory
duty to conduct a hearing despite the
prosecution's
refusal
to
adduce
evidence
in
opposition
to
the
application to grant and fix bail.
(Joselito V. Narciso v. Flor Marie
Sta.
Romana-Cruz,
G.R.
No.
134504, March 17, 2000, 3rd Div.
[Panganiban])
220. What are the duties of the judge
in cases of bail applications where the
accused is charged with capital
offense?
Held: Basco v. Rapatalo (269 SCRA
220, March 5, 1997) enunciated the

following duties of the trial judge in


such petition for bail:
1)

Notify the prosecutor of the


hearing of the application for bail or
require
him
to
submit
his
recommendation;
2)
Conduct a hearing of the
application for bail regardless of
whether or not the prosecution refuses
to present evidence to show that the
guilt of the accused is strong for the
purpose of enabling the court to
exercise its sound discretion;
3)
Decide whether the evidence
of guilt of the accused is strong based
on the summary of evidence of the
prosecution;
4)
If the guilt of the accused is
not strong, discharge the accused
upon the approval of the bailbond.
Otherwise, petition should be denied.
The Court added:
"The aboveenumerated procedure should now
leave no room for doubt as to the
duties of the trial judge in cases of bail
applications.
So
basic
and
fundamental is it to conduct a hearing
in connection with the grant of bail in
the proper cases that it would amount
to judicial apostasy for any member of
the judiciary to disclaim knowledge or
awareness thereof."
Additionally, the court's grant or
refusal of bail must contain a
summary of the evidence for the
prosecution, on the basis of which
should be formulated the judge's own
conclusion on whether such evidence
is strong enough to indicate the guilt
of the accused. The summary thereof
is considered an aspect of procedural
due process for both the prosecution
and the defense; its absence will
invalidate the grant or the denial of
the application for bail. (Joselito V.
Narciso v. Flor Marie Sta. RomanaCruz, G.R. No. 134504, March 17,
2000, 3rd Div. [Panganiban])
221. Should
the
accused
who
remained
at
large
after
their
conviction be allowed provisional
liberty? Can the bail bond that the
accused previously posted be used
during the entire period of appeal?
Held: Despite an order of arrest from
the trial court and two warnings from
the Court of Appeals, petitioners had
remained at large. It is axiomatic that
for one to be entitled to bail, he should
be in the custody of the law, or
otherwise, deprived of liberty.
The
purpose of bail is to secure ones

release and it would be incongruous to


grant bail to one who is free.
Petitioners Compliance and Motion x x
x came short of an unconditional
submission to respondent courts
lawful order and to its jurisdiction.
The trial court correctly denied
petitioners motion that they be
allowed provisional liberty after their
conviction, under their respective bail
bonds. Apart from the fact that they
were at large, Section 5, Rule 114 of
the Rules of Court, as amended by
Supreme Court Administrative Circular
12-94, provides that:
Xxx
The Court, in its discretion, may allow
the accused to continue on provisional
liberty under the same bail bond
during the period to appeal subject to
the consent of the bondsman.
The bail bond that the accused
previously posted can only be used
during the 15-day period to appeal
(Rule 122) and not during the entire
period of appeal. This is consistent
with Section 2(a) of Rule 114 which
provides that the bail shall be
effective upon approval and remain in
force at all stages of the case, unless
sooner
cancelled,
until
the
promulgation of the judgment of the
Regional Trial Court, irrespective of
whether the case was originally filed in
or appealed to it. This amendment,
introduced by SC Administrative
Circular 12-94 is a departure from the
old rules which then provided that bail
shall be effective and remain in force
at all stages of the case until its full
determination, and thus even during
the period of appeal. Moreover, under
the present rule, for the accused to
continue his provisional liberty on the
same bail bond during the period to
appeal, consent of the bondsman is
necessary. From the record, it appears
that the bondsman x x x filed a motion
in the trial court x x x for the
cancellation of petitioners bail bond
for the latters failure to renew the
same upon its expiration. Obtaining
the consent of the bondsman was,
thus, foreclosed.
(Maguddatu v.
Court of Appeals, 326 SCRA 362,
Feb. 23, 2000, 1st Div. [Kapunan])
223. Is a condition in an application
for bail that accused be first arraigned
before he could be granted bail valid?
Held: In requiring that petitioner be
first arraigned before he could be

granted
bail,
the
trial
court
apprehended that if petitioner were
released on bail he could, by being
absent, prevent his early arraignment
and thereby delay his trial until the
complainants got tired and lost
interest in their cases.
Hence, to
ensure
his
presence
at
the
arraignment, approval of petitioners
bail bonds should be deferred until he
could be arraigned. After that, even if
petitioner does not appear, trial can
proceed as long as he is notified of the
date of the hearing and his failure to
appear is unjustified, since under Art.
III, Sec. 14(2) of the Constitution, trial
in absencia is authorized. This seems
to be the theory of the trial court in its
x x x order conditioning the grant of
bail to petitioner on his arraignment.
This theory is mistaken. In the first
place x x x in cases where it is
authorized, bail should be granted
before arraignment, otherwise the
accused may be precluded from filing
a motion to quash.
For if the
information is quashed and the case is
dismissed, there would then be no
need for the arraignment of the
accused. In the second place, the trial
court could ensure the presence of
petitioner at the arraignment precisely
by granting bail and ordering his
presence at any stage of the
proceedings, such as arraignment.
Under Rule 114, Sec. 2(b) of the Rules
on Criminal Procedure, one of the
conditions of bail is that the accused
shall appear before the proper court
whenever so required by the court or
these Rules, while under Rule 116,
Sec. 1(b) the presence of the accused
at the arraignment is required.
On the other hand, to condition the
grant of bail to an accused on his
arraignment would be to place him in
a position where he has to choose
between (1) filing a motion to quash
and thus delay his release on bail
because until his motion to quash can
be resolved, his arraignment cannot
be held, and (2) foregoing the filing of
a motion to quash so that he can be
arraigned at once and thereafter be
released on bail.
These scenarios
certainly undermine the accuseds
constitutional right not to be put on
trial except upon valid complaint or
information sufficient to charge him
with a crime and his right to bail.
(Lavides v. CA, 324 SCRA 321,
Feb. 1, 2000, 2nd Div. [Mendoza])

The Right to be Informed of the


Nature and Cause of Accusation
against the Accused
224. What are the objectives of the
right to be informed of the nature and
cause of accusations against the
accused?
Held:
Instructive in this regard is
Section 6, Rule 110 of the Rules of
Court x x x.
The purpose of the above-quoted rule
is to inform the accused of the nature
and cause of the accusation against
him, a right guaranteed by no less
than the fundamental law of the land
(Article III, Section 14[2], 1987
Constitution).
Elaborating on the
defendants right to be informed, the
Court held in Pecho v. People (262
SCRA 518) that the objectives of this
right are:
1)

To furnish the accused with


such a description of the charge
against him as will enable him to make
the defense;
2)
To avail himself of his
conviction or acquittal for protection
against a further prosecution for the
same cause; and
3)
To inform the court of the
facts alleged, so that it may decide
whether they are sufficient in law to
support a conviction, if one should be
had.
It is thus imperative that the
Information filed with the trial court be
complete to the end that the
accused may suitably prepare for his
defense.
Corollary to this, an
indictment must fully state the
elements of the specific offense
alleged to have been committed as it
is the recital of the essentials of a
crime which delineates the nature and
cause of accusation against the
accused.
Xxx
In the case under scrutiny, the
information does not allege the
minority of the victim x x x although
the same was proven during the trial x
x x. The omission is not merely formal
in nature since doctrinally, an accused
cannot be held liable for more than
what he is indicted for. It matters not
how conclusive and convincing the
evidence of guilt may be, but an
accused cannot be convicted of any
offense, not charged in the Complaint
or Information on which he is tried or

therein necessarily included. He has a


right to be informed of the nature of
the offense with which he is charged
before he is put on trial. To convict an
accused of an offense higher than that
charged
in
the
Complaint
or
Information on which he is tried would
constitute unauthorized denial of that
right. (People v. Bayya, 327 SCRA
771, March 10, 2000, En Banc
[Purisima])
The Right to a Fair Trial
225. What is the purpose of the rule
barring trial or sentence of an insane
person?
What are the reasons
underlying it?
Held:
The rule barring trial or
sentence of an insane person is for the
protection of the accused, rather than
of the public. It has been held that it
is inhuman to require an accused
disabled by God to make a just
defense for his life or liberty. To put a
legally incompetent person on trial or
to convict and sentence him is a
violation of the constitutional rights to
a fair trial; and this has several
reasons underlying it. For one, the
accuracy of the proceedings may not
be assured, as an incompetent
defendant who cannot comprehend
the proceedings may not appreciate
what information is relevant to the
proof of his innocence. Moreover, he
is not in a position to exercise many of
the rights afforded a defendant in a
criminal case, e.g., the right to
effectively consult with counsel, the
right to testify in his own behalf, and
the right to confront opposing
witnesses, which rights are safeguards
for the accuracy of the trial result.
Second,
the
fairness
of
the
proceedings may be questioned, as
there are certain basic decisions in the
course of a criminal proceeding which
a defendant is expected to make for
himself, and one of these is his plea.
Third, the dignity of the proceedings
may be disrupted, for an incompetent
defendant is likely to conduct himself
in the courtroom in a manner which
may destroy the decorum of the court.
Even if the defendant remains passive,
his
lack
of
comprehension
fundamentally impairs the functioning
of the trial process.
A criminal
proceeding
is
essentially
an
adversarial proceeding.
If the
defendant is not a conscious and
intelligent participant, the adjudication
loses its character as a reasoned
interaction between an individual and

his community and becomes and


invective against an insensible object.
Fourth, it is important that the
defendant knows why he is being
punished, a comprehension which is
greatly
dependent
upon
his
understanding of what occurs at trial.
An incompetent defendant may not
realize the moral reprehensibility of his
conduct.
The societal goal of
institutionalized retribution may be
frustrated when the force of the state
is brought to bear against one who
cannot comprehend its significance.
(People v. Estrada, 333 SCRA 699,
718-719, June 19, 2000, En Banc
[Puno])
The Right to an Impartial Trial
226. What are the two principal legal
and philosophical schools of thought
on how to deal with the rain of
unrestrained publicity during the
investigation and trial of high profile
cases?
Held:
There are two (2) principal
legal and philosophical schools of
thought on how to deal with the rain of
unrestrained publicity during the
investigation and trial of high profile
cases.
The British approach the
problem with the presumption that
publicity will prejudice a jury. Thus,
English courts readily stay and stop
criminal trials when the right of an
accused to fair trial suffers a threat.
The American approach is different.
US
courts
assume
a
skeptical
approach about the potential effect of
pervasive publicity on the right of an
accused to a fair trial. They have
developed different strains of tests to
resolve this issue, i.e., substantial
probability of irreparable harm, strong
likelihood, clear and present danger,
etc. (Estrada v. Desierto, G.R. Nos.
146710-15, March 2, 2001, En
Banc [Puno])
227. Should the Ombudsman be
stopped
from
conducting
the
investigation of the cases filed against
petitioner (former President) Estrada
due to the barrage of prejudicial
publicity on his guilt?
Held: Petitioner x x x contends that
the respondent Ombudsman should be
stopped
from
conducting
the
investigation of the cases filed against
him due to the barrage of prejudicial
publicity on his guilt. He submits that
the respondent Ombudsman has
developed bias and is all set to file the

criminal cases in violation of his right


to due process.
Xxx
This is not the first time the
issue of trial by publicity has been
raised in this Court to stop the trials or
annul convictions in high profile
criminal cases.
In People v.
Teehankee, Jr. (249 SCRA 54 [1995]),
later reiterated in the case of
Larranaga v. Court of Appeals, et al.
(287 SCRA 581 at pp. 596-597 [1998]),
we laid down the doctrine that:
We cannot sustain appellants claim
that he was denied the right to
impartial trial due to prejudicial
publicity. It is true that the print and
broadcast media gave the case at bar
pervasive publicity, just like all high
profile and high stake criminal trials.
Then and now, we rule that the right
of an accused to a fair trial is not
incompatible to a free press. To be
sure, responsible reporting enhances
an accuseds right to a fair trial for, as
well pointed out, a responsible press
has always been regarded as the
handmaiden
of
effective
judicial
administration,
especially
in
the
criminal field x x x. The press does
not simply publish information about
trials
but
guards
against
the
miscarriage of justice by subjecting
the police, prosecutors, and judicial
processes to extensive public scrutiny
and criticism.
Pervasive publicity is not per se
prejudicial to the right of an accused
to fair trial. The mere fact that the
trial of appellant was given a day-today, gavel-to-gavel coverage does not
by itself prove that the publicity so
permeated the mind of the trial judge
and impaired his impartiality. For one,
it is impossible to seal the minds of
members of the bench from pre-trial
and other off-court publicity of
sensational criminal cases. The state
of the art of our communication
system brings news as they happen
straight to our breakfast tables and
right to our bedrooms. These news
form part of our everyday menu of the
facts and fictions of life. For another,
our idea of a fair and impartial judge is
not that of a hermit who is out of
touch with the world. We have not
installed the jury system whose
members are overly protected from
publicity
lest
they
lose
their
impartiality. x x x. Our judges are
learned in the law and trained to
disregard off-court evidence and on-

camera performances of parties to a


litigation.
Their mere exposure to
publications and publicity stunts does
not per se fatally infect their
impartiality.
At best, appellant can only conjure
possibility of prejudice on the part of
the trial judge due to the barrage of
publicity
that
characterized
the
investigation and trial of the case. In
Martelino, et al. v. Alejandro, et al., we
rejected this standard of possibility of
prejudice and adopted the test of
actual prejudice as we ruled that to
warrant a finding of prejudicial
publicity, there must be allegation and
proof that the judges have been
unduly influenced, not simply that
they might be, by the barrage of
publicity.
In the case at bar, the
records do not show that the trial
judge developed actual bias against
appellant as a consequence of the
extensive media coverage of the pretrial and trial of his case. The totality
of circumstances of the case does not
prove that the trial judge acquired a
fixed opinion as a result of prejudicial
publicity which is incapable of change
even by evidence presented during
the trial. Appellant has the burden to
prove this actual bias and he has not
discharged the burden.

principal actors in the case the NBI,


the respondents, their lawyers and
their sympathizers have participated
in this media blitz. The possibility of
media abuses and their threat to a fair
trial notwithstanding, criminal trials
cannot be completely closed to the
press and public. In the seminal case
of Richmond Newspapers, Inc. v.
Virginia, it was wisely held:
x x x

Xxx

(a) The historical evidence of the


evolution of the criminal trial in AngloAmerican
justice
demonstrates
conclusively that at the time this
Nations organic laws were adopted,
criminal trials both here and in
England had long been presumptively
open, thus giving assurance that the
proceedings were conducted fairly to
all
concerned
and
discouraging
perjury,
the
misconduct
of
participants, or decisions based on
secret bias or partiality. In addition,
the significant community therapeutic
value of public trials was recognized:
when a shocking crime occurs, a
community reaction of outrage and
public protest often follows, and
thereafter the open processes of
justice
serve
an
important
prophylactic purpose, providing an
outlet
for
community
concern,
hostility, and emotion.
To work
effectively, it is important that
societys criminal process satisfy the
appearance of justice, Offutt v. United
States, 348 US 11, 14, 99 L Ed 11, 75
S Ct 11, which can best be provided by
allowing people to observe such
process.
From
this
unbroken,
uncontradicted history, supported by
reasons as valid today as in centuries
past, it must be concluded that a
presumption of openness inheres in
the very nature of a criminal trial
under this Nations system of justice,
Cf., e.g., Levine v. United States, 362
US 610, 4 L Ed 2d 989, 80 S Ct 1038.

The
democratic
settings,
media
coverage of trials of sensational cases
cannot be avoided and oftentimes, its
excessiveness has been aggravated
by kinetic developments in the
telecommunications industry.
For
sure, few cases can match the high
volume and high velocity of publicity
that
attended
the
preliminary
investigation of the case at bar. Our
daily diet of facts and fiction about the
case continues unabated even today.
Commentators still bombard the public
with views not too many of which are
sober and sublime. Indeed, even the

(b) The freedoms of speech, press, and


assembly, expressly guaranteed by
the First Amendment, share a common
core purpose of assuring freedom of
communication on matters relating to
the functioning of government.
In
guaranteeing freedoms such as those
of speech and press, the First
Amendment can be read as protecting
the right of everyone to attend trials
so as give meaning to those explicit
guarantees; the First Amendment right
to receive information and ideas
means, in the context of trials, that
the guarantees of speech and press,

We expounded further on this


doctrine in the subsequent case of
Webb v. Hon. Raul de Leon, etc. (247
SCRA 652 [1995]) and its companion
cases, viz.:
Again, petitioners raise the effect of
prejudicial publicity on their right to
due
process
while
undergoing
preliminary investigation. We find no
procedural impediment to its early
invocation considering the substantial
risk to their liberty whole undergoing a
preliminary investigation.

standing alone, prohibit government


from summarily closing courtroom
doors which had long been open to the
public
at
the
time
the
First
Amendment was adopted. Moreover,
the right of assembly is also relevant,
having been regarded not only as an
independent right but also as a
catalyst to augment the free exercise
of the other First Amendment rights
with which it was deliberately linked
by the draftsmen. A trial courtroom is
a public place where the people
generally and representatives of the
media have a right to be present,
and where their presence historically
has been thought to enhance the
integrity and quality of what takes
place.
(c) Even though the Constitution
contains no provision which by its
terms guarantees to the public the
right to attend criminal trials, various
fundamental rights, not expressly
guaranteed, have been recognized as
indispensable to the enjoyment of
enumerated rights.
The right to
attend criminal trial is implicit in the
guarantees of the First Amendment:
without the freedom to attend such
trials, which people have exercised for
centuries,
important
aspects
of
freedom of speech and of the press
could be eviscerated.
Be that as it may, we recognize that
pervasive and prejudicial publicity
under certain circumstances can
deprive an accused of his due process
right to fair trial. Thus, in Martelino, et
al. v. Alejandro, et al., we held that to
warrant a finding of prejudicial
publicity there must be allegation and
proof that the judges have been
unduly influenced, not simply that
they might be, by the barrage of
publicity. In the case at bar, we find
nothing in the records that will prove
that the tone and content of the
publicity
that
attended
the
investigation of petitioners fatally
infected the fairness and impartiality
of the DOJ Panel. Petitioners cannot
just rely on the subliminal effects of
publicity on the sense of fairness of
the DOJ Panel, for these are basically
unbeknown and beyond knowing. To
be sure, the DOJ Panel is composed of
an Assistant Chief State Prosecutor
and Senior State Prosecutors. Their
long
experience
in
criminal
investigation is a factor to consider in
determining whether they can easily
be blinded by the klieg lights of
publicity.
Indeed, their 26-page
Resolution carries no indubitable

indicia of bias for it does not appear


that they considered any extra-record
evidence except evidence properly
adduced by the parties. The length of
time the investigation was conducted
despite it summary nature and the
generosity
with
which
they
accommodated the discovery motions
of petitioners speak well of their
fairness. At no instance, we note, did
petitioners seek the disqualification of
any member of the DOJ Panel on the
ground of bias resulting from their
bombardment of prejudicial publicity.
Applying the above ruling, we hold
that there is not enough evidence to
warrant this Court to enjoin the
preliminary
investigation
of
the
petitioner
by
the
respondent
Ombudsman. Petitioner needs to offer
more than hostile headlines to
discharge his burden of proof.
He
needs to show more than weighty
social science evidence to successfully
prove the impaired capacity of a judge
to render a bias-free decision. Well to
note, the cases against the petitioner
are
still
undergoing
preliminary
investigation by a special panel of
prosecutors in the office of the
respondent
Ombudsman.
No
allegation whatsoever has been made
by the petitioner that the minds of the
members of this special panel have
already been infected by bias because
of the pervasive prejudicial publicity
against him. Indeed, the special panel
has yet to come out with its findings
and the Court cannot second guess
whether its recommendation will be
unfavorable
to
the
petitioner.
(Estrada v. Desierto, G.R. Nos.
146710-15, March 2, 2001, En
Banc [Puno])
The Right against SelfIncrimination
228. Discuss the types of immunity
statutes. Which has broader scope of
protection?
Held: Our immunity statutes are of
American origin. In the United States,
there are two types of statutory
immunity granted to a witness. They
are the transactional immunity and
the use-and-derivative-use immunity.
Transactional immunity is broader in
the scope of its protection. By its
grant, a witness can no longer be
prosecuted for any offense whatsoever
arising out of the act or transaction.
In contrast, by the grant of use-andderivative-use immunity, a witness is

only assured that his or her particular


testimony and evidence derived from
it will not be used against him or her
in a subsequent prosecution. (Mapa,
Jr. v. Sandiganbayan, 231 SCRA
783, 797-798, April 26, 1994, En
Banc [Puno])
229. Is the grant of immunity to an
accused willing to testify for the
government a special privilege and
therefore must be strictly construed
against the accused?
Held: [W]e reject respondent courts
ruling that the grant of section 5
immunity must be strictly construed
against
the
petitioners.
It
simplistically characterized the grant
as a special privilege, as if it was
gifted by the government, ex gratia.
In taking this posture, it misread the
raison d etre and the long pedigree of
the right against self-incrimination vis-vis immunity statutes.
The days of inquisition brought about
the most despicable abuses against
human rights. Not the least of these
abuses is the expert use of coerced
confessions to send to the guillotine
even the guiltless. To guard against
the recurrence of this totalitarian
method, the right against selfincrimination was ensconced in the
fundamental laws of all civilized
countries. Over the years, however,
came the need to assist government
in its task of containing crime for
peace and order is a necessary matrix
of public welfare. To accommodate
the need, the right against selfincrimination was stripped of its
absoluteness. Immunity statutes in
varying shapes were enacted which
would allow government to compel a
witness to testify despite his plea of
the right against self-incrimination. To
insulate these statutes from the virus
of unconstitutionality, a witness is
given what has come to be known as
transactional or a use-derivative-use
immunity x x x. Quite clearly, these
immunity statutes are not a bonanza
from government. Those given the
privilege of immunity paid a high price
for it the surrender of their precious
right to be silent. Our hierarchy of
values demands that the right against
self-incrimination and the right to be
silent should be accorded greater
respect and protection.
Laws that
tend to erode the force of these
preeminent rights must necessarily be
given a liberal interpretation in favor
of the individual. The government has
a right to solve crimes but it must do

it,
rightly.
(Mapa,
Jr.
v.
Sandiganbayan, 231 SCRA 783,
805-806, April 26, 1994, En Banc
[Puno])
230. May the right against selfincrimination be validly invoked during
inquiry in aid of legislation?
Held: Now to another matter.
It has been held that a congressional
committees right to inquire is subject
to all relevant limitations placed by
the Constitution on governmental
action,
including
the
relevant
limitations of the Bill of Rights.
(Maurice A. Hutcheson v. U.S., 369 US
599)
In another case
x x x the mere semblance of
legislative purpose would not justify
an inquiry in the face of the Bill of
Rights. The critical element is the
existence of, and the weight to be
ascribed to, the interest of the
Congress in demanding disclosures
from an unwilling witness. We cannot
simply assume, however, that every
congressional investigation is justified
by a public need that over-balances
any private rights affected. To do so
would be to abdicate the responsibility
placed by the Constitution upon the
judiciary to insure that the Congress
does not unjustifiably encroach upon
an individuals right to privacy nor
abridge his liberty of speech, press,
religion or assembly. (Watkins v. US,
354 USS 178 citing US v. Rumely, 345
US 41)
One
of
the
basic
rights
guaranteed by the Constitution to an
individual is the right against selfincrimination (Sec. 17, Art. III of the
Constitution). This right construed as
the right to remain completely silent
may be availed of by the accused in a
criminal case; but it may be invoked
by other witnesses only as questions
are asked of them.
This distinction is enunciated by
the Court in Romeo Chavez v. The
Honorable Court of Appeals, et al.
(G.R. No. L-29169, August 19, 1968,
24 SCRA 663) thus
Petitioner, as accused, occupies a
different tier of protection from an
ordinary witness.
Whereas an
ordinary witness may be compelled to
take the witness stand and claim the
privilege as each question requiring an

incriminating answer is shot at him, an


accused may altogether refuse to take
the witness stand and refuse to
answer any and all questions.
Moreover, this right of the
accused is extended to respondents in
administrative investigations but only
if they partake of the nature of a
criminal proceeding or analogous to a
criminal proceeding.
In Galman v.
Pamaran (G.R. Nos. 71208-09, August
30, 1985, 138 SCRA 294), the Court
reiterated the doctrine in Cabal v.
Kapunan (6 SCRA 1059) to illustrate
the right of witnesses to invoke the
right against self-incrimination not
only in criminal proceedings but also
in all other types of suit.
It was held that:
We did not therein state that since he
is not an accused and the case is not a
criminal case, Cabal cannot refuse to
take the witness stand and testify, and
that he can invoke his right against
self-incrimination
only
when
a
question which tends to elicit an
answer that will incriminate him is
propounded to him. Clearly then, it is
not the character of the suit involved
but the nature of the proceedings that
controls.
The
privilege
has
consistently been held to extend to all
proceedings sanctioned by law and to
all cases in which punishment is
sought to be visited upon a witness,
whether a party or not.
We do not here modify these
doctrines. If we presently rule that
petitioners may not be compelled by
the respondent Committee to appear,
testify and produce evidence before it,
it is only because we hold that the
questioned inquiry is not in aid of
legislation and, if pursued, would be
violative of the principle of separation
of powers between the legislative and
the
judicial
departments
of
government,
ordained
by
the
Constitution.
(Bengzon, Jr. v.
Senate Blue Ribbon Committee,
203 SCRA 767, Nov. 20, 1991, En
Banc [Padilla])
The Right against Double Jeopardy
231. Discuss the two kinds of double
jeopardy.
Held: Our Bill of Rights deals with
two (2) kinds of double jeopardy. The
first sentence of Clause 20, Section 1,
Article III of the Constitution ordains
that no person shall be twice put in

jeopardy of punishment for the same


offense. The second sentence of said
clause provides that if an act is
punishable by a law and an ordinance,
conviction or acquittal under either
shall constitute a bar to another
prosecution for the same act. Thus,
the first sentence prohibits double
jeopardy of punishment for the same
offense
whereas,
the
second
contemplates double jeopardy of
punishment for the same act. Under
the first sentence, one may be twice
put in jeopardy of punishment of the
same act, provided that he is charged
with different offenses, or the offense
charged in one case is not included in,
or does not include, the crime charged
in the other case.
The second
sentence applies, even if the offense
charged are not the same, owing to
the fact that one constitutes a
violation of an ordinance and the other
a violation of statute.
If the two
charges are based on one and the
same act, conviction or acquittal
under either the law or the ordinance
shall bar a prosecution under the
other. Incidentally, such conviction or
acquittal is not indispensable to
sustain the plea of double jeopardy of
punishment or the same offense. So
long as jeopardy has been attached
under one of the informations
charging said offense, the defense
may be availed of in the other case
involving the same offense, even if
there has been neither conviction nor
acquittal in either case.
Elsewhere stated, where the offense
charged are penalized either by
different sections of the same statute
or by different statutes, the important
inquiry relates to the identity of
offenses charged. The constitutional
protection against double jeopardy is
available only where an identity is
shown to exist between the earlier and
the subsequent offenses charged. The
question of identity or lack of identity
of offenses is addressed by examining
the essential elements of each of the
two offenses charged, as such
elements are set out in the respective
legislative definitions of the offenses
involved. (People v. Quijada, 259
SCRA 191, July 24, 1996)
232. What must be proved to
substantiate a claim of double
jeopardy? When may legal jeopardy
attach?
Held:
To substantiate a claim of
double jeopardy, the following must be
proven:

(1) A first jeopardy must have


attached prior to the second; (2) the
first jeopardy must have been validly
terminated; (3) the second jeopardy
must be for the same offense, or the
second
offense
includes
or
is
necessarily included in the offense
charged in the first information, or is
an attempt to commit the same or is a
frustration thereof.
Legal jeopardy attaches only: (1) upon
a valid indictment; (b) before a
competent
court;
(c)
after
arraignment; (d) when a valid plea has
been entered; and (e) the case was
dismissed or otherwise terminated
without the express consent of the
accused. (Cuison v. CA, 289 SCRA
159, April 15, 1998 [Panganiban])
233. In its decision in a criminal case,
the Judge promulgated only the civil
aspect of the case, but not the
criminal. Will the promulgation of the
criminal aspect later constitute double
jeopardy?
Held: Petitioner contends that "the
promulgation by Judge Ramos on April
4, 1995 of the Respondent Court's
decision of June 30, 1991 by reading
its dispositive portion has effectively
terminated the criminal cases against
the petitioner x x x." In other words,
petitioner claims that the first
jeopardy attached at that point.
The Court is not persuaded. As a rule,
a criminal prosecution includes a civil
action for the recovery of indemnity.
Hence, a decision in such case
disposes of both the criminal as well
as the civil liabilities of an accused.
Here, trial court promulgated only the
civil aspect of the case, but not the
criminal.
[T]he promulgation of the CA Decision
was not complete. In fact and in truth,
the promulgation was not merely
incomplete; it was also void. In excess
of its jurisdiction, the trial judge
rendered a substantially incomplete
promulgation on April 4, 1995, and he
repeated his mistake in his April 12,
1996 Order. We emphasize that grave
abuse of discretion rendered the
aforementioned act of the trial court
void. Since the criminal cases have
not yet been terminated, the first
jeopardy has not yet attached. Hence,
double jeopardy cannot prosper as a
defense.

We must stress that Respondent


Court's questioned Decision did not
modify or amend its July 30, 1991
Decision.
It merely ordered the
promulgation of the judgment of
conviction and the full execution of the
penalty it had earlier imposed on
petitioner. (Cuison v. CA, 289 SCRA
159, April 15, 1998 [Panganiban])
234. What are the exceptions to the
rule that the dismissal of a criminal
case resulting in acquittal made with
the express consent of the accused or
upon his own motion will not place the
accused in double jeopardy?
Held: In the cases at bar, the
order of dismissal based on a violation
of the right to speedy trial was made
upon motion by counsel for petitioner
before the trial court. It was made at
the instance of the accused before the
trial court, and with his express
consent. Generally, the dismissal of a
criminal case resulting in acquittal
made with the express consent of the
accused or upon his own motion will
not place the accused in double
jeopardy. However, this rule admits of
two exceptions, namely: insufficiency
of evidence and denial of the right to
speedy trial (People v. Bans, 239 SCRA
48, 55 [1994]). Double jeopardy may
attach when the proceedings have
been prolonged unreasonably, in
violation of the accuseds right to
speedy trial (Commission on Elections
v. Court of Appeals, 229 SCRA 501,
507 [1994]). (Almario v. Court of
Appeals, 355 SCRA 1, Mar. 22,
2001, 2nd Div. [Quisumbing]
235. If the criminal case was
dismissed predicated on the right of
the accused to speedy trial, but later
the trial court reconsidered its decision
and allowed the case to be reinstated
as it noted that the delay in the trial
was due to circumstances beyond the
control of the parties and of the trial
court, i.e., the presiding judge was
promoted to the Court of Appeals, and
his successor as trial judge was not
immediately appointed, nor another
judge detailed to his sala, is there
violation of the accuseds right against
double jeopardy?
Held: Here we must inquire
whether there was unreasonable delay

in the conduct of the trial so that


violation of the right to speedy trial of
the accused x x x resulted. For it must
be recalled that in the application of
the constitutional guaranty of the right
to speedy disposition of cases,
particular regard must also be taken of
the facts and circumstances peculiar
to
each
case
(Socrates
v.
Sandiganbayan, 253 SCRA 773, 788
[1996]). Both the trial court and the
appellate court noted that after pretrial
of
petitioners
case
was
terminated x x x continuous trial was
set x x x. The scheduled hearings,
however, were cancelled when the
presiding judge was promoted to the
Court of Appeals, and his successor as
trial judge was not immediately
appointed, nor another judge detailed
to his sala.
Xxx
As observed by respondent
appellate court, delay in the trial was
due to circumstances beyond the
control of the parties and of the trial
court. X x x. Thus, after a closer
analysis of these successive events,
the trial court realized that the dates
of the hearings were transferred for
valid grounds. Hence, the trial court
set aside its initial order and
reinstated
the
cases
against
petitioner, which order the appellate
court later sustained.
That there was no unreasonable
delay of the proceedings is apparent
from the chronology of the hearings
with
the
reasons
for
their
postponements or transfers. X x x
There being no oppressive delay
in
the
proceedings,
and
no
postponements unjustifiably sought,
we concur with the conclusion reached
by the Court of Appeals that
petitioners right to speedy trial had
not been infringed. Where the right of
the accused to speedy trial had not
been violated, there was no reason to
support the initial order of dismissal.
It follows that petitioner cannot
invoke the constitutional right against
double jeopardy when that order was
reconsidered seasonably (People v.
Leviste, 255 SCRA 238, 249 [1996]).
For as petitioners right to speedy trial
was not transgressed, this exception
to the fifth element of double jeopardy
that the defendant was acquitted or
convicted, or the case was dismissed
or otherwise terminated without the
express consent of the accused was

not met. The trial courts initial order


of dismissal was upon motion of
petitioners counsel, hence made with
the express consent of petitioner.
That being the case, despite the
reconsideration of said order, double
jeopardy did not attach. As this Court
had occasion to rule in People v.
Tampal (244 SCRA 202), reiterated in
People v. Leviste (Ibid.), where we
overturned an order of dismissal by
the trial court predicated on the right
to speedy trial
It is true that in an unbroken line of
cases, we have held that the dismissal
of cases on the ground of failure to
prosecute is equivalent to an acquittal
that would bar further prosecution of
the accused for the same offense. It
must be stressed, however, that these
dismissals were predicated on the
clear right of the accused to speedy
trial. These cases are not applicable
to the petition at bench considering
that the right of the private
respondents to speedy trial has not
been violated by the State. For this
reason, private respondents cannot
invoke their right against double
jeopardy.
Both the trial court and the
Court of Appeals were thus not in error
when they allowed reinstatement of
the cases against petitioner. (Almario
v. Court of Appeals, 355 SCRA 1,
Mar. 22, 2001, 2nd Div.
[Quisumbing]
236. Is there double jeopardy when
an accused was acquitted in a criminal
case for reckless imprudence but the
civil aspect of the case was elevated
to the Court of Appeals and the latter
found him liable for indemnity and
damages?
Held: Petitioner opines that the Court
of Appeals should not have disturbed
the findings of the trial court on the
lack of negligence or reckless
imprudence under the guise of
determining his civil liability. He
argues that the trial courts finding
that he was neither imprudent nor
negligent was the basis for his
acquittal, and not reasonable doubt.
He submits that in finding him liable
for indemnity and damages, the
appellate court not only placed his

acquittal in suspicion, but also put him


in double jeopardy.
Private respondents contend that
while the trial court found that
petitioners guilt had not been proven
beyond reasonable doubt, it did not
state in clear and unequivocal terms
that petitioner was not recklessly
imprudent or negligent. Hence,
impliedly the trial court acquitted him
on reasonable doubt. Since civil
liability is not extinguished in criminal
cases, if the acquittal is based on
reasonable doubt, the Court of
Appeals had to review the findings of
the trial court to determine if there
was a basis for awarding indemnity
and damages.
Preliminarily, petitioners claim that
the decision of the appellate court
awarding indemnity placed him in
double jeopardy is misplaced. X x x.
When a person is charged with an
offense and the case is terminated
either by acquittal or conviction or in
any manner without the consent of the
accused, the latter cannot again be
charged with the same or identical
offense (Melo v. People, 85 Phil. 766,
768 [1950]). This is double jeopardy.
For double jeopardy to exist, the
following elements must be
established: (1) a first jeopardy must
have attached prior to the second; (2)
the first jeopardy must have
terminated; and (3) the second
jeopardy must be for the same offense
as the first (People v. Bocar, 138 SCRA
166, 171 [1985]). In the instant case,
petitioner had once been placed in
jeopardy by the filing of Criminal Case
No. 066 and the jeopardy was
terminated by his discharge. The
judgment of acquittal became
immediately final. Note, however,
that what was elevated to the Court of
Appeals by private respondents was
the civil aspect of Criminal Case No.
066. Petitioner was not charged anew
in CA-G.R. CV No. 19240 with a second
criminal offense identical to the first
offense. The records clearly show that
no second criminal offense was being
imputed to petitioner on appeal. In

modifying the lower courts judgment,


the appellate court did not modify the
judgment of acquittal. Nor did it order
the filing of a second criminal cases
against petitioner for the same
offense. Obviously, therefore, there
was no second jeopardy to speak of.
Petitioners claim of having been
placed in double jeopardy is incorrect.
Our law recognizes two kinds of
acquittal, with different effects on the
civil liability of the accused. First is an
acquittal on the ground that the
accused is not the author of the act or
omission complained of. This instance
closes the door to civil liability, for a
person who has been found to be not
the perpetrator of any act or omission
cannot and can never be held liable
for such or omission (Almeida, et al. v.
Abaroa, 8 Phil. 178, 181 [1907]; other
citations omitted.) There being no
delict, civil liability ex delicto is out of
the question, and the civil action, if
any, which may be instituted must be
based on grounds other than the delict
complained of. This is the situation
contemplated in Rule 111 of the Rules
of Court. The second instance is an
acquittal based on reasonable doubt
on the guilt of the accused. In this
case, even if the guilt of the accused
has not been satisfactorily
established, he is not exempt from
civil liability which may be proved by
preponderance of evidence only
(Manahan, Jr. v. Court of Appeals, 255
SCRA 202, 214 [1996], citing Padilla v.
Court of Appeals, 129 SCRA 558
[1984]). This is the situation
contemplated in Article 29 of the Civil
Code, where the civil action for
damages is for the same act or
omission. Although the two actions
have different purposes, the matters
discussed in the civil case are similar
to those discussed in the criminal
case. However, the judgment in the
criminal proceeding cannot be read in
evidence in the civil action to establish
any fact there determined, even
though both actions involve the same
act or omission (Almeida Chantangco
and Lete v. Abaroa, supra note 13, at
1061). The reason for this rule is that

the parties are not the same and


secondarily, different rules of evidence
are applicable. Hence,
notwithstanding herein petitioners
acquittal, the Court of Appeals in
determining whether Article 29
applied, was not precluded from
looking into the question of
petitioners negligence or reckless
imprudence. (Manantan v. Court of
Appeals, 350 SCRA 387, Jan. 29,
2001, 2nd Div. [Quisumbing])

The Right against Ex Post Facto


Laws and Bills of Attainder
237. What is a bill of attainder?
P.D. 1866 a bill of attainder?

Is

Held: [T]he Court, in People v. Ferrer


(G.R. Nos. L-32613-14, December 27,
1972, 48 SCRA 382), defined a bill of
attainder as a legislative act which
inflicts punishment on individuals or
members of a particular group without
a judicial trial. Essential to a bill of
attainder are a specification of certain
individuals or a group of individuals,
the imposition of a punishment, penal
or otherwise, and the lack of judicial
trial. This last element, the total lack
of court intervention in the finding of
guilt and the determination of the
actual penalty to be imposed, is the
most essential. P.D. No. 1866 does not
possess the elements of a bill of
attainder. It does not seek to inflict
punishment without a judicial trial.
Nowhere in the measure is there a
finding of guilt and an imposition of a
corresponding punishment. What the
decree does is to define the offense
and provide for the penalty that may
be imposed, specifying the qualifying
circumstances that would aggravate
the
offense.
There
is
no
encroachment on the power of the
court to determine after due hearing
whether the prosecution has proved
beyond reasonable doubt that the
offense of illegal possession of
firearms has been committed and that
the qualifying circumstances attached
to it has been established also beyond
reasonable doubt as the Constitution
and
judicial
precedents
require.
(Misolas v. Panga, 181 SCRA 648,
659-660, Jan. 30, 1990, En Banc
[Cortes])
238. What is an ex post facto law? Is
R.A. No. 8249 an ex post facto law?

Held: Ex post facto law, generally,


prohibits retrospectivity of penal laws.
R.A. 8249 is not a penal law. It is a
substantive law on jurisdiction which is
not penal in character. Penal laws are
those acts of the Legislature which
prohibit certain acts and establish
penalties for their violations; or those
that define crimes, treat of their
nature,
and
provide
for
their
punishment.
R.A. 7975, which
amended P.D. 1606 as regards the
Sandiganbayans jurisdiction, its mode
of appeal and other procedural
matters, has been declared by the
Court as not a penal law, but clearly a
procedural statute, i.e., one which
prescribes rules of procedure by which
courts applying laws of all kinds can
properly administer justice. Not being
a penal law, the retroactive application
of R.A. 8249 cannot be challenged as
unconstitutional.
Petitioners
and
intervenors
contention that their right to a twotiered appeal which they acquired
under R.A. 7975 has been diluted by
the enactment of R.A. 8249, is
incorrect. The same contention has
already been rejected by the court
several times considering that the
right to appeal is not a natural right
but statutory in nature that can be
regulated by law.
The mode of
procedure provided for in the statutory
right of appeal is not included in the
prohibition against ex post facto laws.
R.A. 8249 pertains only to matters of
procedure, and being merely an
amendatory statute it does not
partake the nature of an ex post facto
law. It does not mete out a penalty
and, therefore, does not come within
the prohibition. Moreover, the law did
not alter the rules of evidence or the
mode of trial. It has been ruled that
adjective statutes may be made
applicable to actions pending and
unresolved at the time of their
passage.
At any rate, R.A. 8249 has preserved
the accuseds right to appeal to the
Supreme Court to review questions of
law.
On the removal of the
intermediate review of facts, the
Supreme Court still has the power of
review
to
determine
if
the
presumption of innocence has been
convincingly overcome. (Panfilo M.
Lacson v. The Executive Secretary,
et. al., G.R. No. 128096, Jan. 20,
1999 [Martinez])

ADMINISTRATIVE LAW
239. Describe
Code of 1987

the

Administrative

Held: The Code is a general law and


incorporates in a unified document
the major structural, functional and
procedural principles of governance
(Third Whereas Clause, Administrative
Code of 1987) and embodies changes
in
administrative
structures
and
procedures designed to serve the
people. (Fourth Whereas Clause,
Administrative Code of 1987)
The
Code is divided into seven (7) books.
These books contain provisions on the
organization, powers and general
administration
of
departments,
bureaus and offices under the
executive branch, the organization
and functions of the Constitutional
Commissions and other constitutional
bodies, the rules on the national
government budget, as well as
guidelines
for
the
exercise
by
administrative agencies of quasilegislative and quasi-judicial powers.
The Code covers both the internal
administration,
i.e.,
internal
organization,
personnel
and
recruitment,
supervision
and
discipline, and the effects of the
functions performed by administrative
officials on private individuals or
parties outside government. (Ople v.
Torres, G.R. No. 127685, July 23,
1998 [Puno])
240.

What is administrative power?

Held:
Administrative power is
concerned with the work of applying
policies and enforcing orders as
determined by proper governmental
organs. It enables the President to fix
a uniform standard of administrative
efficiency and check the official
conduct of his agents. To this end, he
can issue administrative orders, rules
and regulations.
(Ople v. Torres,
G.R. No. 127685, July 23, 1998
[Puno])
241. What
order?

is

an

administrative

242. What is the Government of the


Republic of the Philippines?
Answer:
The Government of the
Republic of the Philippines refers to
the corporate governmental entity
through which the functions of the
government are exercised throughout
the Philippines, including, save as the
contrary appears from the context, the
various arms through which political
authority is made effective in the
Philippines, whether pertaining to the
autonomous regions, the provincial,
city,
municipal
or
barangay
subdivisions or other forms of local
government.
(Sec.
2[1],
Introductory Provisions, Executive
Order No. 292)
243. What
is
a
government
instrumentality? What are included in
the term government instrumentality?
Answer:
A
government
instrumentality refers to any agency of
the
national
government,
not
integrated within the department
framework,
vested
with
special
functions or jurisdiction by law,
endowed with some if not all corporate
powers, administering special funds,
enjoying
operational
autonomy,
usually through a charter. The term
includes
regulatory
agencies,
chartered
institutions
and
government-owned
or
controlled
corporations.
(Sec.
2[10],
Introductory Provisions, Executive
Order No. 292)
244.

Answer: A regulatory agency refers


to any agency expressly vested with
jurisdiction to regulate, administer or
adjudicate
matters
affecting
substantial rights and interest of
private persons, the principal powers
of which are exercised by a collective
body, such as a commission, board or
council. (Sec. 2[11], Introductory
Provisions, Executive Order No.
292)
245.

Held: An administrative order is an


ordinance issued by the President
which relates to specific aspects in the
administrative
operation
of
government. It must be in harmony
with the law and should be for the sole
purpose of implementing the law and
carrying out the legislative policy.
(Ople v. Torres, G.R. No. 127685,
July 23, 1998 [Puno])

What is a regulatory agency?

What is a chartered institution?

Answer:
A chartered institution
refers to any agency organized or
operating under a special charter, and
vested by law with functions relating
to specific constitutional policies or
objectives. This term includes state
universities and colleges and the
monetary authority of the State.
(Section
2[12],
Introductory

Provisions, Executive Order No.


292)
246. When is a government-owned
or controlled corporation deemed to
be performing proprietary function?
When is it deemed to be performing
governmental function?
Held:
Government-owned
or
controlled corporations may perform
governmental or proprietary functions
or both, depending on the purpose for
which they have been created. If the
purpose is to obtain special corporate
benefits or earn pecuniary profit, the
function is proprietary. If it is in the
interest of health, safety and for the
advancement of public good and
welfare, affecting the public in
general, the function is governmental.
Powers classified as proprietary are
those intended for private advantage
and benefit. (Blaquera v. Alcala,
295 SCRA 366, 425, Sept. 11,
1998, En Banc [Purisima])
247. Does the petition for annulment
of proclamation of a candidate merely
involve the exercise by the COMELEC
of its administrative power to review,
revise and reverse the actions of the
board of canvassers and, therefore,
justifies non-observance of procedural
due process, or does it involve the
exercise of the COMELEC's quasijudicial function?
Held: Taking cognizance of private
respondent's petitions for annulment
of petitioner's proclamation, COMELEC
was not merely performing an
administrative
function.
The
administrative powers of the COMELEC
include the power to determine the
number and location of polling places,
appoint
election
officials
and
inspectors, conduct registration of
voters, deputize law enforcement
agencies
and
governmental
instrumentalities to ensure free,
orderly, honest, peaceful and credible
elections, register political parties,
organizations or coalition, accredit
citizen's arms of the Commission,
prosecute election offenses, and
recommend to the President the
removal of or imposition of any other
disciplinary action upon any officer or
employee it has deputized for violation
or disregard of its directive, order or
decision. In addition, the Commission
also has direct control and supervision
over all personnel involved in the
conduct of election.
However, the
resolution of the adverse claims of
private respondent and petitioner as

regards the existence of a manifest


error in the questioned certificate of
canvass requires the COMELEC to act
as an arbiter.
It behooves the
Commission to hear both parties to
determine the veracity of their
allegations and to decide whether the
alleged error is a manifest error.
Hence, the resolution of this issue calls
for the exercise by the COMELEC of its
quasi-judicial power. It has been said
that where a power rests in judgment
or discretion, so that it is of judicial
nature or character, but does not
involve the exercise of functions of a
judge, or is conferred upon an officer
other than a judicial officer, it is
deemed quasi-judicial. The COMELEC
therefore, acting as quasi-judicial
tribunal,
cannot
ignore
the
requirements
of
procedural
due
process in resolving the petitions filed
by private respondent. (Federico S.
Sandoval v. COMELEC, G.R. No.
133842, Jan. 26, 2000 [Puno])
248. Discuss the Doctrine of Primary
Jurisdiction (or Prior Resort).
Held:
Courts cannot and will not
resolve a controversy involving a
question
which
is
within
the
jurisdiction
of
an
administrative
tribunal, especially where the question
demands the exercise of sound
administrative discretion requiring the
special knowledge, experience and
services of the administrative tribunal
to determine technical and intricate
matters of fact.
In recent years, it has been the
jurisprudential trend to apply this
doctrine to cases involving matters
that demand the special competence
of administrative agencies even if the
question involved is also judicial in
character. It applies where a claim is
originally cognizable in the courts, and
comes
into
play
whenever
enforcement of the claim requires the
resolution of issues which, under a
regulatory scheme, have been placed
within the special competence of an
administrative body; in such case, the
judicial process is suspended pending
referral of such issues to the
administrative body for its view.
In cases where the doctrine of primary
jurisdiction is clearly applicable, the
court cannot arrogate unto itself the
authority to resolve a controversy, the
jurisdiction over which is lodged with
an administrative body of special
competence. (Villaflor v. CA, 280
SCRA 287)

9)
249. Discuss
the
Doctrine
of
Exhaustion
of
Administrative
Remedies.
Enumerate exceptions
thereto.
Held: 1. Before a party is allowed to
seek the intervention of the court, it is
a pre-condition that he should have
availed
of
all
the
means
of
administrative processes afforded him.
Hence, if a remedy within the
administrative machinery can still be
resorted
to
by
giving
the
administrative officer concerned every
opportunity to decide on a matter that
comes within his jurisdiction then such
remedy should be exhausted first
before the courts judicial power can
be sought. The premature invocation
of courts jurisdiction is fatal to ones
cause of action. Accordingly, absent
any finding of waiver or estoppel the
case is susceptible of dismissal for
lack of cause of action. This doctrine
of
exhaustion
of
administrative
remedies was not without its practical
and legal reasons, for one thing,
availment of administrative remedy
entails lesser expenses and provides
for
a
speedier
disposition
of
controversies. It is no less true to
state that the courts of justice for
reasons of comity and convenience
will shy away from a dispute until the
system of administrative redress has
been completed and complied with so
as to give the administrative agency
concerned
every
opportunity
to
correct its error and to dispose of the
case.

when the subject matter is a


private land in land case proceeding;
10)
when the rule does not
provide a plain, speedy and adequate
remedy, and
11)
when
there
are
circumstances indicating the urgency
of judicial intervention.
(Paat v. CA, 266 SCRA 167 [1997])
2. Non-exhaustion of administrative
remedies is not jurisdictional. It only
renders the action premature, i.e.,
claimed cause of action is not ripe for
judicial determination and for that
reason a party has no cause of action
to ventilate in court.
(Carale v.
Abarintos, 269 SCRA 132)
250. When may the Government not
validly
invoke
the
rule
that
prescription does not run against the
State?
Held: While it is true that prescription
does not run against the State, the
same may not be invoked by the
government in this case since it is no
longer interested in the subject
matter.
While Camp Wallace may
have belonged to the government at
the time Rafael Galvezs title was
ordered cancelled in Land Registration
Case No. N-361, the same no longer
holds true today.
Republic Act No. 7227, otherwise
known as the Base Conversion and
Development Act of 1992, created the
Bases Conversion and Development
Authority. X x x

This doctrine is disregarded:


Xxx
1)
2)
3)
4)
5)
6)

7)
8)

when there is a violation of


due process;
when the issue involved is
purely a legal question;
when
the
administrative
action is patently illegal amounting to
lack or excess of jurisdiction;
when there is estoppel on
the part of the administrative agency
concerned;
when there is irreparable
injury;
when the respondent is a
department secretary whose acts as
an alter ego of the President bears the
implied and assumed approval of the
latter;
when to require exhaustion
of administrative remedies would be
unreasonable;
when it would amount to a
nullification of a claim;

With the transfer of Camp Wallace to


the BCDA, the government no longer
has a right or interest to protect.
Consequently, the Republic is not a
real party in interest and it may not
institute the instant action. Nor may it
raise the defense of imprescriptibility,
the same being applicable only in
cases where the government is a party
in interest. X x x. Being the owner of
the areas covered by Camp Wallace, it
is
the
Bases
Conversion
and
Development
Authority,
not
the
Government, which stands to be
benefited if the land covered by TCT
No. T-5710 issued in the name of
petitioner is cancelled.
Nonetheless, it has been posited that
the transfer of military reservations
and their extensions to the BCDA is
basically
for
the
purpose
of

accelerating the sound and balanced


conversion
of
these
military
reservations
into
alternative
productive uses and to enhance the
benefits to be derived from such
property as a measure of promoting
the economic and social development,
particularly of Central Luzon and, in
general, the countrys goal for
enhancement (Section 2, Republic Act
No. 7227). It is contended that the
transfer of these military reservations
to the Conversion Authority does not
amount to an abdication on the part of
the Republic of its interests, but simply
a recognition of the need to create a
body corporate which will act as its
agent for the realization of its
program. It is consequently asserted
that the Republic remains to be the
real party in interest and the
Conversion Authority merely its agent.
We, however, must not lose sight of
the fact that the BCDA is an entity
invested with a personality separate
and distinct from the government. X x
x
It may not be amiss to state at this
point that the functions of government
have
been
classified
into
governmental or constituent and
proprietary or ministrant. While public
benefit
and
public
welfare,
particularly, the promotion of the
economic and social development of
Central Luzon, may be attributable to
the operation of the BCDA, yet it is
certain that the functions performed
by the BCDA are basically proprietary
in nature. The promotion of economic
and social development of Central
Luzon, in particular, and the countrys
goal for enhancement, in general, do
not make the BCDA equivalent to the
Government. Other corporations have
been created by government to act as
its agents for the realization of its
programs, the SSS, GSIS, NAWASA and
the NIA, to count a few, and yet, the
Court has ruled that these entities,
although performing functions aimed
at promoting public interest and public
welfare, are not government-function
corporations
invested
with
governmental attributes. It may thus
be said that the BCDA is not a mere
agency of the Government but a
corporate body performing proprietary
functions.
Xxx
Having the capacity to sue or be sued,
it should thus be the BCDA which may
file an action to cancel petitioners

title, not the Republic, the former


being the real party in interest. One
having no right or interest to protect
cannot invoke the jurisdiction of the
court as a party plaintiff in an action
(Ralla v. Ralla, 199 SCRA 495 [1991]).
A suit may be dismissed if the plaintiff
or the defendant is not a real party in
interest. X x x.
However, E.B. Marcha Transport Co.,
Inc. v. IAC (147 SCRA 276 [1987]) is
cited as authority that the Republic is
the proper party to sue for the
recovery of possession of property
which at the time of the installation of
the suit was no longer held by the
national government body but by the
Philippine Ports Authrotiy.
In E.B.
Marcha, the Court ruled:
It can be said that in suing for the
recovery of the rentals, the Republic of
the Philippines, acted as principal of
the Philippine Ports Authority, directly
exercising the commission it had
earlier conferred on the latter as its
agent.
We may presume that, by
doing so, the Republic of the
Philippines did not intend to retain the
said rentals for its own use,
considering that by its voluntary act it
had transferred the land in question to
the Philippine Ports Authority effective
July 11, 1974. The Republic of the
Philippines had simply sought to
assist, not supplant, the Philippine
Ports Authority, whose title to the
disputed property it continues to
recognize. We may expect the that
the said rentals, once collected by the
Republic of the Philippines, shall be
turned over by it to the Philippine
Ports Authority conformably to the
purposes of P.D. No. 857.
E.B. Marcha is, however, not on all
fours with the case at bar. In the
former, the Court considered the
Republic a proper party to sue since
the claims of the Republic and the
Philippine Ports Authority against the
petitioner therein were the same. To
dismiss the complaint in E.B. Marcha
would have brought needless delay in
the settlement of the matter since the
PPA would have to refile the case on
the same claim already litigated upon.
Such is not the case here since to
allow the government to sue herein
enables it to raise the issue of
imprescriptibility, a claim which is not
available to the BCDA. The rule that
prescription does not run against the
State does not apply to corporations
or artificial bodies created by the State
for special purposes, it being said that

when the title of the Republic has


been divested, its grantees, although
artificial bodies of its own creation, are
in the same category as ordinary
persons (Kingston v. LeHigh Valley
Coal Co., 241 Pa 469). By raising the
claim of imprescriptibility, a claim
which cannot be raised by the BCDA,
the Government not only assists the
BCDA, as it did in E.B. Marcha, it even
supplants the latter, a course of action
proscribed by said case.
Moreover,
to
recognize
the
Government as a proper party to sue
in this case would set a bad precedent
as it would allow the Republic to
prosecute, on behalf of governmentowned or controlled corporations,
causes of action which have already
prescribed, on the pretext that the
Government is the real party in
interest against whom prescription
does not run, said corporations having
been created merely as agents for the
realization of government programs.
It should also be noted that petitioner
is unquestionably a buyer in good faith
and for value, having acquired the
property in 1963, or 5 years after the
issuance of the original certificate of
title, as a third transferee. If only not
to do violence and to give some
measure of respect to the Torrens
System, petitioner must be afforded
some
measure
of
protection.
(Shipside Incorporated v. Court of
Appeals, 352 SCRA 334, Feb. 20,
2001, 3rd Div. [Melo])
251. Discuss the nature and
functions of the NTC, and analyze its
powers and authority as well as the
laws, rules and regulations that govern
its existence and operations.
Held: The NTC was created
pursuant to Executive Order No. 546,
promulgated on July 23, 1979. It
assumed the functions formerly
assigned to the Board of
Communications and the
Communications Control Bureau,
which were both abolished under the
said Executive Order. Previously, the
NTCs function were merely those of
the defunct Public Service Commission
(PSC), created under Commonwealth
Act No. 146, as amended, otherwise
known as the Public Service Act,
considering that the Board of
Communications was the successor-in-

interest of the PSC. Under Executive


Order No. 125-A, issued in April 1987,
the NTC became an attached agency
of the Department of Transportation
and Communications.
In the regulatory
communications industry, the NTC has
the sole authority to issue Certificates
of Public Convenience and Necessity
(CPCN) for the installation, operation,
and maintenance of communications
facilities and services, radio
communications systems, telephone
and telegraph systems. Such power
includes the authority to determine
the areas of operations of applicants
for telecommunications services.
Specifically, Section 16 of the Public
Service Act authorizes the then PSC,
upon notice and hearing, to issue
Certificates of Public Convenience for
the operation of public services within
the Philippines whenever the
Commission finds that the operation of
the public service proposed and the
authorization to do business will
promote the public interests in a
proper and suitable manner.
(Commonwealth Act No. 146, Section
16[a]) The procedure governing the
issuance of such authorizations is set
forth in Section 29 of the said Act x x
x. (Republic v. Express
Telecommunication Co., Inc., 373
SCRA 316, Jan. 15, 2002, 1st Div.
[Ynares-Santiago])
252. Is the filing of the administrative
rules and regulations with the UP Law
Center the operative act that gives the
rules force and effect?
Held: In granting Bayantel the
provisional authority to operate a
CMTS, the NTC applied Rule 15,
Section 3 of its 1978 Rules of Practice
and Procedure, which provides:
Sec. 3. Provisional Relief. Upon the
filing of an application, complaint or
petition or at any stage thereafter, the
Board may grant on motion of the
pleader or on its own initiative, the
relief prayed for, based on the
pleading, together with the affidavits

and supporting documents attached


thereto, without prejudice to a final
decision after completion of the
hearing which shall be called within
thirty (30) days from grant of authority
asked for. (italics ours)
Respondent Extelcom, however,
contends that the NTC should have
applied the Revised Rules which were
filed with the Office of the National
Administrative Register on February 3,
1993. These Revised Rules deleted
the phrase on its own initiative;
accordingly, a provisional authority
may be issued only upon filing of the
proper motion before the Commission.
In answer to this argument, the
NTC, through the Secretary of the
Commission, issued a certification to
the effect that inasmuch as the 1993
Revised Rules have not been
published in a newspaper of general
circulation, the NTC has been applying
the 1978 Rules.
The absence of publication,
coupled with the certification by the
Commissioner of the NTC stating that
the NTC was still governed by the
1987 Rules, clearly indicate that the
1993 Revised Rules have not taken
effect at the time of the grant of the
provisional authority to Bayantel. The
fact that the 1993 Revised Rules were
filed with the UP Law Center on
February 3, 1993 is of no moment.
There is nothing in the Administrative
Code of 1987 which implies that the
filing of the rules with the UP Law
Center is the operative act that gives
the rules force and effect. Book VII,
Chapter 2, Section 3 thereof merely
states:
Filing. (1) Every agency shall file with
the University of the Philippines Law
Center three (3) certified copies of
every rule adopted by it. Rules in
force on the date of effectivity of this
Code which are not filed within three
(3) months from the date shall not
thereafter be the basis of any sanction
against any party or persons.

(2) The records officer of the agency,


or his equivalent functionary, shall
carry out the requirements of this
section under pain of disciplinary
action.
(3) A permanent register of all rules
shall be kept by the issuing agency
and shall be open to public inspection.
The National Administrative
Register is merely a bulletin of codified
rules and it is furnished only to the
Office of the President, Congress, all
appellate courts, the National Library,
other public offices or agencies as the
Congress may select, and to other
persons at a price sufficient to cover
publication and mailing or distribution
costs (Administrative Code of 1987,
Book VII, Chapter 2, Section 7). In a
similar case, we held:
This does not imply, however, that the
subject Administrative Order is a valid
exercise of such quasi-legislative
power. The original Administrative
Order issued on August 30, 1989,
under which the respondents filed
their applications for importations,
was not published in the Official
Gazette or in a newspaper of general
circulation. The questioned
Administrative Order, legally, until it is
published, is invalid within the context
of Article 2 of Civil Code, which reads:
Article 2. Laws shall take effect after
fifteen days following the completion
of their publication in the Official
Gazette (or in a newspaper of general
circulation in the Philippines), unless it
is otherwise provided. X x x
The fact that the amendments to
Administrative Order No. SOCPEC 8908-01 were filed with, and published
by the UP Law Center in the National
Administrative Register, does not cure
the defect related to the effectivity of
the Administrative Order.
This Court, in Tanada v. Tuvera (G.R.
No. L-63915, December 29, 1986, 146
SCRA 446) stated, thus:

We hold therefore that all statutes,


including those of local application and
private laws, shall be published as a
condition for their effectivity, which
shall begin fifteen days after
publication unless a different
effectivity is fixed by the legislature.
Covered by this rule are presidential
decrees and executive orders
promulgated by the President in the
exercise of legislative power or, at
present, directly conferred by the
Constitution. Administrative Rules and
Regulations must also be published if
their purpose is to enforce or
implement existing law pursuant also
to a valid delegation.
Interpretative regulations and those
merely internal in nature, that is,
regulating only the personnel of the
administrative agency and not the
public, need not be published. Neither
is publication required of the so-called
letters of instructions issued by
administrative superiors concerning
the rules or guidelines to be followed
by their subordinates in the
performance of their duties.
Xxx
We agree that the publication must be
in full or it is no publication at all since
its purpose is to inform the public of
the contents of the laws.
The Administrative Order under
consideration is one of those
issuances which should be published
for its effectivity, since its purpose is
to enforce and implement an existing
law pursuant to a valid delegation, i.e.,
P.D. 1071, in relation to LOI 444 and
EO 133 (Philippine International
Trading Corp. v. Angeles, 263 SCRA
421, 446-447 [1996]).
Thus, publication in the Official
Gazette or a newspaper of general
circulation is a condition sine qua non
before statutes, rules or regulations
can take effect. This is explicit from
Executive Order No. 200, which

repealed Article 2 of the Civil Code,


and which states that:
Laws shall take effect after fifteen
days following the completion of their
publication either in the Official
Gazette or in a newspaper of general
circulation in the Philippines, unless it
is otherwise provided (E.O. 200,
Section 1).
The Rules of Practice and
Procedure of the NTC, which
implements Section 29 of the Public
Service Act (C.A. 146, as amended),
fall squarely within the scope of these
laws, as explicitly mentioned in the
case of Tanada v. Tuvera (146 SCRA
446 [1986]).
Our pronouncement in Tanada v.
Tuvera is clear and categorical.
Administrative rules and regulations
must be published if their purpose is
to enforce or implement existing law
pursuant to a valid delegation. The
only exception are interpretative
regulations, those merely internal in
nature, or those so-called letters of
instructions issued by administrative
superiors concerning the rules and
guidelines to be followed by their
subordinates in the performance of
their duties (PHILSA International
Placement & Services Corp. v.
Secretary of Labor, G.R. No. 103144,
April 4, 2001, 356 SCRA 174).
Hence, the 1993 Revised Rules
should be published in the Official
Gazette or in a newspaper of general
circulation before it can take effect.
Even the 1993 Revised Rules itself
mandates that said Rules shall take
effect only after their publication in a
newspaper of general circulation
(Section 20 thereof). In the absence
of such publication, therefore, it is the
1978 Rules that governs. (Republic
v. Express Telecommunication Co.,
Inc., 373 SCRA 316, Jan. 15, 2002,
1st Div. [Ynares-Santiago])
253. May a person be held liable for
violation of an administrative
regulation which was not published?

Held: Petitioner insists,


however, that it cannot be held liable
for illegal exaction as POEA
Memorandum Circular No. II, Series of
1983, which enumerated the allowable
fees which may be collected from
applicants, is void for lack of
publication.
There is merit in the argument.
In Tanada v. Tuvera (136 SCRA
27 [1985]), the Court held, as follows:
We hold therefore that all statutes,
including those of local application and
private laws, shall be published as a
condition for their effectivity, which
shall begin fifteen days after
publication unless a different
effectivity date is fixed by the
legislature.
Covered by this rule are presidential
decrees and executive orders
promulgated by the President in the
exercise of legislative powers
whenever the same are validly
delegated by the legislature or, at
present, directly conferred by the
Constitution. Administrative rules and
regulations must also be published if
their purpose is to enforce or
implement existing law pursuant to a
valid delegation.
Interpretative regulations and those
merely internal in nature, that is,
regulating only the personnel of the
administrative agency and the public,
need not be published. Neither is
publication required of the so-called
letter of instructions issued by the
administrative superiors concerning
the rules or guidelines to be followed
by their subordinates in the
performance of their duties.
Applying this doctrine, we have
previously declared as having no force
and effect the following administrative
issuances: a) Rules and Regulations
issued by the Joint Ministry of HealthMinistry of Labor and Employment
Accreditation Committee regarding the

accreditation of hospitals, medical


clinics and laboratories (Joint Ministry
of Health-Ministry of Labor and
Employment Accreditation Committee
v. Court of Appeals, 196 SCRA 263
[1991]); b) Letter of Instruction No.
416 ordering the suspension of
payments due and payable by
distressed copper mining companies
to the national government (Caltex
Philippines, Inc. v. Court of Appeals,
208 SCRA 726 [1992]); c)
Memorandum Circulars issued by the
POEA regulating the recruitment of
domestic helpers to Hong Kong (Phil.
Association of Service Exporters v.
Torres, 212 SCRA 298 [1992]); d)
Administrative Order No. SOCPEC 8908-01 issued by the Philippine
International Trading Corporation
regulating applications for importation
from the Peoples Republic of China
(Philippine International Trading
Corporation v. Angeles, 263 SCRA 421
[1996]); and e) Corporate
Compensation Circular No. 10 issued
by the Department of Budget and
Management discontinuing the
payment of other allowances and
fringe benefits to government officials
and employees (De Jesus v.
Commission on Audit, 294 SCRA 152
[1998). In all these cited cases, the
administrative issuances questioned
therein were uniformly struck down as
they were not published or filed with
the National Administrative Register as
required by the Administrative Code of
1987 (Administrative Code of 1987,
Book VII, chapter 2, Section 3).
POEA memorandum Circular No.
2, Series of 1983 must likewise be
declared ineffective as the same was
never published or filed with the
National Administrative Register.
POEA Memorandum Circular No.
2, Series of 1983 provides for the
applicable schedule of placement and
documentation fees for private
employment agencies or authority
holders. Under the said Order, the
maximum amount which may be
collected from prospective Filipino
overseas workers is P2,500.00. The

said circular was apparently issued in


compliance with the provisions of
Article 32 of the Labor Code x x x.

for lack of proper publication, the said


circulars may not be enforced or
implemented.

It is thus clear that the


administrative circular under
consideration is one of those
issuances which should be published
for its effectivity, since its purpose is
to enforce and implement an existing
law pursuant to a valid delegation
(Philippine International Trading
Corporation v. Angeles, supra.).
Considering that POEA Administrative
Circular No. 2, Series of 1983 has not
as yet been published or filed with the
National Administrative Register, the
same is ineffective and may not be
enforced. (Philsa International
Placement and Services
Corporation v. Secretary of Labor
and Employment, 356 SCRA 174,
April 4, 2001, 3rd Div., [GonzagaReyes])

Our pronouncement in Tanada v.


Tuvera is clear and categorical.
Administrative rules and regulations
must be published if their purpose is
to enforce or implement existing law
pursuant to a valid delegation. The
only exceptions are interpretative
regulations, those merely internal in
nature, or those so-called letters of
instructions issued by administrative
superiors concerning the rules and
guidelines to be followed by their
subordinates in the performance of
their duties. Administrative Circular
No. 2, Series of 1983 has not been
shown to fall under any of these
exceptions.

254. Does the publication


requirement apply as well to
administrative regulations addressed
only to a specific group and not to the
general public?
Held: The Office of the Solicitor
General likewise argues that the
questioned administrative circular is
not among those requiring publication
contemplated by Tanada v. Tuvera as it
is addressed only to a specific group of
persons and not to the general public.
Again, there is no merit in this
argument.
The fact that the said circular is
addressed only to a specified group,
namely private employment agencies
or authority holders, does not take it
away from the ambit of our ruling in
Tanada v. Tuvera. In the case of Phil.
Association of Service Exporters v.
Torres ((212 SCRA 298 [1992]), the
administrative circulars questioned
therein were addressed to an even
smaller group, namely Philippine and
Hong Kong agencies engaged in the
recruitment of workers for Hong Kong,
and still the Court ruled therein that,

In this regard, the Solicitor Generals


reliance on the case of Yaokasin v.
Commissioner of Customs (180 SCRA
599 [1989]) is misplaced. In the said
case, the validity of certain Customs
Memorandum Orders were upheld
despite their lack of publication as
they were addressed to a particular
class of persons, the customs
collectors, who were also the
subordinates of the Commissioner of
the Bureau of Customs. As such, the
said Memorandum Orders clearly fall
under one of the exceptions to the
publication requirement, namely those
dealing with instructions from an
administrative superior to a
subordinate regarding the
performance of their duties, a
circumstance which does not obtain in
the case at bench.
Xxx
To summarize, petitioner should be
absolved from the three (3) counts of
exaction as POEA Administrative
Circular No. 2, Series of 1983 could not
be the basis of administrative
sanctions against petitioner for lack of
publication. (Philsa International
Placement and Services
Corporation v. Secretary of Labor
and Employment, 356 SCRA 174,

April 4, 2001, 3rd Div., [GonzagaReyes])


255. May a successful bidder compel
a government agency to formalize a
contract with it notwithstanding that
its bid exceeds the amount
appropriated by Congress for the
project?
Held: Enshrined in the 1987
Philippine Constitution is the mandate
that no money shall be paid out of
the Treasury except in pursuance of an
appropriation made by law. (Sec.
29[1], Article VI of the 1987
Constitution) Thus, in the execution of
government contracts, the precise
import of this constitutional restriction
is to require the various agencies to
limit their expenditures within the
appropriations made by law for each
fiscal year.
Xxx
It is quite evident from the tenor
of the language of the law that the
existence of appropriations and the
availability of funds are indispensable
pre-requisites to or conditions sine qua
non for the execution of government
contracts. The obvious intent is to
impose such conditions as a priori
requisites to the validity of the
proposed contract (Fernandez, A
Treatise on Government Contracts
Under Philippine Law, 2001, pp. 4041). Using this as our premise, we
cannot accede to PHOTOKINAs
contention that there is already a
perfected contract. While we held in
Metropolitan Manila Development
Authority v. Jancom Environmental
Corporation (Supra) that the effect of
an unqualified acceptance of the offer
or proposal of the bidder is to perfect
a contract, upon notice of the award to
the bidder, however, such statement
would be inconsequential in a
government where the acceptance
referred to is yet to meet certain
conditions. To hold otherwise is to
allow a public officer to execute a
binding contract that would obligate
the government in an amount in

excess of the appropriations for the


purpose for which the contract was
attempted to be made (64 Am Jur 2d
Sec. 11). This is a dangerous
precedent.
In the case at bar, there seems
to be an oversight of the legal
requirements as early as the bidding
stage. The first step of a Bids and
Awards Committee (BAC) is to
determine whether the bids comply
with the requirements. The BAC shall
rate a bid passed only if it complies
with all the requirements and the
submitted price does not exceed the
approved budget for the contract.
(Implementing Rules and Regulations
[IRR] for Executive Order No. 262,
supra.)
Extant on the record is the fact
that the VRIS Project was awarded to
PHOTOKINA on account of its bid in
the amount of P6.588 Billion Pesos.
However, under Republic Act No. 8760
(General Appropriations Act, FY 2000,
p. 1018, supra.), the only fund
appropriated for the project was P1
Billion Pesos and under the
Certification of Available Funds (CAF)
only P1.2 Billion Pesos was available.
Clearly, the amount appropriated is
insufficient to cover the cost of the
entire VRIS Project. There is no way
that the COMELEC could enter into a
contract with PHOTOKINA whose
accepted bid was way beyond the
amount appropriated by law for the
project. This being the case, the BAC
should have rejected the bid for being
excessive or should have withdrawn
the Notice of Award on the ground that
in the eyes of the law, the same is null
and void.
Xxx
Even the draft contract
submitted by Commissioner Sadain,
that provides for a contract price in
the amount of P1.2 Billion Pesos is
unacceptable. X x x While the
contract price under the draft contract
is only P1.2 Billion and, thus, within
the certified available funds, the same

covers only Phase I of the VRIS Project,


i.e., the issuance of identification
cards for only 1,000,000 voters in
specified areas (Ibid., p. 382). In
effect, the implementation of the VRIS
Project will be segmented or
chopped into several phases. Not
only is such arrangement disallowed
by our budgetary laws and practices, it
is also disadvantageous to the
COMELEC because of the uncertainty
that will loom over its modernization
project for an indefinite period of time.
Should Congress fail to appropriate
the amount necessary for the
completion of the entire project, what
good will the accomplished Phase I
serve? As expected, the project failed
to sell with the Department of
Budget and Management. Thus,
Secretary Benjamin Diokno, per his
letter of December 1, 2000, declined
the COMELECs request for the
issuance of the Notice of Cash
Availability (NCA) and a multi-year
obligatory authority to assume
payment of the total VRIS Project for
lack of legal basis. Corollarily, under
Section 33 of R.A. No. 8760, no agency
shall enter into a multi-year contract
without a multi-year obligational
authority, thus:
SECTION 33. Contracting Multi-Year
Projects. - In the implementation of
multi-year projects, no agency shall
enter into a multi-year contract
without a multi-year Obligational
Authority issued by the Department of
Budget and Management for the
purpose. Notwithstanding the
issuance of the multi-year Obligational
Authority, the obligation to be incurred
in any given calendar year, shall in no
case exceed the amount programmed
for implementation during said
calendar year.
Petitioners are justified in refusing to
formalize the contract with
PHOTOKINA. Prudence dictated them
not to enter into a contract not backed
up by sufficient appropriation and
available funds. Definitely, to act
otherwise would be a futile exercise

for the contract would inevitably suffer


the vice of nullity. X x x
Xxx
Verily, the contract, as expressly
declared by law, is inexistent and void
ab initio (Article 1409 of the Civil Code
of the Philippines). This is to say that
the proposed contract is without force
and effect from the very beginning or
from its incipiency, as if it had never
been entered into, and hence, cannot
be validated either by lapse of time or
ratification (Manila Lodge v. Court of
Appeals, 73 SCRA 162 [1976]; See also
Tongoy v. Court of Appeals, 123 SCRA
99 [1983]).
Xxx
In fine, we rule that PHOTOKINA,
though the winning bidder, cannot
compel the COMELEC to formalize the
contract. Since PHOTOKINAs bid is
beyond the amount appropriated by
Congress for the VRIS Project, the
proposed contract is not binding upon
the COMELEC and is considered void x
x x. (Commission on Elections v.
Judge Ma. Luisa Quijano-Padilla,
G.R. No. 151992, Sept. 18, 2002,
En Banc [Sandoval-Gutierrez])
256. What is the remedy available to
a party who contracts with the
government contrary to the
requirements of the law and,
therefore, void ab initio?
Held: Of course, we are not saying
that the party who contracts with the
government has no other recourse in
law. The law itself affords him the
remedy. Section 48 of E.O. No. 292
explicitly provides that any contract
entered into contrary to the abovementioned requirements shall be void,
and the officers entering into the
contract shall be liable to the
Government or other contracting party
for any consequent damage to the
same as if the transaction had been
wholly between private parties. So
when the contracting officer
transcends his lawful and legitimate

powers by acting in excess of or


beyond the limits of his contracting
authority, the Government is not
bound under the contract. It would be
as if the contract in such case were a
private one, whereupon, he binds
himself, and thus, assumes personal
liability thereunder. (Fernandez, a
Treatise on Government Contracts
Under Philippine Law, 2001, supra.,
pp. 22-23). Otherwise stated, the
proposed contract is unenforceable as
to the Government.
While this is not the proceeding
to determine where the culpability lies,
however, the constitutional mandate
cited above constrains us to remind all
public officers that public office is a
public trust and all public officers must
at all times be accountable to the
people. The authority of public
officers to enter into government
contracts is circumscribed with a
heavy burden of responsibility. In the
exercise of their contracting
prerogative, they should be the first
judges of the legality, propriety and
wisdom of the contract they entered
into. They must exercise a high
degree of caution so that the
Government may not be the victim of
ill-advised or improvident action
(Rivera v. Maclang, 7 SCRA 57 [1963]).
(Commission on Elections v. Judge
Ma. Luisa Quijano-Padilla, G.R. No.
151992, Sept. 18, 2002, En Banc
[Sandoval-Gutierrez])

adjudicatory power itself.


explained:

The Court

x x x [T]he Commission on Human


Rights x x x was not meant by the
fundamental law to be another court
or quasi-judicial agency in this
country, or duplicate much less take
over the functions of the latter.
The most that may be conceded to
the Commission in the way of
adjudicative power is that it may
investigate, i.e., receive evidence and
make findings of fact as regards
claimed
human
rights
violations
involving civil and political rights. But
fact finding is not adjudication, and
cannot be likened to the judicial
function of a court of justice, or even a
quasi-judicial agency or official. The
function of receiving evidence and
ascertaining therefrom the facts of a
controversy is not a judicial function,
properly speaking. To be considered
such,
the
faculty
of
receiving
evidence
and
making
factual
conclusions in a controversy must be
accompanied by the authority of
applying the law to those factual
conclusions to the end that the
controversy may be decided or
determined authoritatively, finally and
definitively, subject to such appeals or
modes of review as may be provided
by law. This function, to repeat, the
Commission does not have. (Simon,
Jr. v. Commission on Human
Rights, 229 SCRA 117, 125, Jan. 5,
1994, En Banc [Vitug, J.])
258. Does
the
Commission
on
Human Rights have jurisdiction to
issue TRO or writ of preliminary
injunction?

257. Does
the
Commission
on
Human Rights have the power to
adjudicate?

Held:
In Export Processing
Zone Authority v. Commission on
Human rights (208 SCRA 125, 131),
the Court x x x explained:

Held: In its Order x x x denying


petitioners motion to dismiss, the CHR
theorizes that the intention of the
members
of
the
Constitutional
Commission is to make CHR a quasijudicial body. This view, however, has
not heretofore been shared by this
Court. In Carino v. Commission on
Human Rights (204 SCRA 483, 492),
the Court x x x has observed that it is
only the first of the enumerated
powers and functions that bears any
resemblance
to
adjudication
of
adjudgment, but that resemblance
can in no way be synonymous to the

The constitutional provision directing


the CHR to provide for preventive
measures and legal aid services to the
underprivileged whose human rights
have been violated or need protection
may not be construed to confer
jurisdiction on the Commission to
issue a restraining order or writ of
injunction for, if that were the
intention, the Constitution would have
expressly said so.
Jurisdiction is
conferred only by the Constitution or
by law.
It is never derived by
implication.

Evidently, the preventive measures


and legal aid services mentioned in
the Constitution refer to extrajudicial
and judicial remedies (including a writ
of preliminary injunction) which the
CHR may seek from the proper courts
on behalf of the victims of human
rights violations. Not being a court of
justice, the CHR itself has no
jurisdiction to issue the writ, for a writ
of preliminary injunction may only be
issued by the judge of any court in
which the action is pending [within his
district], or by a Justice of the Court of
Appeals, or of the Supreme Court. x x
x. A writ of preliminary injunction is
an ancillary remedy. It is available
only in a pending principal action, for
the preservation or protection of the
rights and interest of a party thereto,
and for no other purpose.
The Commission does have legal
standing to indorse, for appropriate
action,
its
findings
and
recommendations to any appropriate
agency of government (See Export
Processing
Zone
Authority
v.
Commission on Human Rights, 208
SCRA
125).
(Simon,
Jr.
v.
Commission on Human Rights, 229
SCRA 117, 134-135, Jan. 5, 1994,
En Banc [Vitug, J.])
259. Discuss the contempt power of
the Commission on Human Rights
(CHR).
When may it be validly
exercised?
Held: On its contempt powers,
the CHR is constitutionally authorized
to adopt its operational guidelines
and rules of procedure, and cite for
contempt for violations thereof in
accordance with the Rules of Court.
Accordingly, the CHR acted within its
authority in providing in its revised
rules, its power to cite or hold any
person in direct or indirect contempt,
and to impose the appropriate
penalties in accordance with the
procedure and sanctions provided for
in the Rules of Court. That power to
cite for contempt, however, should be
understood to apply only to violations
of its adopted operational guidelines
and rules of procedure essential to
carry out its investigatorial powers. To
exemplify, the power to cite for
contempt could be exercised against
persons who refuse to cooperate with
the said body, or who unduly withhold
relevant information, or who decline to
honor summons, and the like, in
pursuing its investigative work. The
order to desist (a semantic interplay
for a restraining order) in the instance

before
us,
however,
is
not
investigatorial
in
character
but
prescinds from an adjudicative power
that it does not possess.
x x x
(Simon, Jr. v. Commission on
Human Rights, 229 SCRA 117,
134, Jan. 5, 1994, En Banc [Vitug,
J.])

THE LAW OF PUBLIC OFFICERS


260. How are positions in the Civil
Service classified? Discuss the
characteristics of each.
Ans.: Positions in the Civil
Service may be classified into: 1)
Career Positions, and 2) Non-Career
Positions.
Career Positions are characterized by
(1) entrance based on merit and
fitness to be determined as far as
practicable by competitive
examination, or based on highly
technical qualifications; (2)
opportunity for advancement to higher
career positions; and (3) security of
tenure (Sec. 7, Chap. 2, Subtitle A,
Title I, Bk. V, E.O. No. 292).
The Non-Career Service shall be
characterized by (1) entrance on
bases other than of the usual tests of
merit or fitness utilized for the career
service; and (2) tenure which is limited
to a period specified by law, or which
is coterminous with that of the
appointing authority or subject to his
pleasure, or which is limited to the
duration of a particular project for
which purpose employment was made
(Sec. 9, Chap. 2, Subtitle A, Title I,
Bk. V, E.O. No. 292).

261. Define Appointment. Discuss its


nature.
Held: An appointment to a public
office is the unequivocal act of
designating or selecting by one having
the authority therefor of an individual
to discharge and perform the duties
and functions of an office or trust. The
appointment is deemed complete
once the last act required of the

appointing
authority
has
been
complied with and its acceptance
thereafter by the appointee in order to
render it effective.
Appointment
necessarily calls for an exercise of
discretion on the part of the
appointing authority. In Pamantasan
ng Lungsod ng Maynila v. Intermediate
Appellate Court (140 SCRA 22),
reiterated in Flores v. Drilon (223 SCRA
568), this Court has held:
The power to appoint is, in essence,
discretionary. The appointing power
has the right of choice which he may
exercise freely according to his
judgment, deciding for himself who is
best qualified among those who have
the necessary qualifications and
eligibilities. It is a prerogative of the
appointing power x x x. (At p. 579)
Indeed, it may rightly be said that the
right of choice is the heart of the
power to appoint. In the exercise of
the power of appointment, discretion
is an integral thereof. (Bermudez v.
Torres, 311 SCRA 733, Aug. 4,
1999, 3rd Div. [Vitug])
262. May
the
Civil
Service
Commission, or the Supreme Court,
validly nullify an appointment on the
ground that somebody else is better
qualified?
Held: The head of an agency who is
the appointing power is the one most
knowledgeable to decide who can best
perform the functions of the office.
Appointment
is
an
essentially
discretionary power and must be
performed by the officer vested with
such power according to his best
lights, the only condition being that
the appointee should possess the
qualifications required by law. If he
does, then the appointment cannot be
faulted on the ground that there are
others better qualified who should
have been preferred. Indeed, this is a
prerogative of the appointing authority
which he alone can decide.
The
choice of an appointee from among
those who possess the required
qualifications is a political and
administrative decision calling for
considerations
of
wisdom,
convenience, utility and the interests
of the service which can best be made
by the head of the office concerned,
the person most familiar with the
organizational
structure
and
environmental circumstances within
which the appointee must function.

As long as the appointee is qualified


the Civil Service Commission has no
choice but to attest to and respect the
appointment even if it be proved that
there are others with superior
credentials.
The law limits the
Commissions
authority
only
to
whether or not the appointees possess
the legal qualifications and the
appropriate civil service eligibility,
nothing else.
If they do then the
appointments are approved because
the Commission cannot exceed its
power by substituting its will for that
of the appointing authority. Neither
can we.
(Rimonte v. CSC, 244
SCRA 504-505, May 29, 1995, En
Banc [Bellosillo, J.])
263. Does the next-in-rank rule
import any mandatory or peremptory
requirement that the person next-inrank must be appointed to the
vacancy?
Held: The next-in-rank rule is not
absolute; it only applies in cases of
promotion, a process which denotes a
scalar ascent of an officer to another
position higher either in rank or salary.
And even in promotions, it can be
disregarded for sound reasons made
known to the next-in-rank, as the
concept
does
not
import
any
mandatory or peremptory requirement
that the person next-in-rank must be
appointed to the vacancy.
The
appointing authority, under the Civil
Service Law, is allowed to fill
vacancies by promotion, transfer of
present employees, reinstatement,
reemployment, and appointment of
outsiders who have appropriate civil
service eligibility, not necessarily in
that order. There is no legal fiat that a
vacancy must be filled only by
promotion; the appointing authority is
given wide discretion to fill a vacancy
from among the several alternatives
provided by law.
What the Civil Service Law provides is
that if a vacancy is filled by promotion,
the person holding the position next in
rank thereto shall be considered for
promotion.
In Taduran v. Civil Service Commission
(131 SCRA 66 [1984]), the Court
construed that phrase to mean that
the person next-in-rank would be
among the first to be considered for
the vacancy, if qualified. In Santiago,
Jr. v. Civil Service Commission (178
SCRA
733
[1989]),
the
Court
elaborated the import of the rule in
the following manner:

One who is next-in-rank is entitled to


preferential
consideration
for
promotion to the higher vacancy but it
does not necessarily follow that he
and no one else can be appointed.
The rule neither grants a vested right
to the holder nor imposes a ministerial
duty on the appointing authority to
promote such person to the next
higher position x x x
(Abila v. CSC, 198 SCRA 102, June
3, 1991, En Banc [Feliciano])
263. Can a person who lacks the
necessary qualifications for a public
position be appointed to it in a
permanent capacity? Illustrative case.
Held: At the outset, it must be
stressed that the position of Ministry
Legal Counsel-CESO IV is embraced in
the Career Executive Service. X x x
In the case at bar, there is no
question that private respondent does
not have the required CES eligibility.
As admitted by private respondent in
his Comment, he is not a CESO or a
member of the Career Executive
Service.
In the case of Achacoso v.
Macaraig, et al. (195 SCRA 235, 239240 [1991]), the Court held:
It is settled that a permanent
appointment can be issued only to a
person who meets all the
requirements for the position to which
he s being appointed, including the
appropriate eligibility prescribed.
Achacoso did not. At best, therefore,
his appointment could be regarded
only as temporary. And being so, it
could be withdrawn at will by the
appointing authority and at a
moments notice, conformably to
established jurisprudence.
The Court, having considered these
submissions and the additional
arguments of the parties in the
petitioners Reply and of the SolicitorGenerals Rejoinder, must find for the
respondents.

The mere fact that a position belongs


to the Career Service does not
automatically confer security of tenure
in its occupant even if he does not
possess the required qualifications.
Such right will have to depend on the
nature of his appointment, which in
turn depends on his eligibility or lack
of it. A person who does not have the
requisite qualifications for the position
cannot be appointed to it in the first
place or, only as an exception to the
rule, may be appointed to it merely in
an acting capacity in the absence of
appropriate eligibles. The
appointment extended to him cannot
be regarded as permanent even if it
may be so designated.
Evidently, private respondents
appointment did not attain
permanency. Not having taken the
necessary Career Executive Service
examination to obtain the requisite
eligibility, he did not at the time of his
appointment and up to the present,
possess the needed eligibility for a
position in the Career Executive
Service. Consequently, his
appointment as Ministry Legal
Counsel-CESO IV/Department Legal
Counsel and/or Director III, was merely
temporary. Such being the case, he
could be transferred or reassigned
without violating the constitutionally
guaranteed right to security of tenure.
Private respondent capitalizes
on his lack of CES eligibility by
adamantly contending that the
mobility and flexibility concepts in the
assignment of personnels under the
Career Executive Service do not apply
to him because he s not a Career
Executive Service Officer. Obviously,
the contention is without merit. As
correctly pointed out by the Solicitor
General, non-eligibles holding
permanent appointments to CES
positions were never meant to remain
immobile in their status. Otherwise,
their lack of eligibility would be a
premium vesting them with
permanency in the CES positions, a
privilege even their eligible
counterparts do not enjoy.

Then too, the cases on


unconsented transfer invoked by
private respondent find no application
in the present case. To reiterate,
private respondents appointment is
merely temporary; hence, he could be
transferred or reassigned to other
positions without violating his right to
security of tenure. (De Leon v. Court
of Appeals, 350 SCRA 1, Jan. 22,
2001, En Banc [Ynares-Santiago])
264. In the career executive service,
is a career executive service (CES)
eligibility all that an employee needs
to acquire security of tenure? Is
appointment to a CES rank necessary
for the acquisition of such security of
tenure?
Held: The petitions are
impressed with merit.
In the career executive service,
the acquisition of security of tenure
which presupposes a permanent
appointment is governed by the rules
and regulations promulgated by the
CES Board x x x.
As clearly set forth in the foregoing
provisions, two requisites must concur
in order that an employee in the
career executive service may attain
security of tenure, to wit:
a
b

CES eligibility; and


Appointment to the appropriate CES
rank.
In addition, it must be stressed that
the security of tenure of employees in
the career executive service (except
first and second level employees in
the civil service), pertains only to rank
and not to the office or to the position
to which they may be appointed.
Thus, a career executive service
officer may be transferred or
reassigned from one position to
another without losing his rank which
follows him wherever he is transferred
or reassigned. In fact, a CESO suffers
no diminution of salary even if
assigned to a CES position with lower

salary grade, as he is compensated


according to his CES rank and not on
the basis of the position or office he
occupies.
In the case at bar, there is no question
that respondent Ramon S. Roco,
though a CES eligible, does not
possess the appropriate CES rank,
which is CES rank level V, for the
position of Regional Director of the LTO
(Region V). Falling short of one of the
qualifications that would complete his
membership in the CES, respondent
cannot successfully interpose violation
of security of tenure. Accordingly, he
could be validly reassigned to other
positions in the career executive
service. X x x
Moreover, under the mobility and
flexibility principles of the Integrated
Reorganization Plan, CES personnel
may be reassigned or transferred from
one position to another x x x.
One last point. Respondent capitalizes
on the fact that petitioner Luis Mario
M. General is not a CES eligible. The
absence, however, of such CES
eligibility is of no moment. As stated
in Part III, Chapter I, Article IV,
paragraph 5(c), of the Integrated
Reorganization Plan
x x x the President may, in
exceptional cases, appoint any person
who is not a Career Executive Service
eligible; provided that such appointee
shall subsequently take the required
Career Executive Service examination
and that he shall not be promoted to a
higher class until he qualified in such
examination.
Evidently, the law allows appointment
of those who are not CES eligible,
subject to the obtention of said
eligibility, in the same manner that the
appointment of respondent who does
not possess the required CES rank
(CES rank level V) for the position of
Regional Director of the LTO, is
permitted in a temporary capacity.
(General v. Roco, 350 SCRA 528,
Jan. 29, 2001, 1st Div. [YnaresSantiago])

265. May an elective public official


be validly appointed or designated to
any public office or position during his
tenure?
Ans.: No elective official shall
be eligible for appointment or
designation in any capacity to any
public office or position during his
tenure. (Sec. 7, 1st par., Art. IX-B,
1987 Constitution)
266. May an appointive public official
hold any other office or employment?
Ans.: Unless otherwise allowed
by law or by the primary functions of
his position, no appointive official shall
hold any other office or employment in
the Government or any subdivision,
agency or instrumentality thereof,
including
government-owned
or
controlled corporation. (Sec. 7, 2nd
par., Art. IX-B, 1987 Constitution)
267. May
the
President,
VicePresident, Members of the Cabinet,
their deputies or assistants hold any
other office or employment?
Ans.:
The President, VicePresident, the Members of the
Cabinet, and their deputies or
assistants shall not, unless otherwise
provided in this Constitution, hold any
other office or employment during
their tenure.
(Sec. 13, Art. VII,
1987 Constitution)
268. Does the prohibition in Section
13, Article VII of the 1987 Constitution
insofar as Cabinet members, their
deputies or assistants are concerned
admit of the broad exceptions made
for appointive officials in general
under Section 7, par. (2), Article IX-B?
Held: The threshold question
therefore is: does the prohibition in
Section 13, Article VII of the 1987
Constitution
insofar
as
Cabinet
members, their deputies or assistants
are concerned admit of the broad
exceptions
made
for
appointive
officials in general under Section 7,
par. (2), Article IX-B which, for easy
reference is quoted anew, thus:
Unless otherwise allowed by law or
by the primary functions of his
position, no appointive official shall
hold any other office or employment in
the government or any subdivision,
agency or instrumentality thereof,
including
government-owned
or
controlled
corporation
or
their
subsidiaries.

We rule in the negative.


Xxx
The practice of designating
members of the Cabinet, their
deputies and assistants as members
of the governing bodies or boards of
various government agencies and
instrumentalities,
including
government-owned and controlled
corporations, became prevalent during
the time legislative powers in this
country were exercised by former
President
Ferdinand
E.
Marcos
pursuant to his martial law authority.
There was a proliferation of newlycreated agencies, instrumentalities
and government-owned and controlled
corporations created by presidential
decrees
and
other
modes
of
presidential issuances where Cabinet
members, their deputies or assistants
were designated to head or sit as
members of the board with the
corresponding salaries, emoluments,
per diems, allowances and other
perquisites of office. X x x
This practice of holding multiple
offices or positions in the government
soon led to abuses by unscrupulous
public officials who took advantage of
this scheme for purposes of selfenrichment. X x x
Particularly odious and revolting
to the peoples sense of propriety and
morality in government service were
the data contained therein that
Roberto v. Ongpin was a member of
the governing boards of twenty-nine
(29)
governmental
agencies,
instrumentalities and corporations;
Imelda R. Marcos of twenty-three (23);
Cesar E.A. Virata of twenty-two (22);
Arturo R. Tanco, Jr. of fifteen (15); Jesus
S. Hipolito and Geronimo Z. Velasco, of
fourteen each (14); Cesar C. Zalamea
of thirteen (13); Ruben B. Ancheta and
Jose A. Rono of twelve (12) each;
Manuel P. Alba, Gilberto O. Teodoro,
and Edgardo Tordesillas of eleven (11)
each; and Lilia Bautista and Teodoro Q.
Pena of ten (10) each.
The blatant betrayal of public
trust evolved into one of the serious
causes of discontent with the Marcos
regime.
It was therefore quite
inevitable and in consonance with the
overwhelming sentiment of the people
that
the
1986
Constitutional
Commission, convened as it was after
the people successfully unseated
former President Marcos, should draft

into its proposed Constitution the


provisions under consideration which
are envisioned to remedy, if not
correct, the evils that flow from the
holding of multiple governmental
offices and employment. X x x
But what is indeed significant is
the fact that although Section 7,
Article IX-B already contains a blanket
prohibition against the holding of
multiple offices or employment in the
government subsuming both elective
and appointive public officials, the
Constitutional Commission should see
it fit to formulate another provision,
Sec. 13, Article VII, specifically
prohibiting
the
President,
VicePresident, members of the Cabinet,
their deputies and assistants from
holding
any
other
office
or
employment during their tenure,
unless otherwise provided in the
Constitution itself.
Evidently, from this move as
well as in the different phraseologies
of the constitutional provisions in
question, the intent of the framers of
the Constitution was to impose a
stricter prohibition on the President
and his official family in so far as
holding other offices or employment in
the government or elsewhere is
concerned.
Moreover,
such
intent
is
underscored by a comparison of
Section 13, Article VII with other
provisions of the Constitution on the
disqualifications of certain public
officials or employees from holding
other offices or employment. Under
Section 13, Article VI, [N]o Senator or
Member
of
the
House
of
Representatives may hold any other
office
or
employment
in
the
Government x x x. Under section
5(4), Article XVI, [N]o member of the
armed forces in the active service
shall, at any time, be appointed in any
capacity to a civilian position in the
Government, including governmentowned or controlled corporations or
any of their subsidiaries.
Even
Section 7(2), Article IX-B, relied upon
by respondents provides [U]nless
otherwise allowed by law or by the
primary functions of his position, no
appointive official shall hold any other
office
or
employment
in
the
Government.
It is quite notable that in all
these provisions on disqualifications to
hold other office or employment, the
prohibition pertains to an office or

employment in the government and


government-owned
or
controlled
corporations or their subsidiaries. In
striking contrast is the wording of
Section 13, Article VII which states
that [T]he President, Vice-President,
the Members of the Cabinet, and their
deputies or assistants shall not, unless
otherwise
provided
in
this
Constitution, hold any other office or
employment during their tenure. In
the
latter
provision,
the
disqualification is absolute, not being
qualified by the phrase in the
Government.
The
prohibition
imposed on the President and his
official
family
is
therefore
allembracing and covers both public and
private office or employment.
Going further into Section 13,
Article VII, the second sentence
provides: They shall not, during said
tenure, directly or indirectly, practice
any other profession, participate in
any business, or be financially
interested in any contract with, or in
any franchise, or special privilege
granted by the Government or any
subdivision, agency or instrumentality
thereof, including government-owned
or controlled corporations or their
subsidiaries.
These sweeping, allembracing prohibitions imposed on
the President and his official family,
which prohibitions are not similarly
imposed on other public officials or
employees such as the Members of
Congress, members of the civil service
in general and members of the armed
forces, are proof of the intent of the
1987 Constitution to treat the
President and his official family as a
class by itself and to impose upon said
class stricter prohibitions.
Xxx
Thus, while all other appointive
officials in the civil service are allowed
to hold other office or employment in
the government during their tenure
when such is allowed by law or by the
primary functions of their positions,
members of the Cabinet, their
deputies and assistants may do so
only when expressly authorized by the
Constitution itself.
In other words,
Section 7, Article IX-B is meant to lay
down the general rule applicable to all
elective and appointive public officials
and employees, while Section 13,
Article VII is meant to be the exception
applicable only to the President, the
Vice-President,
Members
of
the
Cabinet, their deputies and assistants.

This being the case, the


qualifying phrase unless otherwise
provided in this Constitution in
Section 13, Article VII cannot possibly
refer to the broad exceptions provided
under Section 7, Article IX-B of the
1987 Constitution. To construe said
qualifying phrase as respondents
would have us to do, would render
nugatory
and
meaningless
the
manifest intent and purpose of the
framers of the Constitution to impose
a stricter prohibition on the President,
Vice-President,
Members
of
the
Cabinet, their deputies and assistants
with respect to holding other offices or
employment in the government during
their
tenure.
Respondents
interpretation that Section 13 of
Article VII admits of the exceptions
found in Section 7, par. (2) of Article
IX-B would obliterate the distinction so
carefully set by the framers of the
Constitution as to when the highranking officials of the Executive
Branch from the President to assistant
Secretary, on the one hand, and the
generality of civil servants from the
rank immediately below Assistant
Secretary downwards, on the other,
may hold any other office or position
in the government during their tenure.
Moreover, respondents reading
of the provisions in question would
render
certain
parts
of
the
Constitution
inoperative.
This
observation applies particularly to the
Vice-President who, under Section 13
of Article VII is allowed to hold other
office or employment when so
authorized by the Constitution, but
who as an elective public official under
Sec. 7, par. (1) of Article IX-B is
absolutely ineligible for appointment
or designation in any capacity to any
public office or position during his
tenure.
Surely, to say that the
phrase unless otherwise provided in
this Constitution found in Section 13,
Article VII has reference to Section 7,
par. (1) of Article IX-B would render
meaningless the specific provisions of
the Constitution authorizing the VicePresident to become a member of the
Cabinet (Sec. 3, Ibid.), and to act as
President without relinquishing the
Vice-Presidency where the President
shall not have been chosen or fails to
qualify (Sec. 7, Article VII).
Such
absurd consequence can be avoided
only by interpreting the two provisions
under consideration as one, i.e.,
Section 7, par. (1) of Article IX-B
providing the general rule and the
other, i.e., Section 13, Article VII as
constituting the exception thereto. In

the same manner must Section 7, par.


(2) of Article IX-B be construed vis-vis Section 13, Article VII.
Xxx
Since the evident purpose of
the framers of the 1987 Constitution is
to impose a stricter prohibition on the
President, Vice-President, members of
the Cabinet, their deputies and
assistants with respect to holding
multiple offices or employment in the
government during their tenure, the
exception to this prohibition must be
read with equal severity. On its face,
the language of Section 13, Article VII
is prohibitory so that it must be
understood as intended to be a
positive and unequivocal negation of
the privilege of holding multiple
government offices and employment.
Verily, wherever the language used in
the constitution is prohibitory, it is to
be understood as intended to be a
positive and unequivocal negation
(Varney v. Justice, 86 Ky 596; 6 S.W.
457; Hunt v. State, 22 Tex. App. 396, 3
S.W. 233).
The phrase unless
otherwise
provided
in
this
Constitution must be given a literal
interpretation to refer only to those
particular instances cited in the
Constitution itself, to wit: the VicePresident being appointed as a
member of the Cabinet under Section
3, par. (2), Article VII; or acting as
President in those instances provided
under Section 7, pars. (2) and (3),
Article VII; and, the Secretary of Justice
being ex-officio member of the Judicial
and Bar Council by virtue of Section 8
(1), Article VIII.
Xxx
It being clear x x x that the
1987 Constitution seeks to prohibit the
President, Vice-President, members of
the
Cabinet,
their
deputies
or
assistants from holding during their
tenure multiple offices or employment
in the government, except in those
cases specified in the Constitution
itself and as above clarified with
respect
to
posts
held
without
additional compensation in an exofficio capacity as provided by law and
as required by the primary functions of
their office, the citation of Cabinet
members (then called Ministers) as
examples during the debate and
deliberation on the general rule laid
down for all appointive officials should
be considered as mere personal
opinions which cannot override the

constitutions manifest intent and the


peoples understanding thereof.
In the light of the construction
given to Section 13, Article VII in
relation to Section 7, par. (2), Article
IX-B
of
the
1987
Constitution,
Executive Order No. 284 dated July 23,
1987 is unconstitutional. Ostensibly
restricting the number of positions
that
Cabinet
members,
undersecretaries
or
assistant
secretaries may hold in addition to
their primary position to not more than
two (2) positions in the government
and
government
corporations,
Executive Order No. 284 actually
allows them to hold multiple offices or
employment in direct contravention of
the express mandate of Section 13,
Article VII of the 1987 Constitution
prohibiting them from doing so, unless
otherwise provided in the 1987
Constitution itself.
The
Court
is
alerted
by
respondents
to
the
impractical
consequences that will result from a
strict application of the prohibition
mandated under Section 13, Article VII
on the operations of the Government,
considering that Cabinet members
would be stripped of their offices held
in an ex-officio capacity, by reason of
their primary positions or by virtue of
legislation. As earlier clarified in this
decision, ex-officio posts held by the
executive official concerned without
additional compensation as provided
by law and as required by the primary
functions of his office do not fall under
the definition of any other office
within the contemplation of the
constitutional
prohibition.
With
respect to other offices or employment
held by virtue of legislation, including
chairmanships or directorships in
government-owned
or
controlled
corporations and their subsidiaries,
suffice it to say that the feared
impractical consequences are more
apparent than real. Being head of an
executive department is no mean job.
It is more than a full-time job,
requiring full attention, specialized
knowledge, skills and expertise.
If
maximum benefits are to be derived
from a department heads ability and
expertise, he should be allowed to
attend
to
his
duties
and
responsibilities without the distraction
of other governmental offices or
employment. He should be precluded
from dissipating his efforts, attention
and energy among too many positions
and responsibility, which may result in
haphazardness
and
inefficiency.

Surely the advantages to be derived


from this concentration of attention,
knowledge and expertise, particularly
at this stage of our national and
economic development, far outweigh
the benefits, if any, that may be
gained from a department head
spreading himself too thin and taking
in more than what he can handle.
Finding Executive Order No. 284
to be constitutionally infirm, the Court
hereby orders respondents x x x to
immediately relinquish their other
offices or employment, as herein
defined, in the government, including
government-owned
or
controlled
corporations and their subsidiaries.
(Civil Liberties Union v. Executive
Secretary, 194 SCRA 317, Feb. 22,
1991, En Banc [Fernan, CJ])
269. Does the prohibition against
holding dual or multiple offices or
employment under Section 13, Article
VII of the Constitution apply to posts
occupied by the Executive officials
specified therein without additional
compensation in an ex-officio capacity
as provided by law and as required by
the primary functions of said officials
office?
Held: The prohibition against
holding dual or multiple offices or
employment under Section 13, Article
VII of the Constitution must not,
however, be construed as applying to
posts occupied by the Executive
officials specified therein without
additional compensation in an exofficio capacity as provided by law and
as required (As opposed to the term
allowed used in Section 7, par. (2),
Article IX-B of the Constitution, which
is permissive. Required suggests an
imposition, and therefore, obligatory in
nature) by the primary functions of
said officials office. The reason is that
these posts do not comprise any
other office within the contemplation
of the constitutional prohibition but
are
properly
an
imposition
of
additional duties and functions on said
officials (Martin v. Smith, 140 A.L.R.
1073; Ashmore v. Greater Greenville
Sewer District, 173 A.L.R. 407). To
characterize these posts otherwise
would lead to absurd consequences,
among which are: The President of the
Philippines cannot chair the National
Security Council reorganized under
Executive Order No. 115 (December
24, 1986).
Neither can the VicePresident, the Executive Secretary,
and the Secretaries of National

Defense,
Justice,
Labor
and
Employment and Local Government sit
in this Council, which would then have
no reason to exist for lack of a
chairperson and members.
The
respective
undersecretaries
and
assistant secretaries, would also be
prohibited.
Xxx
Indeed, the framers of our
Constitution could not have intended
such absurd consequences.
A
Constitution, viewed as a continuously
operative charter of government, is
not to be interpreted as demanding
the impossible or the impracticable;
and
unreasonable
or
absurd
consequences, if possible, should be
avoided (Hirabayashi v. United States,
320 U.S. 81, 87 L. Ed. 1774, 63 S. Ct.
1375; others omitted).
To reiterate, the prohibition
under Section 13, Article VII is not to
be interpreted as covering positions
held without additional compensation
in ex-officio capacities as provided by
law and as required by the primary
functions of the concerned officials
office.
The term ex-officio means
from office; by virtue of office. It
refers to an authority derived from
official character merely, not expressly
conferred
upon
the
individual
character, but rather annexed to the
official position. Ex officio likewise
denotes an act done in an official
character, or as a consequence of
office,
and
without
any
other
appointment or authority than that
conferred by the office. (Blacks Law
Dictionary, p. 516; 15A Words and
Phrases, p. 392) An ex-officio member
of a board is one who is a member by
virtue of his title to a certain office,
and without further warrant or
appointment (15A Words and Phrases,
p. 392).
To illustrate, by express
provision of law, the Secretary of
Transportation and Communications is
the ex-officio Chairman of the Board of
the Philippine Ports Authority (Sec. 7,
E.O. 778), and the Light Rail Transit
Authority (Sec. 1, E.O. 210).
The Court had occasion to
explain the meaning of an ex-officio
position in Rafael v. Embroidery and
Apparel Control and Inspection Board
(21 SCRA 336 [1967]), thus: An
examination of Section 2 of the
questioned statute (R.A. 3137) reveals
that for the chairman and members of
the Board to qualify they need only be
designated
by
the
respective

department
heads.
With
the
exception of the representative from
the private sector, they sit ex-officio. I
order to be designated they must
already be holding positions in the
offices mentioned in the law. Thus, for
instance, one who does not hold a
previous appointment in the Bureau of
Customs, cannot, under the act, be
designated a representative from that
office. The same is true with respect
to the representatives from the other
offices.
No new appointments are
necessary. This is as it should be,
because
the
representatives
so
designated merely perform duties in
the Board in addition to those already
performed
under
their
original
appointments. (Italics supplied)
The term primary used to
describe functions refers to the
order of importance and thus means
chief or principal function. The term is
not restricted to the singular but may
refer to the plural (33A Words and
Phrases, p. 210, citing Collector of
Revenue v. Louisiana Ready Mix Co.,
La. App., 197 S. 2d 141, 145). The
additional duties must not only be
closely related to, but must be
required by the officials primary
functions. Examples of designations
to positions by virtue of ones primary
functions are the Secretaries of
Finance
and
Budget
sitting
as
members of the Monetary Board, and
the Secretary of Transportation and
Communications acting as Chairman
of the Maritime Industry Authority
(Sec. 7, P.D. No. 474) and the Civil
Aeronautics Board.
If the functions to be performed
are
merely
incidental,
remotely
related, inconsistent, incompatible, or
otherwise alien to the primary function
of a cabinet official, such additional
functions would fall under the purview
of any other office prohibited by the
Constitution. An example would be
the Press Undersecretary sitting as a
member of the Board of the Philippine
Amusement and Gaming Corporation.
The same rule applies to such
positions which confer on the cabinet
official management functions and/or
monetary compensation, such as but
not limited to chairmanships or
directorships in government-owned or
controlled corporations and their
subsidiaries.
Mandating additional duties and
functions to the President, VicePresident, Cabinet Members, their
deputies or assistants which are not

inconsistent
with
those
already
prescribed
by
their
offices
or
appointments by virtue of their special
knowledge, expertise and skill in their
respective executive offices is a
practice long-recognized in many
jurisdictions. It is a practice justified
by the demands of efficiency, policy
direction, continuity and coordination
among the different offices in the
Executive Branch in the discharge of
its multifarious tasks of executing and
implementing laws affecting national
interest and general welfare and
delivering basic services to the people.
It is consistent with the power vested
on the President and his alter egos,
the Cabinet members, to have control
of all the executive departments,
bureaus and offices and to ensure that
the laws are faithfully executed
(Section 17, Article VII). Without these
additional duties and functions being
assigned to the President and his
official family to sit in the governing
bodies or boards of governmental
agencies or instrumentalities in an exofficio capacity as provided by law and
as required by their primary functions,
they would be deprived of the means
for control and supervision, thereby
resulting in an unwieldy and confused
bureaucracy.
It bears repeating though that
in order that such additional duties or
functions may not transgress the
prohibition embodied in Section 13,
Article VII of the 1987 Constitution,
such additional duties or functions
must be required by the primary
functions of the official concerned,
who is to perform the same in an exofficio capacity as provided by law,
without receiving
any additional
compensation therefor.
The ex-officio position being
actually and in legal contemplation
part of the principal office, it follows
that the official concerned has no right
to receive additional compensation for
his services in the said position. The
reason is that these services are
already paid for and covered by the
compensation attached to his principal
office. It should be obvious that if,
say, the Secretary of Finance attends a
meeting of the Monetary Board as an
ex-officio member thereof, he is
actually and in legal contemplation
performing the primary function of his
principal office in defining policy in
monetary and banking matters, which
come under the jurisdiction of his
department.
For such attendance,
therefore, he is not entitled to collect

any extra compensation, whether it be


in the form of a per diem or an
honorarium or an allowance, or some
other such euphemism. By whatever
name it is designated, such additional
compensation is prohibited by the
Constitution. (Civil Liberties Union
v. Executive Secretary, 194 SCRA
317, Feb. 22, 1991, En Banc
[Fernan, CJ])
270. Should members of the Cabinet
appointed to other positions in the
government pursuant to Executive
Order No. 284 which later was
declared unconstitutional by the SC for
being violative of Section 13, Article
VII of the Constitution be made to
reimburse
the
government
for
whatever pay and emoluments they
received from holding such other
positions?
Held: During their tenure in
the questioned positions, respondents
may be considered de facto officers
and as such entitled to emoluments
for actual services rendered (Castillo v.
Arrieta, G.R. No. L-31444, November
13, 1974, 61 SCRA 55). It has been
held that in cases where there is no
de jure officer, a de facto officer, who,
in good faith has had possession of the
office and has discharged the duties
pertaining thereto, is legally entitled to
the emoluments of the office, and may
in an appropriate action recover the
salary, fees and other compensations
attached to the office. This doctrine
is,
undoubtedly,
supported
on
equitable grounds since it seems
unjust that the public should benefit
by the services of an officer de facto
and then be freed from all liability to
pay any one for such services
(Patterson v. Benson, 112 Pac. 801, 32
L.R.A. [NS] 949).
Any per diem,
allowances or other emoluments
received by the respondents by virtue
of actual services rendered in the
questioned positions may therefore be
retained by them. (Civil Liberties
Union v. Executive Secretary, 194
SCRA 317, Feb. 22, 1991, En Banc
[Fernan, CJ])
271. May a Senator or Congressman
hold any other office or employment?
Ans.: No Senator or Member of
the House of Representatives may
hold any other office or employment in
the government, or any subdivision,
agency, or instrumentality thereof,
including government-owned or
controlled corporations or their
subsidiaries, during his term without

forfeiting his seat. Neither shall he be


appointed to any office which may
have been created or the emoluments
thereof increased during the term for
which he was elected (Sec. 13, Art. VI,
1987 Constitution). The first sentence
is referred to as an incompatible
office; the second is a forbidden office.

272. Petitioner claims that Benipayo


has no authority to remove her as
Director IV of the EID and reassign her
to the Law Department. Petitioner
further argues that only the COMELEC,
acting as a collegial body, can
authorize such reappointment.
Moreover, petitioner maintains that a
reassignment without her consent
amounts to removal from office
without due process and therefore
illegal.

does not enjoy security of tenure as


Director IV. X x x
Xxx
Having been appointed merely
in a temporary or acting capacity, and
not possessed of the necessary
qualifications to hold the position of
Director IV, petitioner has no legal
basis in claiming that her
reassignment was contrary to the Civil
Service Law. X x x
Still, petitioner assails her
reassignment, carried out during the
election period, as a prohibited act
under Section 261 (h) of the Omnibus
Election Code x x x.
Xxx

Held: Petitioners posturing will


hold water if Benipayo does not
possess any color of title to the office
of Chairman of the COMELEC. We
have ruled, however, that Benipayo is
the de jure COMELEC Chairman, and
consequently he has full authority to
exercise all the powers of that office
for so long as his ad interim
appointment remains effective. X x x.
The Chairman, as the Chief Executive
of the COMELEC, is expressly
empowered on his own authority to
transfer or reassign COMELEC
personnel in accordance with the Civil
Service Law. In the exercise of this
power, the Chairman is not required
by law to secure the approval of the
COMELEC en banc.

Petitioner claims that Benipayo failed


to secure the approval of the
COMELEC en banc to effect transfers
or reassignments of COMELEC
personnel during the election period.
Moreover, petitioner insists that the
COMELEC en banc must concur to
every transfer or reassignment of
COMELEC personnel during the
election period.

Petitioners appointment papers


x x x indisputably show that she held
her Director IV position in the EID only
in an acting or temporary capacity.
Petitioner is not a Career Executive
Service (CES), and neither does she
hold Career Executive Service
Eligibility, which are necessary
qualifications for holding the position
of Director IV as prescribed in the
Qualifications Standards (Revised
1987) issued by the Civil Service
Commission. Obviously, petitioner

The proviso in COMELEC Resolution


No. 3300, requiring due notice and
hearing before any transfer or
reassignment can be made within
thirty days prior to election day, refers
only to COMELEC field personnel and
not to head office personnel like the
petitioner. Under the Revised
Administrative Code (see Section 7
[4], Chapter 2, Subtitle C, Book V of
the Revised Administrative Code), the
COMELEC Chairman is the sole officer
specifically vested with the power to
transfer or reassign COMELEC

Contrary to petitioners allegation, the


COMELEC did in fact issue COMELEC
Resolution No. 3300 dated November
6, 2000, exempting the COMELEC from
Section 261 (h) of the Omnibus
Election Code. X x x
Xxx

personnel. The COMELEC Chairman


will logically exercise the authority to
transfer or reassign COMELEC
personnel pursuant to COMELEC
Resolution No. 3300. The COMELEC en
banc cannot arrogate unto itself this
power because that will mean
amending the Revised Administrative
Code, an act the COMELEC en banc
cannot legally do.
COMELEC Resolution No. 3300 does
not require that every transfer or
reassignment of COMELEC personnel
should carry the concurrence of the
COMELEC as a collegial body.
Interpreting Resolution No. 3300 to
require such concurrence will render
the resolution meaningless since the
COMELEC en banc will have to
approve every personnel transfer or
reassignment, making the resolution
utterly useless. Resolution No. 3300
should be interpreted for what it is, an
approval to effect transfers and
reassignments of personnel, without
need of securing a second approval
from the COMELEC en banc to actually
implement such transfer or
reassignment.
The COMELEC Chairman is the official
expressly authorized by law to transfer
or reassign COMELEC personnel. The
person holding that office, in a de jure
capacity, is Benipayo. The COMELEC
en banc, in COMELEC Resolution No.
3300, approved the transfer or
reassignment of COMELEC personnel
during the election period. Thus,
Benipayos order reassigning
petitioner from the EID to the Law
Department does not violate Section
261 (h) of the Omnibus Election Code.
For the same reason, Benipayos order
designating Cinco Officer-in-Charge of
the EID is legally unassailable.
(Matibag v. Benipayo, 380 SCRA
49, April 2, 2002, En Banc
[Carpio])
273. May the appointment of a
person assuming a position in the civil
service
under
a
completed
appointment be validly recalled or
revoked?

Held: It has been held that upon the


issuance of an appointment and the
appointees assumption of the position
in the civil service, he acquires a
legal right which cannot be taken
away either by revocation of the
appointment or by removal except for
cause and with previous notice and
hearing.
(Mauna v. Civil Service
Commission, 232 SCRA 388, 398
[1994]) Moreover, it is well-settled
that the person assuming a position in
the civil service under a completed
appointment acquires a legal, not just
an equitable, right to the position.
This right is protected not only by
statute, but by the Constitution as
well, which right cannot be taken away
by
either
revocation
of
the
appointment, or by removal, unless
there is valid cause to do so, provided
that there is previous notice and
hearing. (Aquino v. Civil Service
Commission, 208 SCRA 240, 248
[1992])
Petitioner admits that his very first
official act upon assuming the position
of town mayor was to issue Office
Order No. 95-01 which recalled the
appointments
of
the
private
respondents. There was no previous
notice, much less a hearing accorded
to the latter. Clearly, it was petitioner
who acted in undue haste to remove
the private
respondents without
regard for the simple requirements of
due process of law. While he argues
that the appointing power has the sole
authority to revoke said appointments,
there is no debate that he does not
have blanket authority to do so.
Neither can he question the CSCs
jurisdiction to affirm or revoke the
recall.
Rule V, Section 9 of the Omnibus
Implementing Regulations of the
Revised
Administrative
Code
specifically
provides
that
an
appointment
accepted
by
the
appointee cannot be withdrawn or
revoked by the appointing authority
and shall remain in force and in effect
until disapproved by the Commission.
Thus, it is the CSC that is authorized to
recall
an
appointment
initially
approved, but only when such
appointment and approval are proven
to be in disregard of applicable
provisions of the civil service law and
regulations (Debulgado v. Civil
Service Commission, 237 SCRA
184, 200 [1994]).
Moreover, Section 10 of the same rule
provides:

Sec. 10. An appointment issued in


accordance with pertinent laws and
rules shall take effect immediately
upon its issuance by the appointing
authority, and if the appointee has
assumed the duties of the position, he
shall be entitled to receive his salary
at once without awaiting the approval
of
his
appointment
by
the
Commission. The appointment shall
remain effective until disapproved by
the Commission. In no case shall an
appointment take effect earlier than
the date of its issuance.
Section 20 of Rule VI also provides:
Sec. 20. Notwithstanding the initial
approval of an appointment, the same
may be recalled on any of the
following grounds:
a
b
c
d

Non-compliance
with
the
procedures/criteria provided in the
agencys Merit Promotion Plan;
Failure to pass through the agencys
Selection/Promotion Board;
Violation of the existing collective
agreement between management and
employees relative to promotion; or
Violation of other existing civil service
law, rules and regulations.
Accordingly, the appointments of the
private respondents may only be
recalled on the above-cited grounds.
And yet, the only reason advanced by
the petitioner to justify the recall was
that
these
were
midnight
appointments.
The CSC correctly
ruled, however, that the constitutional
prohibition on so-called midnight
appointments,
specifically
those
made
within
two
(2)
months
immediately
prior
to
the
next
presidential elections, applies only to
the President or Acting President. (De
Rama v. Court of Appeals, 353
SCRA 94, Feb. 28, 2001, En Banc
[Ynares-Santiago])
274. The Philippine National Red
Cross (PNRC) is a government-owned
and controlled corporation with an
original charter under R.A. No. 95, as
amended. Its charter, however, was
amended to vest in it the authority to
secure loans, be exempted from
payment of all duties, taxes, fees and
other charges, etc.
With the
amendment of its charter, has it been
impliedly converted to a private
corporation?

Held: The test to determine whether


a corporation is government owned or
controlled, or private in nature is
simple.
Is it created by its own
charter for the exercise of a public
function, or by incorporation under the
general corporation law? Those with
special charters are government
corporations subject to its provisions,
and its employees are under the
jurisdiction of the Civil Service
Commission.
The PNRC was not
impliedly converted to a private
corporation
simply
because
its
charter was amended to vest in it the
authority
to
secure
loans,
be
exempted from payment of all duties,
taxes, fees and other charges, etc.
(Camporedondo v. NLRC, G.R. No.
129049, Aug. 6, 1999, 1st Div.
[Pardo])
275. What is a primarily confidential
position?
What is the test to
determine whether a position is
primarily confidential or not?
Held: A primarily confidential position
is one which denotes not only
confidence in the aptitude of the
appointee for the duties of the office
but primarily close intimacy which
ensures freedom from intercourse
without embarrassment or freedom
from misgivings or betrayals of
personal trust or confidential matters
of state. (De los Santos v. Mallare, 87
Phil. 289 [1950])
Under the proximity rule, the occupant
of a particular position could be
considered a confidential employee if
the predominant reason why he was
chosen by the appointing authority
was the latters belief that he can
share a close intimate relationship
with the occupant which ensures
freedom of discussion without fear or
embarrassment or misgivings of
possible betrayal of personal trust or
confidential matters of state. Withal,
where the position occupied is more
remote from that of the appointing
authority, the element of trust
between
them
is
no
longer
predominant.
(CSC v. Salas, 274
SCRA 414, June 19, 1997)
276. Does the Civil Service Law
contemplate a review of decisions
exonerating officers or employees
from administrative charges?
Held:
By this ruling, we now
expressly abandon
and
overrule
extant jurisprudence that the phrase
party adversely affected by the

decision refers to the government


employee
against
whom
the
administrative case is filed for the
purpose of disciplinary action which
may take the form of suspension,
demotion in rank or salary, transfer,
removal or dismissal from office and
not included are cases where the
penalty imposed is suspension for not
more than thirty (30) days or fine in an
amount not exceeding thirty days
salary (Paredes v. Civil Service
Commission, 192 SCRA 84, 85) or
when respondent is exonerated of the
charges, there is no occasion for
appeal. (Mendez v. Civil Service
Commission, 204 SCRA 965, 968) In
other words, we overrule prior
decisions holding that the Civil Service
Law does not contemplate a review
of decisions exonerating officers or
employees
from
administrative
charges enunciated in Paredes v. Civil
Service Commission (192 SCRA 84);
Mendez v. Civil Service Commission
(204 SCRA 965); Magpale v. Civil
Service Commission (215 SCRA 398);
Navarro v. Civil Service Commission
and Export Processing Zone Authority
(226 SCRA 207) and more recently Del
Castillo v. Civil Service Commission
(237 SCRA 184). (CSC v. Pedro O.
Dacoycoy, G.R. No. 135805, April
29, 1999, En Banc [Pardo])
277. What is preventive suspension?
Discuss its nature.
Held: Imposed during the pendency
of an administrative investigation,
preventive suspension is not a penalty
in itself. It is merely a measure of
precaution so that the employee who
is charged may be separated, for
obvious reasons, from the scene of his
alleged misfeasance while the same is
being investigated. Thus preventive
suspension is distinct from the
administrative penalty of removal from
office such as the one mentioned in
Sec. 8(d) of P.D. No. 807. While the
former may be imposed on a
respondent during the investigation of
the charges against him, the latter is
the penalty which may only be meted
upon him at the termination of the
investigation or the final disposition of
the case.
(Beja, Sr. v. CA, 207
SCRA
689,
March
31,
1992
[Romero])
278. Discuss the kinds of preventive
suspension under the Civil Service
Law.
When may a civil service
employee placed under preventive
suspension
be
entitled
to
compensation?

Held:
There are two kinds of
preventive suspension of civil service
employees who are charged with
offenses punishable by removal or
suspension: (1) preventive suspension
pending investigation (Sec. 51, Civil
Service Law, EO No. 292) and (2)
preventive suspension pending appeal
if the penalty imposed by the
disciplining authority is suspension or
dismissal and, after review, the
respondent is exonerated (Section 47,
par. 4, Civil Service Law, EO No. 292).
Preventive
suspension
pending
investigation is not a penalty. It is a
measure intended to enable the
disciplining authority to investigate
charges
against
respondent
by
preventing the latter from intimidating
or in any way influencing witnesses
against him. If the investigation is not
finished and a decision is not rendered
within that period, the suspension will
be lifted and the respondent will
automatically be reinstated. If after
investigation respondent is found
innocent of the charges and is
exonerated, he should be reinstated.
However, no compensation was due
for
the
period
of
preventive
suspension pending investigation. The
Civil Service Act of 1959 (R.A. No.
2260) providing for compensation in
such a case once the respondent was
exonerated was revised in 1975 and
the provision on the payment of
salaries
during
suspension
was
deleted.
But although it is held that employees
who are preventively suspended
pending investigation are not entitled
to the payment of their salaries even if
they are exonerated, they are entitled
to compensation for the period of their
suspension
pending
appeal
if
eventually they are found innocent.
Preventive
suspension
pending
investigation x x x is not a penalty but
only a means of enabling the
disciplining authority to conduct an
unhampered investigation.
On the
other hand, preventive suspension
pending appeal is actually punitive
although it is in effect subsequently
considered illegal if respondent is
exonerated and the administrative
decision finding him guilty is reversed.
Hence, he should be reinstated with
full pay for the period of the
suspension. (Gloria v. CA, G.R. No.
131012, April 21, 1999, En Banc
[Mendoza])

279. Discuss
the
power
of
Ombudsman to conduct administrative
investigations,
and
to
impose
preventive suspension.
Held: Worth stressing, to resolve the
present controversy, we must recall
that the authority of the Ombudsman
to
conduct
administrative
investigations is mandated by no less
than the Constitution. x x x
R.A. 6770, the Ombudsman Law,
further grants the Office of the
Ombudsman the statutory power to
conduct administrative investigations.
Xxx
Section 21 of R.A. 6770 names the
officials subject to the Ombudsmans
disciplinary authority x x x.
Petitioner is an elective local official
accused of grave misconduct and
dishonesty.
That the Office of the
Ombudsman
may
conduct
an
administrative investigation into the
acts complained of, appears clear from
the foregoing provisions of R.A. 6770.
However, the question of whether or
not the Ombudsman may conduct an
investigation over a particular act or
omission, is different from the
question of whether or not petitioner,
after investigation, may be held
administratively liable. This distinction
ought here to be kept in mind, even as
we must also take note that the power
to investigate is distinct from the
power to suspend preventively an
erring public officer.
Likewise worthy of note, the power of
the Office of the Ombudsman to
preventively
suspend
an
official
subject
to
its
administrative
investigation is provided by specific
provision of law. X x x
We have previously interpreted the
phrase under his authority to mean
that the Ombudsman can preventively
suspend
all
officials
under
investigation by his office, regardless
of the branch of government in which
they are employed (Buenaseda v.
Flavier, 226 SCRA 645, 654 [1993]),
excepting of course those removable
by
impeachment,
members
of
Congress and the Judiciary.
The power to preventively suspend is
available not only to the Ombudsman
but also to the Deputy Ombudsman.
This is the clear import of Section 24
of R.A. 6770 abovecited.

There can be no question in this case


as to the power and authority of
respondent Deputy Ombudsman to
issue
an
order
of
preventive
suspension against an official like the
petitioner, to prevent that official from
using his office to intimidate or
influence witnesses (Gloria v. CA, et
al., G.R. No. 131012, April 21, 1999, p.
7, 306 SCRA 287) or to tamper with
records that might be vital to the
prosecution of the case against him
(Yasay, Jr. v. Desierto, et al., G.R. No.
134495, December 28, 1998, p. 9, 300
SCRA 494). In our view, the present
controversy simply boils down to this
pivotal question: Given the purpose of
preventive
suspension
and
the
circumstances of this case, did
respondent
Deputy
Ombudsman
commit a grave abuse of discretion
when he set the period of preventive
suspension at six months?
Preventive suspension under Sec. 24,
R.A. 6770 x x x may be imposed when,
among other factors, the evidence of
guilt is strong. The period for which
an official may be preventively
suspended must not exceed six
months. In this case, petitioner was
preventively suspended and ordered
to cease and desist from holding office
for the entire period of six months,
which is the maximum provided by
law.
The determination of whether or not
the evidence of guilt is strong as to
warrant preventive suspension rests
with the Ombudsman (Nera v. Garcia,
106
Phil.
1031
[1960];
others
omitted.). The discretion as regards
the period of such suspension also
necessarily
belongs
to
the
Ombudsman, except that he cannot
extend the period of suspension
beyond that provided by law (CastilloCo v. Barbers, supra.). But, in our
view, both the strength of the
evidence to warrant said suspension
and the propriety of the length or
period of suspension imposed on
petitioner are properly raised in this
petition for certiorari and prohibition.
Xxx
Xxx
Given these findings, we cannot say
now that there is no evidence
sufficiently strong to justify the
imposition of preventive suspension
against petitioner. But considering its
purpose and the circumstances in the
case brought before us, it does appear

to us that the imposition of the


maximum period of six months is
unwarranted.
X x x [G]ranting that now the evidence
against petitioner is already strong,
even without conceding that initially it
was weak, it is clear to us that the
maximum
six-month
period
is
excessive and definitely longer than
necessary for the Ombudsman to
make its legitimate case against
petitioner. We must conclude that the
period during which petitioner was
already preventively suspended, has
been sufficient for the lawful purpose
of preventing petitioner from hiding
and destroying needed documents, or
harassing and preventing witnesses
who wish to appear against him.
(Garcia v. Mojica, 314 SCRA 207,
Sept.
10,
1999,
2nd
Div.
[Quisumbing])
280. Distinguish
preventive
suspension
under
the
Local
Government Code from preventive
suspension under the Ombudsman
Act.
Held:
We reach the foregoing
conclusion,
however,
without
necessarily subscribing to petitioners
claim that the Local Government
Code, which he averred should apply
to this case of an elective local official,
has been violated. True, under said
Code, preventive suspension may only
be imposed after the issues are joined,
and only for a maximum period of
sixty days.
Here, petitioner was
suspended without having had the
chance to refute first the charges
against him, and for the maximum
period of six months provided by the
Ombudsman Law. But as respondents
argue,
administrative
complaints
commenced under the Ombudsman
Law are distinct from those initiated
under the Local Government Code.
Respondents point out that the shorter
period of suspension under the Local
Government Code is intended to limit
the period of suspension that may be
imposed by a mayor, a governor, or
the President, who may be motivated
by partisan political considerations. In
contrast the Ombudsman, who can
impose a longer period of preventive
suspension, is not likely to be similarly
motivated
because
it
is
a
constitutional body. The distinction is
valid but not decisive, in our view, of
whether there has been grave abuse
of discretion in a specific case of
preventive suspension.

Xxx
Respondents may be correct in
pointing out the reason for the shorter
period
of
preventive
suspension
imposable
under
the
Local
Government Code.
Political color
could taint the exercise of the power
to suspend local officials by the
mayor, governor, or Presidents office.
In
contrast
the
Ombudsman,
considering the constitutional origin of
his Office, always ought to be
insulated from the vagaries of politics,
as respondents would have us believe.
In Hagad v. Gozo-Dadole (251
SCRA 242 [1995]), on the matter of
whether or not the Ombudsman has
been stripped of his power to
investigate local elective officials by
virtue of the Local Government Code,
we said:
Indeed, there is nothing in the Local
Government Code to indicate that it
has repealed, whether expressly or
impliedly, the pertinent provisions of
the Ombudsman Act.
The two
statutes on the specific matter in
question are not so inconsistent, let
alone irreconcilable, as to compel us
to only uphold one and strike down the
other. (Hagad v. Gozo-Dadole,
supra, 251-252)
It was also argued in Hagad,
that
the
six-month
preventive
suspension under the Ombudsman
Law is much too repugnant to the
60-day period that may be imposed
under the Local Government Code.
But per J. Vitug, the two provisions
govern differently. (Id., at 253-254)
However,
petitioner
now
contends that Hagad did not settle the
question of whether a local elective
official
may
be
preventively
suspended even before the issues
could be joined. Indeed it did not, but
we have held in other cases that there
could be preventive suspension even
before the charges against the official
are heard, or before the official is
given an opportunity to prove his
innocence (supra at note 14,
excluding the case of Buenaseda
v. Flavier). Preventive suspension is
merely a preliminary step in an
administrative investigation and is not
in any way the final determination of
the guilt of the official concerned.
Petitioner also avers that the
suspension order against him was

issued in violation of Section 26[2] of


the Ombudsman Law x x x.
Petitioner argues that before an
inquiry may be converted into a fullblown administrative investigation, the
official concerned must be given 72
hours to answer the charges against
him. In his case, petitioner says the
inquiry was
converted
into an
administrative investigation without
him being given the required number
of hours to answer.
Indeed, it does not appear that
petitioner was given the requisite 72
hours to submit a written answer to
the complaint against him.
This,
however, does not make invalid the
preventive suspension order issued
against him.
As we have earlier
stated, a preventive suspension order
may be issued even before the
charges against the official concerned
is heard.
Moreover, respondents state that
petitioner was given 10 days to submit
his counter-affidavit to the complaint
filed by respondent Tagaan. We find
this 10-day period is in keeping with
Section 5[a] of the Rules of Procedure
of the Office of the Ombudsman x x x.
(Garcia v. Mojica, 314 SCRA 207,
Sept.
10,
1999,
2nd
Div.
[Quisumbing])
281. Does Section 13, Republic Act
No. 3019 exclude from its coverage
the members of Congress and,
therefore, the Sandiganbayan erred in
decreeing the preventive suspension
order against Senator Miriam
Defensor-Santiago? Will the order of
suspension prescribed by Republic Act
No. 3019 not encroach on the power of
Congress to discipline its own ranks
under the Constitution?
Held: The petition assails the
authority of the Sandiganbayan to
decree a ninety-day preventive
suspension of Mme. Miriam DefensorSantiago, a Senator of the Republic of
the Philippines, from any government
position, and furnishing a copy thereof
to the Senate of the Philippines for the
implementation of the suspension
order.
The authority of the
Sandiganbayan to order the

preventive suspension of an
incumbent public official charged with
violation of the provisions of Republic
Act No. 3019 has both legal and
jurisprudential support. X x x
In the relatively recent case of
Segovia v. Sandiganbayan (288 SCRA
328 [1998]), the Court reiterated:
The validity of Section 13, R.A. 3019,
as amended treating of the
suspension pendente lite of an
accused public officer may no longer
be put at issue, having been
repeatedly upheld by this Court.
X x x
The provision of suspension pendente
lite applies to all persons indicted
upon a valid information under the
Act, whether they be appointive or
elective officials; or permanent or
temporary employees, or pertaining to
the career or non-career service. (At
pp. 336-337)
It would appear, indeed, to be a
ministerial duty of the court to issue
an order of suspension upon
determination of the validity of the
information filed before it. Once the
information is found to be sufficient in
form and substance, the court is
bound to issue an order of suspension
as a matter of course, and there
seems to be no ifs and buts about it.
(Libanan v. Sandiganbayan, 163 SCRA
163 [1988]) Explaining the nature of
the preventive suspension, the Court
in the case of Bayot v. Sandiganbayan
(128 SCRA 383 [1984]) observed:
x x x It is not a penalty because it is
not imposed as a result of judicial
proceedings. In fact, if acquitted, the
official concerned shall be entitled to
reinstatement and to the salaries and
benefits which he failed to receive
during suspension. (At p. 386)
In issuing the preventive
suspension of petitioner, the
Sandiganbayan merely adhered to the
clear and unequivocal mandate of the

law, as well as the jurisprudence in


which the Court has, more than once,
upheld Sandiganbayans authority to
decree the suspension of public
officials and employees indicted
before it.
Section 13 of Republic Act No.
3019 does not state that the public
officer concerned must be suspended
only in the office where he is alleged
to have committed the acts with which
he has been charged. Thus, it has
been held that the use of the word
office would indicate that it applies
to any office which the officer charged
may be holding, and not only the
particular office under which he stands
accused. (Bayot v. Sandiganbayan,
supra; Segovia v. Sandiganbayan,
supra.)
En passant, while the imposition
of suspension is not automatic or selfoperative as the validity of the
information must be determined in a
pre-suspension hearing, there is no
hard and fast rule as to the conduct
thereof. It has been said that
x x x No specific rules need be laid
down for such pre-suspension hearing.
Suffice it to state that the accused
should be given a fair and adequate
opportunity to challenge the VALIDITY
OF THE CRIMINAL PROCEEDINGS
against him, e.g., that he has not been
afforded the right of due preliminary
investigation; that the acts for which
he stands charged do not constitute a
violation of the provisions of Republic
Act 3019 or the bribery provisions of
the Revised Penal Code which would
warrant his mandatory suspension
from office under Section 13 of the
Act; or he may present a motion to
quash the information on any of the
grounds provided for in Rule 117 of
the Rules of Court x x x.
x x x
Likewise, he is accorded the right to
challenge the propriety of his
prosecution on the ground that the
acts for which he is charged do not

constitute a violation of Rep. Act 3019,


or of the provisions on bribery of the
Revised Penal Code, and the right to
present a motion to quash the
information on any other grounds
provided in Rule 117 of the Rules of
Court.
However, a challenge to the validity
of the criminal proceedings on the
ground that the acts for which the
accused is charged do not constitute a
violation of the provisions of Rep. Act
No. 3019, or of the provisions on
bribery of the Revised Penal Code,
should be treated only in the same
manner as a challenge to the criminal
proceeding by way of a motion to
quash on the ground provided in
Paragraph (a), Section 2 of Rule 117 of
the Rules of Court, i.e., that the facts
charged do not constitute an offense.
In other words, a resolution of the
challenge to the validity of the
criminal proceeding, on such ground,
should be limited to an inquiry
whether the facts alleged in the
information, if hypothetically admitted,
constitute the elements of an offense
punishable under Rep. Act 3019 or the
provisions on bribery of the Revised
Penal Code. (Luciano v. Mariano, 40
SCRA 187 [1971]; People v. Albano,
163 SCRA 511, 517-519 [1988])
The law does not require that
the guilt of the accused must be
established in a pre-suspension
proceeding before trial on the merits
proceeds. Neither does it contemplate
a proceeding to determine (1) the
strength of the evidence of culpability
against him, (2) the gravity of the
offense charged, or (3) whether or not
his continuance in office could
influence the witnesses or pose a
threat to the safety and integrity of
the records and other evidence before
the court could have a valid basis in
decreeing preventive suspension
pending the trial of the case. All it
secures to the accused is adequate
opportunity to challenge the validity or
regularity of the proceedings against
him, such as, that he has not been
afforded the right to due preliminary

investigation, that the acts imputed to


him do not constitute a specific crime
warranting his mandatory suspension
from office under Section 13 of
Republic Act No. 3019, or that the
information is subject to quashal on
any of the grounds set out in Section
3, Rule 117, of the Revised Rules on
Criminal Procedure (Segovia v.
Sandiganbayan, supra; Resolution of
the Supreme Court in A.M. No. 00-0503-SC, dated 03 October 2000, which
became effective on 01 December
2000)
Xxx
The pronouncement, upholding
the validity of the information filed
against petitioner, behooved
Sandiganbayan to discharge its
mandated duty to forthwith issue the
order of preventive suspension.
The order of suspension
prescribed by Republic Act No. 3019 is
distinct from the power of Congress to
discipline its own ranks under the
Constitution which provides that each

x x x house may determine the rules


of its proceedings, punish its Members
for disorderly behavior, and, with the
concurrence of two-thirds of all its
Members, suspend or expel a Member.
A penalty of suspension, when
imposed, shall not exceed sixty days.
(Section 16[3], Article VI, 1987
Constitution)
The suspension contemplated in
the above constitutional provision is a
punitive measure that is imposed upon
determination by the Senate or the
House of Representatives, as the case
may be, upon an erring member.
Thus, in its resolution in the case of
Ceferino Paredes, Jr. v. Sandiganbayan,
et al. (G.R. No. 118364, 08 August
1995), the Court affirmed the order of
suspension of Congressman Paredes
by the Sandiganbayan, despite his
protestations on the encroachment by
the court on the prerogatives of
Congress. The Court ruled:

x x x. Petitioners invocation of
Section 16 (3), Article VI of the
Constitution which deals with the
power of each House of Congress inter
alia to punish its Members for
disorderly behavior, and suspend or
expel a Member by a vote of twothirds of all its Members subject to the
qualification that the penalty of
suspension, when imposed, should not
exceed sixty days in unavailing, as it
appears to be quite distinct from the
suspension spoken of in Section 13 of
RA 3019, which is not a penalty but a
preliminary, preventive measure,
prescinding from the fact that the
latter is not being imposed on
petitioner for misbehavior as a
Member of the House of
Representatives.
The doctrine of separation of powers
by itself may not be deemed to have
effectively excluded Members of
Congress from Republic Act No. 3019
nor from its sanctions. The maxim
simply recognizes each of the three
co-equal and independent, albeit
coordinate, branches of the
government the Legislative, the
Executive and the Judiciary has
exclusive prerogatives and cognizance
within its own sphere of influence and
effectively prevents one branch from
unduly intruding into the internal
affairs of either branch.
Parenthetically, it might be well to
elaborate a bit. Section 1, Article VIII,
of the 1987 Constitution, empowers
the Court to act not only in the
settlement of actual controversies
involving rights which are legally
demandable and enforceable, but
also in the determination of whether
or not there has been a grave abuse of
discretion amounting to lack or excess
of jurisdiction on the part of any
branch or instrumentality of the
government. The provision allowing
the Court to look into any possible
grave abuse of discretion committed
by any government instrumentality
has evidently been couched in general
terms in order to make it malleable to

judicial interpretation in the light of


any emerging milieu. In its normal
concept, the term has been said to
imply an arbitrary, despotic, capricious
or whimsical exercise of judgment
amounting to lack or excess of
jurisdiction. When the question,
however, pertains to an affair internal
to either of Congress or the Executive,
the Court subscribes to the view that
unless an infringement of any specific
Constitutional proscription thereby
inheres the Court should not deign
substitute its own judgment over that
of any of the other two branches of
government. It is an impairment or a
clear disregard of a specific
constitutional precept or provision that
can unbolt the steel door for judicial
intervention. If any part of the
Constitution is not, or ceases to be,
responsive to contemporary needs, it
is the people, not the Court, who must
promptly react in the manner
prescribed by the Charter itself.
Republic Act No. 3019 does not
exclude from its coverage the
members of Congress and that,
therefore, the Sandiganbayan did not
err in thus decreeing the assailed
preventive suspension order.
Attention might be called to the fact
that Criminal Case No. 16698 has
been decided by the First Division of
the Sandiganbayan on 06 December
1999, acquitting herein petitioner. The
Court, nevertheless, deems it
appropriate to render this decision for
future guidance on the significant
issue raised by petitioner. (Santiago
v. Sandiganbayan, 356 SCRA 636,
April 18, 2001, En Banc [Vitug])

282. What
is
the
doctrine
of
forgiveness or condonation? Does it
apply to pending criminal cases?
Held: 1. A public official cannot be
removed
for
administrative
misconduct committed during a prior
term, since his re-election to office
operates as a condonation of the
officers previous misconduct to the
extent of cutting off the right to
remove him therefor. The foregoing

rule, however, finds no application to


criminal
cases
pending
against
petitioner.
(Aguinaldo v. Santos,
212 SCRA 768, 773 [1992])
2. A reelected local official may not be
held administratively accountable for
misconduct committed during his prior
term of office. The rationale for this
holding is that when the electorate put
him back into office, it is presumed
that it did so with full knowledge of his
life and character, including his past
misconduct.
If, armed with such
knowledge, it still reelects him, then
such reelection is considered a
condonation of his past misdeeds.
(Mayor Alvin B. Garcia v. Hon.
Arturo C. Mojica, et al., G.R. No.
139043,
Sept.
10,
1999
[Quisumbing])
283. What
is
the
Doctrine
Condonation? Illustrative case.

of

Held: Petitioner contends that, per


our ruling in Aguinaldo v. Santos (212
SCRA 768 [1992]), his reelection has
rendered the administrative case filed
against him moot and academic. This
is because his reelection operates as a
condonation by the electorate of the
misconduct committed by an elective
official during his previous term.
Petitioner further cites the ruling of
this Court in Pascual v. Hon. Provincial
Board of Nueva Ecija (106 Phil. 466,
472 [1959], citing Conant v. Brogan, 6
N.Y.S.R. 332 [1887], cited in 17 A.L.R.
281, 63, So. 559, 50 LRA [NS] 553),
that
x x x When the people have elected
a man to office, it must be assumed
that they did this with knowledge of
his life and character, and that they
disregarded or forgave his faults or
misconduct, if he had been guilty of
any. It is not for the court, by reason
of such faults or misconduct to
practically overrule the will of the
people.
Respondents, on the other hand,
contend that while the contract in
question was signed during the
previous term of petitioner, it was to
commence or be effective only on
September 1998 or during his current
term.
It is the respondents
submission that petitioner went
beyond the protective confines of
jurisprudence when he agreed to
extend his act to his current term of
office.
Aguinaldo cannot apply,
according to respondents, because
what is involved in this case is a

misconduct
committed
during
a
previous term but to be effective
during the current term.
Respondents maintain that,
x x x petitioner performed two acts
with respect to the contract: he
provided for a suspensive period
making the supply contract commence
or be effective during his succeeding
or current term and during his current
term of office he acceded to the
suspensive period making the contract
effective during his current term by
causing the implementation of the
contract.
Hence, petitioner cannot take
refuge in the fact of his reelection,
according to respondents.
Further, respondents point out
that the contract in question was
signed just four days before the date
of the 1998 election and so it could
not be presumed that when the people
of Cebu City voted petitioner to office,
they did so with full knowledge of
petitioners character.
On
this
point,
petitioner
responds that knowledge of an
officials previous acts is presumed
and the court need not inquire
whether, in reelecting him, the
electorate was actually aware of his
prior misdeeds.
Petitioner cites our ruling in
Salalima v. Guingona (257 SCRA 55
[1996]), wherein we absolved Albay
governor Ramon R. Salalima of his
administrative liability as regards a
retainer agreement he signed in favor
of a law firm during his previous term,
although disbursements of public
funds to cover payments under the
agreement were still being done
during his subsequent term. Petitioner
argues that, following Salalima, the
doctrine of Aguinaldo applies even
where the effects of the acts
complained of are still evident during
the subsequent term of the reelected
official. The implementation of the
contract is a mere incident of its
execution.
Besides, according to
petitioner, the sole act for which he
has been administratively charged is
the signing of the contract with F.E.
Zuellig.
The charge, in his view,
excludes the contracts execution or
implementation,
or
any
act
subsequent to the perfection of the
contract.

In Salalima, we recall that the


Solicitor General maintained that
Aguinaldo did not apply to that case
because the administrative case
against Governor Rodolfo Aguinaldo of
Cagayan was already pending when
he filed his certificate of candidacy for
his reelection bid. Nevertheless, in
Salalima, the Court applied the
Aguinaldo doctrine, even if the
administrative case against Governor
Salalima was filed after his reelection.
Xxx
We now come to the concluding
inquiry. Granting that the Office of the
Ombudsman may investigate, for
purposes provided for by law, the acts
of petitioner committed prior to his
present term of office; and that it may
preventively suspend him for a
reasonable period, can that office hold
him administratively liable for said
acts?
In a number of cases, we have
repeatedly held that a reelected local
official
may
not
be
held
administratively
accountable
for
misconduct committed during his prior
term of office (Pascual v. Hon.
Provincial Board of Nueva Ecija, 106
Phil. 466 [1959]; others omitted). The
rationale for this holding is that when
the electorate put him back into office,
it is resumed that it did so with full
knowledge of his life and character,
including his past misconduct.
If,
armed with such knowledge, it still
reelects him, then such reelection is
considered a condonation of his past
misdeeds.
However, in the present case,
respondents point out that the
contract entered into by petitioner
with F.E. Zuellig was signed just four
days before the date of the elections.
It was not made an issue during the
election, and so the electorate could
not be said to have voted for
petitioner with knowledge of this
particular aspect of his life and
character.
For his part, petitioner contends
that the only conclusive determining
factor as regards the peoples
thinking on the matter is an election.
On this point we agree with petitioner.
That the people voted for an official
with knowledge of his character is
presumed, precisely to eliminate the
need to determine, in factual terms,
the extent of this knowledge. Such an
undertaking
will
obviously
be

impossible. Our rulings on the matter


do not distinguish the precise timing
or period when the misconduct was
committed, reckoned from the date of
the officials reelection, except that it
must be prior to said date.

contract
was
signed,
during
petitioners prior term.
At that
moment, petitioner already acceded to
the terms of the contract, including
stipulations now alleged to be
prejudicial to the city government.
Thus, any culpability petitioner may
have in signing the contract already
became extant on the day the
contract was signed. It hardly matters
that the deliveries under the contract
are supposed to have been made
months later.

As held in Salalima,
The rule adopted in Pascual, qualified
in Aguinaldo insofar as criminal cases
are concerned, is still a good law.
Such a rule is not only founded on the
theory that an officials reelection
expresses the sovereign will of the
electorate to forgive or condone any
act or omission constituting a ground
for administrative discipline which was
committed during his previous term.
We may add that sound policy dictates
it. To rule otherwise would open the
floodgates to exacerbating endless
partisan
contests
between
the
reelected official and his political
enemies, who may not stop to hound
the former during his new term with
administrative cases for acts alleged
to have been committed during his
previous term. His second term may
thus be devoted to defending himself
in the said cases to the detriment of
public service x x x. (Emphasis
added.) (Salalima v. Guingona, supra
at 115)

While petitioner can no longer


be held administratively liable for
signing the contract with F.E. Zuellig,
however, this should not prejudice the
filing of any case other than
administrative against petitioner. Our
ruling in this case, may not be taken
to mean the total exoneration of
petitioner for whatever wrongdoing, if
any, might have been committed in
signing the subject contract.
The
ruling now is limited to the question of
whether or not he may be held
administratively liable therefor, and it
is our considered view that he may
not. (Garcia v. Mojica, 314 SCRA
207, Sept. 10, 1999, 2nd Div.
[Quisumbing])
284. What are the situations covered
by the law on nepotism?

The above ruling in Salalima


applies to this case. Petitioner cannot
anymore be held administratively
liable for an act done during his
previous term, that is, his signing of
the contract with F.E. Zuellig.
The assailed retainer agreement
in Salalima was executed sometime in
1990.
Governor Salalima was
reelected in 1992 and payments for
the retainer continued to be made
during his succeeding term.
This
situation is no different from the one in
the present case, wherein deliveries of
the asphalt under the contract with
F.E. Zuellig and the payments therefor
were supposed to have commenced
on
September
1998,
during
petitioners second term.
However, respondents argue
that the contract, although signed on
May 7, 1998, during petitioners prior
term, is to be made effective only
during his present term.
We fail to see any difference to
justify a valid distinction in the result.
The agreement between petitioner
(representing Cebu City) and F.E.
Zuellig was perfected on the date the

Held:
Under the definition of
nepotism, one is guilty of nepotism if
an appointment is issued in favor of a
relative within the third civil degree of
consanguinity or affinity of any of the
following:
a)
b)
c)

appointing authority;
recommending authority;
chief of the bureau or office;
and

d)

person exercising immediate


supervision over the appointee.
Clearly, there are four situations
covered. In the last two mentioned
situations, it is immaterial who the
appointing or recommending authority
is. To constitute a violation of the law,
it suffices that an appointment is
extended or issued in favor of a
relative within the third civil degree of
consanguinity or affinity of the chief of
the bureau or office, or the person
exercising immediate supervision over
the appointee. (CSC v. Pedro O.
Dacoycoy, G.R. No. 135805, April
29, 1999, En Banc [Pardo])

285. What are the exemptions from


the operation of the rules on
nepotism?
Ans.:
The following are exempted
from the operation of the rules on
nepotism: (a) persons employed in a
confidential capacity, (b) teachers, (c)
physicians, and (d) members of the
Armed Forces of the Philippines.
The rules on nepotism shall
likewise not be applicable to the case
of a member of any family who, after
his or her appointment to any position
in an office or bureau, contracts
marriage with someone in the same
office or bureau, in which event the
employment or retention therein of
both husband and wife may be
allowed. (Sec. 59, Chap. 7, Subtitle
A, Title I, Bk. V, E.O. No. 292)
286. Distinguish term of office from
tenure of the incumbent.
Held: In the law of public officers,
there is a settled distinction between
term and tenure. [T]he term of
an office must be distinguished from
the tenure of the incumbent.
The
term means the time during which the
officer may claim to hold office as of
right, and fixes the interval after which
the several incumbents shall succeed
one another. The tenure represents
the term during which the incumbent
actually holds the office. The term of
office is not affected by the hold-over.
The tenure may be shorter than the
term for reasons within or beyond the
power of the incumbent. (Thelma P.
Gaminde v. COA, G.R. No. 140335,
Dec. 13, 2000, En Banc [Pardo])
287. Discuss the operation of the
rotational plan insofar as the term of
office of the Chairman and Members of
the Constitutional Commissions is
concerned.
Held: In Republic v. Imperial (96 Phil.
770 [1955]), we said that the
operation of the rotational plan
requires
two
conditions,
both
indispensable to its workability: (1)
that the terms of the first three (3)
Commissioners should start on a
common date, and (2) that any
vacancy due to death, resignation or
disability before the expiration of the
term should only be filled only for the
unexpired balance of the term.
Consequently, the terms of the first
Chairmen and Commissioners of the
Constitutional Commissions under the

1987 Constitution must start on a


common date, irrespective of the
variations
in
the
dates
of
appointments and qualifications of the
appointees,
in
order
that
the
expiration of the first terms of seven,
five and three years should lead to the
regular recurrence of the two-year
interval between the expiration of the
terms.
Applying the foregoing conditions x x
x, we rule that the appropriate starting
point of the terms of office of the first
appointees to the Constitutional
Commissions
under
the
1987
Constitution must be on February 2,
1987, the date of the adoption of the
1987 Constitution.
In case of a
belated appointment or qualification,
the interval between the start of the
term and the actual qualification of
the appointee must be counted
against the latter.
(Thelma P.
Gaminde v. COA, G.R. No. 140335,
Dec. 13, 2000, En Banc [Pardo])
288. What is the hold-over doctrine?
What is its purpose?
Held: 1. The concept of holdover
when applied to a public officer
implies that the office has a fixed term
and the incumbent is holding onto the
succeeding term.
It is usually
provided by law that officers elected
or appointed for a fixed term shall
remain in office not only for that term
but until their successors have been
elected and qualified.
Where this
provision is found, the office does not
become vacant upon the expiration of
the term if there is no successor
elected and qualified to assume it, but
the present incumbent will carry over
until his successor is elected and
qualified, even though it be beyond
the term fixed by law.
Absent
an
express
or
implied
constitutional or statutory provision to
the contrary, an officer is entitled to
stay in office until his successor is
appointed or chosen and has qualified.
The legislative intent of not allowing
holdover must be clearly expressed or
at least implied in the legislative
enactment, otherwise it is reasonable
to assume that the law-making body
favors the same.
Indeed, the law abhors a vacuum in
public offices, and courts generally
indulge in the strong presumption
against a legislative intent to create,
by statute, a condition which may
result
in
an
executive
or

administrative office becoming, for


any period of time, wholly vacant or
unoccupied by one lawfully authorized
to exercise its functions.
This is
founded on obvious considerations of
public policy, for the principle of
holdover is specifically intended to
prevent public convenience from
suffering because of a vacancy and to
avoid a hiatus in the performance of
government functions. (Lecaroz v.
Sandiganbayan, 305 SCRA 397,
March
25,
1999,
2nd
Div.
[Bellosillo])
2.
The rule is settled that unless
holding over be expressly or impliedly
prohibited,
the
incumbent
may
continue to hold over until someone
else is elected and qualified to assume
the office. This rule is demanded by
the most obvious requirements of
public policy, for without it there must
frequently be cases where, from a
failure to elect or a refusal or neglect
to qualify, the office would be vacant
and the public service entirely
suspended.
Otherwise stated, the
purpose is to prevent a hiatus in the
government pending the time when
the successor may be chosen and
inducted into office.
(Galarosa v.
Valencia, 227 SCRA 728, Nov. 11,
1993, En Banc [Davide, Jr.])
289. What is resignation? What are
the requisites of a valid resignation?
Held: 1. It is the act of giving up or
the act of an officer by which he
declines his office and renounces the
further right to use it.
It is an
expression of the incumbent in some
form, express or implied, of the
intention to surrender, renounce, and
relinquish
the
office
and
the
acceptance by competent and lawful
authority. To constitute a complete
and operative resignation from public
office, there must be: (a) an intention
to relinquish a part of the term; (b) an
act of relinquishment; and (c) an
acceptance by the proper authority.
The last one is required by reason of
Article 238 of the Revised Penal Code.
(Sangguniang
Bayan
of
San
Andres, Catanduanes v. CA, 284
SCRA 276, Jan. 16, 1998)
2.
Resignation x x x is a factual
question and its elements are beyond
quibble: there must be an intent to
resign and the intent must be coupled
by acts of relinquishment (Gonzales v.
Hernandez, 2 SCRA 228 [1961]). The
validity of a resignation is not
governed by any formal requirement

as to form. It can be oral. It can be


written. It can be express. It can be
implied. As long as the resignation is
clear, it must be given legal effect.
(Estrada v. Desierto, G.R. Nos.
146710-15, March 2, 2001, en
Banc [Puno])
290. What is abandonment of an
office? What are its requisites? How
is it distinguished from resignation?
Held: Abandonment of an office has
been defined as the voluntary
relinquishment of an office by the
holder,
with
the
intention
of
terminating his possession and control
thereof.
Indeed, abandonment of
office is a species of resignation; while
resignation in general is a formal
relinquishment, abandonment is a
voluntary
relinquishment
through
nonuser.
Abandonment springs from and is
accompanied by deliberation and
freedom of choice. Its concomitant
effect is that the former holder of an
office can no longer legally repossess
it even by forcible reoccupancy.
Clear intention to abandon should be
manifested by the officer concerned.
Such intention may be express or
inferred from his own conduct. Thus,
the failure to perform the duties
pertaining to the office must be with
the officers actual or imputed
intention to abandon and relinquish
the office. Abandonment of an office
is not wholly a matter of intention; it
results from a complete abandonment
of duties of such continuance that the
law will infer a relinquishment.
Therefore, there are two essential
elements of abandonment; first, an
intention to abandon and, second, an
overt or external act by which the
intention is carried into effect.
(Sangguniang
Bayan
of
San
Andres, Catanduanes v. CA, 284
SCRA 276, Jan. 16, 1998)
291. Petitioner claims that Benipayo
has no authority to remove her as
Director IV of the EID and reassign her
to the Law Department. Petitioner
further argues that only the COMELEC,
acting as a collegial body, can
authorize such reappointment.
Moreover, petitioner maintains that a
reassignment without her consent
amounts to removal from office

without due process and therefore


illegal.
Held: Petitioners posturing will
hold water if Benipayo does not
possess any color of title to the office
of Chairman of the COMELEC. We
have ruled, however, that Benipayo is
the de jure COMELEC Chairman, and
consequently he has full authority to
exercise all the powers of that office
for so long as his ad interim
appointment remains effective. X x x.
The Chairman, as the Chief Executive
of the COMELEC, is expressly
empowered on his own authority to
transfer or reassign COMELEC
personnel in accordance with the Civil
Service Law. In the exercise of this
power, the Chairman is not required
by law to secure the approval of the
COMELEC en banc.
Petitioners appointment papers
x x x indisputably show that she held
her Director IV position in the EID only
in an acting or temporary capacity.
Petitioner is not a Career Executive
Service (CES), and neither does she
hold Career Executive Service
Eligibility, which are necessary
qualifications for holding the position
of Director IV as prescribed in the
Qualifications Standards (Revised
1987) issued by the Civil Service
Commission. Obviously, petitioner
does not enjoy security of tenure as
Director IV. X x x
Xxx
Having been appointed merely
in a temporary or acting capacity, and
not possessed of the necessary
qualifications to hold the position of
Director IV, petitioner has no legal
basis in claiming that her
reassignment was contrary to the Civil
Service Law. X x x
Still, petitioner assails her
reassignment, carried out during the
election period, as a prohibited act
under Section 261 (h) of the Omnibus
Election Code x x x.

Xxx
Petitioner claims that Benipayo failed
to secure the approval of the
COMELEC en banc to effect transfers
or reassignments of COMELEC
personnel during the election period.
Moreover, petitioner insists that the
COMELEC en banc must concur to
every transfer or reassignment of
COMELEC personnel during the
election period.
Contrary to petitioners allegation, the
COMELEC did in fact issue COMELEC
Resolution No. 3300 dated November
6, 2000, exempting the COMELEC from
Section 261 (h) of the Omnibus
Election Code. X x x
Xxx
The proviso in COMELEC Resolution
No. 3300, requiring due notice and
hearing before any transfer or
reassignment can be made within
thirty days prior to election day, refers
only to COMELEC field personnel and
not to head office personnel like the
petitioner. Under the Revised
Administrative Code (see Section 7
[4], Chapter 2, Subtitle C, Book V of
the Revised Administrative Code), the
COMELEC Chairman is the sole officer
specifically vested with the power to
transfer or reassign COMELEC
personnel. The COMELEC Chairman
will logically exercise the authority to
transfer or reassign COMELEC
personnel pursuant to COMELEC
Resolution No. 3300. The COMELEC en
banc cannot arrogate unto itself this
power because that will mean
amending the Revised Administrative
Code, an act the COMELEC en banc
cannot legally do.
COMELEC Resolution No. 3300 does
not require that every transfer or
reassignment of COMELEC personnel
should carry the concurrence of the
COMELEC as a collegial body.
Interpreting Resolution No. 3300 to
require such concurrence will render
the resolution meaningless since the
COMELEC en banc will have to

approve every personnel transfer or


reassignment, making the resolution
utterly useless. Resolution No. 3300
should be interpreted for what it is, an
approval to effect transfers and
reassignments of personnel, without
need of securing a second approval
from the COMELEC en banc to actually
implement such transfer or
reassignment.
The COMELEC Chairman is the official
expressly authorized by law to transfer
or reassign COMELEC personnel. The
person holding that office, in a de jure
capacity, is Benipayo. The COMELEC
en banc, in COMELEC Resolution No.
3300, approved the transfer or
reassignment of COMELEC personnel
during the election period. Thus,
Benipayos order reassigning
petitioner from the EID to the Law
Department does not violate Section
261 (h) of the Omnibus Election Code.
For the same reason, Benipayos order
designating Cinco Officer-in-Charge of
the EID is legally unassailable.
(Matibag v. Benipayo, 380 SCRA
49, April 2, 2002, En Banc
[Carpio])
292. Is a government employee who
has been ordered arrested and
detained for a non-bailable offense
and for which he was suspended for
his inability to report for work until the
termination of his case, still required
to file a formal application for leave of
absence to ensure his reinstatement
upon his acquittal and thus protect his
security of tenure? Concomitantly, will
his prolonged absence from office for
more than one (1) year automatically
justify his being dropped from the rolls
without prior notice despite his being
allegedly placed under suspension by
his employer until the termination of
his case, which finally resulted in his
acquittal for lack of evidence?
EUSEBIA R. GALZOTE was
employed as a lowly clerk in the
service of the City Government of
Makati City. With her meager income
she was the lone provider for her
children. But her simple life was

disrupted abruptly when she was


arrested without warrant and detained
for more than three (3) years for a
crime she did not commit. Throughout
her ordeal she trusted the city
government that the suspension
imposed on her was only until the final
disposition of her case. As she drew
near her vindication she never did
expect the worst to come to her. On
the third year of her detention the city
government lifted her suspension,
dropped her from the rolls without
prior notice and without her
knowledge, much less gave her an
opportunity to forthwith correct the
omission of an application for leave of
absence belatedly laid on her.
Upon her acquittal for lack of
evidence and her release from
detention she was denied
reinstatement to her position. She
was forced to seek recourse in the
Civil Service Commission which
ordered her immediate reinstatement
with back wages from 19 October
1994, the date when she presented
herself for reassumption of duties but
was turned back by the city
government, up to the time of her
actual reinstatement.
Xxx
Plainly, the case of petitioner
City Government of Makati City
revolves around a rotunda of doubt, a
dilemma concerning the legal status
and implications of its suspension of
private respondent Eusebia R. Galzote
and the automatic leave of absence
espoused by the Civil Service
Commission. Against this concern is
the punctilious adherence to
technicality, the requirement that
private respondent should have filed
an application for leave of absence in
proper form. The instant case is
therefore a dispute between, at its
worst, private respondents substantial
compliance with the standing rules,
and the City Governments insistence
that the lowly clerk should have still
gone through the formalities of
applying for leave despite her

detention, of which petitioner had


actual notice, and the suspension
order couched in simple language that
she was being suspended until the
final disposition of her criminal case.
The meaning of suspension until
the final disposition of her case is that
should her case be dismissed she
should be reinstated to her position
with payment of back wages. She did
not have to apply for leave of absence
since she was already suspended by
her employer until her case would be
terminated. We have done justice to
the workingman in the past; today we
will do no less by resolving all doubts
in favor of the humble employee in
faithful obeisance to the constitutional
mandate to afford full protection to
labor (Const., Art. XIII, Sec. 3, par. 1;
Art. II, Sec. 18)
Xxx
As may be gleaned from the
pleadings of the parties, the issues
are: (1) whether private respondent
Eusebia R. Galzote may be considered
absent without leave; (b) whether due
process had been observed before she
was dropped from the rolls; and, (3)
whether she may be deemed to have
abandoned her position, hence, not
entitled to reinstatement with back
salaries for not having filed a formal
application for leave. Encapsulated,
the issues may be reduced to whether
private respondent may be considered
absent without leave or whether she
abandoned her job as to justify being
dropped from the service for not filing
a formal application for leave.
Petitioner would have private
respondent declared on AWOL and
faults her for failing to file an
application for leave of absence under
Secs. 20 (Now Sec. 52 of Rule XVI,
Leave of Absence, of Res. No. 91-1631
dated 27 December 1991, as
amended by CSC MC No. 41, s. 1998)
and 35 (Now Sec. 63 of Rule XVI,
Leave of Absence, of Res. No. 91-1631
dated 27 December 1991, as
amended by CSC MC Nos. 41, s. 1998

and 14, s. 1999) of the CSC Rules and


rejects the CSCs ruling of an
automatic leave of absence for the
period of her detention since the
current Civil Service Law and Rules
do not contain any specific provision
on automatic leave of absence.
The Court believes that private
respondent cannot be faulted for
failing to file prior to her detention an
application for leave and obtain
approval thereof. The records clearly
show that she had been advised three
(3) days after her arrest, or on 9
September 1991, that petitioner City
government of Makati City had placed
her under suspension until the final
disposition of her criminal case. This
act of petitioner indubitably
recognized private respondents
predicament and thus allowed her to
forego reporting for work during the
pendency of her criminal case without
the needless exercise of strict
formalities. At the very least, this
official communication should be
taken as an equivalent of a prior
approved leave of absence since it
was her employer itself which placed
her under suspension and thus
excused her from further formalities in
applying for such leave. Moreover, the
arrangement bound the City
Government to allow private
respondent to return to her work after
the termination of her case, i.e., if
acquitted of the criminal charge. This
pledge sufficiently served as
legitimate reason for her to altogether
dispense with the formal application
for leave; there was no reason to, as in
fact it was not required, since she was
for all practical purposes incapacitated
or disabled to do so.
Indeed, private respondent did not
have the least intention to go on
AWOL from her post as Clerk III of
petitioner, for AWOL means the
employee leaving or abandoning his
post without justifiable reason and
without notifying his employer. In the
instant case, private respondent had a
valid reason for failing to report for
work as she was detained without bail.

Hence, right after her release from


detention, and when finally able to do
so, she presented herself to the
Municipal Personnel Officer of
petitioner City Government to report
for work. Certainly, had she been told
that it was still necessary for her to file
an application for leave despite the 9
September 1991 assurance from
petitioner, private respondent would
have lost no time in filing such piece
of document. But the situation
momentarily suspending her from
work persisted: petitioner City
Government did not alter the modus
vivendi with private respondent and
lulled her into believing that its
commitment that her suspension was
only until the termination of her case
was true and reliable. Under the
circumstances private respondent was
in, prudence would have dictated
petitioner, more particularly the
incumbent city executive, in patria
potestas, to advise her that it was still
necessary although indeed
unnecessary and a useless ceremony
to file such application despite the
suspension order, before depriving her
of her legitimate right to return to her
position. Patria potestas in piatate
debet, non in atrocitate, consistere.
Paternal power should consist or be
exercised in affection, not in atrocity.
It is clear from the records that private
respondent Galzote was arrested and
detained without a warrant on 6
September 1991 for which reason she
and her co-accused were subjected
immediately to inquest proceedings.
This fact is evident from the instant
petition itself and its attachments x x
x. Hence, her ordeal in jail began on 6
September 1991 and ended only after
her acquittal, thus leaving her no time
to attend to the formality of filing a
leave of absence.
But petitioner City Government
would unceremoniously set aside its 9
September 1991 suspension order
claiming that it was superseded three
(3) years later by a memorandum
dropping her from the rolls effective
21 January 1993 for absence for more

than one (1) year without official


leave. Hence, the suspension order
was void since there was no pending
administrative charge against private
respondent so that she was not
excused from filing an application for
leave.
We do not agree. In placing
private respondent under suspension
until the final disposition of her
criminal case, the Municipal Personnel
Officer acted with competence, so he
presumably knew that his order of
suspension was not akin to either
suspension as penalty or preventive
suspension since there was no
administrative case against private
respondent. As competence on the
part of the MPO is presumed, any error
on his part should not prejudice
private respondent, and that what he
had in mind was to consider her as
being on leave of absence without pay
and their employer-employee
relationship being merely suspended,
not severed, in the meantime. This
construction of the order of
suspension is actually more consistent
with logic as well as fairness and
kindness to its author, the MPO.
Significantly, the idea of a suspended
employer-employee relationship is
widely accepted in labor law to
account for situations wherein laborers
would have no work to perform for
causes not attributable to them (see
e.g., Visayan Stevedore Transportation
Company v. Court of Industrial
Relations, No. L-21696, 25 February
1967, 19 SCRA 426; Tomas Lao
Construction v. NLRC, G.R. No. 116781,
5 September 1997, 278 SCRA 716).
We find no basis for denying the
application of this principle to the
instant case which also involves a
lowly worker in the public service.
Moreover, we certainly cannot
nullify the City Governments order of
suspension, as we have no reason to
do so, much less retroactively apply
such nullification to deprive private
respondent of a compelling and valid
reason for not filing the leave
application. For as we have held, a

void act though in law a mere scrap of


paper nonetheless confers legitimacy
upon past acts or omissions done in
reliance thereof (De Agbayani v.
Philippine National Bank, G.R. No.
231127, 29 April 1971, 38 SCRA 429;
Municipality of Malabang v. Benito, G.R
No. 28113, 28 March 1969, 27 SCRA
545). Consequently, the existence of
a statute or executive order prior to its
being adjudged void is an operative
fact to which legal consequences are
attached (De Agbayani, supra, p. 435).
It would indeed be ghastly unfair to
prevent private respondent from
relying upon the order of suspension in
lieu of a formal leave application.
At any rate, statements are, or
should be, construed against the one
responsible for the confusion;
otherwise stated, petitioner must
assume full responsibility for the
consequences of its own act, hence,
he should be made to answer for the
mix-up of private respondent as
regards the leave application. At the
very least, it should be considered
estopped from claiming that its order
of suspension is void or that it did not
excuse private respondent from filing
an application for leave on account of
her incarceration. It is a fact that she
relied upon this order, issued barely
three (3) days from the date of her
arrest, and assumed that when the
criminal case would be settled she
could return to work without need of
any prior act. X x x
Xxx
The holding of the Civil Service
Commission that private respondent
was on automatic leave of absence
during the period of her detention
must be sustained. The CSC is the
constitutionally mandated central
personnel agency of the Government
tasked to establish a career service
and adopt measures to promote
morale, efficiency, integrity,
responsiveness, progressiveness and
courtesy in the civil service (Const.,
Art. IX-B, Sec. 3) and strengthen the
merit and rewards system, integrate

all human resources development


programs for all levels and ranks, and
institutionalize a management climate
conducive to public accountability.
(Ibid.) Besides, the Administrative
Code of 1987 further empowers the
CSC to prescribe, amend, and enforce
rules and regulations for carrying into
effect the provisions of the Civil
Service Law and other pertinent laws,
(Bk. V, I (A), Ch. 3, Sec. 12) and for
matters concerning leaves of absence,
the Code specifically vests the CSC to
ordain
Sec. 60. Leave of absence. Officers
and employees in the Civil Service
shall be entitled to leave of absence,
with or without pay, as may be
provided by law and the rules and
regulations of the Civil Service
Commission in the interest of the
service.
Pursuant thereto the CSC
promulgated Resolution No. 91-1631
dated 27 December 1991 entitled
Rules Implementing Book V of
Executive Order No. 292 and Other
Pertinent Civil Service Laws which it
has several times amended through
memorandum circulars. It devotes
Rule XVI to leaves of absence.
Petitioner City Government relies upon
Secs. 20 and 35 to debunk the CSC
ruling of an automatic leave of
absence. Significantly, these
provisions have been amended so that
Sec. 20 of the Civil Service Rules is
now Sec. 52 of Rule XVI, on Leave of
Absence, of Resolution No. 91-1631
dated 27 December 1991 as amended
by CSC MC No. 41, s. 1998, and Sec.
35 is now Sec. 63 as amended by CSC
MC Nos. 41, s. 1998 and 14, s. 1999.
Xxx
As a general rule, Secs. 20 and
52, as well as Secs. 35 and 63, require
an approved leave of absence to avoid
being an AWOL. However, these
provisions cannot be interpreted as
exclusive and referring only to one
mode of securing the approval of a
leave of absence which would require

an employee to apply for it, formalities


and all, before exceeding thirty (30)
days of absence in order to avoid from
being dropped from the rolls. There
are, after all, other means of seeking
and granting an approved leave of
absence, one of which is the CSC
recognized rule of automatic leave of
absence under specified
circumstances. X x x
Xxx
As properly noted, the CSC was
only interpreting its own rules on leave
of absence and not a statutory
provision (As a matter of fact, Sec. 60
of the Administrative Code does not
provide for any rule on leave of
absence other than that civil servants
are entitled to leave of absence) in
coming up with this uniform rule.
Undoubtedly, the CSC like any other
agency has the power to interpret its
own rules and any phrase contained in
them (Norwegian Nitrogen Products
Co. v. United States of America, 288
SCRA 294, 325, 77 Led. 796, 812
[1933]) with its interpretation
significantly becoming part of the
rules themselves. X x x
Xxx
Under RA 6656 (An Act to
Protect the Security of Tenure of Civil
Service Officers and Employees in the
Implementation of Government
Reorganization) and RA 7160 (The
Local Government Code of 1991), civil
servants who are found illegally
dismissed or retrenched are entitled to
full pay for the period of their
separation.
Our final point. An efficient and
honest bureaucracy is never
inconsistent with the emphasis on and
the recognition of the basic rights and
privileges of our civil servants or, for
that matter, the constitutional
mandates of the Civil Service
Commission. In fact only from an
enlightened corps of government
workers and an effective CSC grows
the professionalization of the

bureaucracy. Indeed the government


cannot be left in the lurch; but neither
could we decree that government
personnel be separated from their jobs
indiscriminately regardless of fault.
The fine line between these concerns
may be difficult to clearly draw but if
we only exerted extra effort to rebel
against the allure of legal oversimplification, justice would have been
done where it is truly due. (City
Government of Makati City v. Civil
Service Commission, 376 SCRA
248, Feb. 6, 2002, En Banc
[Bellosillo])
293. What is abandonment of office?
What are its essential elements?
Held: Abandonment of an office is
the voluntary relinquishment of an
office by the holder, with the intention
of terminating his possession and
control thereof (Sangguniang Bayan of
San Andres, Catanduanes v. Court of
Appeals, 284 SCRA 276 [1998]). In
order to constitute abandonment of an
office, it must be total and under such
circumstances as clearly to indicate an
absolute relinquishment (Airoso v. De
Guzman, 49 Phil. 371 [1926]). There
must be a complete abandonment of
duties of such continuance that the
law will infer a relinquishment (67
C.J.S. Officers Sec. 100, citing Cosby v.
Moore, 65 So.2d 178, 259 Ala. 41).
Abandonment of duties is a voluntary
act (Ibid., citing Steingruber v. San
Antonio, Comm. App., 220 S.W. 77,
78);
it
springs
from
and
is
accompanied by deliberation and
freedom of choice (Jorge v. Mayor, 10
SCRA 331 [1964], citing Teves v.
Sindiong, 81 Ohil. 658 [1948]). There
are, therefore, two essential elements
of abandonment: first, an intention to
abandon and second, an overt or
external act by which the intention
is carried into effect (67 C.J.S. Officers
Sec. 100, citing Rainwater v. State ex
rel. Strickland, 178 So. 484, 237 Ala.
482, 121 A.L.R. 981)
Generally speaking, a person holding a
public office may abandon such office
by nonuser or acquiescence (Ibid.,
citing Herbert v. State Oil and Gas Bd.,
250 So.2d 597, 287 Ala. 221). Nonuser refers to a neglect to use a right
or privilege or to exercise an office
(Sangguniang Bayan of San Andres,
Catanduanes v. Court of Appeals,
supra). However, nonperformance of
the duties of an office does not

constitute abandonment where such


nonperformance
results
from
temporary
disability
or
from
involuntary failure to perform (67
C.J.S. Sec. 100, citing Doris v. Heroux,
47
A.2d
633,
71
R.I.
491).
Abandonment may also result from an
acquiescence by the officer in his
wrongful removal or discharge, for
instance, after a summary removal, an
unreasonable delay by an officer
illegally removed in taking steps to
vindicate his rights may constitute an
abandonment of the office (Ibid., citing
Nicholas v. U.S., Ct. Cl., 42 S.Ct. 7, 257
U.S. 71, 66 L. Ed. 133). Where, while
desiring and intending to hold the
office, and with no willful desire or
intention to abandon it, the public
officer vacates it in deference to the
requirements of a statute which is
afterwards declared unconstitutional,
such a surrender will not be deemed
an abandonment and the officer may
recover the effect.
(Mechem, A
Treatise on the Law of Public Offices
and Officers, 1890 edition, p. 279,
citing Turnipseed v. Hudson, 50 Miss.
429, 19 Am. Rep. 15) (Canonizado v.
Aguirre, 351 SCRA 659, 665-668,
Feb. 15, 2001, En Banc [GonzagaReyes])
294. By accepting another position in
the government during the pendency
of a case brought precisely to assail
the constitutionality of his removal may a person be deemed to have
abandoned
his
claim
for
reinstatement?
Held:
Although petitioners do not
deny the appointment of Canonizado
as Inspector General, they maintain
that
Canonizados
initiation
and
tenacious pursuance of the present
case would belie any intention to
abandon his former office. Petitioners
assert that Canonizado should not be
faulted
for
seeking
gainful
employment during the pendency of
this case.
Furthermore, petitioners
point out that from the time
Canonizado
assumed
office
as
Inspector General he never received
the salary pertaining to such position x
x x.
Xxx
By accepting the position of
Inspector
General
during
the
pendency of the present case
brought precisely to assail the
constitutionality of his removal from
the NAPOLCOM Canonizado cannot
be deemed to have abandoned his

claim for reinstatement to the latter


position. First of all, Canonizado did
not voluntarily leave his post as
Commissioner, but was compelled to
do so on the strength of Section 8 of
RA 8551 x x x
In our decision of 25 January
2000, we struck down the
abovequoted provision for being
violative of petitioners constitutionally
guaranteed right to security of tenure.
Thus, Canonizado harbored no willful
desire or intention to abandon his
official duties. In fact, Canonizado,
together with petitioners x x x lost no
time disputing what they perceived to
be an illegal removal; a few weeks
after RA 8551 took effect x x x
petitioners instituted the current
action x x x assailing the
constitutionality of certain provisions
of said law. The removal of petitioners
from their positions by virtue of a
constitutionally infirm act necessarily
negates a finding of voluntary
relinquishment. (Canonizado v.
Aguirre, 351 SCRA 659, 665-668,
Feb. 15, 2001, En Banc [GonzagaReyes])
295. What is the effect of acceptance
of an incompatible office to a claim for
reinstatement?
Held:
The next issue is whether
Canonizados appointment to and
acceptance of the position of Inspector
General
should
result
in
an
abandonment
of
his
claim
for
reinstatement to the NAPOLCOM. It is
a well-settled rule that he who, while
occupying one office, accepts another
incompatible with the first, ipso facto
vacates the first office and his title is
thereby terminated without any other
act or proceeding (Mechem, A Treatise
on the Law of Public Offices and
Officers, 1890 edition, p. 267). Public
policy considerations dictate against
allowing the same individual to
perform inconsistent and incompatible
duties (Ibid.).
The incompatibility
contemplated is not the mere physical
impossibility
of
one
persons
performing the duties of the two
offices due to a lack of time or the
inability to be in two places at the
same moment, but that which
proceeds from the nature and relations
of the two positions to each other as
to give rise to contrariety and

antagonism
should
one
person
attempt to faithfully and impartially
discharge the duties of one toward the
incumbent of the other. (Ibid.)
There is no question that the
positions of NAPOLCOM Commissioner
and Inspector General of the IAS are
incompatible with each other. As
pointed out by respondents, RA 8551
prohibits any personnel of the IAS from
sitting in a committee charged with
the task of deliberating on the
appointment, promotion, or
assignment of any PNP personnel,
whereas the NAPOLCOM has the
power of control and supervision over
the PNP. However, the rule on
incompatibility of duties will not apply
to the case at bar because at no point
did Canonizado discharge the
functions of the two offices
simultaneously. Canonizado was
forced out of his first office by the
enactment of Section 8 of RA 8551.
Thus, when Canonizado was appointed
as Inspector General x x x he had
ceased to discharge his official
functions as NAPOLCOM
Commissioner. X x x. Thus, to
reiterate, the incompatibility of duties
rule never had a chance to come into
play for petitioner never occupied the
two positions, of Commissioner and
Inspector General, nor discharged
their respective functions,
concurrently.
Xxx
As in the Tan (Tan v. Gimenez, 107
Phil. 17 [1960]) and Gonzales
(Gonzales v. Hernandez, 2 SCRA 228
[1961]) cases, Canonizado was
compelled to leave his position as
Commissioner, not by an erroneous
decision, but by an unconstitutional
provision of law. Canonizado, like the
petitioners in the above mentioned
cases, held a second office during the
period that his appeal was pending.
As stated in the Comment filed by
petitioners, Canonizado was impelled
to accept this subsequent position by
a desire to continue serving the
country, in whatever capacity. Surely,
this selfless and noble aspiration

deserves to be placed on at least


equal footing with the worthy goal of
providing for oneself and ones family,
either of which are sufficient to justify
Canonizados acceptance of the
position of Inspector General. A
Contrary ruling would deprive
petitioner of his right to live, which
contemplates not only a right to earn
a living, as held in previous cases, but
also a right to lead a useful and
productive life. Furthermore,
prohibiting Canonizado from accepting
a second position during the pendency
of his petition would be to unjustly
compel him to bear the consequences
of an unconstitutional act which under
no circumstance can be attributed to
him. However, before Canonizado can
re-assume his post as Commissioner,
he should first resign as Inspector
General of the IAS-PNP. (Canonizado
v. Aguirre, 351 SCRA 659, Feb. 15,
2001, En Banc [Gonzaga-Reyes])

296. May the appointment of a


person assuming a position in the civil
service
under
a
completed
appointment be validly recalled or
revoked?
Held: It has been held that upon the
issuance of an appointment and the
appointees assumption of the position
in the civil service, he acquires a
legal right which cannot be taken
away either by revocation of the
appointment or by removal except for
cause and with previous notice and
hearing. (Mauna v. Civil Service
Commission, 232 SCRA 388, 398
[1994]) Moreover, it is well-settled
that the person assuming a position in
the civil service under a completed
appointment acquires a legal, not just
an equitable, right to the position.
This right is protected not only by
statute, but by the Constitution as
well, which right cannot be taken away
by
either
revocation
of
the
appointment, or by removal, unless
there is valid cause to do so, provided
that there is previous notice and
hearing. (Aquino v. Civil Service
Commission, 208 SCRA 240, 248
[1992])
Petitioner admits that his very first
official act upon assuming the position
of town mayor was to issue Office
Order No. 95-01 which recalled the

appointments
of
the
private
respondents. There was no previous
notice, much less a hearing accorded
to the latter. Clearly, it was petitioner
who acted in undue haste to remove
the private
respondents without
regard for the simple requirements of
due process of law. While he argues
that the appointing power has the sole
authority to revoke said appointments,
there is no debate that he does not
have blanket authority to do so.
Neither can he question the CSCs
jurisdiction to affirm or revoke the
recall.
Rule V, Section 9 of the Omnibus
Implementing Regulations of the
Revised
Administrative
Code
specifically
provides
that
an
appointment
accepted
by
the
appointee cannot be withdrawn or
revoked by the appointing authority
and shall remain in force and in effect
until disapproved by the Commission.
Thus, it is the CSC that is authorized to
recall
an
appointment
initially
approved, but only when such
appointment and approval are proven
to be in disregard of applicable
provisions of the civil service law and
regulations (Debulgado v. Civil
Service Commission, 237 SCRA
184, 200 [1994]).
Moreover, Section 10 of the same rule
provides:
Sec. 10. An appointment issued in
accordance with pertinent laws and
rules shall take effect immediately
upon its issuance by the appointing
authority, and if the appointee has
assumed the duties of the position, he
shall be entitled to receive his salary
at once without awaiting the approval
of
his
appointment
by
the
Commission. The appointment shall
remain effective until disapproved by
the Commission. In no case shall an
appointment take effect earlier than
the date of its issuance.
Section 20
provides:

of

Rule

VI

also

Sec. 20. Notwithstanding the initial


approval of an appointment, the same
may be recalled on any of the
following grounds:
e
f

Non-compliance
with
the
procedures/criteria provided in the
agencys Merit Promotion Plan;
Failure to pass through the agencys
Selection/Promotion Board;

g
h

Violation of the existing collective


agreement between management and
employees relative to promotion; or
Violation of other existing civil service
law, rules and regulations.
Accordingly, the appointments of the
private respondents may only be
recalled on the above-cited grounds.
And yet, the only reason advanced by
the petitioner to justify the recall was
that
these
were
midnight
appointments.
The CSC correctly
ruled, however, that the constitutional
prohibition on so-called midnight
appointments,
specifically
those
made
within
two
(2)
months
immediately
prior
to
the
next
presidential elections, applies only to
the President or Acting President. (De
Rama v. Court of Appeals, 353
SCRA 94, Feb. 28, 2001, En Banc
[Ynares-Santiago])
297. When
may
unconsented
transfers be considered anathema to
security of tenure?
Held: As held in Sta. Maria v. Lopez
(31 SCRA 637, 653 citing Ibanez v.
Commission on Elections, L-26558,
April 27, 1967, 19 SCRA 1002, 1012
and Section 12 of the Tax Code).
"x x x
the rule that outlaws
unconsented transfers as anathema to
security of tenure applies only to an
officer who is appointed - not merely
assigned - to a particular station.
Such a rule does not pr[o]scribe a
transfer carried out under a specific
statute that empowers the head of an
agency to periodically reassign the
employees and officers in order to
improve the service of the agency. x x
x"
The guarantee of security of tenure
under the Constitution is not a
guarantee of perpetual employment.
It only means that an employee
cannot be dismissed (or transferred)
from the service for causes other than
those provided by law and after due
process is accorded the employee.
What it seeks to prevent is capricious
exercise of the power to dismiss. But
where it is the law-making authority
itself which furnishes the ground for
the transfer of a class of employees,
no such capriciousness can be raised
for so long as the remedy proposed to
cure a perceived evil is germane to
the purposes of the law. (Agripino A.
De Guzman, Jr., et al. v. COMELEC,
G.R. No. 129118, July 19, 2000, En
Banc [Purisima])

298.

Discuss Abolition of Office?

Held: The creation and abolition of


public offices is primarily a legislative
function.
It is acknowledged that
Congress may abolish any office it
creates without impairing the officer's
right to continue in the position held
and that such power may be exercised
for various reasons, such as the lack of
funds or in the interest of economy.
However, in order for the abolition to
be valid, it must be made in good
faith, not for political or personal
reasons, or in order to circumvent the
constitutional security of tenure of civil
service employees.
An abolition of office connotes an
intention to do away with such office
wholly and permanently, as the word
"abolished" denotes. Where one office
is abolished and replaced with another
office vested with similar functions,
the abolition is a legal nullity. Thus, in
U.P. Board of Regents v. Rasul (200
SCRA 685 [1991]) we said:
It is true that a valid and bona fide
abolition of an office denies to the
incumbent the right to security of
tenure (De la Llana v. Alba, 112 SCRA
294 [1982]). However, in this case,
the renaming and restructuring of the
PGH and its component units cannot
give rise to a valid and bona fide
abolition of the position of PGH
Director. This is because where the
abolished office and the offices
created in its place have similar
functions, the abolition lacks good
faith (Jose L. Guerrero v. Hon. Antonio
V. Arizabal, G.R. No. 81928, June 4,
1990, 186 SCRA 108 [1990]).
We
hereby apply the principle enunciated
in Cezar Z. Dario v. Hon. Salvador M.
Mison (176 SCRA 84 [1989]) that
abolition which merely changes the
nomenclature of positions is invalid
and does not result in the removal of
the incumbent.
The above notwithstanding, and
assuming that the abolition of the
position of the PGH Director and the
creation of a UP-PGH Medical Center
Director are valid, the removal of the
incumbent is still not justified for the
reason that the duties and functions of
the two positions are basically the
same.
This was also our ruling in Guerrero v.
Arizabal (186 SCRA 108 [1990]),
wherein
we
declared
that
the
substantial identity in the functions

between the two offices was indicia of


bad faith in the removal of petitioner
pursuant to a reorganization. (Alexis
C. Canonizado, et al. v. Hon.
Alexander P. Aguirre, et al., G.R.
No. 133132, Jan. 25, 2000, En
Banc [Gonzaga-Reyes])
299. What is reorganization?
is it valid? When is it invalid?

When

Held: 1. Reorganization takes place


when there is an alteration of the
existing structure of government
offices or units therein, including the
lines
of
control,
authority
and
responsibility between them.
It
involves a reduction of personnel,
consolidation of offices, or abolition
thereof by reason of economy or
redundancy of functions. Naturally, it
may result in the loss of one's position
through removal or abolition of an
office. However, for a reorganization
to be valid, it must also pass the test
of good faith, laid down in Dario v.
Mison (176 SCRA 84 [1989]):
x x x As a general rule, a
reorganization is carried out in "good
faith" if it is for the purpose of
economy or to make bureaucracy
more efficient.
In that event, no
dismissal (in case of dismissal) or
separation actually occurs because
the position itself ceases to exist. And
in that case, security of tenure would
not be a Chinese wall. Be that as it
may, if the "abolition" which is nothing
else but a separation or removal, is
done for political reasons or purposely
to defeat security of tenure, or
otherwise not in good faith, no valid
"abolition" takes place and whatever
"abolition" is done, is void ab initio.
There is an invalid "abolition" as where
there is merely a change of
nomenclature of positions, or where
claims of economy are belied by the
existence of ample funds.
(Alexis C. Canonizado, et al. v.
Hon. Alexander P. Aguirre, et al.,
G.R. No. 133132,
Jan. 25, 2000,
En Banc [Gonzaga-Reyes])
2. While the Presidents power
to reorganize can not be denied, this
does not mean however that the
reorganization itself is properly made
in accordance with law. Well-settled is
the
rule
that
reorganization
is
regarded as valid provided it is
pursued in good faith. Thus, in Dario
v. Mison, this Court has had the
occasion to clarify that:

As a general rule, a reorganization is


carried out in good faith if it is for the
purpose of economy or to make the
bureaucracy more efficient. In that
event no dismissal or separation
actually occurs because the position
itself ceases to exist. And in that case
the security of tenure would not be a
Chinese wall. Be that as it may, if the
abolition which is nothing else but a
separation or removal, is done for
political reasons or purposely to defeat
security of tenure, or otherwise not in
good faith, no valid abolition takes
place and whatever abolition done is
void ab initio.
There is an invalid
abolition as where there is merely a
change of nomenclature of positions
or where claims of economy are belied
by the existence of ample funds.
(176 SCRA 84)
(Larin v. Executive Secretary, 280
SCRA 713, Oct. 16, 1997)
300. What are the circumstances
evidencing bad faith in the removal of
employees
as
a
result
of
reorganization and which may give
rise to a claim for reinstatement or
reappointment)?
Held:
1)

2)
3)

4)

5)

Where there is a significant


increase in the number of positions in
the new staffing pattern of the
department or agency concerned;
Where an office is abolished
and another performing substantially
the same functions is created;
Where
incumbents
are
replaced by those less qualified in
terms of status of appointment,
performance and merit;
Where
there
is
a
reclassification of offices in the
department or agency concerned and
the
reclassified
offices
perform
substantially the same functions as
the original offices;
Where the removal violates
the order of separation provided in
Section 3 hereof.
(Sec. 2, R.A. No. 6656; Larin v.
Executive Secretary, 280 SCRA
713, Oct. 16, 1997)
ELECTION LAWS
301. Discuss the Right of Suffrage,
and its substantive and procedural
requirements.
Held: In a representative
democracy such as ours, the right of
suffrage, although accorded a prime

niche in the hierarchy of rights


embodied in the fundamental law,
ought to be exercised within the
proper bounds and framework of the
Constitution and must properly yield
to pertinent laws skillfully enacted by
the Legislature, which statutes for all
intents and purposes, are crafted to
effectively insulate such so cherished
right from ravishment and preserve
the democratic institutions our people
have, for so long, guarded against the
spoils of opportunism, debauchery and
abuse.
To be sure, the right of suffrage
x x x is not at all absolute. Needless
to say, the exercise of the right of
suffrage, as in the enjoyment of all
other rights, is subject to existing
substantive
and
procedural
requirements
embodied
in
our
Constitution, statute books and other
repositories of law. Thus, as to the
substantive aspect, Section 1, Article V
of the Constitution provides:
SECTION 1. SUFFRAGE MAY BE
EXERCISED BY ALL CITIZENS OF THE
PHILIPPINES
NOT
OTHERWISE
DISQUALIFIED BY LAW, WHO ARE AT
LEAST EIGHTEEN YEARS OF AGE, AND
WHO SHALL HAVE RESIDED IN THE
PHILIPPINES FOR AT LEAST ONE YEAR
AND IN THE PLACE WHEREIN THEY
PROPOSE TO VOTE FOR AT LAST SIX
MONTHS IMMEDIATELY PRECEDING
THE ELECTION.
NO LITERACY,
PROPERTY, OR OTHER SUBSTANTIVE
REQUIREMENT SHALL BE IMPOSED ON
THE EXERCISE OF SUFFRAGE.
As to the procedural limitation,
the right of a citizen to vote is
necessarily conditioned upon certain
procedural requirements he must
undergo: among others, the process of
registration. Specifically, a citizen in
order to be qualified to exercise his
right to vote, in addition to the
minimum requirements set by the
fundamental charter, is obliged by law
to register, at present, under the
provisions of Republic Act No. 8189,
otherwise known as the Voters
Registration Act of 1996. (AkbayanYouth v. COMELEC, 355 SCRA 318,
Mar. 26, 2001, En Banc [Buena])
302. Discuss the reason behind the
principle of ballot secrecy. May the
conduct of exit polls transgress the
sanctity and the secrecy of the ballot
to justify its prohibition?

Held: The reason behind the principle


of ballot secrecy is to avoid vote
buying through voter identification.
Thus, voters are prohibited from
exhibiting the contents of their official
ballots to other persons, from making
copies thereof, or from putting
distinguishing marks thereon so as to
be identified.
Also proscribed is
finding out the contents of the ballots
cast by particular voters or disclosing
those of disabled or illiterate voters
who have been assisted. Clearly, what
is forbidden is the association of
voters with their respective votes, for
the purpose of assuring that the votes
have been cast in accordance with the
instructions of a third party.
This
result cannot, however, be achieved
merely through the voters verbal and
confidential disclosure to a pollster of
whom they have voted for.
In exit polls, the contents of the official
ballot are not actually exposed.
Furthermore, the revelation of whom
an elector has voted for is not
compulsory, but voluntary.
Voters
may also choose not to reveal their
identities. Indeed, narrowly tailored
countermeasures may be prescribed
by the Comelec, so as to minimize or
suppress incidental problems in the
conduct
of
exit
polls,
without
transgressing the fundamental rights
of
our
people.
(ABS-CBN
Broadcasting
Corporation
v.
COMELEC, G.R. No. 133486, Jan.
28, 2000, En Banc [Panganiban])
303. Discuss
the
meaning
and
purpose of residency requirement in
Election Law.
Held: 1. The meaning and purpose
of the residency requirement were
explained recently in our decision in
Aquino v. Comelec (248 SCRA 400,
420-421 [1995]), as follows:
X x x [T]he place where a party
actually or constructively has his
permanent home, where he, no
matter where he may be found at any
given time, eventually intends to
return and remain, i.e., his domicile, is
that to which the Constitution refers
when it speaks of residence for the
purposes of election law. The manifest
purpose of this deviation from the
usual conceptions of residency in law
as explained in Gallego v. Vera is to
exclude strangers or newcomers
unfamiliar with the conditions and
needs of the community from taking
advantage of favorable circumstances

existing in that community for


electoral gain. While there is nothing
wrong with the practice of establishing
residence in a given area for meeting
election
law
requirements,
this
nonetheless defeats the essence of
representation, which is to place
through the assent of voters those
most cognizant and sensitive to the
needs of a particular district, if a
candidate falls short of the period of
residency mandated by law for him to
qualify.
That purpose could be
obviously best met by individuals who
have either had actual residence in
the area for a given period or who
have been domiciled in the same area
either by origin or by choice.
(Marcita
Mamba
Perez
v.
COMELEC, G.R. No. 133944, Oct.
28, 1999, En Banc [Mendoza])
2. The Constitution and the law
requires residence as a qualification
for seeking and holding elective public
office, in order to give candidates the
opportunity to be familiar with the
needs,
difficulties,
aspirations,
potentials for growth and all matters
vital
to
the
welfare
of
their
constituencies; likewise, it enables the
electorate to evaluate the office
seekers qualifications and fitness for
the job they aspire for. Inasmuch as
Vicente Y. Emano has proven that he,
together with his family, (1) had
actually resided in a house he bought
in 1973 in Cagayan de Oro City; (2)
had actually held office there during
his three terms as provincial governor
of Misamis Oriental, the provincial
capitol being located therein; and (3)
has registered as voter in the city
during the period required by law, he
could not be deemed a stranger or
newcomer when he ran for and was
overwhelmingly voted as city mayor.
Election laws must be liberally
construed to give effect to the popular
mandate.
(Torayno,
Sr.
v.
COMELEC, 337 SCRA 574, Aug. 9,
2000, En Banc [Panganiban])
3. Generally, in requiring candidates
to have a minimum period of
residence in the area in which they
seek to be elected, the Constitution or
the law intends to prevent the
possibility of a stranger or newcomer
unacquainted with the conditions and
needs of a community and not
identified
with
the
latter
from
[seeking] an elective office to serve
that community. Such provision is
aimed at excluding outsiders from
taking
advantage
of
favorable
circumstances
existing
in
that

community
for
electoral
gain.
Establishing residence in a community
merely to meet an election law
requirement defeats the purpose of
representation: to elect through the
assent of voters those most cognizant
and sensitive to the needs of the
community. This purpose is best met
by individuals who have either had
actual residence in the area for a
given period or who have been
domiciled in the same area either by
origin or by choice. (Torayno, Sr. v.
COMELEC, 337 SCRA 574, Aug. 9,
2000, En Banc [Panganiban])

SCRA 318, Mar. 26, 2001, En Banc


[Buena])
306. What is the Lone Candidate
Law? What are its salient provisions?
Answer: The Lone Candidate Law is
Republic Act No. 8295, enacted on
June 6, 1997.
Section 2 thereof
provides that Upon the expiration of
the deadline for the filing of the
certificate of candidacy in a special
election called to fill a vacancy in an
elective position other than for
President and Vice-President, when
there is only one (1) qualified
candidate for such position, the lone
candidate shall be proclaimed elected
to the position by proper proclaiming
body of the Commission on Elections
without holding the special election
upon certification by the Commission
on Elections that he is the only
candidate for the office and is thereby
deemed elected.

304. Does the fact that a person is


registered as a voter in one district
proof that he is not domiciled in
another district?
Held:
The fact that a person is
registered as a voter in one district is
not proof that he is not domiciled in
another district. Thus, in Faypon v.
Quirino (96 Phil. 294 [1954]), this
Court held that the registration of a
voter in a place other than his
residence of origin is not sufficient to
consider him to have abandoned or
lost his residence. (Marcita Mamba
Perez v. COMELEC, G.R. No.
133944, Oct. 28, 1999, En Banc
[Mendoza])

Section 3 thereof provides that the


lone candidate so proclaimed shall
assume office not earlier than the
scheduled election day, in the absence
of any lawful ground to deny due
course or cancel the certificate of
candidacy in order to prevent such
proclamation, as provided for under
Sections 69 and 78 of Batas Pambansa
Bilang 881 also known as the Omnibus
Election Code.

305. Discuss the nature of Voters


Registration.
Held:
Stated differently, the
act of registration is an indispensable
precondition to the right of suffrage.
For registration is part and parcel of
the right to vote and an indispensable
element in the election process. Thus,
x x x registration cannot and should
not be denigrated to the lowly stature
of a mere statutory requirement.
Proceeding from the significance of
registration as a necessary requisite to
the
right
to
vote,
the
State
undoubtedly, in the exercise of its
inherent police power, may then enact
laws to safeguard and regulate the act
of voters registration for the ultimate
purpose of conducting honest, orderly
and peaceful election, to the incidental
yet generally important end, that even
pre-election
activities
could
be
performed by the duly constituted
authorities in a realistic and orderly
manner one which is not indifferent
and so far removed from the pressing
order of the day and the prevalent
circumstances
of
the
times.
(Akbayan-Youth v. COMELEC, 355

307. Who are disqualified to run in a


special election under the Lone
Candidate Law?
Answer:
Section 4 of the Lone
Candidate Law provides that In
addition to the
disqualifications
mentioned in Sections 12 and 68 of
the Omnibus Election Code and
Section 40 of Republic Act No. 7160,
otherwise
known
as
the
Local
Government Code, whenever the
evidence of guilt is strong, the
following persons are disqualified to
run in a special election called to fill
the vacancy in an elective office, to
wit:
a)

Any elective official who has


resigned from his office by accepting
an appointive office or for whatever
reason which he previously occupied
but has caused to become vacant due
to his resignation; and
b)
Any person who, directly or
indirectly, coerces, bribes, threatens,
harasses,
intimidates or actually
causes, inflicts or produces any
violence, injury, punishment, torture,

damage, loss or disadvantage to any


person or persons aspiring to become
a candidate or that of the immediate
member of his family, his honor or
property that is meant to eliminate all
other potential candidate.
308. What is the purpose of the law
in requiring the filing of certificate of
candidacy and in fixing the time limit
therefor?
Held: The evident purpose of the law
in requiring the filing of certificate of
candidacy and in fixing the time limit
therefor are: (a) to enable the voters
to know, at least sixty days before the
regular
election,
the
candidates
among whom they are to make the
choice, and (b) to avoid confusion and
inconvenience in the tabulation of the
votes cast. For if the law did not
confine the choice or election by the
voters
to
the
duly
registered
candidates, there might be as many
persons voted for as there are voters,
and votes might be cast even for
unknown or fictitious persons as a
mark to identify the votes in favor of a
candidate for another office in the
same election. (Miranda v. Abaya,
G.R. No. 136351, July 28, 1999)
309. May a disqualified candidate
and whose certificate of candidacy
was denied due course and/or
canceled by the Comelec be validly
substituted?
Held: Even on the most basic and
fundamental principles, it is readily
understood that the concept of a
substitute presupposes the existence
of the person to be substituted, for
how can a person take the place of
somebody who does not exist or who
never was. The Court has no other
choice but to rule that in all instances
enumerated in Section 77 of the
Omnibus Election Code, the existence
of a valid certificate of candidacy
seasonably filed is a requisite sine qua
non.
All told, a disqualified candidate may
only be substituted if he had a valid
certificate of candidacy in the first
place because, if the disqualified
candidate did not have a valid and
seasonably
filed
certificate
of
candidacy, he is and was not a
candidate at all. If a person was not a
candidate, he cannot be substituted
under Section 77 of the Code.
(Miranda v. Abaya, G.R. No.
136351, July 28, 1999, en Banc
[Melo])

310. Should the votes cast for the


substituted candidate be considered
votes for the substitute candidate?
Answer: Republic Act No. 9006,
otherwise known as the Fair Election
Act, provides in Section 12 thereof:
In case of valid substitutions after the
official ballots have been printed, the
votes
cast
for
the
substituted
candidates shall be considered as
stray votes but shall not invalidate the
whole ballot. For this purpose, the
official ballots shall provide spaces
where the voters may write the name
of the substitute candidates if they are
voting for the latter: Provided,
however, That if the substitute
candidate is of the same family name,
this provision shall not apply.
311. What is the effect of the filing of
certificate of candidacy by elective
officials?
Answer: COMELEC Resolution No.
3636, promulgated March 1, 2001,
implementing the Fair Election Act
(R.A. No. 9006) provides in Section 26
thereof: any elective official, whether
national or local, who has filed a
certificate of candidacy for the same
or any other office shall not be
considered resigned from his office.
Note that Section 67 of the Omnibus
Election Code and the first proviso in
the third paragraph of Section 11 of
Republic Act No. 8436 which modified
said Section 67, were expressly
repealed and rendered ineffective,
respectively, by Section 14 (Repealing
Clause) of The Fair Election Act (R.A.
No. 9006).
312. What
kind
of
material
misrepresentation is contemplated by
Section 78 of the Omnibus Election
Code as a ground for disqualification
of a candidate? Does it include the
use of surname?
Held: Therefore, it may be concluded
that the material misrepresentation
contemplated by Section 78 of the
(Omnibus Election) Code refers to
qualifications for elective office. This
conclusion is strengthened by the fact
that the consequences imposed upon
a candidate guilty of having made a
false representation in his certificate
of candidacy are grave to prevent
the candidate from running or, if
elected, from serving, or to prosecute
him for violation of the election laws.
It could not have been the intention of

the law to deprive a person of such a


basic and substantial political right to
be voted for a public office upon just
any innocuous mistake.
[A]side from the requirement of
materiality, a false representation
under Section 78 must consist of a
deliberate
attempt
to
mislead,
misinform, or hide a fact which would
otherwise
render
a
candidate
ineligible. In other words, it must be
made with an intention to deceive the
electorate as to ones qualifications for
public office. The use of a surname,
when not intended to mislead or
deceive the public as to ones identity,
is not within the scope of the
provision. (Victorino Salcedo II v.
COMELEC, G.R. No. 135886, Aug.
16, 1999, En Banc [GonzagaReyes])
313. Who has authority to declare
failure of elections and the calling of
special election? What are the three
instances where a failure of election
may be declared?
Held: The COMELECs authority to
declare failure of elections is provided
in our election laws. Section 4 of RA
7166 provides that the Comelec sitting
en banc by a majority vote of its
members may decide, among others,
the declaration of failure of election
and the calling of special election as
provided in Section 6 of the Omnibus
Election Code. X x x
There are three instances where a
failure of election may be declared,
namely, (a) the election in any polling
place has not been held on the date
fixed on account of force majeure,
violence, terrorism, fraud or other
analogous causes; (b) the election in
any polling place has been suspended
before the hour fixed by law for the
closing of the voting on account of
force majeure, violence, terrorism,
fraud or other analogous causes; or (c)
after the voting and during the
preparation and transmission of the
election returns or in the custody or
canvass thereof, such election results
in a failure to elect on account of force
majeure, violence, terrorism, fraud or
other analogous causes.
In these
instances, there is a resulting failure to
elect. This is obvious in the first two
scenarios, where the election was not

held and where the election was


suspended. As to the third scenario,
where the preparation and the
transmission of the election returns
give rise to the consequence of failure
to elect, it must x x x, be interpreted
to mean that nobody emerged as a
winner. (Banaga, Jr. v. COMELEC,
336 SCRA 701, July 31, 2000, En
Banc [Quisumbing])
314. What are the two conditions
that must concur before the COMELEC
can act on a verified petition seeking
to declare a afailure of election?
Held: Before the COMELEC can act
on a verified petition seeking to
declare a failure of election two
conditions must concur, namely: (1) no
voting took place in the precinct or
precincts on the date fixed by law, or
even if there was voting, the election
resulted in a failure to elect; and (2)
the votes not cast would have affected
the result of the election. Note that
the cause of such failure of election
could only be any of the following:
force majeure, violence, terrorism,
fraud or other analogous causes.
Thus, in Banaga, Jr. v. COMELEC
(336 SCRA 701, July 31, 2000, En Banc
[Quisumbing]), the SC held:
We have painstakingly examined the
petition filed by petitioner Banaga
before the Comelec. But we found
that petitioner did not allege at all that
elections were either not held or
suspended. Neither did he aver that
although there was voting, nobody
was elected.
On the contrary, he
conceded that an election took place
for the office of vice-mayor of
Paranaque City, and that private
respondent was, in fact, proclaimed
elected to that post. While petitioner
contends that the election was tainted
with widespread anomalies, it must be
noted that to warrant a declaration of
failure of election the commission of
fraud must be such that it prevented
or suspended the holding of an
election, or marred fatally the
preparation and transmission, custody

and canvass of the election returns.


These essential facts ought to have
been alleged clearly by the petitioner
below, but he did not.
315. Cite instances when Comelec
may or may not validly declare failure
of elections.
Held: In Mitmug v. COMELEC (230
SCRA 54 [1994]), petitioner instituted
with the COMELEC an action to declare
failure of election in forty-nine
precincts where less than a quarter of
the electorate were able to cast their
votes.
He also lodged an election
protest with the Regional Trial Court
disputing the result of the election in
all precincts in his municipality. The
Comelec denied motu proprio and
without due notice and hearing the
petition to declare failure of election
despite petitioners argument that he
has meritorious grounds in support
thereto,
that
is,
massive
disenfranchisement of voters due to
terrorism. On review, we ruled that
the Comelec did not gravely abuse its
discretion in denying the petition. It
was not proven that no actual voting
took place. Neither was it shown that
even if there was voting, the results
thereon would be tantamount to
failure to elect. Considering that there
is no concurrence of the conditions
seeking to declare failure of election,
there is no longer need to receive
evidence
on
alleged
election
irregularities.
In Sardea v. COMELEC (225 SCRA 374
[1993]), all election materials and
paraphernalia with the municipal
board of canvassers were destroyed
by the sympathizers of the losing
mayoralty candidate. The board then
decided to use the copies of election
returns furnished to the municipal trial
court.
Petitioner therein filed a
petition to stop the proceedings of the
board of canvassers on the ground
that it had no authority to use said
election returns obtained from the
municipal trial court. The petition was
denied.
Next, he filed a petition
assailing the composition of the board

of canvassers. Despite that petition,


the board of canvassers proclaimed
the winning candidates.
Later on,
petitioner filed a petition to declare a
failure of election alleging that the
attendant
facts
would
justify
declaration of such failure. On review,
we ruled that petitioners first two
actions
involved
pre-proclamation
controversies which can no longer be
entertained
after
the
winning
candidates have been proclaimed.
Regarding the petition to declare a
failure of election, we held that the
destruction and loss of copies of
election returns intended for the
municipal board of canvassers on
account of violence is not one of the
causes that would warrant the
declaration of failure of election. The
reason is that voting actually took
place as scheduled and other valid
election
returns
still
existed.
Moreover, the destruction or loss did
not affect the result of the election.
We also declared that there is failure
of elections only when the will of the
electorate has been muted and cannot
be ascertained.
If the will of the
people is determinable, the same
must as far as possible be respected.
Xxx
In Loong v. COMELEC (257 SCRA 1
[1996]), the petition for annulment of
election results or to declare failure of
elections in Parang, Sulu, on the
ground of statistical improbability and
massive fraud was granted by the
COMELEC. Even before the technical
examination of election documents
was conducted, the Comelec already
observed badges of fraud just by
looking at the election results in
Parang. Nevertheless, the Comelec
dismissed the petition for annulment
of election results or to declare failure
of elections in the municipalities of
Tapul, Panglima Estino, Pata, Siasi and
Kalinggalang Calauag. The COMELEC
dismissed the latter action on ground
of untimeliness of the petition, despite
a finding that the same badges of
fraud evident from the results of the
election based on the certificates of

canvass of votes in Parang, are also


evident in the election results of the
five mentioned municipalities.
We
ruled that Comelec committed grave
abuse of discretion in dismissing the
petition as there is no law which
provides for a reglementary period to
file annulment of elections when there
is yet no proclamation. The election
resulted in a failure to elect on
account of fraud.
Accordingly, we
ordered the Comelec to reinstate the
aforesaid
petition.
Those
circumstances, however, are not
present in this case, so that reliance
on Loong by petitioner Banaga is
misplaced. (Banaga, Jr. v. COMELEC,
336 SCRA 701, July 31, 2000, En
Banc [Quisumbing])
316. What acts of a Division of the
COMELEC may be subject of a motion
for reconsideration of the COMELEC en
banc?
Held:
Section 5, Rule 19 of the
COMELEC
Rules
of
Procedure,
provides:
SEC.
5.
How
Motion
for
Reconsideration Disposed of. - Upon
the filing of a motion to reconsider a
decision, resolution, order or ruling of
a Division, the Clerk of Court
concerned shall, within twenty-four
(24) hours from the filing thereof,
notify the presiding Commissioner.
The latter shall within two (2) days
thereafter certify the case to the
Commission en banc.
Under the above-quoted rule,
the acts of a Division that are subject
of a motion for reconsideration must
have a character of finality before the
same can be elevated to the COMELEC
en banc. The elementary rule is that
an order is final in nature if it
completely disposes of the entire case.
But if there is something more to be
done in the case after its issuance,
that order is interlocutory.
As correctly pointed out by
public respondent in its assailed order
of November 29, 1999, the October
11, 1999 did not dispose of the case
completely as there is something more
to be done which is to decide the
election protest. As such, it is the
herein public respondent (Second
Division of the COMELEC) which issued
the interlocutory order of October 11,

1999 that should resolve petitioners


motion for reconsideration, not the
COMELEC en banc (Bulaong v.
COMELEC, First Division, 220 SCRA
745, 749 [1993]). Accordingly, the
applicable rule on the subject is
Section 5(c), Rule 3 of the COMELEC
Rules of Procedure, which states:
Rule 3, Section 5(c). Any motion to
reconsider a decision, resolution, order
or ruling of a Division shall be resolved
by the Commission en banc except
motions on interlocutory orders of the
division, which shall be resolved by
the divisions which issued the order.
That only final orders of a
Division may be raised before the
COMELEC en banc is in accordance
with Article IX-C, Section 3 of the
Constitution which mandates that only
motions for reconsideration of final
decisions shall be decided by the
Commission on Elections en banc,
thus:
Sec. 3. The Commission on Elections
may sit en banc or in two divisions,
and shall promulgate its rules of
procedure in order to expedite
disposition of election cases, including
pre-proclamation controversies. All
such election cases shall be heard and
decided in division, provided that
motions for reconsideration of
decisions shall be decided by the
Commission en banc.
It bears stressing that under
this constitutional
provision, the
COMELEC en banc shall decide
motions for reconsideration only of
decisions of a Division, meaning
those acts of final character. Clearly,
the assailed order denying petitioner's
demurrer
to
evidence,
being
interlocutory, may not, be resolved by
the COMELEC en banc (Ambil, Jr. v.
Commission on elections, G.R. No.
143398, Oct. 25, 2000, 344 SCRA
358). (Gementiza v. Commission
on Elections, 353 SCRA 724,
March 6, 2001, En Banc [SandovalGutierrez])
317. Is a petition to declare failure of
election different from a petition to
annul the election results?
Held: A prayer to declare failure of
elections and a prayer to annul the
election results x x x are actually of

the same nature. Whether an action is


for declaration of failure of elections or
for annulment of election results,
based on allegations of fraud,
terrorism, violence or analogous, the
Omnibus Election Code denominates
them similarly.
(Banaga, Jr. v.
COMELEC, 336 SCRA 701, July 31,
2000, En Banc [Quisumbing])
318. What conditions must concur
before the Comelec can act on a
verified petition seeking to declare a
failure of election? Is low turn-out of
voters enough basis to grant the
petition?
Held: Before COMELEC can act on a
verified petition seeking to declare a
failure of election, two (2) conditions
must concur: first, no voting has taken
place in the precinct or precincts on
the date fixed by law or, even if there
was voting, the election nevertheless
results in failure to elect; and, second,
the votes not cast would affect the
result of the election.
There can be failure of election in a
political unit only if the will of the
majority has been defiled and cannot
be ascertained.
But, if it can be
determined, it must be accorded
respect. After all, there is no provision
in our election laws which requires
that a majority of registered voters
must cast their votes. All the law
requires is that a winning candidate
must be elected by a plurality of valid
votes, regardless of the actual number
of ballots cast. Thus, even if less than
25% of the electorate in the
questioned precincts cast their votes,
the same must still be respected.
(Mitmug v. COMELEC, 230 SCRA
54, Feb. 10, 1994, En Banc
[Bellosillo])
319. Distinguish a petition to declare
failure of elections from an election
protest.
Held:
While petitioner may have
intended to institute an election
protest by praying that said action
may also be considered an election
protest, in our view, petitioners action
is a petition to declare a failure of
elections or annul election results. It
is not an election protest.
First, his petition before the Comelec
was instituted pursuant to Section 4 of
Republic Act No. 7166 in relation to
Section 6 of the Omnibus Election

Code. Section 4 of RA 7166 refers to


postponement, failure of election and
special elections while Section 6 of
the Omnibus Election Code relates to
failure of election.
It is simply
captioned as Petition to Declare
Failure
of
Elections
and/or
For
Annulment of Elections.
Second, an election protest is an
ordinary action while a petition to
declare a failure of elections is a
special action under the 1993 Comelec
Rules of Procedure as amended. An
election protest is governed by Rule
20 on ordinary actions, while a petition
to declare failure of elections is
covered by Rule 26 under special
actions.
In this case, petitioner filed his petition
as a special action and paid the
corresponding fee therefor. Thus, the
petition was docketed as SPA-98-383.
This
conforms
to
petitioners
categorization of his petition as one to
declare a failure of elections or annul
election results.
In contrast, an
election protest is assigned a docket
number starting with EPC, meaning
election protest case.
Third, petitioner did not comply with
the requirements for filing an election
protest. He failed to pay the required
filing fee and cash deposits for an
election protest. Failure to pay filing
fees will not vest the election tribunal
jurisdiction over the case.
Such
procedural lapse on the part of a
petitioner would clearly warrant the
outright dismissal of his action.
Fourth, an en banc decision of
Comelec in an ordinary action
becomes final and executory after
thirty (30) days from its promulgation,
while an en banc decision in a special
action becomes final and executory
after five (5) days from promulgation,
unless restrained by the Supreme
Court (Comelec Rules of Procedure,
Rule 18, Section 13 [a], [b]). For that
reason, a petition cannot be treated as
both an election protest and a petition
to declare failure of elections.
Fifth, the allegations in the petition
decisively
determine
its
nature.
Petitioner alleged that the local
elections for the office of vice-mayor
in Paranaque City held on May 11,
1998, denigrates the true will of the
people as it was marred with
widespread anomalies on account of
vote buying, flying voters and glaring
discrepancies in the election returns.

He averred that those incidents


warrant the declaration of a failure of
elections.
Given these circumstances, public
respondent cannot be said to have
gravely erred in treating petitioners
action as a petition to declare failure
of elections or to annul election
results. (Banaga, Jr. v. COMELEC,
336 SCRA 701, July 31, 2000, En
Banc [Quisumbing])
320. What
are
pre-proclamation
cases, and exceptions thereto? What
Court has jurisdiction over preproclamation cases?
Held: As a general rule, candidates
and
registered
political
parties
involved in an election are allowed to
file pre-proclamation cases before the
Comelec.
Pre-proclamation cases
refer to any question pertaining to or
affecting the proceedings of the board
of canvassers which may be raised by
any candidate or by any registered
political party or coalition of political
parties before the board or directly
with the Commission, or any matter
raised under Sections 233, 234, 235
and 236 in relation to the preparation,
transmission, receipt, custody and
appreciation
of
election
returns
(Section 241, Omnibus Election Code).
The Comelec has exclusive jurisdiction
over
all
pre-proclamation
controversies (Section 242, supra). As
an exception, however, to the general
rule, Section 15 of Republic Act 7166
prohibits
candidates
in
the
presidential,
vice-presidential,
senatorial and congressional elections
from filing pre-proclamation cases. It
states:
Sec. 15. Pre-Proclamation Cases Not
Allowed in Elections for President,
Vice-President, Senator, and Members
of the House of Representatives. - For
purposes
of
the
elections
for
President, Vice-President, Senator and
Member
of
the
House
of
Representatives, no pre-proclamation
cases shall be allowed on matters
relating
to
the
preparation,
transmission, receipt, custody and
appreciation of election returns or the
certificates of canvass, as the case
may be.
However, this does not
preclude
the
authority
of
the
appropriate canvassing body motu
proprio or upon written complaint of
an interested person to correct

manifest errors in the certificate of


canvass or election returns before it.
The prohibition aims to avoid delay in
the proclamation of the winner in the
election, which delay might result in a
vacuum in these sensitive posts. The
law,
nonetheless,
provides
an
exception to the exception.
The
second sentence of Section 15 allows
the filing of petitions for correction of
manifest errors in the certificate of
canvass or election returns even in
elections for president, vice-president
and members of the House of
Representatives for the simple reason
that the correction of manifest error
will not prolong the process of
canvassing nor delay the proclamation
of the winner in the election. The rule
is consistent with and complements
the authority of the Comelec under the
Constitution
to
"enforce
and
administer all laws and regulations
relative to the conduct of an election,
plebiscite, initiative, referendum and
recall" (Section 2[1], Article IX-C, 1987
Constitution) and its power to "decide,
except those involving the right to
vote, all questions affecting elections."
(Section 2[3], Article IX-C, supra)
(Federico S. Sandoval v. COMELEC,
G.R. No. 133842, Jan. 26, 2000
[Puno])
321. Who has authority to rule on
petitions for correction of manifest
error in the certificate of canvass or
election returns?
Held:
The authority to rule on
petitions for correction of manifest
error is vested in the Comelec en
banc. Section 7 of Rule 27 of the 1993
COMELEC Rules of Procedure (took
effect on February 15, 1993) provides
that if the error is discovered before
proclamation, the board of canvassers
may motu proprio, or upon verified
petition by any candidate, political
party, organization or coalition of
political parties, after due notice and
hearing, correct the errors committed.
The aggrieved party may appeal the
decision of the board to the
Commission and said appeal shall be
heard and decided by the Commission
en banc. Section 5, however, of the
same rule states that a petition for
correction of manifest error may be
filed directly with the Commission en
banc provided that such errors could
not have been discovered during the
canvassing despite the exercise of due
diligence and proclamation of the
winning candidate had already been
made. (Federico S. Sandoval v.

COMELEC, G.R. No. 133842, Jan.


26, 2000 [Puno])
322. Distinguish Election Protest from
Petition for Quo Warranto.
Held:
In Samad v. COMELEC, we
explained that a petition for quo
warranto under the Omnibus Election
Code raises in issue the disloyalty or
ineligibility of the winning candidate.
It is a proceeding to unseat the
respondent from office but not
necessarily to install the petitioner in
his place. An election protest is a
contest between the defeated and
winning candidates on the ground of
frauds or irregularities in the casting
and counting of the ballots, or in the
preparation of the returns. It raises
the question of who actually obtained
the plurality of the legal votes and
therefore is entitled to hold the office.
(Dumayas, Jr. v. COMELEC, G.R.
Nos. 141952-53, April 20, 2001, En
Banc [Quisumbing])
323. What is a counter-protest?
When should it be filed?
Held: Under the Comelec Rules of
Procedure,
the
protestee
may
incorporate in his answer a counterprotest.
It has been said that a
counter-protest is tantamount to a
counterclaim in a civil action and may
be presented as a part of the answer
within the time he is required to
answer the protest, i.e., within five (5)
days upon receipt of the protest,
unless a motion for extension is
granted, in which case it must be filed
before the expiration of the extended
time.
As early as in the case of Arrieta v.
Rodriguez (57 Phil. 717), the SC had
firmly settled the rule that the counterprotest must be filed within the period
provided by law, otherwise, the forum
loses its jurisdiction to entertain the
belatedly filed counter-protest. (Kho
v. COMELEC, 279 SCRA 463, Sept.
25, 1997, En Banc [Torres])
324. What is the effect of death of a
party in an election protest? Should it
warrant the dismissal of the protest?
Held:
An election protest involves
both the private interests of the rival
candidates and the public interest in
the final determination of the real
choice of the electorate, and for this
reason, an election contest necessarily
survives the death of the protestant or
the protestee. It is true that a public

office is personal to the public officer


and is not a property transmissible to
his heirs upon death, thus, upon the
death of the incumbent, no heir of his
may be allowed to continue holding
his office in his place. But while the
right to a public office is personal and
exclusive to the public officer, an
election protest is not purely personal
and exclusive to the protestant or to
the protestee such that after the death
of either would oust the court of all
authority to continue the protest
proceedings.
An election contest,
after
all,
involves
not
merely
conflicting private aspirations but is
imbued
with
paramount
public
interests. The death of the protestant
neither constitutes a ground for the
dismissal of the contest nor ousts the
trial court of its jurisdiction to decide
the election contest. (De Castro v.
COMELEC, 267 SCRA 806, Feb. 7,
1997)
325. Does the fact that one or a few
candidates in an election got zero
votes in one or a few precincts
adequately support a finding that the
election
returns
are
statistically
improbable?
Held:
From experiences in past
elections, it is possible for one
candidate or even a few candidates to
get zero votes in one or a few
precincts.
Standing alone and without more, the
bare fact that a candidate for public
office received zero votes in one or
two precincts can not adequately
support a finding that the subject
election
returns
are
statistically
improbable. A no-vote for a particular
candidate in election returns is but
one
strand
in
the
web
of
circumstantial evidence that those
election returns were prepared under
duress, force and intimidation.
In
the case of Una Kibad v. Comelec (23
SCRA 588 [1968]), the SC warned that
the doctrine of statistical improbability
must be viewed restrictively, the
utmost care being taken lest in
penalizing the fraudulent and corrupt
practices, innocent voters become
disenfranchised, a result which hardly
commends itself.
Moreover, the
doctrine of statistical improbability
involves a question of fact and a more
prudential approach prohibits its
determination ex parte. (Arthur V.
Velayo v. COMELEC, G.R. No.
135613, March 9, 2000, En Banc
[Puno])

326. What Court has jurisdiction over


election protests and quo warranto
proceedings involving Sangguniang
Kabataan (SK) elections?
Held:
Any contest relating to the
election of members of
the
Sangguniang Kabataan (including the
chairman) whether pertaining to
their eligibility or the manner of their
election is cognizable by MTCs,
MCTCs, and MeTCs.
Section 6 of
Comelec Resolution No. 2824 which
provides that cases involving the
eligibility or qualification of SK
candidates shall be decided by the
City/Municipal Election Officer whose
decision shall be final, applies only to
proceedings before the election.
Before proclamation, cases concerning
eligibility of SK officers and members
are cognizable by the Election Officer.
But
after
the
election
and
proclamation, the same cases become
quo warranto cases cognizable by
MTCs, MCTCs, and MeTCs.
The
distinction is based on the principle
that it is the proclamation which
marks off the jurisdiction of the courts
from the jurisdiction of election
officials.
The case of Jose M. Mercado v. Board
of Election Supervisors (243 SCRA 423,
G.R. No. 109713, April 6, 1995), in
which this Court ruled that election
protests involving SK elections are to
be determined by the Board of
Election Supervisors was decided
under the aegis of Comelec Resolution
No. 2499, which took effect on August
27,
1992.
However,
Comelec
Resolution No. 2824, which took effect
on February 6, 1996 and was passed
pursuant to R.A. 7808, in relation to
Arts. 252-253 of the Omnibus Election
Code, has since transferred the
cognizance of such cases from the
Board of Election Supervisors to the
MTCs, MCTCs and MeTCs. Thus, the
doctrine of Mercado is no longer
controlling.
(Francis
King
L.
Marquez v. COMELEC, G.R. No.
127318, Aug. 25, 1999, En Banc
[Purisima])
327. What acts of a Division of the
COMELEC may be subject of a motion
for reconsideration of the COMELEC en
banc?
Held:
Section 5, Rule 19 of the
COMELEC
Rules
of
Procedure,
provides:
SEC.
5.
How
Motion
for
Reconsideration Disposed of. - Upon

the filing of a motion to reconsider a


decision, resolution, order or ruling of
a Division, the Clerk of Court
concerned shall, within twenty-four
(24) hours from the filing thereof,
notify the presiding Commissioner.
The latter shall within two (2) days
thereafter certify the case to the
Commission en banc.
Under the above-quoted rule,
the acts of a Division that are subject
of a motion for reconsideration must
have a character of finality before the
same can be elevated to the COMELEC
en banc. The elementary rule is that
an order is final in nature if it
completely disposes of the entire case.
But if there is something more to be
done in the case after its issuance,
that order is interlocutory.
As correctly pointed out by
public respondent in its assailed order
of November 29, 1999, the October
11, 1999 did not dispose of the case
completely as there is something more
to be done which is to decide the
election protest. As such, it is the
herein public respondent (Second
Division of the COMELEC) which issued
the interlocutory order of October 11,
1999 that should resolve petitioners
motion for reconsideration, not the
COMELEC en banc (Bulaong v.
COMELEC, First Division, 220 SCRA
745, 749 [1993]). Accordingly, the
applicable rule on the subject is
Section 5(c), Rule 3 of the COMELEC
Rules of Procedure, which states:
Rule 3, Section 5(c). Any motion to
reconsider a decision, resolution, order
or ruling of a Division shall be resolved
by the Commission en banc except
motions on interlocutory orders of the
division, which shall be resolved by
the divisions which issued the order.
That only final orders of a
Division may be raised before the
COMELEC en banc is in accordance
with Article IX-C, Section 3 of the
Constitution which mandates that only
motions for reconsideration of final
decisions shall be decided by the
Commission on Elections en banc,
thus:
Sec. 3. The Commission on Elections
may sit en banc or in two divisions,
and shall promulgate its rules of
procedure in order to expedite
disposition of election cases, including
pre-proclamation controversies. All
such election cases shall be heard and

decided in division, provided that


motions for reconsideration of
decisions shall be decided by the
Commission en banc.
It bears stressing that under
this constitutional
provision, the
COMELEC en banc shall decide
motions for reconsideration only of
decisions of a Division, meaning
those acts of final character. Clearly,
the assailed order denying petitioner's
demurrer
to
evidence,
being
interlocutory, may not, be resolved by
the COMELEC en banc (Ambil, Jr. v.
Commission on elections, G.R. No.
143398, Oct. 25, 2000, 344 SCRA
358). (Gementiza v. Commission
on Elections, 353 SCRA 724,
March 6, 2001, En Banc [SandovalGutierrez])

THE LAW OF PUBLIC


CORPORATIONS
328.

What is an autonomous region?

Answer:
An autonomous region
consists
of
provinces,
cities,
municipalities, and geographical areas
sharing common
and
distinctive
historical
and
cultural
heritage,
economic and social structures, and
other relevant characteristics within
the framework of the Constitution and
the national sovereignty as well as the
territorial integrity of the Republic of
the Philippines. (Sec. 15, Art. X,
1987 Constitution)
329. What
are
administrative
regions?
Are
they considered
territorial and political subdivisions of
the State? Who has the power to
create administrative regions?
Held:
Administrative regions are
mere
groupings
of
contiguous
provinces for administrative purposes.
They are not territorial and political
subdivisions like provinces, cities,
municipalities and barangays. While
the power to merge administrative
regions is not expressly provided for in
the Constitution, it is a power which
has traditionally been lodged with the
President to facilitate the exercise of
the power of general supervision over
local
governments.
(Abbas
v.
COMELEC, 179 SCRA 287, Nov. 10,
1989, En Banc [Cortes])

329. Is there a conflict between the


power of the President to merge
administrative
regions
with
the
constitutional provision requiring a
plebiscite in the merger of local
government units?
Held: There is no conflict between
the power of the President to merge
administrative
regions
with
the
constitutional provision requiring a
plebiscite in the merger of local
government
units
because
the
requirement of a plebiscite in a merger
expressly applies only to provinces,
cities, municipalities or barangays, not
to administrative regions. (Abbas v.
COMELEC, 179 SCRA 287, Nov. 10,
1989, En Banc [Cortes])
330. What is the Metropolitan Manila
Development Authority (MMDA)? Is it
a local government unit or public
corporation endowed with legislative
power? May it validly exercise police
power? How is it distinguished from
the former Metro Manila Council (MMC)
created under PD No. 824?
Held: Metropolitan or Metro Manila is
a body composed of several local
government units i.e., twelve (12)
cities and five (5) municipalities x x x.
With the passage of Republic Act No.
7924 in 1995, Metropolitan Manila was
declared as a special development
and administrative region and the
Administration of metrowide basic
services affecting the region placed
under a development authority
referred to as the MMDA.
The governing board of the MMDA is
the Metro Manila Council. The Council
is composed of the mayors of the
component
12
cities
and
5
municipalities, the president of the
Metro Manila Vice-Mayors League and
the president of the Metro Manila
Councilors League.
The Council is
headed by a Chairman who is
appointed by the President and vested
with the rank of cabinet member. As
the policy-making body of the MMDA,
the Metro Manila Council approves
metro-wide plans, programs and
projects, and issues the necessary
rules
and
regulations
for
the
implementation of said plans; it
approves the annual budget of the
MMDA and promulgates the rules and
regulations for the delivery of basic
services, collection of service and
regulatory fees, fines and penalties. X
xx

Clearly, the scope of the MMDAs


function is limited to the delivery of
the seven (7) basic services. One of
these
is
transport
and
traffic
management x x x.
It will be noted that the powers of the
MMDA are limited to the following
acts:
formulation,
coordination,
regulation,
implementation,
preparation, management, monitoring,
setting of policies, installation of a
system and administration. There is
no syllable in R.A. No. 7924 that grants
the MMDA police power, let alone
legislative power.
Even the Metro
Manila Council has not been delegated
any legislative power.
Unlike the
legislative
bodies
of
the
local
government
units,
there
is
no
provision in R.A. No. 7924 that
empowers the MMDA or its Council to
enact ordinances, approve resolutions
and appropriate funds for the general
welfare of the inhabitants of Metro
Manila.
The MMDA is x x x a
development authority.
It is an
agency created for the purpose of
laying down policies and coordinating
with the various national government
agencies, peoples organizations, nongovernmental organizations and the
private sector for the efficient and
expeditious delivery of basic services
in the vast metropolitan area. All its
functions are administrative in nature
and these are actually summed up in
the charter itself x x x.
Secondly, the MMDA is not the same
entity as the MMC in Sangalang.
Although the MMC is the forerunner of
the present MMDA, an examination of
Presidential Decree No. 824, the
charter of the MMC, shows that the
latter possessed greater powers which
were not bestowed on the present
MMDA.
Metropolitan Manila was first created
in 1975 by Presidential Decree No.
824. It comprised the Greater Manila
Area composed of the contiguous four
(4) cities of Manila, Quezon, Pasay and
Caloocan, and the thirteen (13)
municipalities x x x. Metropolitan
Manila was created as a response to
the finding that the rapid growth of
population and the increase of social
and economic requirements in these
areas demand a call for simultaneous
and unified development; that the
public services rendered by the
respective local governments could be
administered more efficiently and
economically if integrated under a
system of central planning; and this

coordination,
especially
in
the
maintenance of peace and order and
the eradication of social and economic
ills that fanned the flames of rebellion
and discontent [were] part of the
reform measures under Martial Law
essential to the safety and security of
the State.
Metropolitan Manila was established
as a public corporation x x x.
The administration of Metropolitan
Manila was placed under the Metro
Manila Commission (MMC) x x x.
The
MMC
was
the
central
government of Metro Manila for the
purpose
of
establishing
and
administering
programs
providing
services common to the area. As a
central government it had the power
to levy and collect taxes and special
assessments, the power to charge and
collect fees; the power to appropriate
money for its operation, and at the
same time, review appropriations for
the city and municipal units within its
jurisdiction.
It was bestowed the
power to enact or approve ordinances,
resolutions and fix penalties for
violation of such ordinances and
resolutions. It also had the power to
review, amend, revise or repeal all
ordinances, resolutions and acts of
any of the x x x cities and x x x
municipalities
comprising
Metro
Manila.
Xxx
The creation of the MMC also carried
with it the creation of the Sangguniang
Bayan. This was composed of the
members of the component city and
municipal councils, barangay captains
chosen by the MMC and sectoral
representatives appointed by the
President.
The Sangguniang Bayan
had the power to recommend to the
MMC the adoption of ordinances,
resolutions or measures. It was the
MMC itself, however, that possessed
legislative powers.
All ordinances,
resolutions
and
measures
recommended by the Sangguniang
Bayan were subject to the MMCs
approval.
Moreover, the power to
impose taxes and other levies, the
power to appropriate money, and the
power
to
pass
ordinances
or
resolutions with penal sanctions were
vested exclusively in the MMC.
Thus, Metropolitan Manila had a
central government, i.e., the MMC
which fully possessed legislative and

police powers. Whatever legislative


powers the component cities and
municipalities had were all subject to
review and approval by the MMC.
After
President
Corazon
Aquino
assumed power, there was a clamor to
restore the autonomy of the local
government units in Metro Manila.
Hence, Sections 1 and 2 of Article X of
the 1987 Constitution x x x.
The
Constitution, however, recognized the
necessity of creating metropolitan
regions not only in the existing
National Capital Region but also in
potential equivalents in the Visayas
and Mindanao. X x x
The
Constitution
itself
expressly
provides that Congress may, by law,
create special metropolitan political
subdivisions which shall be subject to
approval by a majority of the votes
cast in a plebiscite in the political units
directly affected; the jurisdiction of
this subdivision shall be limited to
basic services requiring coordination;
and the cities and municipalities
comprising this subdivision shall retain
their basic autonomy and their own
local
executive
and
legislative
assemblies (Section 11, Article X, 1987
Constitution). Pending enactment of
this law, the Transitory Provisions of
the Constitution gave the President of
the Philippines the power to constitute
the Metropolitan Authority x x x.
In 1990, President Aquino issued
Executive
Order
No.
392
and
constituted the Metropolitan Manila
Authority (MMA).
The powers and
functions of the MMC were devolved to
the MMA. It ought to be stressed,
however, that not all powers and
functions of the MMC were passed to
the MMA.
The MMAs power was
limited to the delivery of basic urban
services requiring coordination in
Metropolitan Manila.
The MMAs
governing body, the Metropolitan
Manila Council, although composed of
the mayors of the component cities
and municipalities, was merely given
the power of: (1) formulation of
policies on the delivery of basic
services requiring coordination and
consolidation; and (2) promulgation of
resolutions and other issuances,
approval of a code of basic services
and the exercise of its rule-making
power.
Under the 1987 Constitution, the local
government units became primarily
responsible for the governance of their
respective political subdivisions. The

MMAs jurisdiction was limited to


addressing
common
problems
involving
basic
services
that
transcended local boundaries. It did
not have legislative power. Its power
was merely to provide the local
government units technical assistance
in
the
preparation
of
local
development plans. Any semblance of
legislative power it had was confined
to a review [of] legislation proposed
by the local legislative assemblies to
ensure
consistency
among
local
governments
and
with
the
comprehensive development plan of
Metro Manila, and to advise the local
governments accordingly.
When R.A. No. 7924 took effect,
Metropolitan Manila became a special
development
and
administrative
region and the MMDA a special
development
authority
whose
functions were without prejudice to
the autonomy of the affected local
government units. The character of
the MMDA was clearly defined in the
legislative
debates
enacting
its
charter.
Xxx
Clearly, the MMDA is not a political
unit of government.
The power
delegated to the MMDA is that given
to the Metro Manila Council to
promulgate administrative rules and
regulations in the implementation of
the MMDAs functions. There is no
grant of authority to enact ordinances
and regulations for the general welfare
of the inhabitants of the metropolis.
This was explicitly stated in the last
Committee deliberations prior to the
bills presentation to Congress. X x x
It is thus beyond doubt that the MMDA
is not a local government unit or a
public corporation endowed with
legislative power. It is not even a
special
metropolitan
political
subdivision
as
contemplated
in
Section
11,
Article
X
of
the
Constitution.
The creation of a
special
metropolitan
political
subdivision requires the approval by a
majority of the votes cast in a
plebiscite in the political units directly
affected.
R.A. No. 7924 was not
submitted to the inhabitants of Metro
Manila in a plebiscite. The Chairman
of the MMDA is not an official elected
by the people, but appointed by the
President with the rank and privileges
of a cabinet member. In fact, part of
his function is to perform such other
duties as may be assigned to him by

the President, whereas in local


government units, the President
merely exercises supervisory authority.
This emphasizes the administrative
character of the MMDA.
Clearly then, the MMC under P.D. No.
824 is not the same entity as the
MMDA under R.A. No. 7924. Unlike the
MMC, the MMDA has no power to
enact ordinances for the welfare of the
community. It is the local government
units, acting through their respective
legislative councils, that possess
legislative power and police power. In
the case at bar, the Sangguniang
Panlungsod of Makati City did not pass
any ordinance or resolution ordering
the opening of Neptune Street, hence,
its proposed opening by petitioner
MMDA is illegal x x x. (MMDA v. BelAir Village Association, Inc., 328
SCRA 836, March 27, 2000, 1st
Div. [Puno])
331. Discuss the concept of local
autonomy.
Held:
Autonomy
is
either
decentralization of administration or
decentralization of power. There is
decentralization
of
administration
when
the
central
government
delegates administrative powers to
political subdivisions in order to
broaden the base of government and
in the process to make local
governments more responsive and
accountable, and ensure their fullest
development
as
self-reliant
communities and make them more
effective partners in the pursuit of
national development and social
progress. At the same time, it relieves
the central government of the burden
of managing local affairs and enables
it to concentrate on national concerns.
The
President
exercises
general
supervision over them, but only to
ensure
that
local
affairs
are
administered according to law. He has
no control over their acts in the sense
that he can substitute their judgments
with his own.
Decentralization of power, on the
other hand, involves an abdication of
political power in favor of local
government
units
declared
autonomous.
In that case, the
autonomous government is free to
chart its own destiny and shape its
own future with minimum intervention
from central authorities. According to
a
constitutional
author,
decentralization of power amounts to
self-immolation, since in that event,

the autonomous government becomes


accountable not to the central
authorities but to its constituency.
(Limbona v. Mangelin, 170 SCRA
786, Feb. 28, 1989, En Banc
[Sarmiento])
332. What kind of local autonomy is
contemplated by the Constitution?
What about the kind of autonomy
contemplated
insofar
as
the
autonomous regions are concerned?
Held:
1.
The principle of local
autonomy under the 1987 Constitution
simply means decentralization. It
does not make local governments
sovereign within the state or an
imperium in imperio. Remaining to
be an intra sovereign subdivision of
one
sovereign
nation,
but
not
intended, however, to be an imperium
in imperio, the local government unit
is autonomous in the sense that it is
given
more
powers,
authority,
responsibilities and resources. Power
which used to be highly centralized in
Manila, is thereby deconcentrated,
enabling especially the peripheral
local government units to develop not
only at their own pace and discretion
but also with their own resources and
assets. (Alvarez v. Guingona, Jr.,
252 SCRA 695, Jan. 31, 1996, En
Banc [Hermosisima])
2.
The constitutional guarantee of
local autonomy in the Constitution
refers to the administrative autonomy
of local government units or, cast in
more
technical
language,
the
decentralization
of
government
authority.
On the other hand, the creation of
autonomous
regions
in
Muslim
Mindanao and the Cordilleras, which is
peculiar to the 1987 Constitution,
contemplates the grant of political
autonomy and not just administrative
autonomy to these regions. Thus, the
provision in the Constitution for an
autonomous regional government with
a basic structure consisting of an
executive department and a legislative
assembly and special courts with
personal, family and property law
jurisdiction in each of the autonomous
regions. (Cordillera Broad Coalition
v. COA, 181 SCRA 495, Jan. 29,
1990, En Banc [Cortes])
333. Whether or not the Internal
Revenue allotments (IRAs) are to be
included in the computation of the
average
annual
income
of
a
municipality for purposes of its

conversion
into
component city?

an

independent

Held: Yes. The IRAs are items of


income because they form part of the
gross accretion of the funds of the
local government unit.
The IRAs
regularly and automatically accrue to
the local treasury without need of any
further action on the part of the local
government unit. They thus constitute
income which the local government
can invariably rely upon as the source
of much needed funds.
Xxx
[T]o reiterate, IRAs are a regular,
recurring item of income; nil is there a
basis, too, to classify the same as a
special fund or transfer, since IRAs
have a technical definition and
meaning all its own as used in the
Local
Government
Code
that
unequivocally makes it distinct from
special funds or transfers referred to
when the Code speaks of funding
support from the national government,
its instrumentalities and governmentowned or controlled corporations.
Thus, Department of Finance Order
No. 35-93 correctly encapsulizes the
full import of the above disquisition
when it defined ANNUAL INCOME to be
revenues and receipts realized by
provinces, cities and municipalities
from regular sources of the Local
General Fund including the internal
revenue allotment and other shares
provided for in Sections 284, 290 and
291 of the Code, but exclusive of nonrecurring receipts, such as other
national
aids,
grants,
financial
assistance, loan proceeds, sales of
fixed assets, and similar others. Such
order,
constituting
executive
or
contemporaneous construction of a
statute by an administrative agency
charged with the task of interpreting
and applying the same, is entitled to
full respect and should be accorded
great weight by the courts, unless
such construction is clearly shown to
be in sharp conflict with the
Constitution, the governing statute, or
other laws. (Alvarez v. Guingona,
Jr., 252 SCRA 695, Jan. 31, 1996,
En Banc [Hermosisima, Jr., J.])
334. State the importance of drawing
with precise strokes the territorial
boundaries of a local government unit.
Held: The importance of drawing with
precise
strokes
the
territorial
boundaries of a local unit of

government
cannot
be
overemphasized.
The boundaries
must be clear for they define the limits
of the territorial jurisdiction of a local
government unit. It can legitimately
exercise powers of government only
within the limits of its territorial
jurisdiction. Beyond these limits, its
acts are ultra vires. Needless to state,
any uncertainty in the boundaries of
local government units will sow costly
conflicts
in
the
exercise
of
governmental powers which ultimately
will prejudice the peoples welfare.
This is the evil sought to be avoided
by the Local Government Code in
requiring that the land area of a local
government unit must be spelled out
in metes and bounds, with technical
descriptions.
(Mariano,
Jr.
v.
COMELEC, 242 SCRA 211, 217-219,
Mar. 7, 1995, En Banc [Puno])
335. R.A.
7854
was
enacted
converting the Municipality of Makati
into a highly urbanized city. Section 2
thereof did not provide for a cadastral
type of description of its boundary but
merely provided that the boundary of
the new city of Makati shall be the
boundary of the present municipality
of Makati. Petitioners contended in a
petition brought the SC that R.A. 7854
was defective because it did not
comply with the requirement in the
Local Government Code that the
territorial jurisdiction of newly created
or
converted
cities
should
be
described by metes and bounds, with
technical descriptions. Note that at
the time the law was enacted, there
was a pending boundary dispute
between Makati and one of its
neighbors, Taguig, before the regular
court.
Should the contention be
upheld?
Held: Given the facts of the cases at
bench, we cannot perceive how this
evil (uncertainty in the boundaries of
local government units will sow costly
conflicts in the exercise of government
powers which ultimately will prejudice
the peoples welfare) can be brought
about by the description made in
Section 2 of R.A. No. 7854. Petitioners
have not demonstrated that the
delineation of the land area of the
proposed City of Makati will cause
confusion as to its boundaries. We
note that said delineation did not
change even by an inch the land area
previously covered by Makati as a
municipality. Section 2 did not add,
subtract, divide, or multiply the
established land area of Makati. In
language that cannot be any clearer,

Section 2 stated that the citys land


area shall comprise the present
territory of the municipality.
The deliberations of Congress will
reveal that there is a legitimate reason
why the land area of the proposed City
of Makati was not defined by metes
and
bounds,
with
technical
descriptions.
At the time of the
consideration of R.A. No. 7854, the
territorial
dispute
between
the
municipalities of Makati and Taguig
over Fort Bonifacio was under court
litigation. Out of a becoming sense of
respect to a co-equal department of
government, the legislators felt that
the dispute should be left to the courts
to decide.
They did not want to
foreclose the dispute by making a
legislative finding of fact which could
decide the issue. This would have
ensued if they defined the land area of
the proposed city by its exact metes
and
bounds,
with
technical
descriptions. We take judicial notice of
the fact that Congress has also
refrained from using the metes and
bounds description of the land area of
other local government units with
unsettled boundary disputes.
We hold that the existence of a
boundary dispute does not per se
present an insurmountable difficulty
which will prevent Congress from
defining with reasonable certitude the
territorial jurisdiction of a local
government unit.
In the cases at
bench, Congress maintained the
existing boundaries of the proposed
City of Makati but as an act of fairness,
made them subject to the ultimate
resolution by the courts. Considering
these peculiar circumstances, we are
not prepared to hold that Section 2 of
R.A. No. 7854 is unconstitutional. We
sustain the submission of the Solicitor
General in this regard, viz:
Going now to Sections 7 and 450 of
the Local Government Code, it is
beyond cavil that the requirement
started therein, viz: the territorial
jurisdiction of newly created or
converted cities should be described
by metes and bounds, with technical
descriptions was made in order to
provide a means by which the area of
said
cities
may
be
reasonably
ascertained.
In other words, the
requirement on metes and bounds was
meant merely as a tool in the
establishment of local government
units. It is not an end in itself. Ergo,
so long as the territorial jurisdiction of
a city may be reasonably ascertained,

i.e.,
by
referring
to
common
boundaries
with
neighboring
municipalities, as in this case, then, it
may be concluded that the legislative
intent behind the law has been
sufficiently served.
Certainly, Congress did not intend that
laws creating new cities must contain
therein detailed technical descriptions
similar to those appearing in Torrens
titles, as petitioners seem to imply. To
require such description in the law as
a condition sine qua non for its validity
would be to defeat the very purpose
which the Local Government Code
seeks to serve. The manifest intent of
the Code is to empower local
government units and to give them
their rightful due. It seeks to make
local governments more responsive to
the needs of their constituents while
at the same time serving as a vital cog
in national development. To invalidate
R.A. No. 7854 on the mere ground that
no cadastral type of description was
used in the law would serve the letter
but defeat the spirit of the Code. It
then becomes a case of the master
serving the slave, instead of the other
way around. This could not be the
intendment of the law. X x x
(Mariano, Jr. v. COMELEC, 242
SCRA 211, 217-219, Mar. 7, 1995,
En Banc [Puno])
336. What is
"devolution"?

the

meaning

of

Answer: The term "devolution" refers


to the act by which the National
government
confers
power
and
authority upon the various local
government units to perform specific
functions and responsibilities. (Sec.
17[e], 2nd par., Local Government
Code)
337. Have the powers of the Land
Transportation Office (LTO) to register,
tricycles in particular, as well as to
issue licenses for the driving thereof,
been devolved likewise to local
government units?
Held: Only the powers of the Land
Transportation Franchising Regulatory
Board
(LTFRB)
to
regulate
the
operation of tricycles-for-hire and to
grant franchises for the operation
thereof have been devolved to local
governments
under
the
Local
Government Code. Clearly unaffected
by the Local Government Code are the
powers of the LTO under R.A. No. 4136
requiring the registration of all kinds of
motor vehicles used or operated on

or upon any public highway in the


country. This can be gleaned from the
explicit language of the statute itself,
as
well
as
the
corresponding
guidelines issued by the DOTC. In
fact, even the power of LGUs to
regulate the operation of tricycles and
to grant franchises for the operation
thereof is still subject to the guidelines
prescribed by the DOTC. (LTO v. City
of Butuan, G.R. No. 131512, Jan.
20, 2000, 3rd Div. [Vitug])

specifically empowered by law to


supervise the profession, in this case
the
Professional
Regulations
Commission
and
the
Board
of
Examiners in Optometry. (Acebedo
Optical Company, Inc. v. CA, 329
SCRA 314, March 31, 2000, En
Banc [Purisima])
339. May a local government unit
validly authorize an expropriation of
private property through a mere
resolution of its lawmaking body?

338. Distinguish the power to grant a


license or permit to do business and
the power to issue a license to engage
in the practice of a particular
profession.

Held: The Local Government Code


expressly and clearly requires an
ordinance or a local law for that
purpose.
A resolution that merely
expresses the sentiment or opinion of
the Municipal Council will not suffice.
The case of Province of Camarines Sur
v. Court of Appeals which held that a
mere resolution may suffice to support
the exercise of eminent domain by a
local government unit is not in point
because the applicable law at that
time was B.P. 337, the previous Local
Government Code, which had provided
that a mere resolution would enable
an LGU to exercise eminent domain. In
contrast, R.A. 7160, the present Local
Government Code, explicitly required
an ordinance for this purpose.
(Municipality of Paranaque v. V.M.
Realty Corp., 292 SCRA 678, July
20, 1998 [Panganiban])

Held:
Distinction must be made
between the grant of a license or
permit to do business and the
issuance of a license to engage in the
practice of a particular profession.
The first is usually granted by the local
authorities and the second is issued by
the Board or Commission tasked to
regulate the particular profession. A
business permit authorizes the person,
natural or otherwise, to engage in
business or some form of commercial
activity. A professional license, on the
other hand, is the grant of authority to
a natural person to engage in the
practice or exercise of his or her
profession.
In the case at bar, what is sought by
petitioner (Acebedo Optical Company,
Inc.) from respondent City Mayor is a
permit to engage in the business of
running an optical shop. It does not
purport to seek a license to engage in
the practice of optometry as a
corporate body or entity, although it
does have in its employ, persons who
are
duly
licensed
to
practice
optometry by the Board of Examiners
in Optometry.
Xxx
In the present case, the objective of
the imposition of subject conditions on
petitioners business permit could be
attained by requiring the optometrists
in petitioners employ to produce a
valid certificate of registration as
optometrists, from the Board of
Examiners in Optometry. A business
permit is issued primarily to regulate
the conduct of business and the City
Mayor cannot, through the issuance of
such permit, regulate the practice of a
profession, like that of optometry.
Such a function is within the exclusive
domain of the administrative agency

340. What are the requisites before a


Local Government Unit can validly
exercise the power of eminent
domain?
Held:
1)

An ordinance is enacted by
the local legislative council authorizing
the local chief executive, in behalf of
the LGU, to exercise the power of
eminent
domain
or
pursue
expropriation proceedings over a
particular private property;
2)
The
power
of
eminent
domain is exercised for public use,
purpose or welfare, or for the benefit
of the poor and the landless;
3)
There is payment of just
compensation, as required under
Section
9,
Article
III
of
the
Constitution, and other pertinent laws;
4)
A valid and definite offer has
been previously made to the owner of
the
property
sought
to
be
expropriated, but said offer was not
accepted.
(Municipality of Paranaque v. V.M.
Realty Corp., 292 SCRA 678, July
20, 1998 [Panganiban])

341. May
the
Sangguniang
Panlalawigan validly disapprove a
resolution
or
ordinance
of
a
municipality
calling
for
the
expropriation of private property to be
made site of a Farmers Center and
Other Government Sports Facilities on
the ground that said expropriation is
unnecessary considering that there
are still available lots of the
municipality for the establishment of a
government center?
Held: Under the Local Government
Code, the Sangguniang Panlalawigan
is granted the power to declare a
municipal resolution invalid on the
sole ground that it is beyond the
power of the Sangguniang Bayan or
Mayor to issue. As held in Velazco v.
Blas (G.R. No. L-30456, July 30, 1982,
115 SCRA 540, 544-545), The only
ground upon which a provincial board
may declare any municipal resolution,
ordinance or order invalid is when
such resolution, ordinance, or order is
beyond the powers conferred upon
the council or president making the
same.
A strictly legal question is
before the provincial board in its
consideration
of
a
municipal
resolution, ordinance, or order. The
provincial boards disapproval of any
resolution, ordinance, or order must be
premised specifically upon the fact
that such resolution, ordinance, or
order is outside the scope of the legal
powers conferred by law.
If a
provincial board passes these limits, it
usurps the legislative functions of the
municipal council or president. Such
has been the consistent course of
executive authority. (Moday v. CA,
268 SCRA 586, Feb. 20, 1997)
342. Under Section 8, Article X of the
Constitution, "[T]he term of office of
elective local officials x x x shall be
three years and no such official shall
serve for more than three consecutive
terms." How is this term limit for
elective
local
officials
to
be
interpreted?
Held: The term limit for elective local
officials must be taken to refer to the
right to be elected as well as the right
to serve in the same elective position.
Consequently, it is not enough that an
individual
has
served
three
consecutive terms in an elective local
office, he must also have been elected
to the same position for the same
number
of
times
before
the
disqualification can apply. (Borja, Jr.
v. COMELEC and Capco, Jr., G.R.

No. 133495, Sept. 3, 1998, 295


SCRA 157, En Banc [Mendoza])
Case No. 1. Suppose A is a vicemayor who becomes mayor by reason
of the death of the incumbent. Six
months before the next election, he
resigns and is twice elected thereafter.
Can he run again for mayor in the next
election?
Answer: Yes, because although he
has already first served as mayor by
succession and subsequently resigned
from office before the full term
expired, he has not actually served
three full terms in all for the purpose
of applying the term limit. Under Art.
X, Sec. 8, voluntary renunciation of the
office is not considered as an
interruption in the continuity of his
service for the full term only if the
term is one for which he was
elected. Since A is only completing
the service of the term for which the
deceased and not he was elected, A
cannot be
considered
to
have
completed one term. His resignation
constitutes an interruption of the full
term.
Case No. 2. Suppose B is elected
Mayor and, during his first term, he is
twice suspended for misconduct for a
total of 1 year. If he is twice reelected
after that, can he run for one more
term in the next election?
Answer: Yes, because he has served
only two full terms successively.
In both cases, the mayor is entitled to
run for reelection because the two
conditions for the application of the
disqualification provisions have not
concurred, namely, that the local
official concerned has been elected
three consecutive times and that he
has fully served three consecutive
terms. In the first case, even if the
local official is considered to have
served
three
full
terms
notwithstanding his resignation before
the end of the first term, the fact
remains that he has not been elected
three times. In the second case, the
local official has been elected three
consecutive times, but he has not fully
served three consecutive terms.
Case No. 3. The case of vice-mayor
C who becomes mayor by succession
involves a total failure of the two
conditions to concur for the purpose of
applying Art. X, Sec. 8. Suppose he is
twice elected after that term, is he

qualified to run again in the next


election?
Answer: Yes, because he was not
elected to the office of mayor in the
first term but simply found himself
thrust into it by operation of law.
Neither had he served the full term
because he only continued the
service, interrupted by the death, of
the deceased mayor. (Borja, Jr. v.
COMELEC and Capco, Jr., G.R. No.
133495, Sept. 3, 1998, 295 SCRA
157, En Banc [Mendoza])
343. What are the policies embodied
in the constitutional provision barring
elective local officials, with the
exception of barangay officials, from
serving more than three consecutive
terms?
Held: To prevent the establishment of
political dynasties is not the only
policy embodied in the constitutional
provision in question (barring elective
local officials, with the exception of
barangay officials, from serving more
than three consecutive terms). The
other policy is that of enhancing the
freedom of choice of the people. To
consider, therefore, only stay in office
regardless
of
how
the
official
concerned came to that office
whether by election or by succession
by operation of law would be to
disregard one of the purposes of the
constitutional provision in question.
(Borja, Jr. v. COMELEC and Capco,
Jr., G.R. No. 133495, Sept. 3, 1998,
295
SCRA
157,
En
Banc
[Mendoza])
344.
Lonzanida was previously
elected and served two consecutive
terms as mayor of San Antonio,
Zambales prior to the May 1995
mayoral elections. In the May 1995
elections he again ran for mayor of
San Antonio, Zambales and was
proclaimed winner. He assumed office
and discharged the rights and duties
of mayor until March 1998 when he
was ordered to vacate the post by
reason of the COMELEC decision on
the election protest against him which
declared his opponent Juan Alvez the
duly elected mayor. Alvez served the
remaining portion of the 1995-1998
mayoral term.
Is Lonzanida still
qualified to run for mayor of San
Antonio, Zambales in the May 1998
local elections?
Held:
The two requisites for the
application of the three term rule was
absent. First, Lonzanida cannot be

considered as having been duly


elected to the post in the May 1995
elections, and second, he did not fully
serve the 1995-1998 mayoral term by
reason of involuntary relinquishment
of office. After a re-appreciation and
revision of the contested ballots the
COMELEC itself declared by final
judgment that Lonzanida lost in the
May 1995 mayoral elections and his
previous proclamation as winner was
declared
null
and
void.
His
assumption of office as mayor cannot
be deemed to have been by reason of
a valid election but by reason of a void
proclamation. It has been repeatedly
held by the SC that a proclamation
subsequently declared void is no
proclamation at all and while a
proclaimed candidate may assume
office on the strength of the
proclamation
of
the
Board
of
Canvassers he is only a presumptive
winner who assumes office subject to
the final outcome of the election
protest. Lonzanida did not serve a
term as mayor of San Antonio,
Zambales from May 1995 to March
1998 because he was not duly elected
to the post; he merely assumed office
as
presumptive
winner,
which
presumption was later overturned by
the COMELEC when it decided with
finality that Lonzanida lost in the May
1995 mayoral elections.
Second, Lonzanida cannot be deemed
to have served the May 1995 to 1998
term because he was ordered to
vacate his post before the expiration
of the term.
His opponents'
contention that Lonzanida should be
deemed to have served one full term
from May 1995-1998 because he
served the greater portion of that term
has no legal basis to support it; it
disregards the second requisite for the
application of the disqualification, i.e.,
that he has fully served three
consecutive terms.
The second
sentence
of
the
constitutional
provision
under
scrutiny
states,
"Voluntary renunciation of office for
any length of time shall not be
considered as an interruption in the
continuity of service for the full term
for which he was elected." The clear
intent
of
the
framers
of
the
Constitution to bar any attempt to
circumvent the three-term limit by a
voluntary renunciation of office and at
the same time respect the people's
choice and grant their elected official
full service of a term is evident in this
provision. Voluntary renunciation of a
term does not cancel the renounced
term in the computation of the three

term limit; conversely, involuntary


severance from office for any length of
time short of the full term provided by
law amounts to an interruption of
continuity of service.
Lonzanida
vacated his post a few months before
the next mayoral elections, not by
voluntary
renunciation
but
in
compliance with the legal process of
writ of execution issued by the
COMELEC to that effect.
Such
involuntary severance from office is an
interruption of continuity of service
and thus, Lonzanida did not fully serve
the 1995-1998 mayoral term.
In sum, Lonzanida was not the duly
elected mayor and that he did not
hold office for the full term; hence, his
assumption of office from May 1995 to
March 1998 cannot be counted as a
term for purposes of computing the
three term limit.
(Lonzanida v.
COMELEC, 311 SCRA 602, July 28,
1999, En Banc [Gonzaga-Reyes])
345. May
the
President
validly
withhold a portion of the internal
revenue
allotments
of
Local
Government Units legally due them by
administrative fiat?
Held:
The Constitution vests the
President
with
the
power
of
supervision, not control, over local
government units (LGUs). Such power
enables him to see to it that LGUs and
their officials execute their tasks in
accordance with law. While he may
issue advisories and seek their
cooperation in solving economic
difficulties, he cannot prevent them
from performing their tasks and using
available resources to achieve their
goals. He may not withhold or alter
any authority or power given them by
the law. Thus, the withholding of a
portion of internal revenue allotments
legally due them cannot be directed
by administrative fiat.
Xxx
Section 4 of AO 372 cannot x x x be
upheld. A basic feature of local fiscal
autonomy is the automatic release of
the shares of LGUs in the National
internal revenue. This is mandated by
no less than the Constitution. The
Local Government Code (Sec. 286[a])
specifies further that the release shall
be made directly to the LGU
concerned within five (5) days after
every quarter of the year and shall
not be subject to any lien or holdback
that may be imposed by the national
government for whatever purpose.

As a rule, the term shall is a word of


command that must be given a
compulsory meaning. The provision
is, therefore, imperative.
Section 4 of AO 372, however, orders
the withholding, effective January 1,
1998, of 10 percent of the LGUs IRA
pending
the
assessment
and
evaluation
by
the
Development
Budget Coordinating Committee of the
emerging fiscal situation in the
country.
Such withholding clearly
contravenes the Constitution and the
law.
Although, temporary, it is
equivalent to a holdback, which means
something held back or withheld.
Often temporarily.
Hence, the
temporary nature of the retention by
the national government does not
matter. Any retention is prohibited.
In sum, while Section 1 of AO 372 may
be upheld as an advisory effected in
times of national crisis, Section 4
thereof has no color of validity at all.
The
latter
provision
effectively
encroaches on the fiscal autonomy of
local governments. Concededly, the
President was well-intentioned in
issuing his Order to withhold the LGUs
IRA, but the rule of law requires that
even the best intentions must be
carried out within the parameters of
the Constitution and the law. Verily,
laudable purposes must be carried out
by legal methods. (Pimentel, Jr. v.
Aguirre, G.R. No. 132988, 336
SCRA 201, July 19, 2000, En Banc
[Panganiban])
346. What is meant by fiscal
autonomy of Local Governments?
Does it rule out in any manner
national government intervention by
way of supervision in order to ensure
that local programs are consistent with
national goals?
Held:
Under existing law, local
government units, in addition to
having administrative autonomy in the
exercise of their functions, enjoy fiscal
autonomy as well. Fiscal autonomy
means that local governments have
the power to create their own sources
of revenue in addition to their
equitable share in the national taxes
released by the national government,
as well as the power to allocate their
resources in accordance with their own
priorities.
It
extends
to
the
preparation of their budgets, and local
officials in turn have to work within the
constraints thereof.
They are not
formulated at the national level and
imposed
on
local
governments,

whether they are relevant to local


needs and resources or not. Hence,
the necessity of a balancing of
viewpoints and the harmonization of
proposals from both local and national
officials, who in any case are partners
in the attainment of national goals.
Local fiscal autonomy does not,
however, rule out any manner of
national government intervention by
way of supervision, in order to ensure
that local programs, fiscal and
otherwise, are consistent with national
goals. Significantly, the President, by
constitutional fiat, is the head of the
economic and planning agency of the
government (Section 9, Article XII of
the Constitution), primarily responsible
for formulating and implementing
continuing, coordinated and integrated
social and economic policies, plans
and programs (Section 3, Chapter 1,
Subtitle C, Title II, Book V, EO 292
[Administrative Code of 1987]) for the
entire country. However, under the
Constitution, the formulation and the
implementation of such policies and
programs are subject to consultations
with the appropriate public agencies,
various private sectors, and local
government units.
The President
cannot do so unilaterally. (Pimentel,
Jr. v. Aguirre, 336 SCRA 201, July
19, 2000, En Banc [Panganiban])
347. What are the requisites before
the President may interfere in local
fiscal matters?
Held: x x x [T]he Local Government
Code provides (Sec. 284. See also Art.
379 of the Rules and Regulations
Implementing the Local Government
Code of 1991):
x x x [I]n the event the national
government incurs an unmanaged
public sector deficit, the President of
the Philippines is hereby authorized,
upon the recommendation of [the]
Secretary of Finance, Secretary of the
Interior and Local Government and
Secretary of Budget and Management,
and subject to consultation with the
presiding officers of both Houses of
Congress and the presidents of the
liga,
to
make
the
necessary
adjustments in the internal revenue
allotment of local government units
but in no case shall the allotment be
less than thirty percent (30%) of the
collection of national internal revenue
taxes of the third fiscal year preceding
the current fiscal year x x x

There are therefore several requisites


before the President may interfere in
local fiscal matters: (1) an unmanaged
public sector deficit of the national
government; (2) consultations with the
presiding officers of the Senate and
the House of Representatives and the
presidents of the various local
leagues; and (3) the corresponding
recommendation of the secretaries of
the Department of Finance, Interior
and Local Government, and Budget
and Management. Furthermore, any
adjustment in the allotment shall in no
case be less than thirty percent (30%)
of the collection of national internal
revenue taxes of the third fiscal year
preceding the current one. (Pimentel,
Jr. v. Aguirre, 336 SCRA 201, July
19, 2000, En Banc [Panganiban])
348. Distinguish an ordinance from a
mere resolution.
Held:
A municipal ordinance is
different from a resolution.
An
ordinance is a law, but a resolution is
merely a declaration of the sentiment
or opinion of a lawmaking body on a
specific matter.
An ordinance
possesses a general and permanent
character,
but
a
resolution
is
temporary in nature. Additionally, the
two are enacted differently a third
reading is necessary for an ordinance,
but not for a resolution, unless
decided otherwise by a majority of all
the
Sanggunian
members.
(Municipality of Paranaque v. V.M.
Realty Corporation, 292 SCRA 678,
July 20, 1998 [Panganiban])
349. On its first regular session, may
the Sanggunian transact business
other than the matter of adopting or
updating
its
existing
rules
or
procedure?
Held: We cannot infer the mandate of
the (Local Government) Code that no
other business may be transacted on
the first regular session except to take
up the matter of adopting or updating
rules. All that the law requires is that
on the first regular session x x x the
sanggunian concerned shall adopt or
update
its
existing
rules
or
procedures. There is nothing in the
language thereof that restricts the
matters to be taken up during the first
regular session merely to the adoption
or updating of the house rules. If it
were the intent of Congress to limit
the business of the local council to
such matters, then it would have done
so in clear and unequivocal terms. But
as it is, there is no such intent.

Moreover, adopting or updating of


house rules would necessarily entail
work beyond the day of the first
regular session. Does this mean that
prior thereto, the local council's hands
were tied and could not act on any
other matter? That would certainly be
absurd for it would result in a hiatus
and
a
paralysis
in
the
local
legislature's work which could not
have been intended by the law.
(Malonzo v. Zamora, 311 SCRA
224, July 27, 1999, En Banc
[Romero])
350. May
an
incumbent
ViceGovernor, while concurrently the
Acting Governor, continue to preside
over the sessions of the Sangguniang
Panlalawigan (SP)? If no, who may
preside in the meantime?
Held: Being the acting governor, the
Vice-governor cannot continue to
simultaneously exercise the duties of
the latter office, since the nature of
the duties of the Provincial Governor
calls for a full-time occupant to
discharge them.
Such is not only
consistent with but also appears to be
the clear rationale of the new (Local
Government) Code wherein the policy
of performing dual functions in both
offices has already been abandoned.
To repeat, the creation of a temporary
vacancy in the office of the Governor
creates a corresponding vacancy in
the office of the Vice-Governor
whenever the latter acts as Governor
by virtue of such temporary vacancy.
This event constitutes an inability on
the part of the regular presiding officer
(Vice-Governor) to preside during the
SP sessions, which thus calls for the
operation of the remedy set in Article
49(b) of the Local Government Code
concerning the election of a temporary
presiding officer. The continuity of the
Acting
Governors
(Vice-Governor)
powers as presiding officer of the SP is
suspended so long as he is in such
capacity. Under Section 49(b), in the
event of the inability of the regular
presiding officer to preside at the
sanggunian session, the members
present and constituting a quorum
shall elect from among themselves a
temporary
presiding
officer.
(Gamboa, Jr. v. Aguirre, Jr., G.R.
No. 134213, July 20, 1999, En
Banc [Ynares-Santiago])
351.

What is recall?

Held: Recall is a mode of removal of


a public officer by the people before

the end of his term of office. The


people's prerogative to remove a
public officer is an incident of their
sovereign power and in the absence of
constitutional restraint, the power is
implied
in
all
governmental
operations. Such power has been held
to be indispensable for the proper
administration of public affairs. Not
undeservedly,
it
is
frequently
described as a fundamental right of
the people in a representative
democracy.
(Garcia v. COMELEC,
227 SCRA 108, Oct. 5, 1993, En
Banc [Puno])
352. What is the ground for recall? Is
this subject to judicial inquiry?
Held:
Former Senator Aquilino
Pimentel, Jr., a major author of the
subject law in his book The Local
Government Code of 1991: The Key to
National Development, stressed the
same reason why the substantive
content of a vote of lack of confidence
is beyond any inquiry, thus:
There is only one ground for recall of
local government officials: loss of
confidence.
This means that the
people may petition or the Preparatory
Recall Assembly may resolve to recall
any local elective official without
specifying any particular ground
except loss of confidence. There is no
need for them to bring up any charge
of abuse or corruption against the
local elective officials who are subject
of any recall petition.
In the case of Evardone v. Commission
on Elections, et al., 204 SCRA 464, 472
(1991), the Court ruled that loss of
confidence as a ground for recall is a
political question. In the words of the
Court, 'whether or not the electorate
of the municipality of Sulat has lost
confidence in the incumbent mayor is
a political question.
(Garcia v. COMELEC, 227 SCRA
108, Oct. 5, 1993, En Banc [Puno])
353. The members of the Preparatory
Recall Assembly (PRA) of the province
of Bataan adopted a resolution calling
for the recall of Governor Garcia. It
was admitted, however, by the
proponents of the recall resolution that
only those members of the assembly
inclined to agree were notified of the
meeting where said resolution was
adopted as a matter of strategy and
security.
They justified these
selective notices on the ground that
the law (Local Government Code) does
not specifically mandate the giving of

notice.
Should this submission be
sustained?

No. 154512, Nov. 12, 2002, En


Banc [Carpio])

Held: We reject this submission of


the respondents.
The due process
clause of the Constitution requiring
notice as an element of fairness is
inviolable and should always be
considered part and parcel of every
law in case of its silence. The need for
notice to all the members of the
assembly is also imperative for these
members represent the different
sectors of the electorate of Bataan. To
the extent that they are not notified of
the meeting of the assembly, to that
extent is the sovereign voice of the
people they represent nullified. The
resolution to recall should articulate
the majority will of the members of the
assembly but the majority will can be
genuinely determined only after all the
members of the assembly have been
given a fair opportunity to express the
will of their constituents. Needless to
stress, the requirement of notice is
mandatory for it is indispensable in
determining the collective wisdom of
the members of the Preparatory Recall
Assembly. Its non-observance is fatal
to the validity of the resolution to
recall petitioner Garcia as Governor of
the province of Bataan. (Garcia v.
COMELEC, G.R. No. 111511, Sept.
21, 1993; 227 SCRA 100, Oct. 5,
1993, En Banc [Puno]

355. Whether or not an elective


official who became City Mayor by
legal succession can be the subject of
a recall election by virtue of a
Preparatory Recall Assembly
Resolution which was passed or
adopted when the said elective official
was still the Vice-Mayor.

354. The members of the Preparatory


Recall Assembly (PRA) of Puerto
Princesa City met and adopted a
resolution calling for the recall of
incumbent Mayor Dennis Victorino M.
Socrates on the ground of loss of
confidence on July 2, 2002. Mayor
Socrates argued that they have no
authority to adopt said Recall
Resolution because a majority of PRA
members were seeking a new
electoral mandate in the barangay
elections scheduled on July 15, 2002.
Should his contention be sustained?
Held: This argument deserves scant
consideration considering that when
the PRA members adopted the Recall
Resolution their terms of office had not
yet expired. They were all de jure
sangguniang barangay members with
no legal disqualification to participate
in the recall assembly under Section
70 of the Local Government Code.
(Victorino Dennis M. Socrates v.
The Commission on Elections, G.R.

Held: The specific purpose of


the Preparatory Recall Assembly was
to remove Amelita S. Navarro as the
elected Vice-Mayor of Santiago City
since PRA Resolution No. 1 dated July
12, 1999 expressly states that x x x
it is hereby resolved to invoke the
rescission of the electoral mandate of
the incumbent City Vice-Mayor Amelita
S. Navarro for loss of confidence
through a recall election to be set by
the Commission on Election as
provided for under Section 71 of the
Local Government Code of 1991.
However, the said PRA Resolution No.
1 is no longer applicable to her
inasmuch as she had already vacated
the office of Vice-Mayor on October
11, 1999 when she assumed the
position of City Mayor of Santiago City.
Even if the Preparatory Recall
Assembly were to reconvene to adopt
another resolution for the recall of
Amelita Navarro, this time as Mayor of
Santiago City, the same would still not
prosper in view of Section 74 (b) of the
Local Government Code of 1991 which
provides that No recall shall take
place within one (1) year from the
date of the officials assumption of
office or one (1) year immediately
preceding a regular election. There is
no more allowable time in the light of
that law within which to hold recall
elections for that purpose. The then
Vice-Mayor
Amelita
S.
Navarro
assumed office as Mayor of Santiago
City on October 11, 1999. One year
after her assumption of office as
Mayor will be October 11, 2000 which
is already within the one (1) year
prohibited
period
immediately
preceding the next regular election in
May 2001. (Afiado v. Commission
on Elections, 340 SCRA 600, Sept.
18, 2000, En Banc [De Leon]

356. Mayor Edward S. Hagedorn of


Puerto Princesa City was elected for
three consecutive times in the 1992,
1995 and 1998 elections and served in
full his three consecutive terms as
Mayor. In the 2001 elections, he ran
for Governor of the Province of
Palawan and lost. Socrates ran and
won as Mayor of Puerto Princesa in
that election. On July 2, 2002, the
Preparatory Recall Assembly (PRA) of
Puerto Princesa City adopted a
Resolution calling for the recall of
incumbent Mayor Socrates. The
COMELEC scheduled a Special Recall
Election for Mayor of that City on
September 24, 2002. Is Mayor
Hagedorn qualified to run again for
Mayor in that Special Recall Election
considering the circumstances?
Held: The three-term limit
rule for elective local officials is
found in Section 8, Article X of the
Constitution x x x.
This three-term limit rule is
reiterated in Section 43 (b) of RA No.
7160, otherwise known as the Local
Government Code x x x.
These
constitutional
and
statutory provisions have two parts.
The first part provides that an elective
local official cannot serve for more
than three consecutive terms. The
clear intent is that only consecutive
terms count in determining the threeterm limit rule. The second part states
that voluntary renunciation of office
for any length of time does not
interrupt the continuity of service.
The clear intent is that involuntary
severance from office for any length of
time interrupts continuity of service
and prevents the service before and
after the interruption from being
joined together to form a continuous
service or consecutive terms.
After three consecutive terms,
an elective local official cannot seek
immediate reelection for a fourth
term. The prohibited election refers to
the next regular election for the same
office following the end of the third
consecutive term.
Any subsequent
election, like a recall election, is no
longer covered by the prohibition for
two reasons.
First, a subsequent
election like a recall election is no
longer an immediate reelection after
three consecutive terms. Second, the

intervening period constitutes an


involuntary
interruption
in
the
continuity of service.
Xxx
Clearly, what the Constitution
prohibits is an immediate reelection
for a fourth term following three
consecutive terms. The Constitution,
however,
does
not
prohibit
a
subsequent reelection for a fourth
term as long as the reelection is not
immediately after the end of the third
consecutive term. A recall election
mid-way in the term following the third
consecutive term is a subsequent
election but not an immediate
reelection after the third term.
Neither does the Constitution
prohibit one barred from seeking
immediate reelection to run in any
other subsequent election involving
the same term of office. What the
Constitution prohibits is a consecutive
fourth term.
The debates in the
Constitutional Commission evidently
show that the prohibited election
referred to by the framers of the
Constitution
is
the
immediate
reelection after the third term, not any
other subsequent election.
Xxx
In the case of Hagedorn, his
candidacy in the recall election on
September 24, 2002 is not an
immediate reelection after his third
consecutive term which ended on June
30, 2001. The immediate reelection
that the Constitution barred Hagedorn
from seeking referred to the regular
elections in 2001. Hagedorn did not
seek reelection in the 2001 elections.
Xxx
From June 30, 2001 until the
recall election on September 24, 2002,
the mayor of Puerto Princesa was
Socrates. This period is clearly an
interruption in the continuity of
Hagedorns service as mayor, not
because of his voluntary renunciation,
but because of a legal prohibition.
Hagedorns three consecutive terms
ended on June 30, 2001. Hagedorns
new recall term from September 24,
2002 to June 30, 2004 is not a
seamless continuation of his previous
three consecutive terms as mayor.
One cannot stitch together Hagedorns

previous three-terms with his new


recall term to make the recall term a
fourth consecutive term because
factually it is not. An involuntary
interruption occurred from June 30,
2001 to September 24, 2002 which
broke the continuity or consecutive
character of Hagedorns service as
mayor.
X x x In Hagedorns case, the nearly
15-month period he was out of office,
although short of a full term of three
years, constituted an interruption in
the continuity of his service as mayor.
The Constitution does not require the
interruption or hiatus to be a full term
of three years. The clear intent is that
interruption for any length of time,
as long as the cause is involuntary, is
sufficient to break an elective local
officials
continuity
of
service.
(Victorino Dennis M. Socrates v.
The Commission on Elections, G.R.
No. 154512, Nov. 12, 2002, En
Banc [Carpio])

357. What acts of a Division of the


COMELEC may be subject of a motion
for reconsideration of the COMELEC en
banc?
Held:
Section 5, Rule 19 of the
COMELEC
Rules
of
Procedure,
provides:
SEC.
5.
How
Motion
for
Reconsideration Disposed of. - Upon
the filing of a motion to reconsider a
decision, resolution, order or ruling of
a Division, the Clerk of Court
concerned shall, within twenty-four
(24) hours from the filing thereof,
notify the presiding Commissioner.
The latter shall within two (2) days
thereafter certify the case to the
Commission en banc.
Under the above-quoted rule,
the acts of a Division that are subject
of a motion for reconsideration must
have a character of finality before the
same can be elevated to the COMELEC
en banc. The elementary rule is that
an order is final in nature if it
completely disposes of the entire case.
But if there is something more to be
done in the case after its issuance,
that order is interlocutory.
As correctly pointed out by
public respondent in its assailed order
of November 29, 1999, the October

11, 1999 did not dispose of the case


completely as there is something more
to be done which is to decide the
election protest. As such, it is the
herein public respondent (Second
Division of the COMELEC) which issued
the interlocutory order of October 11,
1999 that should resolve petitioners
motion for reconsideration, not the
COMELEC en banc (Bulaong v.
COMELEC, First Division, 220 SCRA
745, 749 [1993]). Accordingly, the
applicable rule on the subject is
Section 5(c), Rule 3 of the COMELEC
Rules of Procedure, which states:
Rule 3, Section 5(c). Any motion to
reconsider a decision, resolution, order
or ruling of a Division shall be resolved
by the Commission en banc except
motions on interlocutory orders of the
division, which shall be resolved by
the divisions which issued the order.
That only final orders of a
Division may be raised before the
COMELEC en banc is in accordance
with Article IX-C, Section 3 of the
Constitution which mandates that only
motions for reconsideration of final
decisions shall be decided by the
Commission on Elections en banc,
thus:
Sec. 3. The Commission on Elections
may sit en banc or in two divisions,
and shall promulgate its rules of
procedure in order to expedite
disposition of election cases, including
pre-proclamation controversies. All
such election cases shall be heard and
decided in division, provided that
motions for reconsideration of
decisions shall be decided by the
Commission en banc.
It bears stressing that under
this constitutional
provision, the
COMELEC en banc shall decide
motions for reconsideration only of
decisions of a Division, meaning
those acts of final character. Clearly,
the assailed order denying petitioner's
demurrer
to
evidence,
being
interlocutory, may not, be resolved by
the COMELEC en banc (Ambil, Jr. v.
Commission on elections, G.R. No.
143398, Oct. 25, 2000, 344 SCRA
358). (Gementiza v. Commission
on Elections, 353 SCRA 724,
March 6, 2001, En Banc [SandovalGutierrez])
358. Will it be proper for the
Commission on Elections to act on a

petition for recall signed by just one


person?

initiate the recall, followed by the


taking of votes by the PRA on May 29,
1999 for the purpose of adopting a
resolution to initiate the recall of
Jovito Claudio as Mayor of Pasay City
for loss of confidence, the process of
recall began and, since May 29, 1999
was less than a year after he had
assumed office, the PRA was illegally
convened and all proceedings held
thereafter, including the filing of the
recall petition on July 2, 1999, were
null and void.

Held: A petition for recall signed by


just one person is in violation of the
statutory 25% minimum requirement
as to the number of signatures
supporting any petition for recall. Sec.
69(d) of the Local Government Code of
1991 expressly provides that 'recall of
any elective x x x municipal x x x
official may also be validly initiated
upon petition of at least twenty-five
percent (25%) of the total number of
registered
voters
in
the
local
government unit concerned during the
election in which the local official
sought to be recalled was elected.'
The law is plain and unequivocal as to
what constitutes recall proceedings:
only a petition of at least 25% of the
total number of registered voters may
validly initiate recall proceedings.
(Angobung v. COMELEC, G.R. No.
126576, March 5, 1997)

The COMELEC, on the other hand,


maintains that the process of recall
starts with the filing of the petition for
recall and ends with the conduct of the
recall election, and that, since the
petition for recall in this case was filed
on July 2, 1999, exactly one year and a
day after petitioners assumption of
office, the recall was validly initiated
outside
the
one-year
prohibited
period.

359. Section
74
of
the
Local
Government Code provides that no
recall shall take place within one year
x x x immediately preceding a regular
local election. What does the term
regular local election, as used in this
section, mean?

Both petitioner Claudio and the


COMELEC thus agree that the term
recall as used in Sec. 74 refers to a
process. They disagree only as to
when the process starts for purpose of
the one-year limitation in paragraph
(b) of Sec. 74.

Held:
The term regular local
election under Sec. 74 of the Local
Government Code of 1991 which
provides that no recall shall take
place within one (1) year x x x
immediately preceding a regular local
election refers to one where the
position of the official sought to be
recalled is to be actually contested
and filled by the electorate (Paras v.
Comelec, G.R. No. 123169, Nov. 4,
1996). The one-year time bar will not
apply where the local official sought to
be recalled is a Mayor and the
approaching election is a barangay
election. (Angobung v. COMELEC,
G.R. No. 126576, March 5, 1997)

We can agree that recall is a process


which begins with the convening of
the preparatory recall assembly or the
gathering of the signatures at least
25% of the registered voters of a local
government unit, and then proceeds
to the filing of a recall resolution or
petition with the COMELEC, the
verification of such resolution or
petition, the fixing of the date of the
recall election, and the holding of the
election on the scheduled date.
However, as used in paragraph (b) of
Sec. 74, recall refers to the election
itself by means of which voters decide
whether they should retain their local
official or elect his replacement.

360. Does the word Recall in


paragraph (b) of Section 74 of the
Local Government Code include the
convening of the Preparatory Recall
Assembly and the filing by it of a recall
resolution? Discuss.

Xxx

Held: Petitioner contends that the


term recall in Sec. 74 (b) refers to a
process, in contrast to the term recall
election found in Sec. 74 (a), which
obviously refers to an election. He
claims that when several barangay
chairmen met and convened on May
19, 1999 and unanimously resolved to

To sum up, the term recall in


paragraph (b) refers to the recall
election and not to the preliminary
proceedings to initiate recall
1)

Because Sec. 74 speaks of


limitations on recall which, according
to Sec. 69, is a power which shall be
exercised by the registered voters of a
local government unit.
Since the
voters do not exercise such right
except in an election, it is clear that
the initiation of recall proceedings is

not prohibited within the one-year


period provided in paragraph (b);
2)
Because the purpose of the
first limitation in paragraph (b) is to
provide voters a sufficient basis for
judging an elective local official, and
final judging is not done until the day
of the election; and
3)
Because to construe the
limitation in paragraph (b) as including
the initiation of recall proceedings
would unduly curtail freedom of
speech and of assembly guaranteed in
the Constitution.
(Jovito O. Claudio v. COMELEC,
G.R. No. 140560, May 4, 2000, En
Banc [Mendoza])
361. Who has the legal authority to
represent a municipality in lawsuits?
Held:
Only the provincial fiscal,
provincial attorney, and municipal
attorney
should
represent
a
municipality in its lawsuits. Only in
exceptional instances may a private
attorney be hired by a municipality to
represent it in lawsuits. (Ramos v.
CA, 269 SCRA 34, March 3, 1997)
362. What
are
the
exceptional
instances when a private attorney may
be validly hired by a municipality in its
lawsuits?
Held: In Alinsug v. RTC Br. 58, San
Carlos City, Negros Occidental (225
SCRA 553, Aug. 23, 1993), it was held
that the law allows a private counsel
to be hired by a municipality only
when the municipality is an adverse
party in a case involving the provincial
government or another municipality or
city within the province. This provision
has its apparent origin in De Guia v.
The Auditor General (44 SCRA 169,
March 29, 1979) where the Court held
that the municipalitys authority to
employ a private attorney is expressly
limited only to situations where the
provincial fiscal would be disqualified
to serve and represent it. (Ramos v.
CA, 269 SCRA 34, March 3, 1997)
363. Cite
instances
when
the
provincial fiscal may be disqualified to
represent in court a particular
municipality.
Held:
As held in Enriquez, Sr. v.
Gimenez (107 Phil. 932 [1960]), the
provincial fiscal may be disqualified to
represent in court a particular
municipality in the following instances:
1)

If
jurisdiction

and
when
original
of case involving the

municipality is vested in the Supreme


Court;
2)
When the municipality is a
party adverse to the provincial
government
or
to
some
other
municipality in the same province; and
3)
When, in a case involving
the municipality, he, or his wife, or
child, is pecuniarily involved, as heir,
legatee, creditor or otherwise.
(Ramos v. CA, 269 SCRA 34, March
3, 1997)
364. May
a
municipality
be
represented by a private law firm
which had volunteered its services
gratis, in collaboration with the
municipal attorney and the fiscal?
Held: No. Such representation will be
violative of Section 1983 of the old
Administrative Code.
This strict
coherence to the letter of the law
appears to have been dictated by the
fact that the municipality should not
be burdened with expenses of hiring a
private lawyer and that the interests
of the municipality would be best
protected if a government lawyer
handles its litigations.
Private lawyers may not represent
municipalities on their own. Neither
may they do so even in collaboration
with authorized government lawyers.
This is anchored on the principle that
only accountable public officers may
act for and in behalf of public entities
and that public funds should not be
expended to hire private lawyers.
(Ramos v. CA, 269 SCRA 34, March
3, 1997)
365. May a municipality adopt the
work already performed in good faith
by a private lawyer, which work
proved beneficial to it?
Held: Although a municipality may
not hire a private lawyer to represent
it in litigations, in the interest of
substantial justice, however, it was
held that a municipality may adopt the
work already performed in good faith
by such private lawyer, which work is
beneficial to it (1) provided that no
injustice is thereby heaped on the
adverse party and (2) provided further
that no compensation in any guise is
paid therefor by said municipality to
the private lawyer.
Unless so
expressly
adopted,
the
private
lawyers work cannot bind the
municipality.
(Ramos v. CA, 269
SCRA 34, March 3, 1997)

366. May the Punong Barangay


validly
appoint
or
remove
the
barangay treasurer, the barangay
secretary,
and
other
appointive
barangay
officials
without
the
concurrence of the majority of all the
members
of
the
Sangguniang
Barangay?
Held: The Local Government Code
explicitly vests on the punong
barangay, upon approval by a majority
of all the members of the sangguniang
barangay, the power to appoint or
replace the barangay treasurer, the
barangay
secretary,
and
other
appointive barangay officials. Verily,
the power of appointment is to be
exercised conjointly by the punong
barangay and a majority of all the
members
of
the
sangguniang
barangay.
Without such conjoint
action, neither an appointment nor a
replacement can be effectual.
Applying the rule that the power to
appoint includes the power to remove,
the questioned dismissal from office of
the barangay officials by the punong
barangay without the concurrence of
the majority of all the members of the
Sangguniang Barangay cannot be
legally justified.
To rule otherwise
could also create an absurd situation
of
the
Sangguniang
Barangay
members refusing to give their
approval to the replacements selected
by the punong barangay who has
unilaterally terminated the services of
the incumbents. It is likely that the
legislature
did
not
intend
this
absurdity to follow from its enactment
of the law. (Ramon Alquizola, Sr. v.
Gallardo Ocol, G.R. No. 132413,
Aug. 27, 1999, 3rd Div. [Vitug])
367. What acts of a Division of the
COMELEC may be subject of a motion
for reconsideration of the COMELEC en
banc?
Held:
Section 5, Rule 19 of the
COMELEC
Rules
of
Procedure,
provides:
SEC.
5.
How
Motion
for
Reconsideration Disposed of. - Upon
the filing of a motion to reconsider a
decision, resolution, order or ruling of
a Division, the Clerk of Court
concerned shall, within twenty-four
(24) hours from the filing thereof,
notify the presiding Commissioner.
The latter shall within two (2) days
thereafter certify the case to the
Commission en banc.

Under the above-quoted rule,


the acts of a Division that are subject
of a motion for reconsideration must
have a character of finality before the
same can be elevated to the COMELEC
en banc. The elementary rule is that
an order is final in nature if it
completely disposes of the entire case.
But if there is something more to be
done in the case after its issuance,
that order is interlocutory.
As correctly pointed out by
public respondent in its assailed order
of November 29, 1999, the October
11, 1999 did not dispose of the case
completely as there is something more
to be done which is to decide the
election protest. As such, it is the
herein public respondent (Second
Division of the COMELEC) which issued
the interlocutory order of October 11,
1999 that should resolve petitioners
motion for reconsideration, not the
COMELEC en banc (Bulaong v.
COMELEC, First Division, 220 SCRA
745, 749 [1993]). Accordingly, the
applicable rule on the subject is
Section 5(c), Rule 3 of the COMELEC
Rules of Procedure, which states:
Rule 3, Section 5(c). Any motion to
reconsider a decision, resolution, order
or ruling of a Division shall be resolved
by the Commission en banc except
motions on interlocutory orders of the
division, which shall be resolved by
the divisions which issued the order.
That only final orders of a
Division may be raised before the
COMELEC en banc is in accordance
with Article IX-C, Section 3 of the
Constitution which mandates that only
motions for reconsideration of final
decisions shall be decided by the
Commission on Elections en banc,
thus:
Sec. 3. The Commission on Elections
may sit en banc or in two divisions,
and shall promulgate its rules of
procedure in order to expedite
disposition of election cases, including
pre-proclamation controversies. All
such election cases shall be heard and
decided in division, provided that
motions for reconsideration of
decisions shall be decided by the
Commission en banc.
It bears stressing that under
this constitutional
provision, the
COMELEC en banc shall decide
motions for reconsideration only of

decisions of a Division, meaning


those acts of final character. Clearly,
the assailed order denying petitioner's
demurrer
to
evidence,
being
interlocutory, may not, be resolved by
the COMELEC en banc (Ambil, Jr. v.
Commission on elections, G.R. No.
143398, Oct. 25, 2000, 344 SCRA
358). (Gementiza v. Commission
on Elections, 353 SCRA 724,
March 6, 2001, En Banc [SandovalGutierrez])
368. Is a contract entered into by the
city mayor involving the expenditure
of public funds by the local
government without prior
appropriation by the city council valid
and binding?
Held: If we are to limit our
disquisition to the cited provisions of
Presidential Decree No. 1445, or the
Auditing Code of the Philippines, in
conjunction with Section 177 (b) of
Batas Pambansa Blg. 337, or the Local
Government Code of 1983, which
empowered the Sangguniang
Panlungsod to appropriate funds for
expenses of the city government, and
fix the salaries of its officers and
employees according to law, there
would be no debate that prior
appropriation by the city council and a
certification that funds are available
therefore is indeed mandatorily
required.
Xxx
However, the very same
Presidential Decree No. 1445, which is
the cornerstone of petitioners
arguments, does not provide that the
absence of an appropriation law ipso
facto makes a contract entered into by
a local government unit null and void.
Section 84 of the statute specifically
provides:
Revenue funds shall not be paid out of
any public treasury or depository
except in pursuance of an
appropriation law or other specific
statutory authority.

Consequently, public funds may


be disbursed not only pursuant to an
appropriation law, but also in
pursuance of other specific statutory
authority, i.e., Section 84 of PD 1445.
Thus, when a contract is entered into
by a city mayor pursuant to specific
statutory authority, the law, i.e., PD
1445 allows the disbursement of funds
from any public treasury or depository
therefor. It can thus be plainly seen
that the law invoked by petitioner
Quezon City itself provides that an
appropriation law is not the only
authority upon which public funds
shall be disbursed.
Furthermore, then Mayor
Brigido Simon, Jr. did not enter into the
subject contract without legal
authority. The Local Government Code
of 1983, or B.P. Blg. 337, which was
then in force, specifically and
exclusively empowered the city mayor
to represent the city in its business
transactions, and sign all warrants
drawn on the city treasury and all
bonds, contracts and obligations of the
city. Such power granted to the city
mayor by B.P. Blg. 337 was not
qualified nor restricted by any prior
action or authority of the city council.
We note that while the subsequent
Local Government Code of 1991,
which took effect after the execution
of the subject contracts, provides that
the mayors representation must be
upon authority of the sangguniang
panlungsod or pursuant to law or
ordinance, there was no such
qualification under the old code.
(Citations omitted)
We must differentiate the
provisions of the old Local Government
Code of 1983, B.P. Blg. 337, which was
then in force, from that of the Local
Government Code of 1991, R.A. No.
7160, which now requires that the
mayors representation of the city in
its business transactions must be
upon authority of the sangguniang
panlungsod or pursuant to law or
ordinance (Section 455 [vi]. No such
prior authority was required under B.P.
Blg. 337. This restriction, therefore,

cannot be imposed on the city mayor


then since the two contracts were
entered into before R.A. No. 7160 was
even enacted.
Under B.P. Blg. 337, while the
city mayor has no power to
appropriate funds to support the
contracts, neither does said law
prohibit him from entering into
contracts unless and until funds are
appropriated therefor. In fact, it is his
bounden duty to so represent the city
in all its business transactions. On the
other hand, the city council must
provide for the depositing, leaving or
throwing of garbage and to
appropriate funds for such expenses.
(Section 177 [b]). It cannot refuse to
so provide and appropriate public
funds for such services which are very
vital to the maintenance of cleanliness
of the city and the good health of its
inhabitants.
By entering into the two
contracts, Mayor Simon did not usurp
the city councils power to provide for
the proper disposal of garbage and to
appropriate funds therefor. The
execution of contracts to address such
a need is his statutory duty, just as it
is the city councils duty to provide for
said services. There is no provision in
B.P. Blg. 337, however, that prohibits
the city mayor from entering into
contracts for the public welfare, unless
and until there is prior authority from
the city council. This requirement was
imposed much later by R.A. No. 7160,
long after the contracts had already
been executed and implemented.
Even the very Charter of
Quezon City, more particularly Section
9(f), Section 12(a)and Section 12(m)
thereof, simply provide that the mayor
shall exercise general powers and
duties, such as signing all warrants
drawn on the city treasurer and all
bonds, contracts, and obligations of
the city, even as it grants the City
Council the power, by ordinance or
resolution, to make all appropriations
for the expenses of the government of
the city, as well as to prohibit the

throwing or depositing of offal,


garbage, refuse, or other offensive
matter in the same, and to provide for
its collection and disposition x x x.
(Citations omitted)
While the powers and duties of
the Mayor and the City Council are
clearly delineated, there is nothing in
the cited provisions, nor even in the
statute itself, that requires prior
authorization by the city council by
proper enactment of an ordinance
before the City Mayor can enter into
contracts.
Private
respondent
Lexber
asserts that the subject contract was
entered into by Mayor Simon in behalf
of the Quezon City government
pursuant
to
specific
statutory
authority,
more
particularly
the
provisions of Executive Order No. 392
(Constituting
the
Metro
Manila
Authority [MMA]). x x x (City of
Quezon v. Lexber Incorporated,
354 SCRA 493, Mar. 15, 2001, 1st
Div. [Ynares-Santiago])
369. Does the Presidents power of
general supervision extend to the liga
ng mga barangay, which is not a local
government unit?
Held: We rule in the
affirmative. In Opinion No. 41, Series
of 1995, the Department of Justice
ruled that the liga ng mga barangay is
a government organization, being an
association, federation, league or
union created by law or by authority of
law, whose members are either
appointed or elected government
officials. The Local Government Code
defines the liga ng mga barangay as
an organization of all barangays for
the primary purpose of determining
the representation of the liga in the
sanggunians, and for ventilating,
articulating and crystallizing issues
affecting barangay government
administration and securing, through
proper and legal means, solutions
thereto (Sec. 491, Local Government
Code). X x x
Xxx

The ligas are primarily governed


by the provisions of the Local
Government Code (Book III, Title VI,
Local Government Code). However,
their respective constitution and bylaws shall govern other matters
affecting internal organization of the
liga not otherwise provided for in the
Local Government Code provided that
the constitution and by-laws shall be
suppletory to the provisions of Book III,
Title VI of the Local Government Code
and shall always conform to the
provisions of the Constitution and
existing laws (Sec. 507, Local
Government Code).
Having in mind the foregoing
principles, we rule that Memorandum
Circular No. 97-193 of the DILG insofar
as it authorizes the filing a Petition for
Review of the BES with the regular
courts in a post proclamation electoral
protest is of doubtful constitutionality.
We agree with both the petitioner and
the Solicitor General that in
authorizing the filing of the petition for
review of the decision of the BES with
the regular courts, the DILG Secretary
in effect amended and modified the
GUIDELINES promulgated by the
National Liga Board and adopted by
the LIGA which provides that the
decision of the BES shall be subject to
review by the National Liga Board.
The amendment of the GUIDELINES is
more than an exercise of the power of
supervision but is an exercise of the
power of control, which the President
does not have over the LIGA.
Although the DILG is given the power
to prescribe rules, regulations and
other issuances, the Administrative
Code limits its authority to merely
monitoring compliance by local
government units of such issuances
(Taule v. Santos, 200 SCRA 512, 523
[1991]). To monitor means to watch,
observe or check and is compatible
with the power of supervision of the
DILG Secretary over local
governments, which is limited to
checking whether the local
government unit concerned or the
officers thereof perform their duties as

per statutory enactments (Ibid.).


Besides, any doubt as to the power of
the DILG Secretary to interfere with
local affairs should be resolved in
favor of the greater autonomy of the
local government (Ibid.)
The public respondent judge
therefore committed grave abuse of
discretion amounting to lack or excess
of jurisdiction in not dismissing the
respondents Petition for Review for
failure to exhaust all administrative
remedies and for lack of jurisdiction.
(Bito-Onon v. Fernandez, 350
SCRA 732, Jan. 31, 2001, 3rd Div.
[Gonzaga-Reyes])
370. Petitioners would seek the
disqualification of respondent
Leonardo B. Roman on the ground of
his having transgressed the three-term
limit under Section 8, Article X, of the
1987 Constitution and Section 43 of
Republic Act No. 7160 (Local
Government Code). The focal issue
presented before the Court x x x
would revolve on the question of
whether or not private respondent
Roman exceeded the three-term limit
for elective local officials, expressed in
the Constitution and the Local
Government Code, when he again ran
for the position of Governor in the
14th of May 2001 elections, having
occupied and served in that position
following the 1993 recall elections, as
well as the 1995 and 1998 regular
elections, immediately prior to the
2001 elections. In fine, should
respondents incumbency to the post
of Governor following the recall
elections be included in determining
the three-consecutive term limit fixed
by law?
Held: After due deliberation, the
Court voted 8 to 7 to DISMISS the
petition.
VITUG,
J.,
joined
by
YNARESSANTIAGO, J., voted to dismiss the
petition.
He contended that as
revealed by the records of the
Constitutional
Commission,
the
Constitution envisions a continuous
and an uninterrupted service for three
full terms before the proscription
applies. Therefore, not being a full

term, a recall term should not be


counted or used as a basis for the
disqualification whether served prior
(as in this case) or subsequent (as in
the Socrates case) to the nine-year,
full three-term limit.
MENDOZA, J., in whose opinion
QUISUMBING, J., joined, voted to
dismiss the petition on the ground
that, in accordance with the ruling in
Borja, Jr. v. COMELEC, 295 SCRA 157
[1998]; Arcos v. COMELEC, G.R. No.
133639, Oct. 6, 1998 (res.); Lonzanida
v. COMELEC, 311 SCRA 602 [1999];
and Adormeo v. COMELEC, G.R. No.
147927, Feb. 4, 2002, a term during
which succession to a local elective
office takes place or a recall election is
held should not be counted in
determining whether an elective local
official has served more than three
consecutive terms. He argued that
the Constitution does not prohibit
elective local officials from serving for
more than three consecutive terms
because, in fact, it excludes from the
three-term limit interruptions in the
continuity of service, so long as such
interruptions are not due to the
voluntary renunciation of the office by
the incumbent.
Hence, the period
from June 28, 1994 to June 30, 1995,
during which respondent Leonardo B.
Roman served as governor of Bataan
by virtue of a recall election held in
1993, should not be counted. Since
on May 14, 2001 respondent had
previously served as governor of
Bataan for only two consecutive terms
(1995-1998 and 1998-2001), his
election on that day was actually only
his third term for the same position.
PANGANIBAN, J., joined by PUNO, J.,
also voted to dismiss the petition. He
argued that a recall term should not
be considered as one full term,
because a contrary interpretation
would in effect cut short the elected
officials service to less than nine
years
and
shortchange
his
constituents. The desire to prevent
monopoly of political power should be
balanced against the need to uphold
the voters obvious preference who, in
the present case, is Roman who
received 97 percent of the votes cast.
He explained that, in Socrates, he also
voted to affirm the clear choice of the
electorate, because in a democracy
the people should, as much as legally
possible, be governed by leaders
freely chosen by them in credible
elections.
He concluded that, in
election cases, when two conflicting
legal positions are of almost equal

weight, the scales of justice should be


tilted in favor of the peoples
overwhelming choice.
AZCUNA, J., joined by BELLOSILLO, J.,
also voted to dismiss, arguing that it is
clear from the constitutional provision
that the disqualification applies only if
the terms are consecutive and the
service is full and continuous. Hence,
service for less than a term, except
only in case of voluntary renunciation,
should not count to disqualify an
elective local official from running for
the same position.
This case is
different from Socrates, where the full
three consecutive terms had been
continuously
served
so
that
disqualification had clearly attached.
On the other hand, SANDOVALGUTIERREZ, J., with whom DAVIDE,
C.J., and AUSTRIA-MARTINEZ, CORONA,
and CALLEJO, SR., JJ., concurred, holds
the view that the recall term served by
respondent Roman, comprising the
period June 28, 1994 to June 30, 1995,
should be considered as one term.
Since he thereafter served for two
consecutive terms from 1995 to 1998
and from 1998 to 2001, his election on
May 14, 2001 was actually his fourth
term and contravenes Art. X, Sec. 8 of
the Constitution. For this reason, she
voted to grant the petition and to
declare respondents election on May
14, 2002 as null and void.
CARPIO,
J.,
joined
by
CARPI0MORALES, J., also dissented and voted
to grant the petition. He held that a
recall term constitutes one term and
that to totally ignore a recall term in
determining the three-term limit would
allow local officials to serve for more
than nine consecutive years contrary
to the manifest intent of the framers
of the Constitution. He contended that
respondent Romans election in 2001
cannot exempt him from the threeterm
limit
imposed
by
the
Constitution.
In his Separate Opinion, Justice
Vitug voted to dismiss the petition on
the following considerations:
In order that the threeconsecutive term limit can apply, two
conditions must concur, i.e., (1) that
the elective local official concerned
has been elected for three consecutive
terms to the same local government
position, and (2) that he has served
three consecutive full terms, albeit a

voluntary renunciation of the office


for any length of time shall not be
deemed to be an interruption in the
continuity of the service for the full
term for which he is elected. The
constitutional provision does not
appear to be all that imprecise for and
in its application. Section 8, Article X,
of the Constitution is explicit that the
term of office of elective local officials
x x x shall be three years which
phrase is forthwith followed by its
mandate that no such official shall
serve for more than three consecutive
terms, and that [v]oluntary
renunciation of the office for any
length of time shall not be considered
as an interruption in the continuity of
his service for the full term for which
he [is] elected. The law evidently
contemplates a continuous full threeyear term before the proscription can
apply.
The Constitutional Commission,
in its deliberations, referred to a full
nine (9) years of service for each
elective local government official in
the application of the prohibition,
envisioning at the same time a
continuous and uninterrupted period
of nine years by providing for only one
exception, i.e., when an incumbent
voluntarily gives up the office.
Xxx
A winner who dislodges in a
recall election an incumbent elective
local official merely serves the balance
of the latters term of office; it is not a
full three-year term. It also goes
without saying that an incumbent
elective local official against whom a
recall election is initiated and who
nevertheless wins in a recall election
must be viewed as being a continuing
term of office and not as a break in
reckoning his three consecutive terms.
Xxx
If involuntary severance from
the service which results in the
incumbents being unable to finish his
term of office because of his ouster
through valid recall proceedings

negates one term for purposes of


applying the three-term limit, as so
intimated in Lonzanida, it stands to
reason that the balance of the term
assumed by the newly elected local
official in a recall election should not
also be held to be one term in
reckoning the three-term limit. In both
situations, neither the elective local
official who is unable to finish his term
nor the elected local official who only
assumes the balance of the term of
the ousted local official following the
recall election could be considered to
have served a full three-year term set
by the Constitution.
This view is not inconsistent,
but indeed in line, with the conclusion
ultimately reached in Socrates v.
Commission on Elections, where
the Court has considered Hagedorn,
following his three full terms of nine
years, still qualified to run in a recall
election conducted about a year and a
half after the most recent regular local
elections. A recall term then, not
being a full three-year term, is not to
be counted or used as a basis for
disqualification whether it is held prior
or subsequent to the nine year full
three-term limit.
This same issue has been
passed and ruled upon by the
Commission on Elections no less than
five times. Consistently, it has held
that the term of a newcomer in recall
elections cannot be counted as a full
term and may not thus be included in
counting the three-term limit
prescribed under the law. The
Commission on Elections, with its factfinding facilities, its familiarity with
political realities, and its peculiar
expertise in dealing with election
controversies, should be in a good
vantage point to resolve issues of this
nature. Concededly, no ready made
formulae are always extant to address
occasional complex issues, allowing
time and experience to merely evolve
and ultimately provide acceptable
solutions. In the administration of
election laws, it would be unsound by
an excessive zeal to remove from the

Commission on Elections the initiative


it takes on such questions which, in
fact, by legal mandate properly belong
to it (See Loong v. COMELEC, 305
SCRA 832, Pangandaman v. COMELEC,
319 SCRA 283).
Nor should it be ignored that
the law here involved is a limitation on
the right of suffrage not only on the
candidate for office but also, and most
importantly, on the electorate.
Respondent Roman has won the
election to the post of Governor of
Bataan with a comfortable margin
against his closest opponent. Where a
candidate appears to be the clear
choice of the people, doubts on the
candidates eligibility, even only as a
practical matter, must be so resolved
as to respect and carry out, not
defeat, the paramount will of the
electorate. While the Constitution
would attempt to prevent the
monopolization of political power,
indeed a wise rule, the precept of
preserving the freedom of choice of
the people on who shall rightfully hold
the reins of government for them is no
less than fundamental in looking at its
overriding intent. (Melanio L.
Mendoza and Mario E. Ibarra v.
Commission on Elections and
Leonardo B. Roman, G.R. No.
149736, Dec. 17, 2002, En Banc)
371. On May 3, 2001, petitioner filed
with the Provincial Election Supervisor
in Pagadian City a petition for the
disqualification of respondent Sulong,
pursuant to Sec. 40[b] of Republic Act
No. 7160 (Local Government Code),
which disqualifies from running for any
elective local position those removed
from office as a result of an
administrative case. It appears that
respondent Sulong had previously won
as mayor of Lapuyan on January 18,
1988. In the May 11, 1992, and again
in the May 8, 1995 elections, he was
reelected. In a petition for
disqualification, petitioner alleged that
in 1991, during his first term as mayor
of Lapuyan, respondent Sulong, along
with a municipal councilor of Lapuyan
and several other individuals, was

administratively charged (AC No. 1291) with various offenses, and that, on
February 4, 1992, the Sangguniang
Panlalawigan of Zamboanga del Sur
found him guilty of the charges and
ordered his removal from office.
Petitioner claimed that this decision
had become final and executory, and
consequently the then vice-mayor of
Lapuyan, Vicente Imbing, took his oath
as mayor vice respondent Sulong on
March 3, 1992.
Respondent Sulong denied that the
decision in AC No. 12-91 had become
final and executory. He averred that
after receiving a copy of the decision
on February 17, 1992, he filed a
motion for reconsideration and/or
notice of appeal thereof on February
18, 1992; that on February 27, 1992,
the Sangguniang Panlalawigan
required Jim Lingating, the
complainant in AC No. 12-91, to
comment on respondent Sulongs
motion for reconsideration and/or
notice of appeal; that the said
complainant had not yet complied
therewith and his (respondent
Sulongs) motion had consequently
remained pending. Respondent
Sulong denied he had been removed
from office by virtue of the decision in
AC No. 12-91.
Held: Petitioner contends that
the COMELEC en banc erred in
applying the ruling in Aguinaldo v.
Commission on Elections (212 SCRA
768 [1992]) in holding that the
reelection of respondent Sulong in
1992 and 1995 as mayor of Lapuyan
had the effect of condoning the
misconduct for which he was ordered
dismissed by the Sangguniang
Panlalawigan of Zamboanga del Sur.
Petitioner cites Reyes v. Commission
on Elections (254 SCRA 514, 525-526
[1996]) in which we held that an
elective local executive officer, who is
removed before the expiration of the
term for which he was elected, is
disqualified from being a candidate for
a local elective position under Section
40[b] of the Local Government Code.

Xxx
However, Reyes cannot be applied to
this case because it appears that the
1992 decision of the Sangguniang
Panlalawigan, finding respondent
Sulong guilty of dishonesty,
falsification and malversation of public
funds, has not until now become final.
X x x The filing of his motion for
reconsideration prevented the decision
of Sangguniang Panlalawigan from
becoming final.
While R.A. No. 7160 on
disciplinary actions is silent on the
filing of a motion for reconsideration,
the same cannot be interpreted as a
prohibition against the filing of a
motion for reconsideration. x x x.
There is thus no decision finding
respondent guilty to speak of. As
Provincial Secretary of Zamboanga del
Sur Wilfredo Cimafranca attested, the
Sangguniang Panlalawigan simply
considered the matter as having
become moot and academic because
it was overtaken by the local
elections of May [11], 1992.
Neither can the succession of
the then vice-mayor of Lapuyan x x x
and the highest ranking municipal
councilor of Lapuyan x x x to the
offices of mayor and vice-mayor,
respectively, be considered proof that
the decision in AC No. 12-91 had
become final because it appears to
have been made pursuant to Sec. 68
of the Local Government Code, which
makes decisions in administrative
cases immediately executory.
Indeed, considering the failure
of the Sangguniang Panlalawigan to
resolve respondents motion, it is
unfair to the electorate to be told after
they have voted for respondent Sulong
that after all he is disqualified,
especially since at the time of the
elections on May 14, 2001, the
decision of the Sangguniang
Panlalawigan had been rendered
nearly ten years ago. (Atty. Miguel
M. Lingating v. Commission on

Elections and Cesar B. Sulong,


G.R. No. 153475, Nov. 13, 2002,
En Banc [Mendoza])
372. When
may
a
permanent
vacancy arise under Section 44 of the
Local Government Code?
Held:
Under Section 44, a
permanent vacancy arises when an
elective official fills a higher vacant
office, refuses to assume office, fails to
qualify, dies, is removed from office,
voluntarily resigns, or is otherwise
permanently
incapacitated
to
discharge the functions of his office.
(Navarro v. Court of Appeals, 355
SCRA 672, Mar. 28, 2001, 1st Div.
[Kapunan])
373. How is Section 45(b) of the
Local
Government Code
to be
interpreted?
What is the reason
behind the right given to a political
party to nominate a replacement
where a permanent vacancy occurs in
the Sanggunian?
Held: What is crucial is the
interpretation
of
Section
45(b)
providing that x x x only the nominee
of the political party under which the
Sanggunian member concerned has
been elected and whose elevation to
the position next higher in rank
created the last vacancy in the
Sanggunian shall be appointed in the
manner hereinabove provided. The
appointee shall come from the political
party as that of the Sanggunian
member who caused the vacancy x x
x.
The reason behind the right
given to a political party to nominate a
replacement where a permanent
vacancy occurs in the Sanggunian is to
maintain the party representation as
willed by the people in the election
(Aquilino
Pimentel,
the
Local
Government Code of 1991, The Key to
National Development, p. 150).
With the elevation of petitioner
Tamayo, who belonged to REFORMALM, to the position of Vice-Mayor, a
vacancy occurred in the Sanggunian
that should be filled up with someone
who should belong to the political
party of petitioner Tamayo. Otherwise,
REFORMA-LMs representation in the
Sanggunian would be diminished. To
argue that the vacancy created was
that formerly held by Rolando Lalas, a
LAKAS-NUCD-Kampi member, would
result in the increase of that partys

representation in the Sanggunian at


the expense of the REFORMA-LM. This
interpretation is contrary to the letter
and spirit of the law and thus violative
of a fundamental rule in statutory
construction which is to ascertain and
give effect to the intent and purpose
of the law (Manila Lodge No. 761 v.
Court of Appeals, 73 SCRA 12 [1976]).
As earlier pointed out, the reason
behind par. (b), section 44 of the Local
Government Code is the maintenance
of
party
representation
in
the
Sanggunian in accordance with the will
of the electorate.
The last vacancy in the
Sanggunian refers to that created by
the elevation of the member formerly
occupying the next higher in rank
which in turn also had become vacant
by any of the causes already
enumerated. The term last vacancy
is thus used in Sec. 45 (b) to
differentiate it from the other vacancy
previously created. The term by no
means refers to the vacancy in the No.
8 position which occurred with the
elevation of Rolando Lalas to the
seventh position in the Sanggunian.
Such construction will result in
absurdity.
(Navarro v. Court of
Appeals, 355 SCRA 672, Mar. 28,
2001, 1st Div. [Kapunan])
374. May local elective officials
practice their profession or engage in
any occupation?
Ans.: All governors, city and
municipal mayors are prohibited from
practicing their profession or engaging
in any occupation other than the
exercise of their functions as local
chief executives. (Sec. 90[a], LGC)
Sanggunian
members
may
practice their profession, engage in
any occupation, or teach in schools
except during session hours. (Sec.
90[b], LGC)
Sanggunian members who are
also members of the Bar shall not:
(a)

Appear as counsel before


any court in any civil case wherein a
local government unit or any office,
agency, or instrumentality of the
government is the adverse party;
(b)
Appear as counsel in any
criminal case wherein an officer or
employee of the national or local
government is accused of an offense
committed in relation to his office;
(c)
Collect any fee for their
appearance
in
administrative

proceedings
involving
the
local
government unit of which he is an
official; and
(d)
Use property and personnel
of the government except when the
sanggunian member concerned is
defending
the
interest
of
the
Government. (Sec. 90[b], LGC)
375. Whether or not the plebiscites
scheduled
for
the
creation
of
Barangays Karangalan and Napico
should be suspended or cancelled in
view of the pending boundary dispute
between the two local governments.
Held: To begin with, we agree with
the position of the COMELEC that Civil
Case No. 94-3006 involving the
boundary
dispute
between
the
Municipality of Cainta and the City of
Pasig presents a prejudicial question
which must first be decided before the
plebiscites for the creation of the
proposed barangays may be held.
Xxx
In the case at bar, while the City of
Pasig vigorously claims that the areas
covered by the proposed Barangays
Karangalan and Napico are within its
territory, it can not deny that portions
of the same area are included in the
boundary dispute case pending before
the Regional Trial Court of Antipolo.
Surely,
whether
the
areas
in
controversy shall be decided as within
the territorial jurisdiction of the
Municipality of Cainta or the City of
Pasig has material bearing to the
creation of the proposed Barangays
Karangalan and Napico.
Indeed, a
requisite for the creation of a
barangay
is
for
its
territorial
jurisdiction to be properly identified by
metes and bounds or by more or less
permanent natural boundaries (Sec.
386[b], R.A. No. 7160).
Precisely
because territorial jurisdiction is an
issue raised in the pending civil case,
until and unless such issue is resolved
with finality, to define the territorial
jurisdiction of the proposed barangays
would only be an exercise in futility.
Not only that, we would be paving the
way for potentially ultra vires acts of
such barangays. X x x
Moreover, considering the expenses
entailed in the holding of plebiscites, it
is far more prudent to hold in
abeyance the conduct of the same,
pending
final
determination
of
whether or not the entire area of the
proposed barangays are truly within

the territorial jurisdiction of the City of


Pasig.
Neither do we agree that merely
because a plebiscite had already been
held in the case of the proposed
Barangay Napico, the petition of the
Municipality of Cainta has already
been rendered moot and academic.
The issue raised by the Municipality of
Cainta in its petition before the
COMELEC against the holding of the
plebiscite for the creation of Barangay
Napico are still pending determination
before the Antipolo Regional Trial
Court.
Xxx
Therefore, the plebiscite on the
creation of Barangay Karangalan
should be held in abeyance pending
final resolution of the boundary
dispute between the City of Pasig and
the Municipality of Cainta by the
Regional Trial Court of Antipolo City. In
the same vein, the plebiscite held on
March 15, 1997 to ratify the creation
of Barangay Napico, Pasig City, should
be annulled and set aside. (City of
Pasig v. COMELEC, 314 SCRA 179,
Sept. 10, 1999, En Banc [YnaresSantiago])

PUBLIC INTERNATIONAL LAW


376. Discuss the contemporary view
on the rightful place of an Individual in
International Law? Does he remain a
mere object of International Law, or
is he now a proper subject of
International Law?
Held: Then came the long and
still ongoing debate on what should be
the subject of international law. The
20th century saw the dramatic rise
and fall of different types and hues of
authoritarianism the fascism of
Italys Mussolini and Germanys Hitler,
the militarism of Japans Hirohito and
the communism of Russias Stalin, etc.
The sinking of these isms led to the
elevation of the rights of the individual
against the state.
Indeed, some
species of human rights have already
been accorded universal recognition
(See Universal Declaration of Human
Rights
[1948],
The
International
Covenant on Economic, Social and
Cultural Rights [1966] and The
International Covenant on Civil and
Political Rights [1966]). Today, the

drive to internationalize rights of


women and children is also on high
gear
(The
Convention
on
the
Elimination
of
All
Forms
of
Discrimination
Against
Women
[CEDAW] otherwise known as The Bill
of
Rights
for
Women was adopted by the UN
General Assembly in December 1979.
As of November 1999, one hundred
sixty seven [167] states including the
Philippines have ratified or acceded to
it.
See Statement of Angela King,
Special Adviser to the Secretary
General of the UN on Gender Issues
and Advancement of Women, Judicial
Colloquium on the Application of
International Human Rights Law at the
Domestic Level, Vienna, Austria,
October 27, 1999). The higher rating
given to human rights on the hierarchy
of values necessarily led to the reexamination of the rightful place of
the individual in international law.
Given the harshest eye is the mosscovered doctrine that international law
deals only with States and that
individuals are not its subject. For its
undesirable corollary is that subdoctrine that an individuals right in
international law is a near cipher.
Translated in extradition law, the view
that once commanded a consensus is
that since a fugitive is a mere object
and not a subject of international law,
he is bereft of rights. An extraditee,
so it was held, is a mere object
transported from one state to the
other as an exercise of the sovereign
will of the two states involved.
(Blakesley and Lagodny, Finding
Harmony Amidst Disagreement Over
Extradition, Jurisdiction, The Role of
Human
Rights
and
Issues
of
Extraterritoriality Under International
Criminal Law, Vanderbilt Journal of
Transnational Law, Vol. 24, No. 1, p. 44
[1991]) The re-examination consigned
this pernicious doctrine to the
museum of ideas (See generally
Kelsen, Principles of International Law,
2nd ed. [1966]; Korowicz, The Problem
of the International Personality of
Individuals, 50 Am. J., Intl. Law 553
[1966]).
The new thinkers of
international
law
then
gave
a
significant shape to the role and rights
of the individual in state-concluded
treaties
and
other
international
agreements.
X x x (Concurring
Opinion, Puno J., in Jeffrey Liang
[Huefeng] v. People, G.R. No.
125865, Mar. 26, 2001, 1st Div.
[Motion for Reconsideration])

377. What
is
the
doctrine
of
incorporation? How is it applied by
local courts?
Held:
Under the doctrine of
incorporation, rules of international
law form part of the law of the land
and no further legislative action is
needed to make such rules applicable
in the domestic sphere.
The doctrine of incorporation is
applied whenever municipal tribunals
(or local courts) are confronted with
situations in which there appears to be
a
conflict
between
a
rule
of
international law and the provisions of
the Constitution or statute of the local
state. Efforts should first be exerted
to harmonize them, so as to give
effect to both since it is to be
presumed that municipal law was
enacted with proper regard for the
generally
accepted
principles
of
international law in observance of the
Incorporation Clause in Section 2,
Article II of the Constitution.
In a
situation however, where the conflict
is irreconcilable and a choice has to be
made between a rule of international
law and municipal law, jurisprudence
dictates that municipal law should be
upheld by the municipal courts for the
reason that such courts are organs of
municipal law and are accordingly
bound by it in all circumstances. The
fact that international law has been
made part of the law of the land does
not pertain to or imply the primacy of
international law over national or
municipal law in the municipal sphere.
The doctrine of incorporation, as
applied in most countries, decrees that
rules of international law are given
equal standing with, but are not
superior
to,
national
legislative
enactments. Accordingly, the principle
of lex posterior derogat priori takes
effect a treaty may repeal a statute
and a statute may repeal a treaty. In
states where the Constitution is the
highest law of the land, such as the
Republic of the Philippines, both
statutes
and
treaties
may
be
invalidated if they are in conflict with
the Constitution.
(Secretary of
Justice v. Hon. Ralph C. Lantion,
G.R. No. 139465, Jan. 18, 2000, En
Banc [Melo])
378. Is sovereignty really absolute
and all-encompassing? If not, what
are its restrictions and limitations?
Held:
While
sovereignty
has
traditionally been deemed absolute

and all-encompassing on the domestic


level, it is however subject to
restrictions and limitations voluntarily
agreed to by the Philippines, expressly
or impliedly, as a member of the
family of nations. By the doctrine of
incorporation, the country is bound by
generally
accepted
principles
of
international
law,
which
are
considered to be automatically part of
our own laws. One of the oldest and
most
fundamental
rules
in
international law is pacta sunt
servanda international agreements
must be performed in good faith. A
state which has contracted valid
international obligations is bound to
make
in
its
legislations
such
modifications as may be necessary to
ensure
the
fulfillment
of
the
obligations.
By their inherent nature, treaties really
limit or restrict the absoluteness of
sovereignty. By their voluntary act,
nations may surrender some aspects
of their state power in exchange for
greater benefits granted by or derived
from a convention or pact. After all,
states, like individuals, live with
coequals, and in pursuit of mutually
covenanted objectives and benefits,
they also commonly agree to limit the
exercise of their otherwise absolute
rights. Thus, treaties have been used
to record agreements between States
concerning
such
widely
diverse
matters as, for example, the lease of
naval bases, the sale or cession of
territory, the termination of war, the
regulation of conduct of hostilities, the
formation of alliances, the regulation
of commercial relations, the settling of
claims, the laying down of rules
governing conduct in peace and the
establishment
of
international
organizations. The sovereignty of a
state therefore cannot in fact and in
reality
be
considered
absolute.
Certain restrictions enter into the
picture: (1) limitations imposed by the
very nature of membership in the
family of nations and (2) limitations
imposed
by
treaty
stipulations.
(Tanada v. Angara, 272 SCRA 18,
May 2, 1997 [Panganiban])
379. What must a person who feels
aggrieved by the acts of a foreign
sovereign do to espouse his cause?
Held: Private respondent is not
left without any legal remedy for the
redress of its grievances. Under both
Public
International
Law
and
Transnational Law, a person who feels
aggrieved by the acts of a foreign

sovereign can ask his own government


to
espouse
his
cause
through
diplomatic channels.
Private respondent can ask the
Philippine government, through the
Foreign Office, to espouse its claims
against the Holy See. Its first task is
to persuade the Philippine government
to take up with the Holy See the
validity of its claim. Of course, the
Foreign Office shall first make a
determination of the impact of its
espousal on the relations between the
Philippine government and the Holy
See (Young, Remedies of Private
Claimants Against Foreign States,
Selected Readings on Protection by
Law of Private Foreign Investments
905, 919 [1964]). Once the Philippine
government decides to espouse the
claim, the latter ceases to be a private
cause.
According to the Permanent
Court of International Justice, the
forerunner of the International Court of
Justice:
By taking up the case of one of its
subjects and by resorting to diplomatic
action
or
international
judicial
proceedings on his behalf, a State is in
reality asserting its own rights - its
right to ensure, in the person of its
subjects, respect for the rules of
international law (The Mavrommatis
Palestine Concessions, 1 Hudson,
World Court Reports 293, 302 [1924]).
(Holy See, The v. Rosario, Jr., 238
SCRA 524, 538-539, Dec. 1, 1994,
En Banc [Quiason])
380. Discuss the
Vatican and the
International Law.

Status of the
Holy See in

Held: Before the annexation of the


Papal States by Italy in 1870, the Pope
was the monarch and he, as the Holy
See, was considered a subject of
International Law. With the loss of the
Papal States and the limitation of the
territory under the Holy See to an area
of 108.7 acres, the position of the Holy
See in International Law became
controversial.
In 1929, Italy and the Holy See
entered into the Lateran Treaty, where
Italy
recognized
the
exclusive
dominion and sovereign jurisdiction of
the Holy See over the Vatican City. It
also recognized the right of the Holy
See to receive foreign diplomats, to
send its own diplomats to foreign

countries, and to enter into treaties


according to International Law.
The Lateran Treaty established the
statehood of the Vatican City for the
purpose of assuring to the Holy See
absolute and visible independence and
of guaranteeing to it indisputable
sovereignty also in the field of
international relations.
In view of the wordings of the Lateran
Treaty, it is difficult to determine
whether the statehood is vested in the
Holy See or in the Vatican City. Some
writers even suggested that the treaty
created two international persons - the
Holy See and Vatican City.
The Vatican City fits into none of the
established categories of states, and
the attribution to it of sovereignty
must be made in a sense different
from that in which it is applied to other
states. In a community of national
states, the Vatican City represents an
entity organized not for political but
for
ecclesiastical
purposes
and
international objects. Despite its size
and object, the Vatican City has an
independent government of its own,
with the Pope, who is also head of the
Roman Catholic Church, as the Holy
See or Head of State, in conformity
with its traditions, and the demands of
its mission in the world. Indeed, the
world-wide interests and activities of
the Vatican City are such as to make it
in a sense an international state.
One
authority
wrote
that
the
recognition of the Vatican City as a
state has significant implication - that
it is possible for any entity pursuing
objects essentially different from those
pursued by states to be invested with
international personality.
Inasmuch as the Pope prefers to
conduct foreign relations and enter
into transactions as the Holy See and
not in the name of the Vatican City,
one can conclude that in the Pope's
own view, it is the Holy See that is the
international person.
The Republic of the Philippines has
accorded the Holy See the status of a
foreign sovereign.
The Holy See,
through its Ambassador, the Papal
Nuncio,
has
had
diplomatic
representations with the Philippine
government since 1957. This appears
to be the universal practice in
international relations.
(Holy See,
The v. Rosario, Jr., 238 SCRA 524,

533-534, Dec. 1, 1994, En Banc


[Quiason])
381. What
are
international
organizations? Discuss their nature.
Held: International organizations are
institutions
constituted
by
international agreement between two
or more States to accomplish common
goals. The legal personality of these
international organizations has been
recognized not only in municipal law,
but in international law as well.
Permanent international commissions
and administrative bodies have been
created by the agreement of a
considerable number of States for a
variety of international purposes,
economic or social and mainly nonpolitical.
In so far as they are
autonomous and beyond the control of
any one State, they have distinct
juridical personality independent of
the municipal law of the State where
they are situated. As such, they are
deemed to possess a species of
international personality of their own.
(SEAFDEC-AQD v. NLRC, 206 SCRA
283, Feb. 14, 1992)
382. Discuss the basic immunities of
international organizations and the
reason for affording them such
immunities.
Held: One of the basic immunities of
an
international
organization
is
immunity from local jurisdiction, i.e.,
that it is immune from legal writs and
processes issued by the tribunals of
the country where it is found. The
obvious reason for this is that the
subjection of such an organization to
the authority of the local courts would
afford a convenient medium through
which the host government may
interfere in their operations or even
influence or control its policies and
decisions; besides, such subjection to
local jurisdiction would impair the
capacity of such body to discharge its
responsibilities impartially on behalf of
its member-states. (SEAFDEC-AQD
v. NLRC, 206 SCRA 283, Feb. 4,
1992)
383. Discuss the two conflicting
concepts of sovereign immunity from
suit.
Held:
There are two conflicting
concepts of sovereign immunity, each
widely held and firmly established.
According to the classical or absolute
theory, a sovereign cannot, without its

consent, be made a respondent in the


courts
of
another
sovereign.
According to the newer or restrictive
theory, the immunity of the sovereign
is recognized only with regard to
public acts or acts jure imperii of a
state, but not with regard to private
acts or acts jure gestionis.
Some states passed legislation to
serve as guidelines for the executive
or judicial determination when an act
may be considered as jure gestionis.
The United States passed the Foreign
Sovereign Immunities Act of 1976,
which defines a commercial activity as
either a regular course of commercial
conduct or a particular commercial
transaction or act. Furthermore, the
law declared that the commercial
character of the activity shall be
determined by reference to the nature
of the course of conduct or particular
transaction or act, rather than by
reference to its purpose.
The
Canadian Parliament enacted in 1982
an Act to Provide For State Immunity
in Canadian Courts. The Act defines a
commercial activity as any particular
transaction, act or conduct or any
regular course of conduct that by
reason of its nature, is of a
commercial character.
The restrictive theory, which is
intended to be a solution to the host of
problems involving the issue of
sovereign immunity, has created
problems of its own. Legal treatises
and the decisions in countries which
follow the restrictive theory have
difficulty in characterizing whether a
contract of a sovereign state with a
private party is an act jure gestionis or
an act jure imperii.
The restrictive theory came about
because of the entry of sovereign
states
into
purely
commercial
activities remotely connected with the
discharge of governmental functions.
This is particularly true with respect to
the Communist states which took
control
of
nationalized
business
activities and international trading.
(Holy See, The v. Rosario, Jr., 238
SCRA 524, Dec. 1, 1994, En Banc
[Quiason])
384. Cite some transactions by a
foreign state with private parties that
were considered by the Supreme Court
as acts jure imperii and acts jure
gestionis.
Held: This Court has considered the
following transactions by a foreign

state with private parties as acts jure


imperii: (1) the lease by a foreign
government of apartment buildings for
use of its military officers (Syquia v.
Lopez, 84 Phil. 312 [1949]); (2) the
conduct of public bidding for the repair
of a wharf at a United States Naval
Station (United States of America v.
Ruiz, supra.); and (3) the change of
employment status of base employees
(Sanders v. Veridiano, 162 SCRA 88
[1988]).
On the other hand, this Court has
considered the following transactions
by a foreign state with private parties
as acts jure gestionis: (1) the hiring of
a cook in the recreation center,
consisting of three restaurants, a
cafeteria, a bakery, a store, and a
coffee and pastry shop at the John Hay
Air Station in Baguio City, to cater to
American servicemen and the general
public (United States of America v.
Rodrigo, 182 SCRA 644 [1990]; and (2)
the bidding for the operation of barber
shops in Clark Air Base in Angeles City
(United States of America v. Guinto,
182 SCRA 644 [1990]). The operation
of the restaurants and other facilities
open to the general public is
undoubtedly for profit as a commercial
and not a governmental activity. By
entering into the employment contract
with the cook in the discharge of its
proprietary function, the United States
government impliedly divested itself of
it sovereign immunity from suit. (Holy
See, The v. Rosario, Jr., 238 SCRA
524, Dec. 1, 1994, En Banc
[Quiason])
385. What should be the guidelines
to determine what activities and
transactions shall be considered
commercial and as constituting acts
jure gestionis by a foreign state?
Held: In the absence of legislation
defining
what
activities
and
transactions shall be considered
commercial and as constituting acts
jure gestionis, we have to come out
with our own guidelines, tentative they
may be.
Certainly, the mere entering into a
contract by a foreign state with a
private party cannot be the ultimate
test. Such an act can only be the start
of the inquiry. The logical question is
whether the foreign state is engaged
in the activity in the regular course of
business. If the foreign state is not
engaged regularly in a business or
trade, the particular act or transaction
must then be tested by its nature. If

the act is in pursuit of a sovereign


activity, or an incident thereof, then it
is an act jure imperii, especially when
it is not undertaken for gain or profit.
As held in United States of America v.
Guinto (supra.):
There is no question that the United
States of America, like any other state,
will be deemed to have impliedly
waived its non-suability if it has
entered into a contract in its
proprietary or private capacity. It is
only when the contract involves its
sovereign or governmental capacity
that no such waiver may be implied.
(Holy See, The v. Rosario, Jr., 238
SCRA 524, Dec. 1, 1994, En Banc
[Quiason])
386. May the Holy See be sued for
selling the land it acquired by donation
from the Archdiocese of Manila to be
made site of its mission or the
Apostolic Nunciature in the Philippines
but
which
purpose
cannot
be
accomplished as the land was
occupied by squatters who refused to
vacate the area?
Held:
In the case at bench, if
petitioner (Holy See) has bought and
sold lands in the ordinary course of a
real estate business, surely the said
transaction can be categorized as an
act jure gestionis. However, petitioner
has denied that the acquisition and
subsequent disposal of Lot 5-A were
made for profit but claimed that it
acquired said property for the site of
its mission or the Apostolic Nunciature
in the Philippines. X x x
Lot 5-A was acquired by petitioner as a
donation from the Archdiocese of
Manila. The donation was made not
for commercial purpose, but for the
use of petitioner to construct thereon
the official place of residence of the
Papal Nuncio. The right of a foreign
sovereign to acquire property, real or
personal,
in
a
receiving
state,
necessary for the creation and
maintenance of its diplomatic mission,
is recognized in the 1961 Vienna
Convention on Diplomatic Relations.
This treaty was concurred in by the
Philippine Senate and entered into
force in the Philippines on November
15, 1965.
In Article 31(a) of the Convention, a
diplomatic envoy is granted immunity
from the civil and administrative
jurisdiction of the receiving state over
any real action relating to private

immovable property situated in the


territory of the receiving state which
the envoy holds on behalf of the
sending state for the purposes of the
mission. If this immunity is provided
for a diplomatic envoy, with all the
more reason should immunity be
recognized as regards the sovereign
itself, which in this case is the Holy
See.
The decision to transfer the property
and the subsequent disposal thereof
are
likewise
clothed
with
a
governmental character. Petitioner did
not sell Lot 5-A for profit or gain. It
merely wanted to dispose off the same
because the squatters living thereon
made
it almost
impossible
for
petitioner to use it for the purpose of
the donation.
(Holy See, The v.
Rosario, Jr., 238 SCRA 524, Dec. 1,
1994, En Banc [Quiason])
387. How is sovereign or diplomatic
immunity pleaded in a foreign court?
Held:
In Public International Law,
when a state or international agency
wishes
to
plead
sovereign
or
diplomatic immunity in a foreign court,
it requests the Foreign Office of the
state where it is sued to convey to the
court that said defendant is entitled to
immunity.
In the United States, the procedure
followed
is
the
process
of
suggestion, where the foreign state
or the international organization sued
in an American court requests the
Secretary of State to make a
determination as to whether it is
entitled to immunity. If the Secretary
of State finds that the defendant is
immune from suit, he, in turn, asks the
Attorney General to submit to the
court
a
suggestion
that
the
defendant is entitled to immunity. In
England, a similar procedure is
followed, only the Foreign Office issues
a certification to that effect instead of
submitting a suggestion.
In the Philippines, the practice is for
the foreign government or the
international organization to first
secure an executive endorsement of
its claim of sovereign or diplomatic
immunity.
But how the Philippine
Foreign
Office
conveys
its
endorsement to the courts varies. In
International
Catholic
Migration
Commission v. Calleja, 190 SCRA 130
(1990), the Secretary of Foreign Affairs
just sent a letter directly to the
Secretary of Labor and Employment,

informing
the
latter
that
the
respondent-employer could not be
sued because it enjoyed diplomatic
immunity.
In
World
Health
Organization v. Aquino, 48 SCRA 242
(1972), the Secretary of Foreign Affairs
sent the trial court a telegram to that
effect. In Baer v. Tizon, 57 SCRA 1
(1974), the U.S. Embassy asked the
Secretary of Foreign Affairs to request
the Solicitor General to make, in
behalf of the commander of the United
States Naval Base at Olongapo City,
Zambales,
a
suggestion
to
respondent Judge.
The Solicitor
General embodied the suggestion in
a Manifestation and Memorandum as
amicus curiae.
In the case at bench, the Department
of Foreign Affairs, through the Office of
Legal Affairs moved with this Court to
be allowed to intervene on the side of
petitioner. The Court allowed the said
Department to file its memorandum in
support of petitioners claim of
sovereign immunity.
In some cases, the defense of
sovereign immunity was submitted
directly to the local courts by the
respondents through their private
counsels. In cases where the foreign
states bypass the Foreign Office, the
courts can inquire into the facts and
make their own determination as to
the nature of the acts and transactions
involved.
(Holy See, The v.
Rosario, Jr., 238 SCRA 524, Dec. 1,
1994, En Banc [Quiason])
388. Is the determination of the
executive branch of the government
that a state or instrumentality is
entitled to sovereign or diplomatic
immunity subject to judicial review, or
is it a political question and therefore,
conclusive upon the courts?
Held: The issue of petitioners (The
Holy See) non-suability can be
determined by the trial court without
going to trial in light of the pleadings x
x x.
Besides, the privilege of
sovereign immunity in this case was
sufficiently
established
by
the
Memorandum and Certification of the
Department of Foreign Affairs. As the
department tasked with the conduct of
the Philippines foreign relations, the
Department of Foreign Affairs has
formally intervened in this case and
officially certified that the Embassy of
the Holy See is a duly accredited
diplomatic mission to the Republic of
the Philippines exempt from local
jurisdiction and entitled to all the

rights, privileges and immunities of a


diplomatic mission or embassy in this
country.
The determination of the
executive arm of government that a
state or instrumentality is entitled to
sovereign or diplomatic immunity is a
political question that is conclusive
upon the courts. Where the plea of
immunity is recognized and affirmed
by the executive branch, it is the duty
of the courts to accept this claim so as
not to embarrass the executive arm of
the government in conducting the
countrys foreign relations.
As in
International
Catholic
Migration
Commission and in World Health
Organization, we abide by the
certification of the Department of
Foreign Affairs. (Holy See, The v.
Rosario, Jr., 238 SCRA 524, Dec. 1,
1994, En Banc [Quiason])
389. Discuss the
Vatican and the
International Law.

Status of the
Holy See in

Held: Before the annexation of


the Papal States by Italy in 1870, the
Pope was the monarch and he, as the
Holy See, was considered a subject of
International Law. With the loss of the
Papal States and the limitation of the
territory under the Holy See to an area
of 108.7 acres, the position of the Holy
See in International Law became
controversial (Salonga and Yap, Public
International Law 36-37 [1992]).
In 1929, Italy and the Holy See
entered into the Lateran Treaty, where
Italy
recognized
the
exclusive
dominion and sovereign jurisdiction of
the Holy See over the Vatican City. It
also recognized the right of the Holy
See to receive foreign diplomats, to
send its own diplomats to foreign
countries, and to enter into treaties
according to International Law (Garcia,
Questions
and
Problems
in
International Law, Public and Private
81 [1948]).
The Lateran Treaty established
the statehood of the Vatican City "for
the purpose of assuring to the Holy
See absolute and visible independence
and of guaranteeing to it indisputable
sovereignty also in the field of
international relations" (O'Connell, I
International Law 311 [1965]).
In view of the wordings of the
Lateran Treaty, it is difficult to
determine whether the statehood is
vested in the Holy See or in the
Vatican City.
Some writers even
suggested that the treaty created two

international persons - the Holy See


and Vatican City (Salonga and Yap,
supra, 37).
The Vatican City fits into none of
the established categories of states,
and
the
attribution
to
it
of
sovereignty must be made in a
sense different from that in which it is
applied to other states (Fenwick,
International Law 124-125 [1948];
Cruz, International Law 37 [1991]). In
a community of national states, the
Vatican City represents an entity
organized not for political but for
ecclesiastical
purposes
and
international objects. Despite its size
and object, the Vatican City has an
independent government of its own,
with the Pope, who is also head of the
Roman Catholic Church, as the Holy
See or Head of State, in conformity
with its traditions, and the demands of
its mission in the world. Indeed, the
world-wide interests and activities of
the Vatican City are such as to make it
in a sense an international state
(Fenwick,
supra,
125;
Kelsen,
Principles of International Law 160
[1956]).
One authority wrote that the
recognition of the Vatican City as a
state has significant implication - that
it is possible for any entity pursuing
objects essentially different from those
pursued by states to be invested with
international personality (Kunz, The
Status of the Holy See in International
Law, 46 The American Journal of
International Law 308 [1952]).
Inasmuch as the Pope prefers to
conduct foreign relations and enter
into transactions as the Holy See and
not in the name of the Vatican City,
one can conclude that in the Pope's
own view, it is the Holy See that is the
international person.
The Republic of the Philippines
has accorded the Holy See the status
of a foreign sovereign. The Holy See,
through its Ambassador, the Papal
Nuncio,
has
had
diplomatic
representations with the Philippine
government since 1957. This appears
to be the universal practice in
international relations.
(Holy See,
The v. Rosario, Jr., 238 SCRA 524,
533-534, Dec. 1, 1994, En Banc
[Quiason])
390. What is extradition?
does it apply?

To whom

Held: It is the process by which


persons charged with or convicted of
crime against the law of a State and
found in a foreign State are returned
by the latter to the former for trial or
punishment. It applies to those who
are merely charged with an offense
but have not been brought to trial; to
those who have been tried and
convicted and have subsequently
escaped from custody; and those who
have been convicted in absentia. It
does not apply to persons merely
suspected of having committed an
offense but against whom no charge
has been laid or to a person whose
presence is desired as a witness or for
obtaining
or
enforcing
a
civil
judgment.
(Weston,
Falk,
D'
Amato, International Law and
Order, 2nd ed., p. 630 [1990],
cited in Dissenting Opinion, Puno,
J., in Secretary of Justice v. Hon.
Ralph C. Lantion, G.R. No. 139465,
Jan. 18, 2000, En Banc)
391. Discuss the basis for allowing
extradition.
Held: Extradition was first practiced
by the Egyptians, Chinese, Chaldeans
and Assyro-Babylonians but their basis
for allowing extradition was unclear.
Sometimes, it was granted due to
pacts; at other times, due to plain
good will. The classical commentators
on international law thus focused their
early views on the nature of the duty
to surrender an extraditee --- whether
the duty is legal or moral in character.
Grotius and Vattel led the school of
thought that international law imposed
a legal duty called civitas maxima to
extradite criminals. In sharp contrast,
Puffendorf and Billot led the school of
thought that the so-called duty was
but an "imperfect obligation which
could become enforceable only by a
contract or agreement between states.
Modern nations tilted towards the view
of Puffendorf and Billot that under
international law there is no duty to
extradite in the absence of treaty,
whether bilateral or multilateral.
Thus, the US Supreme Court in US v.
Rauscher (119 US 407, 411, 7 S Ct.
234, 236, 30 L. ed. 425 [1886]), held:
x x x it is only in modern times that
the nations of the earth have imposed
upon themselves the obligation of
delivering up these fugitives from
justice to the states where their crimes
were
committed,
for
trial
and
punishment.
This has been done
generally by treaties x x x Prior to
these treaties, and apart from them

there was no well-defined obligation


on one country to deliver up such
fugitives to another; and though such
delivery was often made it was upon
the principle of comity x x x.
(Dissenting Opinion, Puno, J., in
Secretary of Justice v. Hon. Ralph
C. Lantion, G.R. No. 139465, Jan.
18, 2000, En Banc)
392. What is the nature of an
extradition proceeding? Is it akin to a
criminal proceeding?
Held: [A]n extradition proceeding is
sui generis.
It is not a criminal
proceeding which will call into
operation all the rights of an accused
as guaranteed by the Bill of Rights. To
begin with, the process of extradition
does not involve the determination of
the guilt or innocence of an accused.
His guilt or innocence will be adjudged
in the court of the state where he will
be extradited.
Hence, as a rule,
constitutional rights that are only
relevant to determine the guilt or
innocence of an accused cannot be
invoked by an extraditee especially by
one whose extradition papers are still
undergoing evaluation. As held by the
US Supreme Court in United States v.
Galanis:
An extradition proceeding is not a
criminal
prosecution,
and
the
constitutional
safeguards
that
accompany a criminal trial in this
country do not shield an accused from
extradition pursuant to a valid treaty.
(Wiehl,
Extradition
Law
at
the
Crossroads:
The
Trend
Toward
Extending
Greater
Constitutional
Procedural Protections To Fugitives
Fighting Extradition from the United
States, 19 Michigan Journal of
International Law 729, 741 [1998],
citing United States v. Galanis, 429 F.
Supp. 1215 [D. Conn. 1977])
There are other differences between
an extradition proceeding and a
criminal proceeding. An extradition
proceeding is summary in natural
while criminal proceedings involve a
full-blown trial. In contradistinction to
a criminal proceeding, the rules of
evidence in an extradition proceeding
allow admission of evidence under less
stringent standards. In terms of the
quantum of evidence to be satisfied, a
criminal case requires proof beyond
reasonable doubt for conviction while
a fugitive may be ordered extradited
upon showing of the existence of a
prima facie case. Finally, unlike in a
criminal
case
where
judgment

becomes
executory
upon
being
rendered final, in an extradition
proceeding, our courts may adjudge
an individual extraditable but the
President has the final discretion to
extradite him.
The United States
adheres to a similar practice whereby
the Secretary of State exercises wide
discretion in balancing the equities of
the case and the demands of the
nation's
foreign
relations
before
making the ultimate decision to
extradite.
As an extradition proceeding is not
criminal
in
character
and
the
evaluation stage in an extradition
proceeding is not akin to a preliminary
investigation,
the
due
process
safeguards in the latter do not
necessarily apply to the former. This
we hold for the procedural due process
required
by
a
given
set
of
circumstances must begin with a
determination of the precise nature of
the government function involved as
well as the private interest that has
been
affected
by
governmental
action. The concept of due process is
flexible for not all situations calling
for procedural safeguards call for the
same kind of procedure. (Secretary
of Justice v. Hon. Ralph C. Lantion,
G.R. No. 139465, Oct. 17, 2000, En
Banc [Puno])
393. Will the retroactive application
of an extradition treaty violate the
constitutional prohibition against "ex
post facto" laws?
Held: The prohibition against ex post
facto law applies only to criminal
legislation
which
affects
the
substantial rights of the accused. This
being so, there is no merit in the
contention that the ruling sustaining
an extradition treatys retroactive
application violates the constitutional
prohibition against ex post facto laws.
The treaty is neither a piece of
criminal legislation nor a criminal
procedural statute. (Wright v. CA,
235 SCRA 341, Aug. 15, 1994
[Kapunan])
394. Discuss the rules in the
interpretation of extradition treaties.
Held: [A]ll treaties, including the RPUS Extradition Treaty, should be
interpreted in light of their intent.
Nothing
less
than
the
Vienna
Convention on the Law of Treaties to
which the Philippines is a signatory
provides that a treaty shall be
interpreted in good faith in accordance

with the ordinary meaning to be given


to the terms of the treaty in their
context and in light of its object and
purpose. X x x. It cannot be gainsaid
that
today,
countries
like
the
Philippines forge extradition treaties to
arrest
the
dramatic
rise
of
international and transnational crimes
like terrorism and drug trafficking.
Extradition
treaties
provide
the
assurance that the punishment of
these crimes will not be frustrated by
the frontiers of territorial sovereignty.
Implicit in the treaties should be the
unbending commitment that the
perpetrators of these crimes will not
be coddled by any signatory state.
It ought to follow that the RP-US
Extradition
Treaty
calls
for
an
interpretation that will minimize if not
prevent the escape of extraditees from
the long arm of the law and expedite
their trial. X x x
[A]n equally compelling factor to
consider is the understanding of the
parties themselves to the RP-US
Extradition Treaty as well as the
general interpretation of the issue in
question by other countries with
similar treaties with the Philippines.
The rule is recognized that while
courts have the power to interpret
treaties, the meaning given them by
the
departments of
government
particularly
charged
with
their
negotiation
and
enforcement
is
accorded great weight. The reason
for the rule is laid down in Santos III v.
Northwest Orient Airlines, et al. (210
SCRA 256, 261 [1992]), where we
stressed that a treaty is a joint
executive-legislative act which enjoys
the presumption that it was first
carefully studied and determined to be
constitutional before it was adopted
and given the force of law in the
country. (Secretary of Justice v.
Hon. Ralph C. Lantion, G.R. No.
139465, Oct. 17, 2000, En Banc
[Puno])
395. Is respondent Mark Jimenez
entitled to bail during the pendency of
the Extradition Proceeding?
Held: We agree with petitioner. As
suggested by the use of the word
conviction,
the
constitutional
provision on bail x x x, as well as
Section 4 of Rule 114 of the Rules of
Court, applies only when a person has
been arrested and detained for
violation of Philippine criminal laws. It
does
not
apply
to
extradition
proceedings,
because
extradition

courts do not render judgments of


conviction or acquittal.
Moreover, the constitutional
right to bail flows from the
presumption of innocence in favor of
every accused who should not be
subjected to the loss of freedom as
thereafter he would be entitled to
acquittal, unless his guilt be proved
beyond reasonable doubt. (De la
Camara v. Enage, 41 SCRA 1, 6,
September 17, 1971, per Fernando, Jr.
[later CJ])
It follows that the
constitutional provision on bail will not
apply to a case like extradition, where
the presumption of innocence is not
an issue.
The provision in the Constitution
stating that the right to bail shall not
be impaired even when the privilege
of the writ of habeas corpus is
suspended does not detract from the
rule that the constitutional right to bail
is
available
only
in
criminal
proceedings. It must be noted that
the suspension of the privilege of the
writ of habeas corpus finds application
only to persons judicially charged for
rebellion or offenses inherent in or
directly connected with invasion.
(Sec. 18, Article VII, Constitution)
Hence, the second sentence in the
constitutional provision on bail merely
emphasizes the right to bail in criminal
proceedings for the aforementioned
offenses. It cannot be taken to mean
that the right is available even in
extradition proceedings that are not
criminal in nature.
That the offenses for which Jimenez is
sought to be extradited are bailable in
the United States is not an argument
to grant him one in the present case.
To stress, extradition proceedings are
separate and distinct from the trial for
the offenses for which he is charged.
He should apply for bail before the
courts trying the criminal cases
against him, not before the extradition
court. (Government of the United
States
of
America
v.
Hon.
Guillermo Purganan, G.R. No.
148571, Sept. 24, 2002, En Banc
[Panganiban])
396. What is the exception to the
No
Bail
Rule
in
Extradition
Proceedings?
Held: The rule x x x is that bail is not
a matter of right in extradition cases.
However, the judiciary has the
constitutional duty to curb grave
abuse of discretion and tyranny, as

well as the power to promulgate rules


to protect and enforce constitutional
rights. Furthermore, we believe that
the right to due process is broad
enough to include the grant of basic
fairness to extraditees. Indeed, the
right to due process extends to the
life, liberty or property of every
person. It is dynamic and resilient,
adaptable to every situation calling for
its
application.
(I.A.
Cruz,
Constitutional Law, 1998 ed., p. 98)
Accordingly and to best serve
the ends of justice, we believe and so
hold that, after a potential extraditee
has been arrested or placed under the
custody of the law, bail may be
applied for and granted as an
exception, only upon a clear and
convincing showing (1) that, once
granted bail, the applicant will not be
a flight risk or a danger to the
community; and (2) that there exist
special, humanitarian and compelling
circumstances including, as a matter
of reciprocity, those cited by the
highest court in the requesting state
when it grants provisional liberty in
extradition cases therein.
Since this exception has no
express or specific statutory basis, and
since it is derived essentially from
general principles of justice and
fairness, the applicant bears the
burden of proving the above twotiered requirement with clarity,
precision and emphatic forcefulness.
The Court realizes that extradition is
basically an executive, not a judicial,
responsibility arising from the
presidential power to conduct foreign
relations. In its barest concept, it
partakes of the nature of police
assistance amongst states, which is
not normally a judicial prerogative.
Hence, any intrusion by the courts into
the exercise of this power should be
characterized by caution, so that the
vital international and bilateral
interests of our country will not be
unreasonably impeded or
compromised. In short, while this
Court is ever protective of the
sporting idea of fair play, it also
recognizes the limits of its own
prerogatives and the need to fulfill

international obligations.
(Government of the United States
of America v. Hon. Guillermo
Purganan, G.R. No. 148571, Sept.
24, 2002, En Banc [Panganiban])
397. Are there special circumstances
compelling enough for the Court to
grant Mark Jimenezs request for
provisional release on bail?
Held: Along this line, Jimenez
contends that there are special
circumstances that are compelling
enough for the Court to grant his
request for provisional release on bail.
We have carefully examined these
circumstances and shall now discuss
them.
1. Alleged Disenfranchisement
While his extradition was
pending, Respondent Jimenez was
elected as a member of the House of
Representatives. On that basis, he
claims that his detention will
disenfranchise his Manila district of
600,000 residents. We are not
persuaded. In People v. Jalosjos (324
SCRA 689, February 3, 2000, per
Ynares-Santiago, J.), the Court has
already debunked the
disenfranchisement argument x x x.
It must be noted that even
before private respondent ran for and
won a congressional seat in Manila, it
was already of public knowledge that
the United States was requesting his
extradition. Hence, his constituents
were or should have been prepared for
the consequences of the extradition
case against their representative,
including his detention pending the
final resolution of the case. Premises
considered and in line with Jalosjos, we
are constrained to rule against his
claim that his election to public office
is by itself a compelling reason to
grant him bail.
2. Anticipated Delay
Respondent Jimenez further
contends that because the extradition

proceedings are lengthy, it would be


unfair to confine him during the
pendency of the case. Again we are
not convinced. We must emphasize
that extradition cases are summary in
nature. They are resorted to merely to
determine whether the extradition
petition and its annexes conform to
the Extradition Treaty, not to
determine his guilt or innocence.
Neither is it, as a rule, intended to
address issues relevant to the
constitutional rights available to the
accused in a criminal action.
We are not overruling the
possibility that petitioner may, in bad
faith, unduly delay the proceedings.
This is quite another matter that is not
at issue here. Thus, any further
discussion of this point would be
merely anticipatory and academic.
However, if the delay is due to
maneuverings of respondent, with all
the more reason would the grant of
bail not be justified. Giving premium
to delay by considering it as a special
circumstance for the grant of bail
would be tantamount to giving him the
power to grant bail to himself. It
would also encourage him to stretch
out and unreasonably delay the
extradition proceedings even more.
This we cannot allow.
3. Not a Flight Risk?
Jimenez further claims that he is
not a flight risk. To support this claim,
he stresses that he learned of the
extradition request in June 1999; yet,
he has not fled the country. True, he
has not actually fled during the
preliminary stages of the request for
his extradition. Yet, this fact cannot be
taken to mean that he will not flee as
the process moves forward to its
conclusion, as he hears the footsteps
of the requesting government inching
closer and closer. That he has not yet
fled from the Philippines cannot be
taken to mean that he will stand his
ground and still be within reach of our
government if and when it matters;

that is, upon the resolution of the


Petition for Extradition.
In any event, it is settled that
bail may be applied for and granted by
the trial court at anytime after the
applicant has been taken into custody
and prior to judgment, even after bail
has been previously denied. In the
present case, the extradition court
may continue hearing evidence on the
application for bail, which may be
granted in accordance with the
guidelines in this Decision.
(Government of the United States
of America v. Hon. Guillermo
Purganan, G.R. No. 148571, Sept.
24, 2002, En Banc [Panganiban])

398. Discuss the Five Postulates of


Extradition.
Held:
1. Extradition Is a Major Instrument
for the Suppression of Crime.
First, extradition treaties are
entered into for the purpose of
suppressing crime by facilitating the
arrest
and
custodial
transfer
(Bassiouni, International Extradition,
1987 ed., p. 68) of a fugitive from one
state to the other.
With the advent of easier and
faster means of international travel,
the flight of affluent criminals from
one country to another for the purpose
of committing crime and evading
prosecution
has
become
more
frequent.
Accordingly, governments
are adjusting their methods of dealing
with criminals and crimes that
transcend international boundaries.
Today, a majority of nations in
the world community have come to
look upon extradition as the major
effective instrument of international
co-operation in the suppression of
crime. (Bassiouni, supra, p. 21) It is
the only regular system that has been
devised to return fugitives to the
jurisdiction of a court competent to try
them in accordance with municipal
and international law (Id., p. 67).
Xxx

Indeed,
in
this
era
of
globalization,
easier
and
faster
international travel, and an expanding
ring of international crimes and
criminals, we cannot afford to be an
isolationist state.
We need to
cooperate with other states in order to
improve our chances of suppressing
crime in our country.
2. The Requesting State Will
Accord Due Process to the Accused.
Second, an extradition treaty
presupposes that both parties thereto
have examined, and that both accept
and trust, each others legal system
and judicial process (Coquia, On
Implementation
of
the
RP-US
Extradition Treaty, The Lawyers
Review, August 31, 2000, p. 4). More
pointedly,
our
duly
authorized
representatives signature on an
extradition
treaty
signifies
our
confidence in the capacity and
willingness of the other state to
protect the basic rights of the person
sought
to
be
extradited
(See
Bassiouni, p. 546; citing 221 US 508,
512 [1910]). That signature signifies
our full faith that the accused will be
given, upon extradition to the
requesting state, all relevant and basic
rights in the criminal proceedings that
will take place therein; otherwise, the
treaty would not have been signed, or
would have been directly attacked for
its unconstitutionality.
3.
Generis.

The Proceedings Are Sui

Third, as
pointed
out in
Secretary of Justice v. Lantion (Supra),
extradition
proceedings
are
not
criminal in nature.
In criminal
proceedings, the constitutional rights
of the accused are at fore; in
extradition which is sui generis in a
class by itself they are not.
Xxx
Given the foregoing, it is
evident that the extradition court is
not called upon to ascertain the guilt
or the innocence of the person sought
to be extradited (Secretary of Justice v.
Lantion, supra.). Such determination
during the extradition proceedings will
only result in needless duplication and
delay. Extradition is merely a measure
of international judicial assistance
through which a person charged with
or convicted of a crime is restored to a
jurisdiction with the best claim to try
that person.
It is not part of the

function of the assisting authorities to


enter into questions that are the
prerogative
of
that
jurisdiction
(Shearer, Extradition in International
Law, 1971 ed., p. 157). The ultimate
purpose of extradition proceedings in
court is only to determine whether the
extradition request complies with the
Extradition Treaty, and whether the
person sought is extraditable (Id., p.
545).
4

Compliance Shall Be in Good Faith.


Fourth, our executive branch of
government voluntarily entered into
the Extradition Treaty, and our
legislative branch ratified it. Hence,
the Treaty carries the presumption
that its implementation will serve the
national interest.
Fulfilling our obligations under the
Extradition Treaty promotes comity (In
line with the Philippine policy of
cooperation and amity with all nations
set forth in Article II, Section 2,
Constitution).
On the other hand,
failure to fulfill our obligations
thereunder paints at bad image of our
country before the world community.
Such failure would discourage other
states from entering into treaties with
us, particularly an extradition treaty
that hinges on reciprocity.
Verily, we are bound by pacta sunt
servanda to comply in good faith with
our obligations under the Treaty
(Secretary of Justice v. Lantion,
supra.). This principle requires that
we deliver the accused to the
requesting country if the conditions
precedent to extradition, as set forth
in the Treaty, are satisfied. In other
words, [t]he demanding government,
when it has done all that the treaty
and the law require it to do, is entitled
to the delivery of the accused on the
issue of the proper warrant, and the
other government is under obligation
to make the surrender. (Wright v.
Henkel, 190 U.S. 40, 62, March 23,
1903)
Accordingly, the Philippines
must be ready and in a position to
deliver the accused, should it be found
proper.

There Is an Underlying Risk of Flight.


Fifth, persons to be extradited are
presumed to be flight risks.
This
prima
facie
presumption
finds
reinforcement in the experience of the
executive branch: nothing short of
confinement can ensure that the
accused will not flee the jurisdiction of

the requested state in order to thwart


their extradition to the requesting
state.
The present extradition case further
validates the premise that persons
sought to be extradited have a
propensity to flee. Indeed, extradition
hearings would not even begin, if only
the accused were willing to submit to
trial in the requesting country (Persily,
International Extradition and the
Right to Bail, 34 Stan. J. Intl L. 407
[Summer 1988]). Prior acts of herein
respondent

(1)
leaving
the
requesting state right before the
conclusion
of
his
indictment
proceedings there; and (2) remaining
in the requested state despite learning
that the requesting state is seeking his
return and that the crimes he is
charged with are bailable eloquently
speak of his aversion to the processes
in the requesting state, as well as his
predisposition to avoid them at all
cost. These circumstances point to an
ever-present, underlying high risk of
flight. He has demonstrated that he
has the capacity and the will to flee.
Having fled once, what is there to stop
him, given sufficient opportunity, from
fleeing a second time? (Government
of the United States of America v.
Hon. Guillermo Purganan, G.R. No.
148571, Sept. 24, 2002, En Banc
[Panganiban])
399. Discuss the Ten Points to
consider in Extradition Proceedings?
Held: 1. The ultimate purpose of
extradition
proceedings
is
to
determine
whether
the
request
expressed in the petition, supported
by its annexes and the evidence that
may be adduced during the hearing of
the petition, complies with the
Extradition Treaty and Law; and
whether
the
person
sought
is
extraditable.
The proceedings are
intended
merely
to
assist
the
requesting state in bringing the
accused or the fugitive who has
illegally escaped back to its territory,
so that the criminal process may
proceed therein.
2. By entering into an extradition
treaty, the Philippines is deemed to
have reposed its trust in the reliability
or soundness of the legal and judicial
system of its treaty partner; as well as
in the ability and the willingness of the
latter to grant basic rights to the

accused in the pending criminal case


therein.

discretion in the context of the


peculiar facts of each case.

3. By nature then, extradition


proceedings are not equivalent to a
criminal case in which guilt or
innocence is determined.
Consequently, an extradition case is
not one in which the constitutional
rights of the accused are necessarily
available. It is more akin, if at all, to a
courts request to police authorities for
the arrest of the accused who is at
large or has escaped detention or
jumped bail. Having once escaped the
jurisdiction of the requesting state, the
reasonable prima facie presumption is
that the person would escape again if
given the opportunity.

6. Potential extraditees are entitled to


the rights to due process and to
fundamental fairness. Due process
does not always call for a prior
opportunity to be heard. A
subsequent opportunity is sufficient
due to the flight risk involved. Indeed,
available during the hearings on the
petition and the answer is the full
chance to be heard and to enjoy
fundamental fairness that is
compatible with the summary nature
of extradition.

4. Immediately upon receipt of the


petition for extradition and its
supporting documents, the judge shall
make a prima facie finding whether
the petition is sufficient in form and
substance, whether it complies with
the Extradition Treaty and Law, and
whether the person sought is
extraditable. The magistrate has
discretion to require the petitioner to
submit further documentation, or to
personally examine the affiants or
witnesses. If convinced that a prima
facie case exists, the judge
immediately issues a warrant for the
arrest of the potential extraditee and
summons him or her to answer and to
appear at scheduled hearings on the
petition.
5. After being taken into custody,
potential extraditees may apply for
bail. Since the applicants have a
history of absconding, they have the
burden of showing that (a) there is no
flight risk and no danger to the
community; and (b) there exist
special, humanitarian or compelling
circumstances. The grounds used by
the highest court in the requesting
state for the grant of bail therein may
be considered, under the principle of
reciprocity as a special circumstance.
In extradition cases, bail is not a
matter of right; it is subject to judicial

7. This Court will always remain a


protector of human rights, a bastion of
liberty, a bulwark of democracy and
the conscience of society. But it is
also well aware of the limitations of its
authority and of the need for respect
for the prerogatives of the other coequal and co-independent organs of
government.
8. We realize that extradition is
essentially an executive, not a judicial,
responsibility arising out of the
presidential power to conduct foreign
relations and to implement treaties.
Thus, the Executive Department of
government has broad discretion in its
duty and power of implementation.
9. On the other hand, courts merely
perform oversight functions and
exercise review authority to prevent or
excise grave abuse and tyranny. They
should not allow contortions, delays
and over-due process every little
step of the way, lest these summary
extradition proceedings become not
only inutile but also sources of
international embarrassment due to
our inability to comply in good faith
with a treaty partners simple request
to return a fugitive. Worse, our
country should not be converted into a
dubious haven where fugitives and
escapees can unreasonably delay,
mummify, mock, frustrate, checkmate
and defeat the quest for bilateral
justice and international cooperation.

10. At bottom, extradition


proceedings should be conducted with
all deliberate speed to determine
compliance with the Extradition Treaty
and Law; and, while safeguarding
basic individual rights, to avoid the
legalistic contortions, delays and
technicalities that may negate that
purpose. (Government of the
United States of America v. Hon.
Guillermo Purganan, G.R. No.
148571, Sept. 24, 2002, En Banc
[Panganiban])

400.

What is a Treaty? Discuss.

Held: A treaty, as defined by the


Vienna Convention on the Law of
Treaties,
is
an
international
instrument concluded between States
in written form and governed by
international law, whether embodied
in a single instrument or in two or
more
related
instruments,
and
whatever its particular designation.
There are many other terms used for a
treaty or international agreement,
some of which are: act, protocol,
agreement, compromis d' arbitrage,
concordat, convention, declaration,
exchange of notes, pact, statute,
charter and modus vivendi.
All
writers, from Hugo Grotius onward,
have pointed out that the names or
titles of international agreements
included under the general term treaty
have little or no significance. Certain
terms are useful, but they furnish little
more than mere description
Article 2(2) of the Vienna Convention
provides that the provisions of
paragraph 1 regarding the use of
terms in the present Convention are
without prejudice to the use of those
terms, or to the meanings which may
be given to them in the internal law of
the State.
(BAYAN [Bagong
Alyansang
Makabayan]
v.
Executive
Secretary
Ronaldo
Zamora, G.R. No. 138570, Oct. 10,
2000, En Banc [Buena])
401. Discuss the binding effect of
treaties and executive agreements in
international law.
Held: [I]n international law, there is
no difference between treaties and
executive agreements in their binding
effect upon states concerned, as long
as the functionaries have remained

within their powers. International law


continues to make no distinction
between
treaties
and
executive
agreements: they are equally binding
obligations upon nations.
(BAYAN
[Bagong Alyansang Makabayan] v.
Executive
Secretary
Ronaldo
Zamora, G.R. No. 138570, Oct. 10,
2000, En Banc [Buena])
402. Does the Philippines recognize
the binding effect of executive
agreements
even
without
the
concurrence
of
the
Senate
or
Congress?
Held:
In our jurisdiction, we have
recognized the binding effect of
executive agreements even without
the concurrence of the Senate or
Congress.
In Commissioner of
Customs v. Eastern Sea Trading (3
SCRA 351, 356-357 [1961]), we had
occasion to pronounce:
x x x the right of the Executive to
enter into binding agreements without
the
necessity
of
subsequent
Congressional approval has been
confirmed by long usage. From the
earliest days of our history we have
entered into executive agreements
covering such subjects as commercial
and consular relations, most-favorednation rights, patent rights, trademark
and copyright protection, postal and
navigation arrangements and the
settlement of claims. The validity of
these has never been seriously
questioned by our courts. "
(BAYAN
[Bagong
Alyansang
Makabayan]
v.
Executive
Secretary Ronaldo Zamora, G.R.
No. 138570, Oct. 10, 2000, En
Banc [Buena])
403. What is a "protocol de cloture"?
Will it require concurrence by the
Senate?
Held: A final act, sometimes called
protocol de cloture, is an instrument
which records the winding up of the
proceedings
of
a
diplomatic
conference and usually includes a
reproduction of the texts of treaties,
conventions, recommendations and
other acts agreed upon and signed by
the plenipotentiaries attending the
conference. It is not the treaty itself.
It is rather a summary of the
proceedings
of
a
protracted
conference which may have taken
place over several years. It will not
require the concurrence of the Senate.
The documents contained therein are
deemed adopted without need for

ratification. (Tanada v. Angara, 272


SCRA
18,
May
2,
1997
[Panganiban])
404. What is the most-favorednation clause? What is its purpose?
Answer: 1. The most-favored-nation
clause may be defined, in general, as
a pledge by a contracting party to a
treaty to grant to the other party
treatment not less favorable than that
which has been or may be granted to
the most favored among other
countries.
The clause has been
commonly included in treaties of
commercial nature.
There are generally two types of mostfavored-nation
clause,
namely,
conditional
and
unconditional.
According to the clause in its
unconditional form, any advantage of
whatever kind which has been or may
in future be granted by either of the
contracting parties to a third State
shall
simultaneously
and
unconditionally be extended to the
other under the same or equivalent
conditions as those under which it has
been granted to the third State.
(Salonga
&
Yap,
Public
International Law, 5th Edition,
1992, pp. 141-142)
2. The purpose of a most favored
nation clause is to grant to the
contracting party treatment not less
favorable than that which has been or
may be granted to the "most favored"
among other countries.
The most
favored nation clause is intended to
establish the principle of equality of
international treatment by providing
that the citizens or subjects of the
contracting nations may enjoy the
privileges accorded by either party to
those of the most favored nation
(Commissioner
of
Internal
Revenue v. S.C. Johnson and Son,
Inc., 309 SCRA 87, 107-108, June
25, 1999, 3rd Div.
[GonzagaReyes])
405. What is the essence of the
principle behind the "most-favorednation" clause as applied to tax
treaties?
Held: The essence of the principle is
to allow the taxpayer in one state to
avail of more liberal provisions granted
in another tax treaty to which the
country of residence of such taxpayer
is also a party provided that the
subject matter of taxation x x x is the

same as that in the tax treaty under


which the taxpayer is liable.
In
Commissioner
of
Internal
Revenue v. S.C. Johnson and Son,
Inc., 309 SCRA 87, June 25, 1999,
the SC did not grant the claim filed by
S.C. Johnson and Son, Inc., a nonresident foreign corporation based in
the USA, with the BIR for refund of
overpaid withholding tax on royalties
pursuant to the most-favored-nation
clause of the RP-US Tax Treaty in
relation to the RP-West Germany Tax
Treaty. It held:
Given the purpose underlying tax
treaties and the rationale for the most
favored
nation
clause,
the
concessional tax rate of 10 percent
provided for in the RP-Germany Tax
Treaty should apply only if the taxes
imposed upon royalties in the RP-US
Tax Treaty and in the RP-Germany Tax
Treaty
are
paid
under
similar
circumstances. This would mean that
private respondent (S.C. Johnson and
Son, Inc.) must prove that the RP-US
Tax Treaty grants similar tax reliefs to
residents of the United States in
respect of the taxes imposable upon
royalties earned from sources within
the Philippines as those allowed to
their German counterparts under the
RP-Germany Tax Treaty.
The RP-US and the RP-West Germany
Tax Treaties do not contain similar
provisions on tax crediting. Article 24
of the RP-Germany Tax Treaty x x x
expressly allows crediting against
German income and corporation tax of
20% of the gross amount of royalties
paid under the law of the Philippines.
On the other hand, Article 23 of the
RP-US Tax Treaty, which is the
counterpart provision with respect to
relief for double taxation, does not
provide for similar crediting of 20% of
the gross amount of royalties paid. X
xx
X x x The entitlement of the 10% rate
by U.S. firms despite the absence of
matching credit (20% for royalties)
would derogate from the design
behind the most favored nation clause
to grant equality of international
treatment since the tax burden laid
upon the income of the investor is not
the same in the two countries. The
similarity in the circumstances of
payment of taxes is a condition for the
enjoyment of most favored nation
treatment precisely to underscore the
need for equality of treatment.

406. What is ratification? Discuss its


function in the treaty-making process.

have foreseen at the time the treaty


was concluded.

Held: Ratification is generally held to


be an executive act, undertaken by
the head of state or of the
government, as the case may be,
through which the formal acceptance
of the treaty is proclaimed. A State
may provide in its domestic legislation
the process of ratification of a treaty.
The consent of the State to be bound
by a treaty is expressed by ratification
when: (a) the treaty provides for such
ratification,
(b)
it
is
otherwise
established that the negotiating States
agreed that ratification should be
required, (c) the representative of the
State has signed the treaty subject to
ratification, or (d) the intention of the
State to sign the treaty subject to
ratification appears from the full
powers of its representative, or was
expressed during the negotiation.
(BAYAN
[Bagong
Alyansang
Makabayan]
v.
Executive
Secretary Ronaldo Zamora, G.R.
No. 138570, Oct. 10, 2000, En
Banc [Buena])

The doctrine of rebus sic stantibus


does not operate automatically to
render the treaty inoperative. There is
a necessity for a formal act of
rejection, usually made by the head of
state, with a statement of the reasons
why compliance with the treaty is no
longer required.
(Santos III v.
Northwest Orient Airlines, 210
SCRA 256, June 23, 1992)

407. Explain
servanda rule.

the

pacta

sunt

Held: One of the oldest and most


fundamental rules in international law
is pacta sunt servanda international
agreements must be performed in
good faith. A treaty engagement is
not a mere moral obligation but
creates a legally binding obligation on
the parties x x x. A state which has
contracted
valid
international
obligations is bound to make in its
legislations such modifications as may
be necessary to ensure the fulfillment
of
the
obligations
undertaken.
(Tanada v. Angara, 272 SCRA 18,
May 2, 1997 [Panganiban])
408. Explain the "rebus sic stantibus"
rule (i.e., things remaining as they
are). Does it operate automatically to
render a treaty inoperative?
Held:
According to Jessup, the
doctrine constitutes an attempt to
formulate a legal principle which
would justify non-performance of a
treaty obligation if the conditions with
relation
to
which
the
parties
contracted have changed so materially
and so unexpectedly as to create a
situation in which the exaction of
performance would be unreasonable.
The key element of this doctrine is the
vital change in the condition of the
contracting parties that they could not

409. What is the


effective nationality
doctrine)?

doctrine of
(genuine link

Held: This principle is expressed in


Article 5 of the Hague Convention of
1930 on the Conflict of Nationality
Laws as follows:
Art. 5. Within a third State a person
having more than one nationality shall
be treated as if he had only one.
Without prejudice to the application of
its law in matters of personal status
and of any convention in force, a third
State shall, of the nationalities which
any such person possesses, recognize
exclusively in its territory either the
nationality of the country in which he
is habitually and principally resident or
the nationality of the country with
which in the circumstances he appears
to be in fact most closely connected.
(Frivaldo v. COMELEC, 174 SCRA
245, June 23, 1989)
410. What are the conditions before
foreign military bases, troops, or
facilities may be allowed in the
Philippines?
Ans.: After the expiration in
1991 of the Agreement between the
Republic of the Philippines and the
United States of America concerning
Military Bases, foreign military bases,
troops, or facilities shall not be allowed
in the Philippines except under a
treaty duly concurred in by the Senate
and, when the Congress so requires,
ratified by a majority of the votes cast
by the people in a national referendum
held for that purpose, and recognized
as a treaty by the other contracting
State. (Sec. 25, Art. XVIII, 1987
Constitution)
411. Which
provision
of
the
Constitution applies with regard to the

exercise by the Senate of its


constitutional power to concur with
the Visiting Forces Agreement (VFA)?
Held: One focal point of inquiry in
this controversy is the determination
of which provision of the Constitution
applies, with regard to the exercise by
the Senate of its constitutional power
to concur with the VFA. Petitioners
argue that Section 25, Article XVIII is
applicable considering that the VFA
has for its subject the presence of
foreign
military
troops
in
the
Philippines.
Respondents, on the
contrary, maintain that Section 21,
Article VII should apply inasmuch as
the VFA is not a basing arrangement
but an agreement which involves
merely the temporary visits of United
States personnel engaged in joint
military exercises.
The 1987 Philippine contains
two
provisions
requiring
the
concurrence of the Senate on treaties
or international agreements. Section
21, Article VII x x x reads:
"No treaty or international agreement
shall be valid and effective unless
concurred in by at least two-thirds of
all the Members of the Senate."
Section
provides:

25,

Article

XVIII,

"After the expiration in 1991 of the


Agreement between the Republic of
the Philippines and the United States
of America concerning Military Bases,
foreign military bases, troops, or
facilities shall not be allowed in the
Philippines except under a treaty duly
concurred in by the Senate and, when
the Congress so requires, ratified by a
majority of the votes cast by the
people in a national referendum held
for that purpose, and recognized as a
treaty by the other contracting State."
Section 21, Article VII deals with
treaties or international agreements in
general,
in
which
case,
the
concurrence of at least two-thirds (2/3)
of all the Members of the Senate is
required to make the subject treaty, or
international agreement, valid and
binding on the part of the Philippines.
This provision lays down the general
rule on treaties or international
agreements and applies to any form of
treaty with a wide variety of subject
matter, such as, but not limited to,
extradition or tax treaties or those
economic in nature. All treaties or
international agreements entered into

by the Philippines, regardless of


subject matter, coverage, or particular
designation or appellation, requires
the concurrence of the Senate to be
valid and effective.
In contrast, Section 25, Article
XVIII is a special provision that applies
to treaties which involve the presence
of foreign military bases, troops or
facilities in the Philippines. Under this
provision, the concurrence of the
Senate is only one of the requisites to
render
compliance
with
the
constitutional requirements and to
consider the agreement binding on the
Philippines. Section 25, Article XVIII
further requires that "foreign military
bases, troops, or facilities" may be
allowed in the Philippines only by
virtue of a treaty duly concurred in by
the Senate, ratified by a majority of
the
votes
cast
in
a
national
referendum held for that purpose if so
required by Congress, and recognized
as such by the other contracting State.
It is our considered view that
both constitutional provisions, far from
contradicting each other, actually
share some common ground. These
constitutional provisions both embody
phrases in the negative and thus, are
deemed prohibitory in mandate and
character. In particular, Section 21
opens with the clause "No treaty x x
x," and Section 25 contains the phrase
"shall not be allowed." Additionally, in
both instances, the concurrence of the
Senate is indispensable to render the
treaty or international agreement valid
and effective.
To our mind, the fact that the
President referred the VFA to the
Senate under Section 21, Article VII,
and that the Senate extended its
concurrence under the same provision,
is immaterial.
For in either case,
whether under Section 21, Article VII
or Section 25, Article XVIII, the
fundamental law is crystalline that the
concurrence
of
the
Senate
is
mandatory to comply with the strict
constitutional requirements.
On the whole, the VFA is an
agreement
which
defines
the
treatment of United States troops and
personnel visiting the Philippines. It
provides for the guidelines to govern
such visits of military personnel, and
further defines the rights of the United
States and the Philippine government
in the matter of criminal jurisdiction,
movement of vessels and aircraft,

importation
and
exportation
of
equipment, materials and supplies.

distinguish - Ubi lex non distinguit nec


nos distinguire debemos.

Undoubtedly, Section 25, Article


XVIII, which specifically deals with
treaties involving foreign military
bases, troops, or facilities, should
apply in the instant case. To a certain
extent and in a limited sense,
however, the provisions of Section 21,
Article VII will find applicability with
regard to the issue and for the sole
purpose of determining the number of
votes required to obtain the valid
concurrence of the Senate x x x.

In like manner, we do not


subscribe to the argument that
Section 25, Article XVIII is not
controlling since no foreign military
bases, but merely foreign troops and
facilities, are involved in the VFA.
Notably,
a
perusal
of
said
constitutional provision revels that the
proscription covers "foreign military
bases, troops, or facilities." Stated
differently, this prohibition is not
limited to the entry of troops and
facilities without any foreign bases
being established. The clause does
not refer to foreign military bases,
troops, or facilities" collectively but
treats
them
as
separate
and
independent subjects.
The use of
comma and disjunctive word "or"
clearly signifies disassociation and
independence of one thing from the
others included in the enumeration
(Castillo-Co v. Barbers, 290 SCRA 717,
723 [1998]), such that, the provision
contemplates three different situations
- a military treaty the subject of which
could be either (a) foreign bases (b)
foreign troops, or (c) foreign facilities any of the three standing alone places
it under the coverage of Section 25,
Article XVIII.

It is a finely-imbedded principle
in statutory construction that a special
provision or law prevails over a
general one. Lex specialis derogat
generali. Thus, where there is in the
same statute a particular enactment
and also a general one which, in its
most comprehensive sense, would
include what is embraced in the
former, the particular enactment must
be operative, and the general
enactment must be taken to affect
only such cases within its general
language which are not within the
provision of the particular enactment
(Manila Railroad Co. v. Collector of
Customs, 52 Phil. 950).
(BAYAN
[Bagong Alyansang Makabayan] v.
Executive
Secretary
Ronaldo
Zamora, G.R. No. 138570 and
Companion Cases, Oct. 10, 2000,
342 SCRA 449, 481-492, En Banc
[Buena])
412. Should the contention that since
the
VFA
merely
involved
the
temporary visits of United States
personnel engaged in joint military
exercises and not a basing agreement,
therefore, Sec. 21, Art. VII of the 1987
Constitution, and not Sec. 25, Art.
XVIII, should apply to the VFA, be
upheld?
Held: It is specious to argue
that Section 25, Article XVIII is
inapplicable
to
mere
transient
agreements for the reason that there
is no permanent placing of structure
for the establishment of a military
base. On this score, the Constitution
makes
no
distinction
between
"transient"
and
"permanent".
Certainly, we find nothing in Section
25, Article XVIII that requires foreign
troops or facilities to be stationed or
placed permanently in the Philippines.
It is a rudiment in legal
hermeneutics that when no distinction
is made by law, the Court should not

To this end, the intention of the


framers of the Charter, as manifested
during the deliberations of the 1986
Constitutional
Commission,
is
consistent with this interpretation x x
x.
Moreover,
military
bases
established within the territory of
another state is no longer viable
because of the alternatives offered by
new means and weapons of warfare
such as nuclear weapons, guided
missiles as well as huge sea vessels
that can stay afloat in the sea even for
months and years without returning to
their home country. These military
warships
are
actually
used
as
substitutes for a land-home base not
only of military aircraft but also of
military
personnel
and
facilities.
Besides, vessels are mobile as
compared to a land-based military
headquarters.
(BAYAN [Bagong
Alyansang
Makabayan]
v.
Executive
Secretary
Ronaldo
Zamora, G.R. No. 138570 and
Companion Cases, Oct. 10, 2000,
342 SCRA 449, 481-492, En Banc
[Buena])

413. Were the requirements of Sec.


25, Art. XVIII of the 1987 Constitution
complied with when the Senate gave
its concurrence to the VFA?
Held:
Section 25, Article XVIII
disallows
foreign
military
bases,
troops, or facilities in the country,
unless the following conditions are
sufficiently met, viz: (a) it must be
under a treaty; (b) the treaty must be
duly concurred in by the Senate and,
when so required by Congress, ratified
by a majority of the votes cast by the
people in a national referendum; and
(c) recognized as a treaty by the other
contracting state.
There is no dispute as to the presence
of the first two requisites in the case of
the VFA. The concurrence handed by
the Senate through Resolution No. 18
is in accordance with the provisions of
the Constitution, whether under the
general requirement in Section 21,
Article VII, or the specific mandate
mentioned in Section 25, Article XVIII,
the provision in the latter article
requiring ratification by a majority of
the
votes
cast
in
a
national
referendum being unnecessary since
Congress has not required it.
As to the matter of voting, Section 21,
Article VII particularly requires that a
treaty or international agreement, to
be valid and effective, must be
concurred in by at least two-thirds of
all the members of the Senate. On the
other hand, Section 25, Article XVIII
simply provides that the treaty be
"duly concurred in by the Senate."
Applying the foregoing constitutional
provisions, a two-thirds vote of all the
members of the Senate is clearly
required so that the concurrence
contemplated by law may be validly
obtained and deemed present. While
it is true that Section 25, Article XVIII
requires, among other things, that the
treaty - the VFA, in the instant case be "duly concurred in by the Senate,"
it is very true however that said
provision must be related and viewed
in light of the clear mandate embodied
in Section 21, Article VII, which in
more specific terms, requires that the
concurrence
of
a
treaty,
or
international agreement, be made by
a two-thirds vote of all the members of
the Senate.
Indeed, Section 25,
Article XVIII must not be treated in
isolation to Section 21, Article VII.
As
noted,
the
"concurrence
requirement" under Section 25, Article

XVIII must be construed in relation to


the provisions of Section 21, Article
VII. In a more particular language, the
concurrence
of
the
Senate
contemplated under Section 25,
Article XVIII means that at least twothirds of all the members of the
Senate favorably vote to concur with
the treaty - the VFA in the instant
case.
xxx
Having resolved that the first two
requisites prescribed in Section 25,
Article XVIII are present, we shall now
pass upon and delve on the
requirement that the VFA should be
recognized as a treaty by the United
States of America.
xxx
This Court is of the firm view that the
phrase "recognized as a treaty" means
that the other contracting party
accepts
or
acknowledges
the
agreement as a treaty (Ballantine's
Legal Dictionary, 1995). To require the
other contracting state, The United
States of America in this case, to
submit the VFA to the United States
Senate for concurrence pursuant to its
Constitution, is to accord strict
meaning to the phrase.
Well-entrenched is the principle that
the words used in the Constitution are
to be given their ordinary meaning
except where technical terms are
employed,
in
which
case
the
significance thus attached to them
prevails.
Its language should be
understood in the sense they have in
common use (J.M. Tuason & Co., Inc. v.
Land Tenure Association, 31 SCRA 413
[1970])
Moreover,
it
is
inconsequential
whether the United States treats the
VFA only as an executive agreement
because, under international law, an
executive agreement is as binding as a
treaty (Altman Co. v. United States,
224 US 263 [1942], cited in Coquia
and Defensor-Santiago, International
Law, 1998 Ed. P. 497). To be sure, as
long as the VFA possesses the
elements of an agreement under
international law, the said agreement
is to be taken equally as a treaty.
xxx
The records reveal that the United
States
Government,
through
Ambassador Thomas C. Hubbard, has

stated
that
the
United
States
government has fully committed to
living up to the terms of the VFA. For
as long as the United States of
America accepts or acknowledges the
VFA as a treaty, and binds itself further
to comply with its obligations under
the treaty, there is indeed marked
compliance with the mandate of the
Constitution.
Worth stressing too, is that the
ratification, by the President, of the
VFA and the concurrence of the Senate
should be taken as a clear and
unequivocal expression of our nation's
consent to be bound by said treaty,
with the concomitant duty to uphold
the obligations and responsibilities
embodied thereunder.
xxx
With the ratification of the VFA, which
is equivalent to final acceptance, and
with the exchange of notes between
the Philippines and the United States
of America, it now becomes obligatory
and incumbent on our part, under the
principles of international law, to be
bound by the terms of the agreement.
Thus, no less than Section 2, Article II
of the Constitution, declares that the
Philippines
adopts
the
generally
accepted principles of international
law as part of the law of the land and
adheres to the policy of peace,
equality, justice, freedom, cooperation
and amity with all nations. (BAYAN
[Bagong Alyansang Makabayan] v.
Executive
Secretary
Ronaldo
Zamora, G.R. No. 138570 and
Companion Cases, Oct. 10, 2000,
342 SCRA 449, 481-492, En Banc
[Buena])
414. Are the Balikatan exercises
covered by the Visiting Forces
Agreement?
Held:
The holding of
Balikatan 02-1 must be studied in
the framework of the treaty
antecedents to which the Philippines
bound itself. The first of these is the
Mutual Defense Treaty (MDT, for
brevity). The MDT has been described
as the core of the defense
relationship between the Philippines
and its traditional ally, the United
States. Its aim is to enhance the
strategic and technological capabilities
of our armed forces through joint
training with its American

counterparts; the Balikatan is the


largest such training exercise directly
supporting the MDTs objectives. It is
this treaty to which the VFA adverts
and the obligations thereunder which
it seeks to reaffirm.
The lapse of the US-Philippine Bases
Agreement in 1992 and the decision
not to renew it created a vacuum in
US-Philippine defense relations, that
is, until it was replaced by the Visiting
Forces Agreement. It should be
recalled that on October 10, 2000, by
a vote of eleven to three, this Court
upheld the validity of the VFA (BAYAN,
et. Al. v. Zamora, et. al., 342 SCRA 449
[2000]). The VFA provides the
regulatory mechanism by which
United States military and civilian
personnel [may visit] temporarily in
the Philippines in connection with
activities approved by the Philippine
Government. It contains provisions
relative to entry and departure of
American personnel, driving and
vehicle registration, criminal
jurisdiction, claims, importation and
exportation, movement of vessels and
aircraft, as well as the duration of the
agreement and its termination. It is
the VFA which gives continued
relevance to the MDT despite the
passage of years. Its primary goal is
to facilitate the promotion of optimal
cooperation between American and
Philippine military forces in the event
of an attack by a common foe.
The first question that should be
addressed is whether Balikatan 02-1
is covered by the Visiting Forces
Agreement. To resolve this, it is
necessary to refer to the VFA itself.
Not much help can be had therefrom,
unfortunately, since the terminology
employed is itself the source of the
problem. The VFA permits United
States personnel to engage, on an
impermanent basis, in activities, the
exact meaning of which was left
undefined. The expression is
ambiguous, permitting a wide scope of
undertakings subject only to the
approval of the Philippine government.
The sole encumbrance placed on its

definition is couched in the negative,


in that United States personnel must
abstain from any activity inconsistent
with the spirit of this agreement, and
in particular, from any political
activity. All other activities, in other
words, are fair game.
We are not completely unaided,
however. The Vienna Convention on
the Law of Treaties, which contains
provisos governing interpretations of
international agreements, state x x x.
It is clear from the foregoing that the
cardinal rule of interpretation must
involve an examination of the text,
which is presumed to verbalize the
parties intentions. The Convention
likewise dictates what may be used as
aids to deduce the meaning of terms,
which it refers to as the context of the
treaty, as well as other elements may
be taken into account alongside the
aforesaid context. X x x
The Terms of Reference rightly fall
within the context of the VFA.
After studied reflection, it appeared
farfetched that the ambiguity
surrounding the meaning of the word
activities arose from accident. In
our view, it was deliberately made
that way to give both parties a certain
leeway in negotiation. In this manner,
visiting US forces may sojourn in
Philippine territory for purposes other
than military. As conceived, the joint
exercises may include training on new
techniques of patrol and surveillance
to protect the nations marine
resources, sea search-and-destroy
operations to assist vessels in distress,
disaster relief operations, civic action
projects such as the building of school
houses, medical and humanitarian
missions, and the like.
Under these auspices, the VFA gives
legitimacy to the current Balikatan
exercises. It is only logical to assume
that Balikatan 02-1, a mutual antiterrorism advising, assisting and
training exercise, falls under the
umbrella of sanctioned or allowable

activities in the context of the


agreement. Both the history and
intent of the Mutual Defense Treaty
and the VFA support the conclusion
that combat-related activities as
opposed to combat itself such as the
one subject of the instant petition, are
indeed authorized. (Arthur D. Lim
and Paulino R. Ersando v.
Honorable Executive Secretary,
G.R. No. 151445, April 11, 2002,
En Banc [De Leon])

415. With the ratification of the


Visiting Forces Agreement (VFA), has it
now become obligatory and incumbent
on our part to be bound by its terms
even if it is asserted that said
agreement
contravenes
the
Constitution?
Held: With the ratification of the VFA,
which
is
equivalent
to
final
acceptance, and with the exchange of
notes between the Philippines and the
United States of America, it now
becomes obligatory and incumbent on
our part, under the principles of
international law, to be bound by the
terms of the agreement. Thus, no less
than Section 2, Article II of the
Constitution,
declares
that
the
Philippines
adopts
the
generally
accepted principles of international
law as part of the law of the land and
adheres to the policy of peace,
equality, justice, freedom, cooperation
and amity with all nations.
As a member of the family of nations,
the Philippines agrees to be bound by
generally accepted rules for the
conduct of its international relations.
While the international obligation
devolves upon the state and not upon
any particular branch, institution, or
individual member of its government,
the
Philippines
is
nonetheless
responsible for violations committed
by any branch or subdivision of its
government or any official thereof. As
an integral part of the community of
nations, we are responsible to assure
that our government, Constitution and
laws will carry out our international
obligation (Louis Henkin, Richard C.
Pugh, Oscar Schachter, Hans Smit,
International
Law,
Cases
and
Materials, 2nd Ed., American Casebook
Series, p. 136). Hence, we cannot
readily plead the Constitution as a
convenient excuse for non-compliance
with our obligations, duties and

responsibilities
law.

under

international

government decides to espouse the


claim, the latter ceases to be a private
cause.

Beyond this, Article 13 of the


Declaration of Rights and Duties of
States adopted by the International
Law Commission in 1949 provides:
Every State has the duty to carry out
in good faith its obligations arising
from treaties and other sources of
international law, and it may not
invoke provisions in its constitution or
its laws as an excuse for failure to
perform this duty. (Gerhard von Glahn,
supra, p. 487)
Equally important is Article 26 of the
Convention which provides that Every
treaty in force is binding upon the
parties to it and must be performed by
them in good faith. This is known as
the principle of pacta sunt servanda
which preserves the sanctity of
treaties and have been one of the
most
fundamental
principles
of
positive international law, supported
by the jurisprudence of international
tribunals (Harris, p. 634 cited in
Coquia, International Law, supra, p.
512). (BAYAN [Bagong Alyansang
Makabayan]
v.
Executive
Secretary Ronaldo Zamora, G.R.
No. 138570, Oct. 10, 2000, 342
SCRA 449, 492-493, En Banc
[Buena])
416. What must a person who feels
aggrieved by the acts of a foreign
sovereign do to espouse his cause?
Held: Private respondent is not
left without any legal remedy for the
redress of its grievances. Under both
Public
International
Law
and
Transnational Law, a person who feels
aggrieved by the acts of a foreign
sovereign can ask his own government
to
espouse
his
cause
through
diplomatic channels.

According to the Permanent


Court of International Justice, the
forerunner of the International Court of
Justice:
By taking up the case of one of its
subjects and by resorting to diplomatic
action
or
international
judicial
proceedings on his behalf, a State is in
reality asserting its own rights - its
right to ensure, in the person of its
subjects, respect for the rules of
international law (The Mavrommatis
Palestine Concessions, 1 Hudson,
World Court Reports 293, 302 [1924]).
(Holy See, The v. Rosario, Jr., 238
SCRA 524, 538-539, Dec. 1, 1994,
En Banc [Quiason])
417. What are the conditions before
the rights of belligerency may be
accorded the rebels?
Ans.:
As a matter of legal
theory, the rebels have to fulfill certain
conditions before the rights of
belligerency are accorded them,
namely:
1

2
3
4

418. Discuss the legal consequences


that follow recognition of belligerency.
Ans.:
1)

Private respondent can ask the


Philippine government, through the
Foreign Office, to espouse its claims
against the Holy See. Its first task is
to persuade the Philippine government
to take up with the Holy See the
validity of its claim. Of course, the
Foreign Office shall first make a
determination of the impact of its
espousal on the relations between the
Philippine government and the Holy
See (Young, Remedies of Private
Claimants Against Foreign States,
Selected Readings on Protection by
Law of Private Foreign Investments
905, 919 [1964]). Once the Philippine

An organized civil government that


has control and direction over the
armed struggle launched by the
rebels;
Occupation of a substantial portion of
the national territory;
Seriousness of the struggle, which
must be so widespread thereby
leaving no doubt as to the outcome;
Willingness on the part of the rebels to
observe the rules and customs of war.

Before recognition as such, it


is the legitimate government that is
responsible for the acts of the rebels
affecting foreign nationals and their
properties. Once recognition is given,
the legitimate government may no
longer be held responsible for their
acts; responsibility is shifted to the
rebel government;
2)
The legitimate government,
once it recognizes the rebels as
belligerents, is bound to observe the
laws and customs of war in conducting
the hostilities;

3)

From the point of view of


third States, the effect of recognition
of belligerency is to put them under
obligation to observe strict neutrality
and abide by the consequences arising
from that position;
4)
On the side of the rebels,
recognition of belligerency puts them
under responsibility to third States and
to the legitimate government for all
their acts which do not conform to the
laws and customs of war. (Salonga &
Yap, Public International Law, 5th
Ed. [1992], p. 33)
419. Should Courts blindly adhere
and
take
on
its
face
the
communication from the Department
of Foreign Affairs (DFA) that a person
is covered by any immunity?
Held:
Courts cannot blindly
adhere and take on its face the
communication from the DFA that
petitioner is covered by any immunity.
The DFAs determination that a certain
person is covered by immunity is only
preliminary which has no binding
effect in courts. In receiving ex parte
the DFAs advice and in motu proprio
dismissing the two criminal cases
without notice to the prosecution, the
latters right to due process was
violated. It should be noted that due
process is a right of the accused as
much as it is of the prosecution. The
needed inquiry in what capacity
petitioner was acting at the time of
the alleged utterances requires for its
resolution evidentiary basis that has
yet to be presented at the proper time
(See United States v. Guinto, 182
SCRA 644 [1990]). At any rate, it has
been ruled that the mere invocation of
the immunity clause does not ipso
facto result in the dropping of the
charges (Chavez v. Sandiganbayan,
193 SCRA 282 [1991]).
(Liang v.
People, 323 SCRA 692, Jan. 28,
2000, 1st Div. [Ynares-Santiago])
420. Discuss the basis of the
argument that a determination by the
DFA that a person is entitled to
diplomatic immunity is a political
question binding on the courts.
Held:
Petitioners argument
that
a
determination
by
the
Department of Foreign Affairs that he
is entitled to diplomatic immunity is a
political question binding on the
courts, is anchored on the ruling
enunciated in the case of WHO, et al.

v. Aquino, et al. (48 SCRA 242 [1972]),


viz:
It is a recognized principle of
international law and under our
system of separation of powers that
diplomatic immunity is essentially a
political question and courts should
refuse to look beyond a determination
by the executive branch of the
government, and where the plea of
diplomatic immunity is recognized and
affirmed by the executive branch of
the government as in the case at bar,
it is then the duty of the courts to
accept the claim of immunity upon
appropriate
suggestion
by
the
principal
law
officer
of
the
government, the Solicitor General in
this case, or other officer acting under
his direction. Hence, in adherence to
the settled principle that courts may
not so exercise their jurisdiction by
seizure and detention of property, as
to embarrass the executive arm of the
government in conducting foreign
relations, it is accepted doctrine that
in such cases the judicial department
of the government follows the action
of the political branch and will not
embarrass the latter by assuming an
antagonistic jurisdiction.
This ruling was reiterated in the
subsequent cases of International
Catholic Migration Commission v.
Calleja (190 SCRA 130 [1990]); The
Holy See v. Rosario, Jr. (238 SCRA 524
[1994]); Lasco v. United Nations (241
SCRA 681 [1995]); and DFA v. NLRC
(262 SCRA 38 [1996]).
The case of WHO v. Aquino
involved the search and seizure of
personal effects of petitioner Leonce
Verstuyft, an official of the WHO.
Verstuyft was certified to be entitled to
diplomatic immunity pursuant to the
Host Agreement executed between
the Philippines and the WHO.
ICMC v. Calleja concerned a
petition for certification election filed
against
ICMC
and
IRRI.
As
international organizations, ICMC and
IRRI were declared to possess
diplomatic immunity. It was held that
they are not subject to local
jurisdictions. It was ruled that the
exercise
of
jurisdiction
by
the
Department of Labor over the case
would defeat the very purpose of
immunity, which is to shield the affairs
of international organizations from
political pressure or control by the
host country and to ensure the

unhampered
functions.

performance

of

their

Holy See v. Rosario, Jr. involved


an action for annulment of sale of land
against the Holy See, as represented
by the Papal Nuncio. The Court upheld
the petitioners defense of sovereign
immunity.
It ruled that where a
diplomatic envoy is granted immunity
from the civil and administrative
jurisdiction of the receiving state over
any real action relating to private
immovable property situated in the
territory of the receiving state, which
the envoy holds on behalf of the
sending state for the purposes of the
mission, with all the more reason
should immunity be recognized as
regards the sovereign itself, which in
that case is the Holy See.
In Lasco v. United Nations, the
United Nations Revolving Fund for
Natural Resources Exploration was
sued before the NLRC for illegal
dismissal. The Court again upheld the
doctrine
of
diplomatic
immunity
invoked by the Fund.
Finally, DFA v. NLRC involved an
illegal dismissal case filed against the
Asian Development Bank. Pursuant to
its Charter and the Headquarters
Agreement, the diplomatic immunity
of the Asian Development Bank was
recognized by the Court.
It bears to stress that all of
these cases pertain to the diplomatic
immunity enjoyed by international
organizations. Petitioner asserts that
he is entitled to the same diplomatic
immunity
and
he
cannot
be
prosecuted for acts allegedly done in
the exercise of his official functions.
The
term
organizations

international

is generally used to describe an


organization set up by agreement
between two or more states. Under
contemporary international law, such
organizations are endowed with some
degree
of
international
legal
personality such that they are capable
of exercising specific rights, duties and
powers. They are organized mainly as
a means for conducting general
international business in which the
member states have an interest.
(ICMC v. Calleja, supra note 2)
International
public
have been defined as:

officials

x x x persons who, on the basis of an


international treaty constituting a
particular international community,
are appointed by this international
community, or by an organ of it, and
are under its control to exercise, in a
continuous way, functions in the
interest of this particular international
community, and who are subject to a
particular personal status. (John Kerry
King, The Privileges and Immunities of
the
Personnel
of
International
Organizations
xiii
[1949],
citing:
Suzanne
Basdevant,
Les
Functionnaires Internationaux [Paris:
1931], Chapter I)
Specialized
agencies
are
international
organizations
having
functions in particular fields, such as
posts, telecommunications, railways,
canals, rivers, sea transport, civil
aviation, meteorology, atomic energy,
finance, trade, education and culture,
health and refugees (ICMC v. Calleja,
et al., citing Articles 57 and 63 of the
United Nations Charter). (Concurring
Opinion, Puno J., in Jeffrey Liang
[Huefeng] v. People, G.R. No.
125865, Mar. 26, 2001, 1st Div.
[Motion for Reconsideration])
421. What
are
the
differences
between Diplomatic and International
Immunities? Discuss.
Held: There are three major
differences between diplomatic and
international immunities. Firstly, one
of the recognized limitations of
diplomatic immunity is that members
of the diplomatic staff of a mission
may be appointed from among the
nationals of the receiving State only
with the express consent of that State;
apart from inviolability and immunity
from jurisdiction in respect of official
acts performed in the exercise of their
functions, nationals enjoy only such
privileges and immunities as may be
granted by the receiving State.
International immunities may be
specially important in relation to the
State of which the official is a national.
Secondly,
the
immunity
of
a
diplomatic agent from the jurisdiction
of the receiving State does not exempt
him from the jurisdiction of the
sending State; in the case of
international immunities there is no
sending State and an equivalent for
the jurisdiction of the Sending State
therefore has to be found either in
waiver of immunity or in some
international disciplinary or judicial
procedure.
Thirdly, the effective
sanctions which secure respect for

diplomatic immunity are the principle


of reciprocity and the danger of
retaliation by the aggrieved State;
international immunities enjoy no
similar protection (C. Wilfred Jenks,
Contemporary
Development
in
International Immunities xxxvii [1961])
(Concurring Opinion, Puno J., in
Jeffrey Liang [Huefeng] v. People,
G.R. No. 125865, Mar. 26, 2001,
1st
Div.
[Motion
for
Reconsideration])
422. Discuss the immunity of
International Officials.
Held:
The generally accepted
principles which are now regarded as
the
foundation
of
international
immunities are contained in the ILO
Memorandum, which reduced them in
three basic propositions, namely: (1)
that international institutions should
have a status which protects them
against control or interference by any
one government in the performance of
functions for the effective discharge of
which
they
are
responsible
to
democratically
constituted
international bodies in which all the
nations concerned are represented; (2)
that no country should derive any
financial advantage by levying fiscal
charges on common international
funds; and (3) that the international
organization should, as a collectivity of
States Members, be accorded the
facilities for the conduct of its official
business customarily extended to each
other by its individual member States.
The
thinking
underlying
these
propositions is essentially institutional
in character. It is not concerned with
the status, dignity or privileges of
individuals, but with the elements of
functional independence necessary to
free international institutions from
national control and to enable them to
discharge
their
responsibilities
impartially on behalf of all their
members (Id. at 17). (Concurring
Opinion, Puno J., in Jeffrey Liang
[Huefeng] v. People, G.R. No.
125865, Mar. 26, 2001, 1st Div.
[Motion for Reconsideration])

organizations. The first is by simple


conventional stipulation, as was the
case in the Hague Conventions of
1899 and 1907. The second is by
internal
legislation
whereby
the
government of a state, upon whose
territory the international organization
is to carry out its functions, recognizes
the international character of the
organization and grants, by unilateral
measures, certain privileges and
immunities to better assure the
successful
functioning
of
the
organization and its personnel. In this
situation, treaty obligation for the
state in question to grant concessions
is lacking. Such was the case with the
Central Commission of the Rhine at
Strasbourg and the International
Institute of Agriculture at Rome. The
third is a combination of the first two.
In this third method, one finds a
conventional obligation to recognize a
certain status of an international
organization and its personnel, but the
status is described in broad and
general terms. The specific definition
and application of those general terms
are determined by an accord between
the organization itself and the state
wherein it is located. This is the case
with the League of Nations, the
Permanent Court of Justice, and the
United Nations (J.K. King, supra note
12, at 81).
The Asian Development Bank
and its Personnel fall under this third
category.

423. What are the three methods of


granting privileges and immunities to
the
personnel
of
international
organizations? Under what category
does the Asian Development Bank and
its Personnel fall?

There is a connection between


diplomatic privileges and immunities
and those extended to international
officials. The connection consists in
the
granting,
by
contractual
provisions, of the relatively wellestablished
body
of
diplomatic
privileges
and
immunities
to
international functionaries.
This
connection is purely historical. Both
types of officials find the basis of their
special status in the necessity of
retaining functional independence and
freedom from interference by the state
of residence.
However, the legal
relationship between an ambassador
and the state to which he is accredited
is
entirely
different
from
the
relationship between the international
official and those states upon whose
territory he might carry out its
functions (See Id. at 255).

Held: Positive international law has


devised three methods of granting
privileges and immunities to the
personnel
of
international

The privileges and immunities of


diplomats and those of international
officials rest upon different legal
foundations.
Whereas
those

immunities awarded to diplomatic


agents are a right of the sending state
based on customary international law,
those granted to international officials
are based on treaty or conventional
law.
Customary international law
places no obligation on a state to
recognize a special status of an
international official or to grant him
jurisdictional immunities.
Such an
obligation can only result from specific
treaty provisions (Id. at 25-26).
The special status of the diplomatic
envoy is regulated by the principle of
reciprocity by which a state is free to
treat the envoy of another state as its
envoys are treated by that state. The
juridical basis of the diplomats
position is firmly established in
customary international law.
The
diplomatic envoy is appointed by the
sending State but it has to make
certain that the agreement of the
receiving State has been given for the
person it proposes to accredit as head
of the mission to that State (Article 4,
Vienna Convention on Diplomatic
Relations).
The staff personnel of an international
organization

the
international
officials assume a different position
as regards their special status. They
are appointed or elected to their
position by the organization itself, or
by a competent organ of it; they are
responsible to the organization and
their official acts are imputed to it.
The juridical basis of their special
position is found in conventional law
(J.K. King, supra note 12, at xiii), since
there is no established basis of usage
or custom in the case of the
international official. Moreover, the
relationship between an international
organization and a member-state does
not admit of the principle of reciprocity
(Id. at 27), for it is contradictory to the
basic principle of equality of states.
An international organization carries
out functions in the interest of every
member
state
equally.
The
international official does not carry out
his functions in the interest of any
state, but in serving the organization
he serves, indirectly, each state
equally. He cannot be, legally, the
object of the operation of the principle
of reciprocity between states under
such circumstances. It is contrary to
the principle of equality of states for
one state member of an international
organization to assert a capacity to
extract special privileges for its
nationals from other member states
on the basis of a status awarded by it

to an international organization. It is
upon this principle of sovereign
equality
that
international
organizations are built.
It follows from this same legal
circumstance that a state called upon
to admit an official of an international
organization does not have a capacity
to declare him persona non grata.
The functions of the diplomat and
those of the international official are
quite different. Those of the diplomat
are functions in the national interest.
The task of the ambassador is to
represent his state, and its specific
interest, at the capital of another
state.
The
functions
of
the
international official are carried out in
the international interest. He does not
represent a state or the interest of any
specific state. He does not usually
represent the organization in the
true sense of that term. His functions
normally are administrative, although
they may be judicial or executive, but
they are rarely political or functions of
representation, such as those of the
diplomat.
There is a difference of degree as well
as of kind. The interruption of the
activities of a diplomatic agent is likely
to produce serious harm to the
purposes for which his immunities
were granted. But the interruption of
the activities of the international
official does not, usually, cause serious
dislocation of the functions of an
international secretariat (id. at 254257).
On the other hand, they are similar in
the sense that acts performed in an
official capacity by either a diplomatic
envoy or an international official are
not attributable to him as an individual
but are imputed to the entity he
represents, the state in the case of the
diplomat, and the organization in the
case of the international official (Id. at
103). ). (Concurring Opinion, Puno
J., in Jeffrey Liang [Huefeng] v.
People, G.R. No. 125865, Mar. 26,
2001,
1st
Div.
[Motion
for
Reconsideration])
424. What is the reason behind the
current
tendency
of
reducing
privileges and immunities of personnel
of international organizations to a
minimum?
Held: Looking back over 150 years of
privileges and immunities granted to
the
personnel
of
international

organizations, it is clear that they were


accorded a wide scope of protection in
the exercise of their functions The
Rhine Treaty of 1804 between the
German Empire and France which
provided all the rights of neutrality
to persons employed in regulating
navigation
in
the
international
interest; The Treaty of Berlin of 1878
which
granted
the
European
Commission of the Danube complete
independence of territorial authorities
in the exercise of its functions; The
Convention of the League which
granted diplomatic immunities and
privileges.
Today, the age of the
United Nations finds the scope of
protection narrowed.
The current
tendency is to reduce privileges and
immunities
of
personnel
of
international
organizations
to
a
minimum. The tendency cannot be
considered as a lowering of the
standard but rather as a recognition
that the problem on the privileges and
immunities of international officials is
new.
The solution to the problem
presented by the extension of
diplomatic
prerogatives
to
international functionaries lies in the
general reduction of the special
position of both types of agents in that
the special status of each agent is
granted in the interest of function.
The
wide
grant
of
diplomatic
prerogatives was curtailed because of
practical necessity and because the
proper functioning of the organization
did not require such extensive
immunity for its officials. While the
current direction of the law seems to
be to narrow the prerogatives of the
personnel
of
international
organizations, the reverse is true with
respect to the prerogatives of the
organizations themselves, considered
as legal entities. Historically, states
have been more generous in granting
privileges
and
immunities
to
organizations than they have to the
personnel of these organizations (J.K.
King, supra note 12, at 253-268).
Thus, Section 2 of the General
Convention on the Privileges and
Immunities of the United Nations
states that the UN shall enjoy
immunity from every form of legal
process except insofar as in any
particular case it has expressly waived
its immunity.
Section 4 of the
Convention on the Privileges and
Immunities
of
the
Specialized
Agencies likewise provides that the
specialized
agencies
shall
enjoy
immunity from every form of legal
process subject to the same exception.

Finally, Article 50[1] of the ADB


Charter and Section 5 of the
Headquarters
Agreement
similarly
provide that the bank shall enjoy
immunity from every form of legal
process, except in cases arising out of
or in connection with the exercise of
its powers to borrow money, to
guarantee obligations, or to buy and
sell or underwrite the sale of
securities.
The phrase immunity from every
form of legal process as used in the
UN General Convention has been
interpreted
to
mean
absolute
immunity from a states jurisdiction to
adjudicate or enforce its law by legal
process, and it is said that states have
not sought to restrict that immunity of
the United Nations by interpretation or
amendment.
Similar provisions are
contained in the Special Agencies
Convention as well as in the ADB
Charter and Headquarters Agreement.
These organizations were accorded
privileges and immunities in their
charters by language similar to that
applicable to the United Nations. It is
clear
therefore
that
these
organizations were intended to have
similar privileges and immunities (1
Restatement of the Law Third 498501).
From this, it can be easily
deduced
that
international
organizations enjoy absolute immunity
similar to the diplomatic prerogatives
granted to diplomatic envoys.
Even in the United States this seems
to be the prevailing rule x x x.
On the other hand, international
officials are governed by a different
rule.
Section 18[a] of the General
Convention
on
Privileges
and
Immunities of the United Nations
states that officials of the United
Nations shall be immune from legal
process in respect of words spoken or
written and all acts performed by
them in their official capacity. The
Convention on Specialized Agencies
carries exactly the same provision.
The Charter of the ADB provides under
Article
55[i]
that
officers
and
employees of the bank shall be
immune from legal process with
respect to acts performed by them in
their official capacity except when the
Bank waives immunity. Section 45 [a]
of the ADB Headquarters Agreement
accords the same immunity to the
officers and staff of the bank. There
can be no dispute that international
officials are entitled to immunity only
with respect to acts performed in their

official capacity, unlike international


organizations which enjoy absolute
immunity.
Clearly, the most important immunity
to an international official, in the
discharge
of
his
international
functions, is immunity from local
jurisdiction. There is no argument in
doctrine or practice with the principle
that an international official is
independent of the jurisdiction of the
local authorities for his official acts.
Those acts are not his, but are
imputed to the organization, and
without waiver the local courts cannot
hold him liable for them. In strict law,
it would seem that even the
organization itself could have no right
to waive an officials immunity for his
official acts.
This permits local
authorities to assume jurisdiction over
an individual for an act which is not, in
the wider sense of the term, his act al
all. It is the organization itself, as a
juristic person, which should waive its
own immunity and appear in court, not
the individual, except insofar as he
appears
in
the
name
of
the
organization. Provisions for immunity
from jurisdiction for official acts
appear,
aside
from
the
aforementioned treatises, in the
constitution
of
most
modern
international organizations.
The
acceptance
of
the
principle
is
sufficiently widespread to be regarded
as declaratory of international law (J.K.
King, supra note 12, at 258-259)
(Concurring Opinion, Puno J., in
Jeffrey Liang [Huefeng] v. People,
G.R. No. 125865, Mar. 26, 2001,
1st
Div.
[Motion
for
Reconsideration])
425. What is the status of the
international official with respect to his
private acts?
Held: Section 18 [a] of the
General
Convention
has
been
interpreted to mean that officials of
the specified categories are denied
immunity from local jurisdiction for
acts of their private life and empowers
local courts to assume jurisdiction in
such cases without the necessity of
waiver (Id. at 186). It has earlier been
mentioned
that
historically,
international officials were granted
diplomatic privileges and immunities
and were thus considered immune for
both private and official acts.
In
practice, this wide grant of diplomatic
prerogatives was curtailed because of
practical necessity and because the
proper functioning of the organization

did not require such exclusive


immunity for its officials. Thus, the
current status of the law does not
maintain
that
states
grant
jurisdictional immunity to international
officials for acts of their private lives
(But see id. at 259. It is important to
note
that
the
submission
of
international
officials
to
local
jurisdiction for private acts is not
completely accepted in doctrine and
theory.
Jenks, in particular, has
argued for complete jurisdictional
immunity, as has Hammarskjold.).
This much is explicit from the charter
and Headquarters Agreement of the
ADB which contain substantially
similar provisions to that of the
General
convention.
(Concurring
Opinion, Puno J., in Jeffrey Liang
[Huefeng] v. People, G.R. No.
125865, Mar. 26, 2001, 1st Div.
[Motion for Reconsideration])
426. Who is competent to determine
whether a given act of international
officials and representatives is private
or official?
Held: In connection with this
question, the current tendency to
narrow the scope of privileges ad
immunities of international officials
and representatives is most apparent.
Prior to the regime of the United
Nations, the determination of this
question rested with the organization
and its decision was final. By the new
formula, the state itself tends to
assume this competence.
If the
organization is dissatisfied with the
decision, under the provisions of the
General Convention of the United
Nations, or the Special Convention for
Specialized
Agencies,
the
Swiss
Arrangement,
and
other
current
dominant instruments, it may appeal
to an international tribunal by
procedures
outlined
in
these
instruments. Thus, the state assumes
this competence in the first instance.
It means that, if a local court assumes
jurisdiction over an act without the
necessity
of
waiver
from
the
organization, the determination of the
nature of the act is made at the
national level (Id. at 260-261).
It appears that the inclination is
to place the competence to determine
the nature of an act as private or
official in the courts of the state
concerned. That the practical notion
seems to be to leave to the local
courts determination of whether or not
a given act is official or private does
not necessarily mean that such

determination is final. If the United


Nations questions the decision of the
Court, it may invoke proceedings for
settlement of disputes between the
organization and the member states
as provided in Section 30 of the
General Convention.
Thus, the
decision as to whether a given act is
official or private is made by the
national courts in the first instance,
but it may be subjected to review in
the international level if questioned by
the United Nations (Id. at 189).
xxx
Under the Third Restatement of the
Law, it is suggested that since an
international official does not enjoy
personal inviolability from arrest or
detention and has immunity only with
respect to official acts, he is subject to
judicial or administrative process and
must claim his immunity in the
proceedings by showing that the act in
question was an official act. Whether
an act was performed in the
individuals official capacity is a
question for the court in which a
proceeding is brought, but if the
international organization disputes the
courts finding, the dispute between
that organization and the state of the
forum is to be resolved by negotiation,
by an agreed mode of settlement or
by
advisory
opinion
of
the
International
Court
of
Justice
(Restatement of the Law Third 512).
Recognizing the difficulty that by
reason of the right of a national court
to assume jurisdiction over private
acts without a waiver of immunity, the
determination of the official or private
character of a particular act may pass
from international to national, Jenks
proposes three ways of avoiding
difficulty in the matter. The first would
be for a municipal court before which
a question of the official or private
character of a particular act arose to
accept as conclusive in the matter any
claim by the international organization
that the act was official in character,
such a claim being regarded as
equivalent to a governmental claim
that a particular act is an act of State.
Such a claim would be in effect a claim
by
the
organization
that
the
proceedings against the official were a
violation of the jurisdictional immunity
of the organization itself which is
unqualified and therefore not subject
to delimitation in the discretion of the
municipal court. The second would be
for a court to accept as conclusive in
the matter a statement by the

executive government of the country


where the matter arises certifying the
official character of the act. The third
would be to have recourse to the
procedure of international arbitration.
Jenks opines that it is possible that
none of these three solutions would be
applicable in all cases; the first might
be readily acceptable only in the
clearest cases and the second is
available only if the executive
government of the country where the
matter arises concurs in the view of
the
international
organization
concerning the official character of the
act. However, he surmises that taken
in
combination,
these
various
possibilities may afford the elements
of a solution to the problem (Jenks,
supra
note
14,
at
117-118).
(Concurring Opinion, Puno J., in
Jeffrey Liang [Huefeng] v. People,
G.R. No. 125865, Mar. 26, 2001,
1st
Div.
[Motion
for
Reconsideration])
427. Discuss the extent of the
international officials immunity for
official acts.
Held: One final point. The
international officials immunity for
official acts may be likened to a
consular officials immunity from
arrest, detention, and criminal or civil
process which is not absolute but
applies only to acts or omissions in the
performance of his official functions, in
the absence of special agreement.
Since a consular officer is not immune
from all legal processes, he must
respond to any process and plead and
prove immunity on the ground that the
act or omission underlying the process
was in the performance of his official
functions. The issue has not been
authoritatively
determined,
but
apparently the burden is on the
consular official to prove his status as
well as his exemption in the
circumstances. In the United States,
the US Department of State generally
has left it to the courts to determine
whether a particular act was within a
consular officers official duties (1
Restatement of the Law Third 475477). (Concurring Opinion, Puno J.,
in Jeffrey Liang [Huefeng] v.
People, G.R. No. 125865, Mar. 26,
2001,
1st
Div.
[Motion
for
Reconsideration])
428. State the occasions when the
use of force may be allowed under the
UN Charter.

Ans.: There are only two occasions


when the use of force is allowed under
the UN Charter. The first is when it is
authorized in pursuance of the
enforcement action that may be
decreed by the Security Council under
Art. 42. The second is when it is
employed in the exercise of the
inherent right of self-defense under
conditions prescribed in Art. 51.
(Justice Isagani A. Cruz, in an
article entitled A New World
Order written in his column
Separate Opinion published in
the March 30, 2003 issue of the
Philippines Daily Inquirer)

made the armed attack first, thus


becoming the aggressor, not Iraq. Iraq
is now not only exercising its inherent
right of self-defense as recognized by
the UN Charter. (Justice Isagani A.
Cruz, in an article entitled A New
World Order written in his
column
Separate
Opinion
published in the March 30, 2003
issue of the Philippines Daily
Inquirer)

429. Is the United States justified in


invading Iraq invoking its right to
defend itself against an expected
attack by Iraq with the use of its
biological and chemical weapons of
mass destruction?

Ans.: Even if Iraqs hidden arsenal is


discovered or actually used and the
United States is justified in its
suspicions, that circumstance will not
validate the procedure taken against
Iraq. It is like searching a person
without warrant and curing the
irregularity with the discovery of
prohibited drugs in his possession.
The process cannot be reversed. The
warrant must first be issued before the
search and seizure can be made.

Ans.: The United States is invoking


its right to defend itself against an
expected attack by Iraq with the use
of its biological and chemical weapons
of mass destruction.
There is no
evidence of such a threat, but Bush is
probably invoking the modern view
that a state does not have to wait until
the potential enemy fires first. The
cowboy
from
Texas
says
that
outdrawing the foe who is about to
shoot is an act of self-defense.
Art. 51 says, however, that there must
first be an armed attack before a
state can exercise its inherent right of
self-defense, and only until the
Security Council, to which
the
aggression should be reported, shall
have taken the necessary measures to
maintain international peace and
security. It was the United States that

430. Will the subsequent discovery of


weapons of mass destruction in Iraq
after its invasion by the US justify the
attack initiated by the latter?

The American invasion was made


without permission from the Security
Council as required by the UN Charter.
Any subsequent discovery of the
prohibited biological and chemical
weapons will not retroactively legalize
that invasion, which was, legally
speaking, null and void ab initio.
(Justice Isagani A. Cruz, in an
article entitled A New World
Order written in his column
Separate Opinion published in
the March 30, 2003 issue of the
Philippines Daily Inquirer)

Potrebbero piacerti anche